Obsfetrics Gynecology: & Department

You might also like

Download as pdf or txt
Download as pdf or txt
You are on page 1of 222

'It is more blessed to give than to receive.

By

,. t

Obsfetrics & Gynecology Department


Faculty of Medicine
Ain Shams Ufiiversity
-'
WhiteKnightLove

t_
Freely you have received; freely give.

Obstetrics and Gynecology Department


Faculty of Medicine
Ain Shams University

OBSTETRICS AND GYNECOLOGY

STUDY GUIDE
FOR UNDERGRADUATE STUDENTS

- MCQs
- Written questions
- lnstruments

Chief Editor:
Prof. Gamal Wafa
Chairman of Obstetrics & Gynecology Department
Faculty of Medicine
Ain Shams University

Co-Editor:
Prof. Ashraf Fawzy Nabhan
Prof. Mohammad Abd Elhameed

Sixth Edition
2012
WhiteKnightLove
'It is more blessed to give than to receive.

First Edition, 2004


Chief Editor:
Prof, Ali Elyan
Co-Editor:
Prof. Essam Ammar

Second Edition,2005
Chief Editor:
Prof. Mahmoud El-Shourbagy
Co-Editors:
Prof. M. Nabegh El-Mahallawi
Prof. Magdy Mohamed Kamal
Prof. Ayman Abul-Nour
Asst. Prof. Mohammad Abdel-Hameed

Third Edition, 2006


Chief Editor:
Prof. Mahmoud El-Shourbagy
Co-Editors:
Prof. M. Nabegh El-Mahallarvi
Prof. Essam M. Ammar
Prof. Magdy Mohamed Kamal
Prof. Ayman Abul-Nour
Asst. Prof. Mohammad Abdel-Hameed

Fourth Edition,2007
Editor:
Prof. M. Nabegh El-Mahallawi

Fifth Edition,2010
Chief Editor:
Prof. Ahmed I. Abou Gabal
Co-Editors:
Prof. Essam M. Ammar
Prof. Ihab H. Abdel Fattah
Prof. Alaa H. Elfeky
Prof. Mohammad Abd Elhameed

Previous editions copyrighted 2004, 2005, 2006, 2007, 2010

All rights rcscrved. No part of, this publication may be reproduced, stored in a retrieval system or
WhiteKnightLove

transmitted in any lrom or by any mcans, clectronic, mechanical, photocopying, recording or otherwise,
rvithout the prior rvritten permission ol the Chiei Editor (Thc Chairman of Obstetrics & Gynecology
Department- A in Shams University).
Freely you have received; freely give.

sxgtrAcE,
" Study guide in obstetrics and gltnecolog/" /s intended
primarily for the undergraduate students preparin7 for the final
obstetrical and gynecological examination, This book is designed
mainly for undergraduate students, yet it would also be a beneficial
guide to postgraduate candidates. The book is designed to assess the
understanding and analysis of the knowledge covered in the
undergraduate books prepared by the staff members of the
Department of Obstetrics and Gynecology, Ain Shams University. It
includes dffirent types of questions, some forms of the previous
edition have been omitted, and others are represented in a new
.for*.

The study guide should not be the first reading, and is best
used at the end of the teaching course to assess the learning process,
The book is provoking for the students and helps to plan the
revision. It highlights ond elaborates some of the important teaching
points, and it mainly helps self-assessment. The questions are good
example to the style appearing at the examination.

The book is the collective effort of the staff members of the


Department of Obstetrics and Gynecology, Ain Shams University.
The editors would like to thank these experts for their time and effirt
in preparing this new edition.

All suggestions and comments are welcome and will be


considered in preparing the next edition.
WhiteKnightLove
'It is more blessed to give than to receive.

Tahle of Contents
PREFACE

MULTIPLE CHOICE QUESTIONS


Obstetrics:
Normal Pregnancy and Normal Labor I
Obstetrical and Non-Obstetrical Cornplications During Pregnancy 23
Abnornral labor 52
Normal and Abnormal Puerperiurn 68
The Fetus and the Newborn 70
Operative Obstetrics 75
ANSWERS for Obstetrical MCQs 82
Gynecology: 85
Applied Basic Sciences 85
Patient Evaluation 96
Benign Conditions Of Reproductive Organs And Menstrual Disorders 98
The Breast in Gynecological Practice I 16
Gynecological Infections n7
Urogynecology and Genital Displacement 125
Gynecological Oncology r30
Reproductive Endocrinology and Inferti lity 141
Farnily Planning 154
ANSWERS for Gynecological MCQs r60

WzuTTEN QUESTIONS t63

INSTRUMENTS 180

WhiteKnightLove
Freely you have received; freely give.

WhiteKnightLove
'It is more blessed to give than to receive.

OBSTETRICS

NORMAL PREGNANCY AND NORMAL LABOR

Direction: Select the ONE BEST lettered ans\yer or completion in each question.

l- Fertilization:
a) Occurs in the uterine cavity.
b) If occuned by two spermatozoa, causes a trisomic conceptus,
c) Is associated with a surge of maternal luteinizing hormone.
d) Is associated with production of the first polar body.
e) Depends on hyaluronidaze release by the sperm.
2- The followlng statement regarding fcrtilization is CORRECT:
a) It occurs in the ampullary region of the fallopian tube.
b) The sperm penetrates the ovum by the action ofprostaglandins.
c) After penetrating the ovum, the head of the sperm form the zygote.
d) The fertilized ovum starts to divide by meiotic division to form the zygote.
e) Multiple sperms succeed in penetrating the ovum.
3- In early development:
a) The amnion is a double layer of fetal mesodermal origin.
b) The decidua capsularis is a component of the chorion,
c) Chorionic villi are the functional honnonal units.
d) Human chorionic gonadotrophin is produced mainly by cytotrophoblast,
e) Relaxin is produced by the chorion levae.
4- As regards fertilization:
a) The ovum is swept into the fallopian tube in stage of secondary oocyte.
b) Fertilization occurs in the ampullary region of fallopian tube.
c) The ovum is carried down by passive fluid currents.
d) All of the above.
5- The sperm penetrates the zona pellcida by action oft
a) Zonaprotein.
b) Hyalouronidaze in acrosomal cap and movelrlent of sperrn tail.
c)
Elastase & proteinase,
d) None ofthe above.
6- Thc morula is:
a) A cell resulting from fusion of rnale & female gametes.
b) A mass of blastomere cells.
c) Having a blastocyst cavity.
d) Consisting of inner cell mass & trophoblast.
e) Consisting of l2-14 cells.
7- The embryonic period extends to:
a) End of 5 weeks of intrauterine life.
b) End of 7 weeks of intrauterine life.
WhiteKnightLove

c) End of l0 weeks of intrauterine life.


d) End of I I weeks of intrauterine life.
e) End of l3 weeks of intrauterine life.
-1-
Freely you have received; freely give.

8- The trophoblast:
a) Develops from inner celI nrass.
b) Forms the villi.
c) Has an outer layer ofcytotrophoblast.
d) Forms the villi in 2 stages.
9- The decidua:
a) It is modified endometrium of pregnancy.
b) It is due to action of estrogen on the endometrium.
c) The decidua basalis is the parl overlying the embryo.
d) Does not protect against the invasive porver oftrophoblast.
e) Is anatomically divided into four parts.
l0- Functions of the decidua include:
a) Implantation and nutrition of blastocyst.
b) Sharing in formation of placenta.
c) Protection against the invasive power oftrophoblast.
d) All of the above.
ll- The normal placenta:
a) Develops from decidua basalis and chorion frondosum.
b) ls usually implanted in lorver uterine segment.
c) Its weight at full term is one kg.
d) The maternal surface is covered by amnion.
e) Normally separates before the end of the second stage of labor.
l2- The placental barrier:
a) Separates maternal blood from fetal blood.
b) Consists of syncytium and Langhan's layers alone.
c) Does not allow the passage ofrubella virus,
d) It increases in thickness as pregnancy advances.
e) Allthe above.
13- Placental transfer of substances may occur by variety of mechanisms including:
a) Simple diffusion.
b) Facilitated diffusion.
c) Active transport.
d) Endocytosis.
e) All of the above.
l4- The succenturiate placenta consists of:
a) Two equal placental lobes connected by placental tissue.
b) Two equal placental lobes connected by membranes.
c) Srnall accessory lobe connected to the main placenta by placental tissue,
d) Small accessory lobe connected to the main placenta by membranes.
e) Small accessory lobe not connected to the main placenta.
15- As regards the umbilical cord:
a) It has two veins and one artery,
b) Its average length at fullterm is 55 cm.
c) It is not covered by amnion.
d) Polyhydramnios may be associated rvith short cord.
e) All of the above.
l6- Abnormalities of the umbilicalcord include:
a) Marginal insertion and battledore placenta.
b) True knots,
c) Velamentous insertion.
WhiteKnightLove

d) All of the above.


e) None ofthe above.

-2-
'It is more blessed to give than to receive.

l7- Concerning fetal circulation:


a) The umbilical vein carries non oxygenated blood,
b) Blood flows from left atrium to right atrium through foramen ovale.
c) Doppler ultrasound is not helpful in studying fetal hemodynamics.
d) Is characterized by 3 shunts via ductus venosus, foramen ovale & ductus arteriosus.
l8- Post natal changes in fetal circulation incturtc:
a) Umbilical vein forms the ligamentum teres.
b) Umbilical arteries form the hypogastric ligaments.
c) Ductus venosus forms the Iigamentum venosum.
d) All of the above.
e) None ofthe above.
l9- The functional unit of the placenta is:
a) The decidua.
b) The chorionic villus.
c) The cotelydon.
d) The cytotrophoblast.
e) The chorion levae.
20- The decidrla:
a) is the endometrium of the upper uterine segment.
b) secretes human placental lactogen.
c) has a relation with trophoblast that follorvs the laws of transplantation immunology.
d) expresses cell adhesion molecules during the process of implantation.
e) parietalis shares in the formation of the placenta.
2l- Which of the following tissues DO NOT lie in between the maternal blood and the fetal blood:
a) Trophoblast.
b) Capillary endothelium.
c) Mesenchyme.
d) Chorion leave.
e) Fetal blood vessel intima.
22- The following statements regarding the placenta are True EXCEPT:
a) lt develops from decidua basalis and chorion fondosum.
b) The placental barrier gets more thin as pregnancy advances.
c) Placenta succenturiata may be a cause ofpost partum hemorrhage.
d) Its maternal surface is not covered by an amniotic membrane.
e) At term the average weight is 100.0 grams.
23- The INCORRECT statement regarding the Placenta:
a) It develops from the decidua vera and the chorion frondosum.
b) Placental hormones are secreted by the Syncytium.
c) Placental transfer occurs by a variety of mechanisms.
d) Hydatidiform mole is a benign placental tumor.
. e) It is usually small in intrauterine grorvth retardation.
24- The placenta secretes:
a) Estriol.
b) Progesterone.
c) Human chorionic gonadotrophin.
d) Human placental lactogen.
e) All of the above.
25- The placenta secretes:
a) Dehydroepiandrosterone.
WhiteKnightLove

b) l7 hydroxy progesterone.
c) Ethinylestradiol.
d) Serotonin.
e) Inhibin,

-3-
Freely you have received; freely give.

26- All of the following hormones are products of placental synthesis or production EXCEPT:
a) Human chorionic gonadotropin (hCG).
b) Human chorionic sornatomarnmotropin (hCS),
c) Prolactin.
d) Progesterone.
e) Estriol.
27- The human chorionic gonadotrophin:
a) Is a steroid hormone.
b) Consists of alpha &beta subunits.
c) Alpha subunit is specific.
d) Beta subnit is identicalto FSH, LH & TSH.
e) Allthe above.
28- The human chorionic gonadotrophin:
a) Is produced one week after implantation.
b) Its doubling time is 4 days.
c) Disappears from urine and blood l-2 rveeks after delivery.
d) The level in multiple pregnancy is not different from that in singleton pregnancy.
e) AII of the above,
29- The human placental lactogen:
a) Is a decapeptide hormone.
b) Is a double glycosylated polypeptide hormone.
c) Has a synergistic action with insulin.
d) Is structurally similar to human prolactin.
30- The following statements regarding human chorionic gonadotrophin (hCG) are correct EXCEPT:
a) ls a glycoprotein.
b) Has an cr subunit similar to FSH.
c) Reaches a peak level at about 20 weeks'gestation.
d) Is thought to stimulate fetal testosterone secretion.
e) Is produced by syncytiotrophoblast.
3l- Regarding human chorionic gonadotrophins (hCG)
a) It inhibits relaxin secretion by the corpus luteum.
b) It can be detected in maternal blood 5 days post conception,
c) Renal clearance of hCG accounts for 90 percent of its metabolic clearance.
d) Placental GnRH is likely involved in the regulation of hCG formation.
32- The following statements regarding progcsterone hormone are correct EXCEPT:
a) Its main source during early pregnancy is the corpus luteum of pregnancy.
b) Its level is an index of the maternal-fetal placental unit.
c) It is the precursor oftestosterone.
d) It is an intermediate product in steroid metabolism,
e) The placenta is the main source of progesterone during late pregnancy.
33- The following statements regarding estrogens in pregnancy' are correct EXCEPT:
a) Estriol (E3) is the main estrogen during pregnancy.
b) They are decreased in cases ofanencephaly.
c) They are secreted by the syncytium.
d) They stimulate the alveolar development in breasts.
e) The fetus contributes to production.
34- Regarding estrogens in pregnancy:
a) Estradiol is the main estrogen during pregnancy.
b) They are decreased in cases ofhydrocephalus.
WhiteKnightLove

c) They are secreted principally by the cytotrophoblast'


d) The fetus contributes to production.
e) They stimulate the alveolar development in breasts.

-4-
'It is more blessed to give than to receive.

35- The CORRECT statemcnt for battledore placenta:


a) The umbilical cord is inserted into fetal membranes.
b) The umbilical cord is inserled at the margin of the placenta.
c) It may cause antepaftum hemorrhage of fetal origin.
d) Fetal vessels could be palpated on vaginal examination.
e) None ofthe above.
35- Succenturiate placenta could be associated rvith:
a) Antepartum hemorrhage.
b) Postpartum hemorrhage.
c) Preterm delivery.
d) Postterm delivery.
e) Congenital fetal malformations.
37- One of the following characteristics about amniotic fluid is CORRECT:
a) The amniotic fluid pH is acidic.
b) Water constitutes 90Yo of its cornposition.
c) It contains fetal cells desquamated from the fetal skin.
d) It reaches its maximum volume at term.
e) In the first trimester, it is mainly of fetal origin.
38- The following statements regarding the amniotic fluid are correct EXCEPT:
a) It has a fetal contribution as well as maternal one.
b) It has functions during pregnancy as rvell as during labor.
c) Its marked diminution may lead to skeletal deformities in the fetus.
d) It is decreased in anencephaly.
e) it keeps the fetal temperature constant.
39- In the fetal circulation, the INCORRECT statement ls:
a) There are two urnbilical veins & one umbilical artery.
b) The ductus venosus connects the urnbilical vein with inferior vena cava.
c) The blood is shifted from the right atrium to the left atrium via the foramen ovale.
d) Prostaglandins maintain patency of ductus afteriosus.
e) The inferior vena cava contains both oxygenated & deoxygenated blood.
40- After delivery of a term infant with Apgar scores of 2 at I minute and 7 at 5 minutes, you ask that
umbilical cord blood be collected for pH. The umblllcal arterles carrl rvhlch of the followlng?
a) Oxygenated blood to the placenta,
b) Oxygenated blood from the placenta.
c) Deoxygenated blood to the placenta.
d) Deoxygenated blood from the placenta.
4I- In the normal fetus and newborn, thc follorving statements are correct EXCEPT:
a) Prostaglandins delay closure ofthe ductus arleriosus after birth.
b) Glucose reaches his circulation by facilitated diffusion.
c) The central nervous system is fully mature at birth.
d) The ductus venosus conveys oxygenated blood before birth.
e) There is high pulmonary vascular resistance before birth.
42- After birth, the intra-abdominal portion of the umbillcal veln becomes the:
a) Lateral umbilical Iigament.
b) Ligamentum teres.
c) Urachus.
d) Right hepatic ligament.
e) Ligamentum venosum.
43- A woman reports that her last menstrual perlod started on May 261h,2012, ended on May 29th,
WhiteKnightLove

2012. The expected date of delivcry is:


a) February 2nd,2013.
b) Februaty 5th,2013.
c) Februaty 26th, 2013.

-5-
Freely you have received; freely give.

d) March 2nd,2013.
e) March 5th,2013.
44- A 3l-year-old woman, gravida 2,para I presents with a positive home pregnancy test. She used to
have regular menstrual cycles every 35 days. She did not use any contraception previously. The
first day of her last menstrual pcriod (LMP) was March 7,2012 and the last day of that menstrual
period rvas Ma rch 12,2012. Hcr pe riods a Iways last 4-5 days. What is the best estimate of expected
date of dclivcry (EDD)?
a) Decernber 25,2012.
b) Decernber 14,2012.
c) January 23,2013.
d) December 21 ,2012.
e) December 18,2012.
45- A lvoman tvho had 3 abortions at 17-19 rveeks of pregnancy and norv pregnant in her 32 weeks
could be described as
a) Nullipara.
b) Multipara,
c) Nulligravida.
d) Primigravida.
e) Priniipara.
46- Naegele's rulc for estimating a woman's due date is NOT related to:
a) Regular rnonthly menstrual cycles.
b) A pregnancy of280 days.
c) Ovulation about day l4 of the menstrual cycle.
d) Birlh control pills stopped within 2 months before conception,
e) Conception at midcycle.
17- Thc part of the utcrus that becomes the lower uterine scgment is:
a) Cervix.
b) Cornu.
c) lsthmus.
d) Corpus.
e) Interstitiurn.
48- Softening of the cervical isthmus that occurs early in gestation is called:
a) Hegar sign.
b) Chadwick sign.
c) Braxton-Hicks contraction.
d) Palmer sign.
e) Cullen sign.
49- Myomctrial changcs during pregnancy include:
a) Decreased vascularity.
b) Contractions detected by bimanual examination in early pregnancy called Braxton Hicks
contractions.
c) Formation of lower uterine segment from uterine isthmus.
d) Upper uterine segment is covered by loose peritoneum.
e) All of the above.
50- The uterus during pregnancy:
a) Has decreased vascularity.
b) Has a loose peritoneum covering the Upper segment.
c) Has a lower uterine segment.
d) Has a basal tone of l2 mm Hg.
5l- The lower uterinc scgment is characterized by:
WhiteKnightLove

a) Thick myometrium.
b) Middle layer of interlacing muscle fibres.
c) Adherent peritoneum.

-6-
'It is more blessed to give than to receive.

d) Being passive during Iabor.


e) It is the site ofclassic cesarean section incision.
52- Values of rctraction of uppcr utcrinc segment include:
a) Dilation of the cervix.
b) Effacernent of the cervix.
c) Expulsion of fetus & afterbirth.
d) Involution ofthe uterus.
e) All of the above.
53- Changes in the urinary system during prcgnancy include:
a) Decreased frequency of micturition.
b) Dilatation of the ureters.
c) Decreased glornellar filteration rate.
d) [ncreased values of blood urea nitrogen & blood urea.
e) All of the above.
54- Signs of early pregnancy includc all of the follorving EXCEPT:
a) Chadwick sign.
b) Braxton Hicks contractions.
c) Goodell sign.
d) Hegar sign.
55- Sure signs of pregnanc), include all of thc following EXCEPT:
a) Palpation offetal parts.
b) lnspection & palpation of fetal tnovernents.
c) Auscultation of uterine souffl6.
d) Detection of fetus by Lrltrasonography.
56- The Leopold maneuvers include all of tlre follorving EXCEPT:
a) Fundal grip.
b) Urnbilical grip.
c) Parvlick grip.
d) Vaginal examination.
57- A primigravida patient, at 8 rvcchs'gcstational agc of uncomplicated pregnancy is seeking
antenatal care. All of following invcstigations should bc donc routinely at the first visit EXCEPT:
a) Hernoglobin concentlation.
b) Blood glucose test.
c) Kidney function tests.
d) Cornplete urine analysis.
e) Rh factor status.
58- All of the following are correct about mcnstruation-delivcry interval EXCEPT:
a) The average duration ofpregnancy is 40 weeks.
b) lt is calculated fronr the last day of the last rnenstrual period until delivery.
c) Naegele's rule states that the expected date of delivery is calculated by substracting 3 months from
the first day of the last menstrual period then adding I rveek.
d) It could be calculated reliably in those with regular cycles.
e) It is longer than ferlilization delivery interval by l4 days.
59- In calculating the gestational age, all of the following are true EXCEPT:
a) The uterus reaches the umbilicus at 20 - 22 weeks.
b) McDonald rule is applied after 24 weeks.
c) Pinard stethoscope can detect fetal hearl sounds starting at I 0 weeks.
d) Ultrasound is rnost accurate in the first l2 weeks of pregnancy.
e) Naegle's rule is applied only when the dates are sure and reliable,
WhiteKnightLove

60- All of the follorving are causes of oversized uterus EXCEPT:


a) Incorrect dates.
b) Transverse lie.
c) Macrosomic fetus.

-7-
Freely you have received; freely give.

d) Hydrocephalus.
e) Polyhydramnios.
6l- Instructions given to pregnant female include all of the follorving EXCEPT:
a) Iron should be supplemented during uncomplicated pregnancy.
b) Heparin and insulin are safe, when indicated.
c) Exercise is avoided in twin pregnancy, pregnancy induced hypertension & growth restricted fetuses,
d) Vaginal douching is recommended.
62- Regarding immunization during pregnancy:
a) Mumps vaccine is safe.
b) Influenza vaccine is contraindicated,
c) Toxoids are harmfu[.
d) Hepatitis B vaccine is safe,
63- The CORRECT statement regarding immunization during pregnancy:
a) Live virus vaccines as rubella, measles and mumps are contraindicated.
b) Inactivated virus vaccines as influenza and rabies are safe.
c) Toxoids as tetanus & diphtheria are safe.
d) All of the above.
64- In biophysical profile, all of the follorving are true EXCEPT:
a) It includes non stress test, fetal breathing movements, fetal trunk movements, fetal tone and amniotic
fluid volume.
b) Interpretation is dependant on gestational age.
c) Testing is usually starled at32- 34 weeks'gestation.
d) Score of 6i l0 with decreased liquor is an indication to repeat the profile after one week.
65- The following signs or symptoms are present in a l2-week pregnancy:
a) Quickening.
b) Ultrasonographic fetal lteart motion.
c) Fundal level midway between the symphysis pubis and the umbilicus,
d) Bra:tton Hicks contraction.
e) Internal ballotment.
66- The decrease in hemoglobin concentration during the second trimester of normal pregnancy is
due to:
a) Reduced iron stores during the first trimester.
b) Reduced erythropoietin activity in the first l2 weeks of pregnancy.
c) Disproportionate increase in plasma volume compared with RBC mass.
d) Nausea and vomiting during early pregnancy.
67- Physiological changes rvith pregnancy include the follorvings EXCEPT:
a) Dilation of the renal pelves.
b) Increase in the size ofthe kidneys.
c) Increased frequency of micturition.
d) Decrease of the renal plasma flow.
e) More compression of the right ureter than the left one.
68- The following statement regarding blood composition in normal pregnancy is CORRECT:
a) The packed cell volume rises.
b) There is a rise in the iron-binding capacity.
c) The blood cholesterol decreases.
d) The total red celI mass falls by about20Yo.
e) The protein bound iodine level falls.
69- During pregnancy, the renal glomerular filtration rate (GFR) can increase by as much as:
a) I 0 percent.
WhiteKnightLove

b) 25 percent.
c) 50 percent.
d) 75 percent.
e) 100 percent,

-8-
'It is more blessed to give than to receive.

70- Tlrc CORRECT state mcnt regarding circulatory changcs in a hcalthy pregnant woman:
a) A uterine blood flow at term of 500 ml/minute.
b) A continuous increase in the stroke volutne until term.
c) A maximum increase in restng cardiac output of 100%.
d) A rise in cardiac output only during the second and third trimesters.
e) The peripheral blood flow is reduced.
7l- As normal pregnancy progresses, the follorving hematological changes occur:
a) Plasma volume increases and red cell volume remains constant.
b) Plasma volume increases proportionately less than the increase in red cell volume.
c) Red cell volume decreases and plasma volume remains constant.
d) Neither plasma volume nor red cell volume changes.
e) Plasma volurne increases and red blood cell mass increases.
72- The INCORRECT statement regarding physiological changes in cardiac output during normal
pregnancy:
a) lt starts to rise in the first trimester.
b) It varies rvith physiological changes in heaft rate.
c) It varies with stroke volume when the heaft rate is constant.
d) It is not reduced in a hot environment.
e) It is greater from the left ventricle than from the right ventricle.
73- A 34-year-old primigravida at27 rvccks of gcstation is seen in the emergency room rvith complaint
ofshortness ofbreath for the past 6 rvccks. She is comfortablc at rest and has no orthopnea, chest
pain, cough, fevcr, or chills. On cxamination, she is in acutc distress and has a pulse of 92 beats
per minute and a rcspiratory ratc of l6 brcnths per minutc, Her chest is clcar during auscultation.
A sample of arterial blood reveals a normalpH, pCO2, and a pO2. The most likely diagnosis is
a) Bronchial asthma.
b) Pneumonia.
c) Pneumothorax.
d) Pulmonary embolism.
e) Physiologic dyspnea.
74- Thc INCORRECT statement regarrling normalpregnancy:
a) Calcium supplementation is not essential to every pregnant woman.
b) No need for iron supplement if the diet is adequately enriched in iron.
c) The percentage of iron absorbed from the GIT is increased.
d) Daily requirements of folic acid are about 0.8- I mg/day.
e) Folic acid supplement decreases the incidence of neural tubal defects.
75- Which of the following IS NOT a component of the routine antenatal care:
a) Rh typing.
b) Quantitative pregnancy test.
c) Hemoglobin estirnation.
d) Urine analysis for sugar and protein.
e) Cervicovaginal smear.
76- Thc CORRECT statement regarding normal pregnancy:
a) Inthemiddletrimester,theglomerularfiltrationrateisincreased 100%abovenon-pregnantvalues.
b) BIood urea slightly increases.
c) A plasma creatinine of 0.4 mg/dl is rvithin norrnal limits.
d) Maternal weight gain is an accurate predictor of a srnall for gestation age (SGA) infant,
e) Maxirnum rnaternal weight gain occurs betrveen 20 and 24 rveeks'gestation.
77- Which of thc follorving IS NOT charactcristic of a 28 rveelt-normal pregnancy:
a) A fetal weight of 1200 g.
WhiteKnightLove

b) Lecithin to sphingornyelin (L/S) ratio of less than 2: I .


c) The absence oftype II fetal lung alveoli cells.
d) The presence of phosphatidylglycerol.
e) The presence oI lanugo hairs alI over the body.

9
Freely you have received; freely give.

78- During normal pregnancy, which of thc follorvings statements is !\!$:


a) Daily requirement of iron is about 4 mglday.
b) Iron supplement is needed even if the diet is adequately enriched in heme iron.
c) The percentage of iron absorbed from the GIT is decreased by 20%.
d) Daily requirement of folic acid is about 0.8-l m/day.
e) Folic acid supplement decreases the incidence ofneural tubal defects.
79- Which of the following dietary instructions IS NOT appropriate for a pregnant woman?
a) Gain at least 7.5 kg during the pregnancy.
b) Restrict salt intake.
c) Intake of 1200 mg calcium daily.
d) Intake of 800 pg folic acid daily.
e) Intake of supplemented iron.
80- Which of the following therapeutic measures are appropriate in the treatment of women who
have edema of pregnancy?
a) Salt restriction.
b) Fluid restriction.
c) Diuretics.
d) Bed rest.
e) Magnesium sulfate.
8l- A pregnant vegetarian is likely to bc deficicnt in which of the follorving substances?
a) Calcium.
b) Folic acid.
c) Iron.
d) Protein.
e) Vitamin B 12.
82- The following investigations are component of the routine antenatal care EXCEPT:
a) Rh typing.
b) Hemoglobin.
c) Urine analysis for sugar and protein.
d) Blood grouping.
e) Urine culture.
83- The following investigations are component of the routine antenatal care EXCEPT:
a) Rh typing.
b) Hemoglobin.
c) Liver function tests.
d) Urine analysis for sugar and protein.
e) Blood grouping.
84- Which one is included in routine screening of pregnant women ?
a) Parvovirus.
b) Toxoplasmosis.
c) Cytomegalovirus (CMV).
d) Syphilis.
e) Herpes simplex virus (HSV).
85- Which of the following IS NOT a usual routine screening test in the standard ideal current
practice of an early uncomplicated pregnancy?
a) Repeat human chorionic gonadotropin (hCG) levels.
b) Hemoglobin.
c) Serology for syphilis.
d) Cervical cytology.
WhiteKnightLove

e) Blood type and Rh factor.

-l 0-
'It is more blessed to give than to receive.

86- A prcgnant woman in the 6th week-gestrtion is anxious about the outbreak of german measles.
Rubella IgG was done 2 months prcviously during pre-conception counseting and showed
positivity. The best advice for her is:
a) Assurance.
b) Rubella vaccine immunization.
c) Rubella immunoglobulin injection.
d) Therapeutic abortion,
e) Early amniocentesis at 15-week-gestation.
87- A 37-year-old woman gravida 3 para 2, presents during antenatal care at 30 weeks'gestation. She
had uncomplicated obstetrical history as rvcll as medical history. She is 162 cm tall, her
prepregnancy rveight was 60 kg and she norv rvcighs 68 kg. Her BP is l15/80, and her temperature
is 37.3oC. Rubella IgG was negativc. Blood group O RH positive rvith no antibodies. What is the
next step in management?
a) IM 300 pg anti-D immune globulin.
b) Follow-up visit after one week.
c) Advice about her abnormal weight gain.
d) Rubella antibody test.
e) One-hour post-prandial glucose test.
88- A 28-year-old primigravida presents for a routine antenatal visit at24 weeks'gestational age. She
is having concern because she has noticed an increasing numbcr ofspidery veins appearing on her
abdomen. She is upset with the unsightly appearance of these veins and wants to know what you
recommend to get rid of them. Horv do you advise this patient?
a) Tell her that this is not a serious condition and give her referral to a vascular surgeon to have the
veins removed.
b) Tell her that you
are concerned that she may have liver disease and order liver function tests.
c)
Refer her to a dermatologist for furlher workup and evaluation.
d) Tell her that the appearance of these blood vessels is a normal occurrence with pregnancy and will
resolve spontaneously after delivery.
e) Recommend that she wears an abdominal supporl to relieve pressure from her abdomen and cause
resolution of the blood vessels.
89- In the evaluation of a 26-year-old patient for secondary amenorrhea, you evaluate serum
prolactin and beta hCG. The pregnancy test is positive and the prolactin comes back of 65 ng/ml.
This patient may require:
a) Routine obstetric care,
b) MRI scan of her sella turcica to rule out pituitary adenoma.
c) Repeat measurements of prolactin levels during pregnancy to ensure it is not over 100 nglmL.
d) Bromocriptine to suppress prolactin.
e) Evaluation for possible hypothyroidism.
90- Which of the following would normally bc expected to increasc during pregnancy?
a) Alanine aminotransferase (ALT).
b) Asparlate aminotransferase (AST).
c) Hematocrit.
d) Plasma creatinine.
e) Thyroxine-binding globulin (TBG).
9l- Heavy smoking during pregnancy may lead to all the follorving EXCEPT:
a) Spontaneous aboftion.
b) Preterm labor.
c) Oligohydramnios.
d) Fetal macrosomia.
WhiteKnightLove

e) Placental abruption.

-l l-
Freely you have received; freely give.

92- A pregnant patient presents very concerned aboutsome skin lesions/changes she is seeing that are
just like her uncle's who has Iiver cirrhosis from hepatitis C. What lesions or changes is she tikely
referring to?
a) Hyperpigmentation and spider angiomata.
b) Linea nigra and chloasma,
c) Spider angiomata and palmar erythema.
d) Striae and chloasma.
e) Striae and linea nigra.
93- Smoking during pregnancy may lead to all thc follorving EXCEPT:
a) Polyhydramnios,
b) Preterm labor,
c) Oligohydramnios.
d) Placental insuffi ciency.
e) Placental abruption.
94- A healthy 25-year-old GlPO at 40 weeks'gestational age comes to your office to see you for a
routine obstetric visit. The patient complains to you that on several occasions shc has experienced
dizziness, light-headedness, and feeling as if she is going to pass out rvhen she lies down on her
back to take a nap. What is the MOST APPROPRIATE plan of management for this patient?
a) Do an ECG.
b) Monitor her for 24 hours with a Holter monitor to rule out an arrhythmia,
c) Do an arterial blood gas analysis.
d) Refer her immediately to a neurologist.
e) Reassure her that nothing is wrong with her and encourage her not to lie flat on her back.
95- You are called in to evaluate the heart of a l9-year-old primigravida at term. Listening carefully
to the heart, you determine that there is a split Sl, normal 52, 53 easily audible with a 2/6 systolic
ejection murmur greater during inspiration, and a soft diastolic murmur. You immediately
recognize which of the following?
a) The presence of the 53 is abnorrnal.
b) The systolic ejection murmur is unusual in a pregnant woman at term.
c) Diastolic murmurs are common in pregnant women.
d) The combination of a prominent 53 and soft diastolic murmur is a significant abnormality.
e) All findings recorded are normal changes in pregnancy.
96- Warning symptoms during pregnancy DO NOT include:
a) Bleeding per vagina.
b) Sudden loss offluid per vagina.
c) Abdominal pain.
d) Leg cramps.
e) Decreased fetal kicks.
97- Warning symptoms during pregnancy DO NOT include:
a) Bleeding per vagina.
b) Sudden loss of fluid per vagina.
c) Abdominal pain.
d) Leg edema.
e) Excessive salivation (ptyalism).
98- A pregnant uterus that is larger than thc period ofamenorrhea could be due to:
a) Generalized edema.
b) Obesity.
c) lntrauterine growth retardation.
d) Breech presentation.
WhiteKnightLove

e) Polyhydramnios.

-12-
'It is more blessed to give than to receive.

99- A Primigravida had her LMP 30 rvceks ago. On Exanrination, the funda! level is 2 fingers below
the xiphsternum. She may havc any of the follorving EXCEpT:
a) Macrosomia.
b) Twin pregnancy.
c) Associated uterine fibroids.
d) Breech presentation.
e) Polyhydramnios.
100- Concerning antepartum assessment of fetal rvellbeing:
a) Nipple stimulation test is an accurate test.
b) Fetus normally reacts with fetal heart rate acceleration in acoustic stimulation test.
c) Fetal breathing movements alone are sensitive indicator of fetal wellbeing.
d) A score of 8/10 in biophysical profile with decreased liquor is reassuring.
l0l- Basic ultrasound in the first trimester is accurate in all of the following EXCEPT:
a) Gestational sac localization.
b) Placental location.
c) Crown rump length.
d) Fetal number.
102- Indications of amniocentesis include:
a) Diagnosis of chromosomal anomalies.
b) Bilirubin estimation in Rh isoimmunization.
c) Estimation of fetal lung maturity.
d) All of the above,
e) B&Conly.
103- The CORRECT statement about biophysical profile is:
a) It is made up of four components.
b) A score of 6 is satisfactory.
c) Perinatal mortality rises with falling biophysical profile score.
d) Decreased amniotic fluid volume is an imporlant sign of acute asphyxia.
e) It consists of Doppler florv readings of the umbilical cord.
104- Thc following statements, regarding Manning biophlslcal profile are correct EXCEPT:
a) Score ten is the rnaximum value.
b) Score zero is given for dead fetus,
c) Score 6 is non-reassuring.
d) It includes five parameters.
e) It can be performed at 32 weeks' gestational age.
105- The count-to-ten chart to record fetal movementsi
a) It is an accurate method to assess the fetal rvell-being.
b) It might indicate the at risk fetuses.
c) It is the number of fetal movements in ten minutes.
d) If the result is unsatisfactory, immediate delivery of the fetus is indicated.
e) Ifthe result is satisfactory, the fetus is definitely in safe condition.
106- The follorving procedure allorvs the earliest rctrieval of DNA for prenatal diagnosis:
a) Fetoscopy.
b) Chorionic villous sampling,
c) Amniocentesis.
d) Percutaneous umbilical blood sampling.
e) Fetal skin biopsy.
107- The following statements regarding obstetric ultrasound are correct EXCEPT!
a) It can be used with amniocentesis.
WhiteKnightLove

b) It might induce a significant risk to the fetus.


c) lt can diagnose placental grading.
d) It is a useful tool in the assessment of amniotic fluid volume.
e) It could estimate the approximate intrauterine fetal weight.

- 13-
Freely you have received; freely give.

108- The INC,ORRECT statement regarding obstetric ultrasound:


a) After 30 weeks' gestation, it is rnore accurate than Naegele's rule in calculating the EDD.
b) It is useful in detection of placentalanomalies.
c) It can differentiate between types of abortion.
d) It is useful in the management of missed intrauterine contraceptive device (IUCD).
e) It is useful in the management of antepaftum hemorrhage.
109- Regarding obstetric ultrasound:
a) it is most accurat e at 28 weeks' gestation in estimating the expected date of delivery.
b) It is a useful tool in the assessment of amniotic fluid volume.
c) It carries a major risk to the fetus.
d) It has low accuracy in estimating the fetal weight.
ll0- A 28-year-old woman presents with vaginal spotting. She states that her last menstrual period was
7 weeks ago. Transvaginal ultrasonogram reveals a normal-appearing 4-mm yolk sac and a 3-mm
crown-rump length (5.9 weeks). No cardiac activity is detected. The most appropriate next step in
management would be to:
a) Reassure the patient and schedule a return appointment in 4 weeks.
b) Discuss options for medical and surgical evaluation of the uterus.
c) Obtain a beta subunit of hurnan chorionic gonadotropin (B-hCG) level.
d) Repeat a transvaginal ultrasonogram in 5-7 days.
e) Corticosteroid therapy.
I I l- The following ultrasound measurement is the MOST RELIABLE index of fetal weight:
a) Biparietal diameter.
b) Abdominal circumf'erence.
c) Femur length.
d) Intrathoracic ratio.
e) Occipitofrontal diameter.
ll2- Ultrasound can be helpful in the diagnosis of all the following EXCEPT:
a) Incompetent cervix.
b) Fetal cardiac anonralies.
c) Type of twins.
d) Site of implantation.
e) Placental location.
113- The following statements regarding the blood flow velocity in the umbilical arteries by Doppler
ultrasound are correct EXCEPT:
a) It increases with advancing gestational age.
b) It decreases in cases of intrauterine groMh retardation.
c) lt increases with maternal hypertension.
d) It decreases with placental insufficiency.
e) Abnormal changes are detected in twin-to-twin transfusion.
ll4- Amniocentesis IS NOT useful in the following situation:
a) Suspected open neural tube defect.
b) Management of unsensitized Rh-negative pregnant women.
c) Tests for fetal lung maturity.
d) Suspected chorioamnionitis.
e) Examination of fetal cells for the chromosomal pattern.
ll5- Fetal lung maturation is ensured by the presence of:
a) Lecithin.
b) Prostaglandin.
c) Sphingomyelin.
WhiteKnightLove

d) Phosphatidylglycerol.
e) Cortisol

-14-
'It is more blessed to give than to receive.

I l6- The CORRECT statement for a feto protein during pregnancy:


a) Its maternal serum level rises in Down's syndrome.
b) Yolk sac shares in its production.
c) It is a component of biophysical profile.
d) It reaches its highest concentration in the maternal serum at about l6 weeks' gestation.
e) Its fetal serum level is sirnilar to the amniotic fluid level.
I l7- Criteria of normal labor include:
a) Delivery of a 39 weeks fetus.
b) Delivery by mid forceps.
c) Spontaneous breech delivery.
d) Twin pregnancy, both delivered vaginally.
e) Induction of labor by oxytocin at 40 weeks pregnancy.
118- As regards fetal presentation, thc INCORRECT statement is:
a) It is the part ofthe fetus that enters the pelvis first.
b) In face presentation, the occiput and back comes into contact.
c) It is usually cephalic.
d) The denomintor is the Iowest part of the presenting area.
e) In cephalic presentation, it is more common to be vertex.
I l9- The CORRECT statement regarding thc fetal skull fontanelles:
a) There are only 2 fontanelles.
b) The posterior fontanelle is closed at 28 weeks' gestational age.
c) The anterior fontanelle is closed at birth.
d) Neonatal brain U/S is done usually through the posterior fontanelle.
e) The anterior fontanelle is diamond or lozenge in shape.
120- The shortest diameter of the pelvic cavity is:
a) The external conjugate.
b) The diagonal conjugate.
c) The bispinous diameter,
d) The true conjugate.
e) The transverse diameter.
12l- An anthropoid pelvis:
a) May be associated with persistent occipitopsterior.
b) May be associated with deep transverse arrest.
c) Has a male-shaped brirr,
d) The bispinous diameter is reduced.
e) Is associated with spondylolisthesis.
122- The following statements regarding ischialspines arc correct EXCEPT:
a) They make the beginning of the fonvard curve of the pelvis,
b) They are landmarks for pudendal nerve block procedure.
c) They indicate a normal pelvis when particularly prominent.
d) They help to assess station ofthe presenting part.
e) They lie at the level of the plane of least pelvic dimensions.
123- The INCORRECT statement rclating to ischial spines:
a) They lie between the greater and lesser sciatic notches.
b) They mark the beginning of the forward curve of the birth canal.
c) The internal pudendal nerve lies in close relation to the spines.
d) When the widest transverse diameter of the fetal skull is at the level of the spines, the head is
engaged.
e) They are of parlicular prominence in the normal female pelvis.
WhiteKnightLove

124- Concerning the pelvic cavity, all of the following are true EXCEPT:
a) lt is bounded by pelvic brim, plane of least pelvic dimensions, symphysis pubis & the sacrum.
b) The plane of least pelvic dimensions is the site of internal rotation during labor.
c) The plane of greatest pelvic dimensions is rounded in shape.

-1 5-
Freely you have received; freely give.

d) All of its diameters equal 12.5 cm.


125- The anatomical outlet is characterized by all of the following W.EEf:
a) It is rounded in shape.
b) Its plane is formed of anterior & posterior sagittal planes.
c) Its antero posterior diameter extends from the lower border of symphysis pubis to the tip of coccyx.
d) Its transverse diameter extends between the inner aspects of ischial tuberosities.
126- The gynecoid pelvis is characterized by:
a) It exists in25% of female population.
b) Pelvic cavity is narrow & deep,
c) Shape of inlet is rounded,
d) Subpubic angle is less than 90 degrees.
127- The CORRECT statement for anterior fontanelle ( Bregma ) is:
a) It is triangular in shape.
b) It is smaller than the posterior fontanelle,
c) It has bony floor,
d) It becomes obliterated l8 months after birth.
128- The CORRECT statement for biparietal diameter of fetal head:
a) It extends from a point below one parietal bone to a point above the opposite eminence.
b) It is the engaging diameter in veftex presentation only,
c) It is 9.5 cm in length.
d) All of the above,
129- The suboccipto bregmatic diameter:
a) Extends from below the occipital protuberance to anterior end of bregma.
b) It is the diameter of engagement in occipto anterior position when the head is fully flexed.
c) It is the diameter distending the vulva when the head extends after crowning.
d) It is ll.5 cm in length.
130- The occipto frontaldiameter:
a) Extends from occipital protuberance to center ofbregma.
b) Measures 9.5 cm at term.
c) Is the diameter of engagement in after coming head of breech.
d) Is the diameter of engagement in face presentation with a fully extended head.
l3l- The mento vertical diameter
a) Measures 10.75 cm atterm.
b) Is the diameter of engagement in face presentation.
c) Extends from the chin to the verlical point.
d) Is rvidest transverse diameter of the fetal head.
132- The following statements regarding true labor pains are correct EXCEPT:
a) They are regular and painful.
b) They are stopped by sedation and increased by enema.
c) They are associated rvith cervical dilatation.
d) They may be associated with rupture of membranes.
e) They increase in frequency and intensity by time.
133- True labor pains
a) Are inhibited by sedation.
b) Are augmented by enema.
c) Are always initiated by rupture of membranes.
d) Increase in frequency and intensity by time.
134- Condition that fS NOT consistent rvith normal labor:
a) Delivery of a 39 weeks baby.
WhiteKnightLove

b) Delivery by midline episiotomy.


c) The presenting parl is vertex.
d) Twin pregnancy delivered vaginally.
e) Augrnentation of labor by oxytocin.

- l6-
'It is more blessed to give than to receive.

135- Characteristics of uterinc contractions include all of the following EXCEPT:


a) They are involuntary, independent ofextrauterine neurogenic control.
b) They starl usually in Ieft cornu then right cornu.
c) They have fundal dominance.
d) They can be assessed by manual palpation.
136- The CORRECT Statement as regards engagement:
a) It is descent of rvidest longitudinal diameter of presenting part below the pelvic brim.
b) It occurs during second stage of labor in primigravidas.
c) It is diagnosed by PV examination when the denominator of the presenting part reaches the level of
ischial spines.
d) It can not be diagnosed by abdominal examination.
137- The COMMONEST cause of non engagemcnt of presenting part is:
a) Face presentation.
b) Occipto posterior.
c) Placenta previa.
d) Polyhydramnios.
138- During normal labor:
a) The head undergoes increased flexion to bring the longest longitudinal diameter of head to be
presented first to the maternal pelvis.
b) In occipto anterior, the head undergoes internal rotation 3/8 ofa circle.
c) Internal rotation is due to sloping shape ofpelvic floor and rifling action ofthe bony pelvis.
d) Restitution is rotation of the head after delivery l/8 oIa circle in same direction of internal rotation.
e) None of the above is true.
139- False labor pains are characterized by all of the following EXCEPT:
a) Irregular.
b) Associated with dilation & effacement of the cervix.
c) Relieved by sedation & enema.
d) Do not increase in frequency, duration or amplitude.
140- Engagement of the fetal heacl is documented when the following event takes place:
a) The fetal head is inside the pelvis.
b) The presenting part is just above the level ofthe ischial spines.
c) The vertex is left occipitoanterior.
d) The biparietal diameter of the fetal head passes through the plane of the pelvic inlet.
e) The fetal head is fully flexed.
l4l- Engagement of the head can be assessed by:
a) lnspection of the abdomen.
b) Estirnating the relationship to ischial tuberosity,
c) Estimating the relationship to ischial spines.
d) Estirnating the degree of moulding.
e) Estirrating the degree of caput formation.
142- Regarding engagement ofthe presenting part
a) Nuchal cord is a usual cause ofnon-engagement.
b) ln nrultiparous wolnen, it occurs only after full cervical dilatation.
c) In primigravida it usually occurs during the last 2 weeks of pregnancy.
d) Cephalopelvic disproporlion is the commonest cause of non-engagement.
143- Concerning synclitism:
a) The sagittal suture of fetal head is midway between the symphysis pubis and the sacral promontory.
b) Both parietal bones are felt vaginally at the same Ievel.
c) The biparietal diameter is the diarneter of engagement.
WhiteKnightLove

d) All of the above.


144- Asynclitism is best defincd as:
a) Flexion ofthe descending fetal head from pelvic floor resistance.
b) Failure of the sagittal suture to Iie exactly midway between the symphysis and sacral Promontory.

-t7 -
Freely you have received; freely give.

c) Failure ofthe descent because ofinadequate uterine contractions,


d) Inability ofthe fetal head to pass through the pelvic inlet.
e) Inability of internal rotation after the fetal head has reached the ischial spines.
145- Caput succedaneum:
a) Resolves spontaneously after labor.
b) May lead to jaundice of the baby postpartum.
c) Indicates a traumatic vaginal delivery.
d) Indicates that the fetal head is engaged.
e) ls a sign ofintrauterine fetal death.
146- As regards moulding of fetal skull, which is the CORRECT statement?
a) The parietal bones overlap the frontal bones in minor degrees.
b) It involves bones offace and base ofthe skull.
c) Rapid rnoulding may cause intra cranial hemorrhage.
d) It accounts for rnarked diminution of biparietaldiameter & suboccipto bregmatic diameter.
147- Caputsuccedaneum
a) is due to prolonged pressure on fetal head by maternal tissues.
b) is always few millimeters in thickness.
c) does not cross suture lines.
d) indicates that the fetus was dead during labor.
148- Which of the following IS NOT characteristic for active-phase uterine contractions?
a) They create 40 mm Hg of pressure.
b) They cause dilation of the cervix.
c) They cause thickening of the lower uterine segment.
d) They occur every 2 to 4 minutes.
e) They last for 45 seconds.
149- The CORRECT statement for the first stage of labor:
a) Lasts for maxirrum 8 hours in primigravida (PG).
b) Starts with true labor pains & ends after delivery of the fetus.
c) Latent phase is a parl of the prelabor pains.
d) It is preceded by cervical dilatation & effacement.
e) Active phase is characterized by increased rate ofcervical dilatation and descent ofthe presenting
part,
150- The CORRECT statement for the first stage of labor:
a) Ends in expulsion of the fetus,
b) Begins when membranes rupture.
c) Is not affected by the use ofoxytocin.
d) Normally lasts for more than 24 hours in a primigravida.
e) ls prolonged in malposition of the head.
l5l- A 25-year-old G2Pl woman at 4l weeks'gestation is noted to change her cervix from 6 cm to 9
cm over 2 hours. Which of the following is the most likely diagnosis?
a) Norrnal labor.
b) Accerelated active phase.
c) Protracted active phase.
d) Arrest of active phase.
e) Arrest of descent.
152- A 20-year-old G I at 38 weeks' gestation presents rvith regular painful contractions every 3 to 4
minutes lasting 60 seconds. On pelvic examination, she is 3 cm dilated and 907o effaced; an
amniotomy is performed and clear fluid is noted. The patient receives epidural analgesia for pain
management. The fetal heart rate tracing is reactive. One hour later on repeat examination, her
WhiteKnightLove

cervix is 5 cm dilated and 1007, effaced. Which of the following is the best next step in her
management?
a) Begin pushing.
b) Initiate Pitocin augmentation for protracted labor.

-18-
'It is more blessed to give than to receive.

c) No intervention; labor is progressing normally.


d) Perform cesarean delivery for inadequate cervical effacement.
e) Stop epidural infusion to enhance contractions and cervical change.
153- Which of the following statements regarding management of tabor in a low-risk pregnancy is
TRUE?
a) Amniotomy may shorten the length of labor slightly, but not as much as spontaneous rupture.
b) Universal electronic letal monitoring improves perinatal outcome.
c) Food and oral fluids are acceptable if labor is progressing normally,
d) An indwelling catheter is frequently needed when the patient is unable to void spontaneously.
154- The following IS NOT a sign of placental separation during the 3rd stage of labor:
a) A gush ofblood.
b) Rise of the uterus in the abdomen.
c) Painful tetanic uterine contractions.
d) Uterus becornes globular.
e) Further protrusion of the umbilical cord out of the vagina.
155- Signs of placental separation include all of the following EXCEPT:
a) Uterus becomes globular, smaller & harder.
b) Suprapubic bulge.
c) Shortening ofthe cord.
d) Strassman's sign.
156- The INCORRECT statement for delivery of the placenta:
a) Placental separation is often by Schultze method.
b) Duncan method of placental separation is more liable to be followed by retained pafts.
c) The earliest sign of delivery of the placenta include a gush of vulval bleeding.
d) Normally it takes l-2 hours in primigravida (PG).
e) Sign ofplacental descent include a suprapubic bulge.
157- Which statement regarding intrapartum cardiotocography is CORRECT?
a) It is useful in high risk pregnancy.
b) It is a relation between fetal heart rate and fetal kicks.
c) Reactive trace rneans a bad sign.
d) Most of abnormal patterns exist in the absence of fetal hypoxia.
e) Most of early decelerations will need cesarean section.
158- Meconium staining of amniotic fluid:
a) It is considered an indication of fetal distress in cephalic presentation.
b) lt is a normal finding in breech presentation.
c) It is due to hypoxia resulting in increased intestinal movements & relaxation of anal sphincter.
d) It is not an indication of CS in all cases.
e) All of the above.
159- Management of intrapartum fetal distress include all of the follorving E&EEI:
a) Patient is turned on her left side.
b) Oxygen administration.
c) Continue oxytocin infusion.
d) Conection of maternal hypotension.
160- Which of the following IS NOT an cxplanation for decreased variability to the fetal heart rate
(FHR) tracing?
a) Fetal "sleep state".
b) Prematurity.
c) Barbiturate ingestion.
d) Fetal stimulation.
e) Asphyxia.
WhiteKnightLove

- 19-
Freely you have received; freely give.

l6l- A 3l-year-old woman has been pushing in the second stage of labor for 2 hours. The vertex is at
the +2 station. Each contraction is associated with a fetal bradycardia as low as 100 beats per
minute (bpm) that lasts for 30 seconds. This clinical scenario suggests which of the foltowing
situation?
a) Systemic fetal hypoxia.
b) Poor fetal outcome.
c) An association with oligohydramnios.
d) Fetal head compression.
e) A depressed fetal pH.
162- A 2S-year-old woman has been in labor for 4 hours. The cervix is 7 cm dilated, vertex is at + I
station. FHS shows bradycardia as low as 100 beats/minute that lasts for 20 seconds with no
relation to uterine contractions. This clinical scenario suggests which of the following situation?
a) Fetal head compression.
b) Uteroplacental insuffi ciency.
c) Fetal metabolic acidosis.
d) Umbilical cord compression.
e) Fetal distress.
163- Management of intrapartum acute fetal distress includes the following EXCEPT::
a) Oxygen administration.
b) Change of maternal position.
c) Intraamniotic oxygen supplement.
d) Correction of maternal hypotension.
e) CS if the cervix is not fully dilated.
164- A woman with ruptured membranes is in the active phase of labor and is 5-cm dilated with
sustained, deep variable decelerations. The dccision is made to perform a cesarean section. Which
of the following IS NOT helpful fetal resuscitative measure before the cesarean section?
a) Increase the intravenous fluids.
b) Place the patient in the supine position.
c) Start nasal oxygen.
d) Start amnio-infusion.
e) Tocolytic agents.
165- A woman with ruptured membranes is in the active phase of labor receiving oxytocin and the
cervix is 5-cm dilated with sustained, deep variable decelerations. The decision is made to perform
a cesarean section. Which of the following would NOT be appropriate measure needed before the
cesarean section?
a) Increase the intravenous fluids.
b) Place the patient in the supine position.
c) Start nasal oxygen.
d) Systemic antibiotic.
e) Discontinue oxytocin drip.
166- A 24-year-old primigravida, in labor for l6 hours and the cervix is arrested at 9 cm for I hour,
position is ROP, station 0 and moulded. There are fetal late decelerations over the last 30 minutes.
Delivery is BEST managed by:
a) Forceps rotation and then traction.
b) Ventouse rotation (vacuum) rotation and then extraction.
c) Augmentation of labor by oxytocin.
d) Lower segment cesarean section (LSCS).
e) Fundal compression with deep episiotomy.
167- Primigravida with a fully dilated cervix for 2 hours, head station is on6 centimeter above the
WhiteKnightLove

ischial spines with molding of the skull and diffuse caput of the fetal scalp. The patient is best
delivered by:
a) Cesarean section.
b) Forceps delivery.

-20-
'It is more blessed to give than to receive.

c) Vacuum extraction.
d) Internal podalic version then breech extraction.
e) Fundal pressure with deep episiotomy.
168- Epidural anesthesia:
a) Does not affect uterine activity.
b) Is contraindicated in patients with hearl valve lesions.
c) Increases the risk ofpostpartum hemorrhage,
d) Should be routinely administered during the first stage of labor.
e) Should be offered to highly selected cases in labor.
169- Complications of epidural anesthesia include all of the following EXCEPT:
a) Ineffective analgesia.
b) Hyperlension,
c) Central nervous stimulation & convulsions.
d) Prolonged Iabor if used before well established labor.
170- Contraindications to spinal anesthesia include:
a) Acute blood loss,
b) Patients on anticoagulants.
c) Neurological disorders.
d) All of the above.

Direction: Each set of matching questions in this section consists of a list of numbered items followed
by several lettered options. For each numbered item, select the ONE best lettered option that is most
closely associated with it. Each lcttered heading may be selected once, more than once, or not at all.

For each of the following substances numbered (l7l-176), select its method of ptacental transfer
lettered (a-e).
EACH LETTERED HEADING MAY BE USED ONCE, MORE THAN ONCE, OR NOT AT ALL.

171- Glucose I a) Active transport.


172- Non fractionated heparin I b) Facilitated diffusion.
173- Fractionated heparin I c) Simple diffusion.
174- Amino-acids I d) Pinocytosis.
175- Oxygen I e) Not transferred.
176- Immunoslobulins

For each of the following fetal structures numbered (177-180), select its adult correspondence lettered
(a-e).
EACH LETTERED HEADING MAY BE USED ONCE, MORE THAN ONCE, OR NOT AT ALL.

177- Umbilical artery I a) Ligamentum venosum.


178- Ductus venosus I b) Ligamentum teres.
179- Ductus arteriosus I c) Ligamentum afteriosum.
180- Urachus I d) Median umbilical ligament.
e) Lateral umbilical li
WhiteKnightLove

-21-
Freely you have received; freely give.

For each of the following physiological conditions (t8l-185), select its change during pregnBncy
lettered (a-c).
EACH LETTERED HEADING MAY BE USED ONCE, MORE THAN ONCE, OR NOT AT ALI,.

l8l- Intestinal peristalsis a) Increases,


182- Serum cortisol level b) Decreases.
183- Serum thyroxine level c) Remains constant.
184- Respiratory rate
185- Cardiac output

For each of the following descriptions numbered (186-I90), select its appropriate term lettered (a-e).
EACH LETTERED HEADING MAY BE USED ONCE, MORE THAN ONCE, OR NOT AT ALL.

186- Left occipitoanterior a) Lie.


187- Mentum b) Station,
188- Longitudinal c) Denominator,
189- Vertex d) Position.
190- Zero e) Presentation.

For each of the following descriptions numbered (l9l-201), select its diameter lettered (a-h).
EACH LETTERED HEADING MAY BE USED ONCE, MORE THAN ONCE, OR NOT AT ALL.

l9l- Right oblique diameter of the pelvic inlet. a) 9 cm.


192- Left oblique diameter of the pelvic inlet. b) 9.5 cm,
193- True conjugate of pelvic inlet. c) 10.5 cm.
194- Obstetric conjugate. d) ll cm.
195- Sacrocotyloid diameter. e) I 1.5 cm.
196- Bispinous diameter. f) l2 cm.
197- Bituberous diameter. g) 12.5 cm.
198- Biparietal diameter. h) 13.5-13.75 cm.
199- Mentovertical diameter.
200- Suboccipitobregamatic diameter.
201- Occipitofrontal diameter.

WhiteKnightLove

-22-
'It is more blessed to give than to receive.

OBSTETRICAL AND NON-OBSTETRICAL


COMPLICATIONS DURING PREGNANCY

Direction: Select the ONE BEST lettered answer or completion in each question.

202- The following is associated with increased abortion rate EXCEPT:


a) Tobacco,
b) Hyperprolactinemia prior to pregnancy.
c) Radiation over 5 rads.
d) Coffee in excess of4 cups per day.
e) Class Al diabetes mellitus.
203- A 26-year-old para l, pregnant at 8 weeks'gestation, presented to the Reception Room with heavy
vaginal bleeding and opened cervical Os. The MOST COMMON cause of her condition is:
a) Anatomic defects of the uterus or cervix,
b) Multiple pregnancy.
c) Endocrine factors.
d) Chromosomal anomalies.
e) Toxoplasmosis.
204- The INCORRECT statement regarding missed abortion is:
a) It means retained dead embryo/ fetus.
b) Ultiasound is helpful in its diagnosis.
c) The uterus is usually felt smaller than the duration of pregnancy.
d) There may be brownish vaginal discharge.
e) There are exaggerated pregnancy symptoms.
205- All the following about blighted ovum are true EXCEPT:
a) It needs extensive workup if occurred.
b) It has a70Yo chance ofrecurrence.
c) 60% of the cases are due to nondisjunctional defects.
d) It comprises absent fetal tissue histologically.
e) Differentiation frorn early recognized fetal demise is of fundamental importance.
206- Which of the following is the most likely mechanism of septic abortion following an induced
abortion ?

a) Instrumental contamination.
b) Ascending infection.
c) Skin organisms,
d) Urinary tract penetration.
e) Hematogenous infection.
207- A 26-year-old patient has had three consecutive spontaneous abortions early in the second
trimester. As part of an evaluation for this problem, the !Q!s useful test would be:
a) Hysterosalpingogram.
b) Chromosomal analysis of the couple.
c) Endometrial biopsy in the luteal phase.
d) Postcoital test.
e) Tests of thyroid function.
208- The following statements, regarding habitual abortion, are true W.EP'I:
a) Anatomical abnormalities account for about 33% of second trimester habitual abortion.
WhiteKnightLove

b) No etiological factor is identified in approxinrately 50%.


c) The incidence is l% of abortions,
d) The most common hormonal cause in habitual abortion is progesterone deficiency.
e) The most common genetic abnormality in habitual abortion is polyploidy.
-L)-
^a
Freely you have received; freely give.

209- Indications of therapeutic abortion DO NOT include:


a) Advanced hypertensive vascular disease.
b) Missed abortion.
c) Active pulmonary T.B.
d) Advanced chronic renal disease.
e) Allcases of uncontrolled diabetes mellitus.
210- Complications of evacuation and curettage of the uterus DO NOT include:
a) Uterine perforation.
b) Infection.
c) Amniotic band syndrome,
d) Hemorrhage.
e) Asherman's syndrome.
2l l- The following statements regarding ectopic pregnancy are correct EXCEPT:
a) Accounts for l0% of all maternal mortality.
b) There has been a four fold increase over the past 20 years.
c) The majority of cases (over 95%) occur in the tube.
d) The incidence ofectopic is directly related to the prevalence ofsalpingitis.
e) The effect of ectopic pregnancy on the future ferlility is insignificant.
212- A patient presents with missed period, lower abdominal pain and vaginal bleeding. Her p-hCG is
4300 mIU/mL and transvaginal ultrasound showed an empty uterus. The INCORRECT
statement for this condition is:
a) There has been an increase in the incidence over the past 20 years.
b) The majority of cases occur in the fallopian tube.
c) The incidence of ectopic is directly related to the prevalence of salpingitis.
d) B-subunit hCG is doubted every 48 hours in ectopic pregnancy.
e) Laparoscopy is the traditional standard for diagnosis.
213- The risk factors for an ectopic pregnancy include the following EXCEPT:
a) History of pelvic inflammatory disease.
b) Previous ectopic pregnancy.
c) Use of combined oral contraceptives.
d) Assisted reproductive techniques.
e) Previous tubal surgery.
214- The COMMONEST site of occurrence ectopic pregnancy is:
a) Isthmic or interstitialportion of the Fallopian tube.
b) Corneal end ofthe tube.
c) Ampullary portion of the fallopian tube.
d) Fimbrial end of the Fallopian tube.
e) Ovary.
215- Diagnosis of ectopic pregnancy can be ruled out in almost all clinically significant cases if:
a) Transvaginal sonography showed no abnormalities in the adnexal region.
b) Serum p-hCG was negative.
c) Patient did not miss her period.
d) Patient is using the IUCD for contraception.
e) There is no pelvic fluid by transvaginal uterine ultrasound.
216- The INCORRECT statement regarding the diagnosis of ectopic pregnancy:
a) Transvaginal ultrasound is helpful in conjunction with B-hCG in diagnosis of early ectopic.
b) Clinical exarnination of chronic type of ectopic pregnancy is usually not conclusive.
c) Laparoscopy is the traditional standard for diagnosis in doubtful cases,
d) B-hCG is not doubled every 48 hours in ectopic pregnancy.
WhiteKnightLove

e) Culdocentesis is an accurate method for early diagnosis.

-24-
'It is more blessed to give than to receive.

217' A Culdocentesis is performed in a l9-year-old Gl P0 woman with lower abdominal pain and
vaginalspotting. A totalof 3 cc of clotted blood is aspirated. Which one of the foltowing is the best
interpretation ?
a) A hemoperitoneum is present.
b) No hemoperitoneum is present.
c) The blood probably came from a blood vessel.
d) The patient probably has an ectopic pregnancy.
218- Which factor is a contraindication to medical treatment of tubal pregnancy?
a) Serum P-hCG is less than 3000 mlU/mL.
b) Positive fetal heart activity,
c) Patient is hemodynamically stable.
d) Ectopic sac is less than 4 cm in diameter.
e) Ectopic pregnancy sac is intact,
219- Medical treatment of ectopic pregnancy is contraindicated in the foliowing condition:
a) Serum B-hCG less than 3000 mlU/mL.
b) No fetal hearl motion on transvaginal ultrasound.
c) Patient is hemodynamically stable.
d) Ectopic sac is more than 4 cm in diameter.
e) The ectopic pregnancy is not disturbed.
220- Contraindication to medical therapy (Methotrexate) in tubal pregnancy is:
a) Size ofthe ectopic gestational sac is 3 cm or less.
b) Desire for future ferlility.
c) History ofactive hepatic or renal disease.
d) No fetal heart motion on ultrasound,
e) Absence of active bleeding.
221- Which is INCORRECT statement regarding the management of ectopic pregnancy:
a) The majority of undisturbed cases can be treated laparoscopically.
b) Cervical pregnancy may need hysterectomy.
c) Few ectopic pregnancies can resolve spontaneously.
d) Salpingectomy is not the only option for surgical management.
e) Laparoscopy is preferred in shocked patients.
222- As regards the management of ectopic pregnancy, the following are true EXCEPT:
a) The majority of undisturbed cases can be treated laparoscopically.
b) Cervical pregnancy may need hysterectomy.
c) Few ectopic pregnancies can resolve spontaneously.
d) Salpingectomy is the only option for surgical management.
e) Transvaginal ultrasound is helpfut in conjunction with hCG in diagnosis of ectopic.
223- The CORRECT statement regarding conservative management of ectopic pregnancy by
intratubal injection of methotrexate:
a) Is contraindicated in the presence oftubal rupture.
b) Should be limited to cases with a tubal diameter of about 0,5 cm.
c) Is particularly effective when ultrasound examination demonstrates fetal cardiac activity.
d) Should be followed by serial measurement of serum progesterone until values fall to the non-
pregnant range.
e) All the above.
224- The MOST RELEVANT feature of ectopic pregnancy is:
a) Pelvic pain.
b) Amenorrhoea.
c) Fainting.
WhiteKnightLove

d) Vaginal bleeding.
e) Pelvic mass.

-25-
Freely you have received; freely give.

225- The CORRECT statement for ectopic pregnancy:


a) Occurs in about l0% ofall pregnancies.
b) The risk is increased in those with a history or pelvic inflammatory disease.
c) It never exceeds 6-8 weeks' gestation.
d) Patients usually have a negative pregnancy test.
e) If the patient is not shocked early laparotomy is essential.
226- Pervaginal bloody discharge and hemoptysis with recent complete history of abortion raises the
suspicion to diagnose:
a) Hydatiform mole.
b) Choriocarcinoma.
c) Krukenberg tumor.
d) Endometrial cancer.
e) Incomplete aborlion.
227- A case of gestational trophoblastic neoplasia belongs to high risk group according to WHO
scoring system if disease develop after:
a) Hydatidiform mole.
b) Full term pregancy.
c) Spontaneous abortion.
d) Ectopic Pregnancy.
228- The following statements regarding hydatidiform moles are correct EXCEPT:
a) Usually have female karyotype.
b) May be complicated by thyrotoxicosis.
c) May be complicated by ovarian cysts.
d) o feto-protein is a good marker of disease post-evacuation.
e) Is more prevalent in the advanced age group.
229- Which is the INCORRECT statement regarding vesicular mole?
a) A fetus may coexist in utero.
b) Invasive mole penetrates the myometrium.
c) Commonly comprises cells with 46,XX chromosomes of paternalorigin.
d) The incidence is higher in pregnant women over the age of 45 years.
e) After evacuation of vesicular mole, the patient can be monitored by prolactin assays,
230- The following statements regarding gestational trophoblastic disease are true EXCEPT:
a) Hydatidiform moles arise form paternal genetic material.
b) Choriocarcinoma is characterized by an absence of chorionic villi.
c) There is a high incidence in South- East Asia.
d) Choriocarcinoma is highly chemosensitive to methotrexate.
e) B-subunit hCC serum level is doubled every 48 hours.
231- Risk factors for the development of persistent trophoblastic disease following molar evacuation
include the following EXCEPT:
a) Young age (<20 years old).
b) Old age (> 40 years old).
c) Increased uterine size.
d) Presence oftheca lutein cysts.
e) p-hCG level > 100.000 mlU/ml..
232- Theriskof recurrenceofmolarandortrophloblasticdiseaseinsubsequentpregnanciesis:
a) 2%.
b) s%.
c) 8%.
d) l0%.
WhiteKnightLove

e) l5%.
233- The karyotype of a complete Hydatidiform mole is:
a) 46,XY.
b) 45,XY.

-26-
'It is more blessed to give than to receive.

c) 46,XX.
d) 69,XXX.
e) None ofthe above.
234- Indications for instituting single-agent chemotherapy following evacuation of a hydatidifrom mole
usually include all the followings EXCEpT:
a) A rise in hCG titers.
b) A plateau of hCG titers for 3 successive weeks.
c) Failure of hCG titers to return to normal 8 weeks after evacuation.
d) Appearance of liver or brain metastases.
e) Failure of hCG titers to return to normal l0 weeks after evacuation.
235- Follow-up in the management of a hydatidiform mole should routinely include alt of the fotlowing
EXCEPT:
a) Human chorionic gonadotrophin level determination.
b) Pelvic examinations.
c) Contraception.
d) Chest x-rays.
e) Chemotherapy.
236- The best contraceptive method after termination of vesicular mole:
a) Tubal ligation.
b) Hysterectomy.
c) Combined oral contraceptive pills.
d) Intra-uterine contraceptive device.
e) Subcutaneous implants.
237- The MOST COMMON site of metastatic disease in choriocarcinoma is:
a) Brain.
b) Liver.
c) Vagina.
d) Lung.
e) Ovary.
238- Theca lutein cyst are characterized by the followings EXCEPT:
a) Are best treated conservatively.
b) Are usually treated surgically.
c) Are seen occasionally in nonnal pregnancy.
d) Are seen in ovarian hyperstimulation syndrome.
e) Are liable to complications including hemorrhage and torsion.
239- The CORRECT statement for placenta previa:
a) Is more common in multiparous patients.
b) Classical caesarean section is best performed for most cases.
c) Immediate hospital admission is indicated only in severe bleeding.
d) Clinical presentation with bleeding is usually before 28 weeks' gestational age.
e) Preterm labor is a rare associated problem.
240- A 32-year- old P2, both deliveries were by CS. Now ultrasound done at 22 weeks shows that the
placenta is grade 0 and located anteriorly down to the level of internal os. She is at increased risk
of which of the following complication5
a) Placenta accreta.
b) Abruption placenta.
c) Cerebrovascular accident.
d) Amniotic fluid embolus.
e) Pulmonary edema,
WhiteKnightLove

241- A 20-year-old primigravida,32 weeks' gestation, presents with profuse vaginal bleeding with pain
and tenderness per abdomen. The most probable diagnosis is:
a) Abruptio placentae.
b) Placenta praevia.

-27-
Freely you have received; freely give.

c) Marginal sinus bleed.


d) Vaginitis.
e) Uterus rupture.
242- Placenta previa
a) Is more common in women who had prior cesarean delivery.
b) Is more common in women who have gestational hypertension.
c) Is not associated with an increased risk of postpartum hemorrhage.
d) Increases the risk of fetal macrsomia.
e) Is managed by vaginal delivery in most cases
243- Placenta previa
a) Is more common in women who had prior cesarean delivery.
b) Is more common in women who have gestational hypertension.
c) Is not associated with an increased risk of postpartum hemorrhage.
d) Increases the risk of fetal macrsomia.
e) Is managed by vaginal delivery in most cases.
244- Each of the following is a typical feature of placenta previa EXCEPT:
a) It is manifested by painless bleeding.
b) Commonly associated with coagulopathy.
c) First episode of bleeding is usually self-limited.
d) Associated with postcoital spotting.
e) It may predispose to postpartum hemorrhage.
245- A 25-year-old woman at 33 weeks, gestation is noted to have a placenta previa. Upon cesarean
section, bluish tissue densely adherent between the uterus and maternal bladder is noted. The
MOST LIKELY added placental abnormalify is
a) Placenta accrete.
b) Placenta increta.
c) Placenta percreta.
d) Placental polyp.
e) Placental hemangioma.
246- A 22-year-old primigravida at 34 rveeks' gestation presents with moderate vaginal bleeding and
no uterine contractions. The following sequence of examinations is the most appropriate:
a) Speculum examinati on, u ltrasound examination, di gital examination.
b) Ultrasound examination, digital examination, speculum examination.
c) Di gital examination, ultrasound examination, speculum examinati on.
d) Ultrasound examination, speculum examination, digital examination.
e) Di gital examinati on, speculum examination, ultrasound exam inati on.
247- The followings are risk factors for development of placental abruption EXCEPT:
a) Short umbilical cord.
b) Folic acid deficiency.
c) Pre-eclampsia.
d) History of threatened abortion.
e) Previous placental abruption,
248- The INCORRECT statement, regarding abruptio placentae, is:
a) May cause consurnptive coagulopathy.
b) May lead to acute renal failure.
c) Maternal mortality rate is l0%.
d) IUFD may occur if the placenta is separated.
e) May cause hemorrhagic shock.
249- A para 5lvoman presented at 36weeks' pregnancy with vaginal bleeding, tonic contracted uterus
WhiteKnightLove

and fetal heart sounds were not audible. Complications of her condition DO NOT include:
a) Hemorrhagic shock.
b) Consumptive coagulopathy.
c) Retraction ring of the uterus.

-28-
'It is more blessed to give than to receive.

d) Sheehan syndrome.
e) Intrauterine fetal loss.
250- Asregards placenta
previa, --'-
a) It is rnanifested
manifestecl hv
bv
., . ' ' the
nei.t-.^
followi
-^^..-,
recurrent .
lngs arc true EXcEPT:
'v"v'vr'rtss
,ralnless
.t_h- ;-:+:^r ,-
h\ The initial
b] vaginal bleediIng.
hemor*rg. i. rrrrl;'#,,
l] j, T., predispose ro postpafturn hernorrhage.
d) Its incidence is affected -"'
Ui p*i,,
e) The placental site can U.
io.u,.jUv ultrasound.
,t,- ,n. fo,owing
X}:l,rlr or..;;;; lomptication rs Nor
associated with acute
renal cortical
a) Retained dead fetus.
b) placental abruption,
c) Endotoxin_induced shock.
d) placenta previa.
e) Disseminated intravascular
coagulation.
252- When a lvor
measures,r,.T"i,X'r.f:}ljilHnat bleeding at 37 rvceks' gestation, which of the fo[owing
a) Cesarean section.
b) Induction of labor.
c) Coagulation profile.
d) ,Rupture of membranes.
e) Urgent ultrasound.

a previa:
he placental circulation.
nta.
tion.
Iivery is Iower than with
cesarean section.
rather than fetal.
statement is:
nt vaginal bleeding.
m hemorrhage.

ally at reception room (RR).

seyere pre_eclampsia
EXCEpT:

bove l60mmHg.
bove 100 mmHg.

"o l,'*t;'#Iij:ff';'::lvida at 16 weeks'gestation rvith


Bp r80/e0 and protein in
urine, is Mosr
a) preeclampsia.
b) Essential hypertension.
c) Pre existing renal disease.
d) pheochromocytoma.
e) Any ofthe above.

relative who had preeclampsia.


WhiteKnightLove

n,
ring the progress ofthe condition.
smoke cigarettes.

-29-
Freely you have received; freely give.

258- A 30-year-old primigravida presents at 34 rveeks' gestational age with blood pressure of 170/100
mmHg, headache, epigastric pain, visual abnormalities and 3* proteinuria. Biophysical profile of
the fetus is 8/8. Which one of the following is the immediate response:
a) Start magnesium sulfate intravenously.
b) Perform an emergency C-section.
c) Give betaclomethasone to induce fetal lung nraturity.
d) Perform an amniocentesis to assess fetal lung maturity.
e) Repeat the biophysical profile daily.
259- Which of the following IS NOT a risk factor for the development of pregnancy-induced
hypertension (PIHX
a) Multiple gestation.
b) Chronic hyperlension.
c) Genetic predisposition.
d) lron deficiency anemia.
e) Diabetes mellitus.
260- Pathophysiologic consequences of preeclampsia include all of the following EXCEPT:
a) Decreased glomerular filtration rate.
b) Increased plasma uric acid.
c) Increased intravascular volume.
d) Oliguria.
e) Decreased placental perfusion.
261- The pathophysiology of preeclampsia is characterized by:
a) Vasospasm.
b) Hypervolemia.
c) Thrornboxane is decreased.
d) Hemodilution.
e) Vasodilatation followed by vasospasm,
262- Whichofthefollowingismostlikelytobeseenonaurinalysisinapatientwithpreeclampsia?
a) Proteinuria.
b) Hernaturia.
c) Glycosuria.
d) Ketonuria.
e) Hemoglobinuria,
263- A diagnosis of severe preeclampsia in a patient at thirty-seven weeks' gestation with a blood
pressure of 160/110 mmHg would be supported by:
a) Proteinuria(2+ or more on the dipstick).
b) Pulmonary edema.
c) Intrauterine growth restriction.
d) Defective uteroplacental circulation assessed by Doppler study.
e) Any ofthe above.
264- With diagnosis of preeclampsia in a patient at thifi-seven weeks' gestation with a blood pressure
of I60/l l0 mmHg and proteinuria:
a) Immediate delivery is essential.
b) There is an increased risk ofbreech presentation.
c) MgSO4 should not be given.
d) The baby is not at a risk.
e) There is increased risk for postmaturity.
265- A 32-year-old woman, gravida 4, para 3, had an uncomplicated obstetric history. At 33 weeks'
gestation, her blood pressure is 150/100 mmHg and 145/100 mmHg when repeated after 30
WhiteKnightLove

minutes. She has 1* proteinuria on urinary dipstick. There is no generalized or lower limb edema.
The total weight gain during current pregnancy was l4 kg. What is the appropriate next step?
a) Strict bed rest at home.
b) Hospitalization for further evaluation.

-30-
'It is more blessed to give than to receive.

c) Outpatient antihyperlensive therapy.


d) Immediate delivery.
266- Which of the followings IS NOT an indication of severe preeclampsia:
a) Serum creatinine of 0.8 mg/dL.
b) Oliguria.
c) Fetal growth restriction,
d) Epigastric pain.
e) Platelets of 75,000/mm3.
267- Which of the following IS NOT a sign of severe pregnancy-induced hypertension?
a) Oligohydramnios.
b) Proteinuria in excess of I g/24 hours.
c) Platelet count of 50,000/mm3.
d) Serum creatinine of 2.3 mg/dl
e) Elevated hepatic transaminases.
268- The mechanism of action of alpha methyldopa is:
a) Acting on the kidneys to increase sodium and water diuresis.
b) Cardiac action to increase baroceptor sensitivity.
c) Peripherally acting to increase vascular resistance.
d) Peripherally acting to relax arterial smooth muscle,
e) Centrally acting to decrease sympathetic outflow.
269- The following statements, regarding cclampsia, are correct KEEI:
a) It rarely occurs in multiparous patients.
b) It is more common during the post-partum period than ante-partum period.
c) It should be managed with Mg. sulphate.
d) It causes hyper-reflexia.
e) It is grand mal seizures.
270- The CORRECT statement for eclampsia:
a) Peripheral edema is courmon.
b) The circulating blood volume is increased above values of normal pregnancy.
c) Immediate delivery under general anesthesia is the management of choice.
d) Beta blockers would be the best antihypertensive therapy.
e) Delivery is contraindicated before 34 weeks.
271- Eclampsia is defined by:
a) Severe, unremitting headache.
b) Hyperreflexia.
c) Grand mal seizures.
d) Petit mal seizures.
e) Visualscotomata.
272- The INCORRECT statement regarding eclampsia:
a) It may be complicated by respiratory failure.
b) It may be complicated by hyperpyrexia.
c) It may be associated with oliguria.
d) Could be treated by intravenous diuretics.
e) Could be treated by intravenous MgSO4.
273- You are called to the delivery suite whcrc an eclamptic primigravida is fitting. The EEI drug to
prevent further seizures is:
a) Diazepam.
b) Phenytoin.
c) Magnesium sulphate.
WhiteKnightLove

d) Lytic cocktail.
274- The following statement concerning eclampsia is CORRECT:
a) Postpartum eclampsia is more common than anetpartum eclampsia.
b) The maternal mortality rate is highest rvhen it occurs before delivery.

-31-
Freely you have received; freely give.

c) Placental abruption is a recognised association,


d) The pregnancy could be continued with proper medications.
e) Disseminated intravascular coagulation is not an associated hazard.
275- The renal lesion most associated with eclampsia is:
a) Glomerular endothelial swelling.
b) Pyelonephritis.
c) Hydroureter.
d) Cortical necrosis.
e) Acute tubular necrosis.
276- The COMMONEST cause of death in eclampsia is:
a) Ruptured liver.
b) Acute renal failure.
c) Cerebral hemorrhage.
d) Pulmonary embolism.
e) Septic shock.
277- ln type I diabetes mellitus (DM), the CORRECT statement is:
a) It is non insulin dependent type.
b) It is fonnerly known adult onset DM.
c) It is due to deficient insulin secretion.
d) Patients are less prone to ketoacidosis.
278- ln type II diabetes mellitus (DM), all the following are true EXCEPT:
a) It is non insulin dependant DM.
b) Formerly known Juvenile onset DM.
c) It is due to tissue resistance to insulin.
d) Patients are usually less prone to ketoacidosis.
279- A 30 years old pregnant woman presented to the antenatal clinic with normal GTT, however she
had 4.5 kg neonatalweight in her last pregnancy. This patient is considered to have:
a) Latent diabetes.
b) Potentia[ diabetes.
c) Chemical diabetes,
d) Overt diabetes.
280- A pregnant woman with abnormal GTT associated with polyuria is considered:
a) Clinically diabetic.
b) Chernically diabetic.
c) Latent diabetic.
d) Potentially diabetic.
281- As regards the normal blood glucose levels during pregnancy the CORRECT statement is:
a) Fasting blood sugar 90-120 mg/dl.
b) Fasting blood sugar 120-140 mg/dl.
c) Fasting blood sugar 60-90 mg/dl,
d) Post prandial blood sugar 90-120 mg/dl.
282- A 33-year-old gravid 2 para I stillborn infant. She had a screening test for diabetes mellitus at l0
weeks' pregnancy. One-hour post-prandial glucose test is 145 mg/dl. Follow-up of this patient
should include which of the following?
a) No further testing and no need for treatment.
b) A diet regimen with daily 2o00-calories.
c) Standard glucose tolerance test.
d) Serial follow up with domiciliary glucose urine testing.
e) Serial follow up with domiciliary blood sugar testing.
WhiteKnightLove

283- The effect of pregnancy on insulin and blood glucose:


a) In the first half of pregnancy, there is increased insulin response to glucose load.
b) In the second halfofpregnancy, there is increased insulin resistance.
c) In the second half of pregnancy, there is increased insulin requirements.

-32-
'It is more blessed to give than to receive.

d) AII of the above.


284- The effect of pregnancy on DM may bc all the folrowing EXCEpT:
a) DM may appear for the first time.
b) DM may improve with pregnancy.
c) DM may become difficult to control.
d) There is liability to hypoglycaernia.
285- The incidence of PIH in diabetic pregnant women is:
a) 5%.
b) s0%.
c) 25%.
d) 7s%.
286- The most specific fetal malformation to diabetes mellitus is:
a) Ventricular septal defect.
b) Coarctation of the aorta.
c) Spina bifida.
d) Sacral agenesis.
287- There is increased risk of postpartum hemorrhage during detiverT of diabetic woman. This can be
attributed to:
a) Polyhydramnios.
b) Macrsomia,
c) Prolonged labor.
d) Obstructed labor.
e) All of the above.
288- The MOST COMMON congenitalfetal malformation in diabetes mellitus is:
a) Ventricular and atrial septal defect (CVS).
b) Anenchephaly.
c) Spina bifida.
d) Sacral agenesis.
289- Macrosomic infants of diabetic mothers are more prone to:
a) Birth trauma,
b) Hypertrophic cardiomyopathy.
c) Neonatal hypoglycaemia.
d) Vascular thrombosis.
e) All of the above.
290- Neonates of diabetic mothers are more liable to the following EXCEpT:
a) Pathological anaemia.
b) Hypoglycaemia.
c) RDS.
d) Neonataljaundice.
291- The infant of a diabetic mother is at risk for all of the following EXCEPT:
a) Increased perinatal death rate.
b) Hypocalcemia.
c) Hyperglycemia,
d) Neural tube defects.
e) Macrosomia.
292- Neonatal jaundice in babies born to diabetic mothers may be due to:
a) Deficiency of liver enzymes.
b) Pathological anaemia.
c) Post maturity.
WhiteKnightLove

d) All of the above.


293- The following is considered high risk group for DM EXCEPT:
a) History of GDM.
b) History of congenitalfetal malformations.

.JJ.
Freely you have received; freely give.

c) History of unexplained stillbirth.


d) History of emesis gravidarum.
294- The most sensitive screening test for gestational DM is:
a) Fasting blood sugar.
b) 50 grams one hour blood sugar.
c) Post-prandial 2 hours blood sugar.
d) Random blood sugar.
295- The only confirmatory test for gestational DM is:
a) Fasting and2hr PP blood sugar.
b) 3-hour glucose tolerance test (OGTT).
c) 50 gms one hour PP blood sugar.
d) Atl of the above.
296- As regards OGTT, the following statement is CORRECT:
a) It can differentiate DM renal and alimentary glucosurea,
b) No smoking for l2 hours before the test.
c) Recording blood sugar Ievel every hour for 3 hours after 100 gm glucose intake.
d) All of the above.
e) None ofthe above.
297- The average caloric intake advised for diabetic patient rvith average weight and height is:
a) I 800-2400 kcal / day,
b) 2400-2800 kcal / day.
c) 1000-1500 kcal / day.
d) 800-1400 kcal / day.
298- The daily dietary intake advised to diabetic pregnant woman should be:
a) 30% protein, 20o/o CHO, 50% fat.
b) 20% protein,50% CHO,30Yofat.
c) 50% protein, 30Yo CHO, 20% fat.
d) 20% protein,30% CHO, 50Yofat.
299- ln the preconceptional management of diabetic vvomen, the following are true W,EP'[:
a) Postpone pregnancy untilgood control of DM,
b) Oral hypoglycemics should be changed to insulin before pregnancy.
c) HbA I c measurernent reflects glycemic control in the past week.
d) Assessment of diabetic complications.
300- A case of pregestational DM is usually controlled during pregnancy by:
a) Diet only.
b) Diet and oral hypoglycaemics.
c) Oral hypoglycaemics only.
d) Diet and insulin therapy.
e) Stoppage oftreatment once pregnancy occurs.
301- The following pregnant women are considered potentially diabetic EXCEPT:
a) Past history of macrosomia,
b) Past history of eclampsia.
c) Past history of congenital malformations.
d) Past history of unexplained stillbirths.
e) Positive family history of overt diabetes mellitus.
302- Effects of pregnancy on diabetes mellitus include the following EXCEPT:
a) Pregnancy has triphasic effects on glucose metabolism.
b) Control of the blood sugar becomes more difficult.
c) Insulin dosage usually increased after 20 weeks' gestational age.
WhiteKnightLove

d) There is increased liability to hypoglycemia.


e) Progression of chronic diabetic complications may be accelerated.

-34-
'It is more blessed to give than to receive.

303' Pregnancy Do Nor has which of the following effects on diabetic women:
a) Tendency toward ketosis in the first trimester.
b) Tendency toward hypoglycemia during early pregnancy.
c) Increase in insulin requirement during early pregnancy.
d) Increase in insulin requirement during late pregnancy.
e) Women, with preexisting but undetected oveft DM, may be initially diagnosed during pregnancy.
304- Obstetrical complications of diabetes mellitus DO NOT include the following:
a) Congenital fetal rnalformations.
b) Intrauterine fetal death.
c) Macrosomia,
d) Post-term delivery,
e) Preterm labor.
305- Which IS NOT a complication of diabetic pregnancy
a) Congenital abnornralities are rnore common than in normal pregnancy.
b) Insulin requirements increase in late pregnancy.
c) Monilial vaginitis is more common than with normal pregnancy.
d) Intrauterine growth retardation is more common than with normal pregnancy.
e) Oligohydrarnnios.
306- Pregnancies complicated by poorly controlled insulin dependent diabetes mellitus may be
complicated by the following EXCEPT:
a) Neonatal respiratory distress syndrome.
b) Neonatal hypomagnesemia.
c) Neonatal coagulation defects.
d) Decreased amniotic fluid magnesium concentration.
e) Faster progression ofretinopathy than in well controlled diabetes.
307- A pregnant woman at 25 week-gestation, rvith a previous intrauterine fetal death has a 50 g l-
hour glucose tolerance test with the follorving value: 149 mgldl. Follow-up for this patient should
include which of the following?
a) Nothing fufther and reassurance.
b) Home glucose urine testing.
c) Repetition of the test at 28 weeks.
d) 100-g glucose tolerance test.
e) A 2-hour postprandial blood sugar.
308- Which of the following is significant in thc management of pregnant diabetic:
a) Stating oral hypoglycenric agents.
b) Cesarean section before 36 weeks' gestational age.
c) Maintenance of one hour post-prandial blood glucose level below 140 mgldL.
d) Bed rest.
e) Carbohydrate free diet.
309- Pre-conceptional management of diabetic patient includes the followings EXCEPT:
a) Laboratory estimation of glycosylated hemoglobin,
b) Achievement of euglycemia for three months before conception.
c) Ultrasound examination.
d) Discontinuation of oral hypoglycemic agents.
e) Assessment of diabetic complications.
310- The condition that DOES NOT entail delivery of insulin-dependent diabetic at 36 weeks'
gestation:
a) A poor biophysical profile.
b) A lecithin-sphingomyelin ratio of 1.8/1.
WhiteKnightLove

c) A positive contraction stress test.


d) Decreased insulin requirements,
e) Spontaneous contractions with Iate decelerations.

-3 5-
Freely you have received; freely give.

3ll- The following statements regarding congenital malformations in babies of diabetic mothers are
correct EXCEPT:
a) Cause half the perinatal deaths with diabetic pregnancy.
b)
Are most often skeletal abnormalities.
Are cornmon if control is poor in the first trimester.
c)
d)
Have fallen significantly in the last 30 years.
e)
Are commoner in insulin-dependent than non-insulin-dependent mothers,
312- The INCQBRECT statement, about patients rvith gestational diabetes mellitus, is:
a) Gestational diabetes mellitus disappears after delivery.
b) Insulin is not essential in the management.
c) There is liability to hypoglycemia during labor,
d) Lactation is contraindicated after delivery.
e) May cause macrosomia as the insulin dependent diabetes mellitus.
313- In diabetic pregnancy the followings can be seen EXCEPT:
a) Congenital abnormalities are commoner.
b) Insulin requirements increase in second half of pregnancy,
c) Monilial vaginitis is cotnmon.
d) Intrauterine fetal death may occur.
e) Oligohydramnios is comtnon.
314- The CORRECT statement for diabetes in pregnancy:
a) Diabetes may develop during pregnancy owing to the anti-insulin effects of pregnancy-related
hormones such as placental latogen and cortisol.
b) Maternal glucose crosses the placenta into the fetal circulation via active transport mechanisms.
c) A glycosylated HbArc of greater than 60h is diagnostic of diabetes.
d) May be controlled by oral hypoglycemic drugs during the first trimester.
e) Neonatal outcome is related only to the control of blood suger in the second half of pregnancy.
315- The CORRECT statemet for gestational diabetes mellitus:
a) It should be screened foi in only high risk pregnancies.
b) It rnay be excluded by a fasting glucose sample.
c) One hour postprandial blood glucose is reliable confirmatory diagnostic method.
d) It is an absolute indication for induction of labor at 38 weeks.
e) It is a precursor for the development of diabetes in later life.
316- lnfant of diabetic mother is at increased risk of all the follorvings EXCEPT:
a) Neonatal polycythemia.
b) Neonatal hypermagnesemia.
c) Birth trauma.
d) Neonataljaundice.
e) Neonatal hypoglycemia.
317- The INCORRECT statement for the control of diabetes during pregnancy:
a) Carbohydrate intake is calculated to form 400 grams per day.
b) Carbohydrate intake is calculated to form 50% ofthe total energy intake.
c) Oral hypoglycernic drugs are not to be used even in mild cases of diabetes.
d) Twice-daily mixtures of short acting and medium acting insulin are suitable for most cases.
e) Elimination of infection helps good control of blood suger.
318- Regarding insulin therapy for diabetics during pregnancy, the @BECI statement is:
a) It makes diet manipulation unnecessary.
b) The evening dose is usually higher than the others.
c) Timing of injection is about 20-30 minutes after meals.
d) Two-injection regimen is needed for most of cases.
WhiteKnightLove

e) Slide scale technique is the best for long-term adjustment.


319- Regarding insulin therapy for diabetics during pregnancy, the CORRECT statement is:
a) Three-injection regimen is needed in most cases'
b) The evening dose is usually higher than the others,

-36-
'It is more blessed to give than to receive.

c) Timing of injection is about 20-30 minutes after meals.


d) Timing of injection is about 20-30 minutes before meals
e) Slide scale technique is the best for long-term adjustment.
320- what is the MOST coMMoN indication for induction of preterm labor?
a) Fetal distress.
b) Preeclampsia,
c) Fetal growth restriction.
d) Placental abruption.
e) Placenta accrete,
321- Detection of which of the following in cervicovaginal secretions is a powerful clinicat predictor of
subsequent preterm birth?
a) Decidual relaxin.
b) Fetal fibronectin.
c) Interleukin- I .
d) Tumor necrosis factor.
322- Which have shown conclusive benefit to perinatal outcome in preterm birth?
a) Bed rest.
b) Emergency cerclage.
c) Hydration and sedation.
d) None ofthe above,
323- Based on current clinical opinion, management of labor prior to 34 weeks of gestation should
include which of the following?
a) Corticosteroid administration.
b) Consideration of tocolytics to delay delivery briefly.
c) Antimicrobial prophylaxis to prevent group B streptococcus infection.
d) All of the above.
324- Which one of the following is MOST LIKELY to be a contraindication for tocoloysis at 28 weeks'
gestation?
a) Suspected abruption.
b) Asymptomatic bacteruria.
c) Group B streptococcal bacteriuria
d) Uterine fibroid.
e) Contracted pelvis.
325- Which of the following is the first sign of magnesium sulfate toxicity?
a) Respiratory depression.
b) Cardiac depression.
c) Loss ofdeep tendon reflexes.
d) Seizures.
e) None ofthe above.
326- Which of the following tocolytics is associated with reversible oligohydramnios?
a) lndomethacin.
b) Magnesium sulfate.
c) Ritodrine.
d) Terbutaline.
327- Which of the following IS NOT a predisposing factor for premature labor?
a) Matemal age older than 30 years.
b) Smoking more than l0 cigarettes per day.
c) Exposure to diethylstilbestrol (DES) in utero with a documented uterine structural abnormality.
d) More than four previous deliveries.
WhiteKnightLove

e) Twin gestation.
328- The following IS NOT a predisposing factor for premature labor:
a) Oligohydramnios.
b) Heavy smoking,

-37-
Freely you have received; freely give.

c) History of previous preterrn bifth.


d) Twin gestation.
e) Pregnancy induced hyperlension,
329- A l6-year-old primigravida reports that she is experiencing regular menstrual Iike-cramping
every2minutes.Sheis32weekspregnantwithmildvaginalbleeding.The@.M@,N
cause of this bleeding is:
a) A vaginal laceration.
b) An endocervical polyp.
c) Cervical dilation.
d) Placenta previa.
e) Placental abruption.
330- A Z2-year-old woman reports regular uterine contractions at34 weeks'gestation. She has not had
bleeding, trauma, or rupture of membranes. A recent genital culture for group B streptococcus
was negative. The fetal heart tones are 140 beats per minute and the woman is afebrile. The
uterus is nontender without active uterine contractions. Cervical examination reveals non
effacement and 2 cm of dilatation of the internal os. On admission and further examination there
is negative ferning, and the Nitrazine test is negative. Tocolytics are administered. Which of the
following is the best choice regarding antibiotic use to prevent or delay preterm birth in this
patient?
a) Broad-spectrum antibiotics.
b) No antibiotics.
c) Antibiotics effective against anerobes.
d) Combination antibiotics,
e) Intravaginal application of antibiotics.
331- A 35-year-old woman (gravida l, para 0) presents at 35 weeks' gestation, complaining of the
abrupt onset of frequent, painful abdominal contractions, back pain, and moderate vaginal
bleeding. On examination, the uterus is firm and moderately tender. The physician should take
which of the following actions?
a) Adrninistration of ritodrine.
b) Rule out placental abruption and advise immediate delivery.
c) Assure the patient that this often happens towards the end ofpregnancy.
d) Immediately send the patient for an ultrasound and a chest x-ray.
e) Advise the patient to rest in the examination room until the pain subsides while performing
hematologic and coagulation studies.
332- A 24-year-old woman (gravida 1, para 0) presents to the labor and delivery floor at 30 weeks'
gestation, complaining of frequent abdominal cramps and vaginal spotting. The fetal membranes
are intact, she is contracting every l0 minutes, and her cervix is 3 cm dilated and 80%o effaced.
The plan of management SHOULD NOT include which of the follorving
a) Intravenous tocolysis.
b) Urine culture.
c) Ultrasound for estirnation of the fetal weight.
d) Cervicalcerclage.
e) Obtaining a coagulation profile.
333- A 26-year-old primigravida reports that she is experiencing regular menstrual like cramping
every 2 minutes. She is 28 weeks pregnant. After taking a history, the first thing that the physician
should do is:
a) Send her to the labor floor immediately.
b) Confirm the frequency of contractions by abdominal palpation.
c) Evaluate fetal well-being with a fetal monitor.
WhiteKnightLove

d) Evaluate the cervix by speculum examination.


e) Immediate cesarean section.

-3 8-
'It is more blessed to give than to receive.

334' The percentage of patients with a history of prematurity who will have another premature infant
is:
a) 0% to t\oh.
b) 17%to 37%.
c) 40%to 50%.
d) 60%to'10%.
e) 80% to 90%.
335- The administration of corticosteroids to women in preterm labor is associated with:
a) Increased maternal mo(ality.
b) Decreased maternal morbidity.
c) Reduced infant growth,
d) Decreased perinatal mortality.
336- Non steroidal anti-inflammatory drugs are not generally used for long term tocolysis because
they:
a) Are ineffective.
b) Produce marked hypertension.
c) Are too expensive.
d) May cause premature closure of fetal ductus arteriosus.
e) Are associated with lactic acidosis,
337- Which of the following drugs IS NOT used to inhibit premature labor?
a) Indomethacin.
b) Magnesium sulfate.
c) Phenobarbital.
d) Ritodrine.
e) Nifedipine.
338- The tocolytic agent that enhances the toxicity of magnesium to produce neuromuscular blockade
is:
a) Nifedipine,
b) Ritodrine.
c) Indomethacin.
d) Isopropranolol.
e) Ethanol,
339- The CORRECT statement, about diagnosis of premature rupture of membranes(PROM) is:
a) Speculum examination is rarely done.
b) History of sudden gush of water from the vagina is a sure symptom of PROM.
c) The yellow-green Nitrazine paper turns deep blue.
d) Positive fern test is rarely used for diagnosis of PRM.
e) C-reactive protein is a specific indicator for chorioamnionitis.
340- The preferred management in rupture of membranes in 32 gestation is:
a) Antibiotics for 10 days.
b) Tocolysis until 36 weeks' gestation.
c) Steroids until 36 weeks' gestation.
d) Induction of labor at 34 weeks' gestation.
e) Expectant management.
341- Termination of pregnancy in cases of preterm premature rupture of membranes occur in the
following conditions EXCEPT:
a) The patient goes in active labor.
b) There are uterine contractions l/15 minutes.
c) Fetal distress occurred.
WhiteKnightLove

d) Presence oflever after excluding extrauterine causes offever.


e) Cervix is 5 cm dilated.

-39-
Freely you have received; freely give.

342- Modern management of prcterm prelabor rupture of membranes at 32 weeks' gestation includes:
a) Prophylactic Antibiotics.
b) Tocolysis till 37 weeks.
c) Induction oflabor at 34 weeks' gestation.
d) Cesarean delivery at 34 weeks' gestation,
343- Preterm prelabor rupture of membranes is rupture of membranes:
a) Before 34 weeks regardless ofthe onset oflabor.
b) Before 37 weeks and before the onset oflabor.
c) At 38 weeks and before the onset of labor.
d) At 39 weeks and before the onset of labor.
e) At 40 weeks' gestation and before the onset of labor.
344- A healthy primigravida is admitted at 30 weeks' gestation with spontaneous rupture of the
membranes. The following statement is CORRECT:
a) Fetal pulmonary hypoplasia may occur due to oligohydramnios.
b) Prophylactic antibiotics are contraindicated.
c) Corticosteroids are contraindicated because ofthe risk of infection.
d) Active management results in a better perinatal outcome than is achieved by expectant management.
e) The risk of preterm prelabor rupture of the membranes in subsequent pregnancies is70%,
345- 33-years-old primigravida at 30 weeks' gestation is admitted for preterm premature rupture of
membranes. Each of the following statements is correct EXCEPT:
a) Intramuscular cofticosteroid therapy should be given to enhance fetal lung maturity if there is no
evidence of infection.
b) Broad-spectrum antibiotic therapy is indicated only with maternal fever.
c) Premature labor is the most common acute complication to be expected,
d) Bacterial vaginosis is arisk factor for preterm premature rupture of membranes.
e) The length ofthe cervix is useful in predictiong preterm labor.
346- 30-years-old G2 Pl woman at 28 weeks' gestation with preterm premature rupture of membranes
is suspected of having intra-amniotic infection based on fetal tachycardia. The maternal
temperature is normal. Which of the following is the most accurate method to confirm the intra-
amniotic infection?
a) Serum maternal leukocyte count.
b) Speculum examination of the vaginal discharge.
c) Amniocentesis and culture of the amniotic fluid.
d) Palpation of the maternal uterus.
e) Oral temperature.
347- Postterm pregnancy is one that extends beyond which period ofamenorrhea?
a) 37 weeks.
b) 40 weeks.
c) 42 weeks.
d) 44 weeks.
348- Postterm pregnancy is defined as greater than or equal to which period ofamenorrhea?
a) 280 days.
b) 287 days.
c) 294 days.
d) 300 days.
349- What happens to perinatal mortality after 42 weeks'gestation?
a) Markedly decreased.
b) Slightly decreased.
c) No change.
WhiteKnightLove

d) Increased,
350- Which of the following IS NOT a description associated with the postmature infant?
a) Smooth skin.
b) Small fontanelles.

-40-
'It is more blessed to give than to receive.

c) Long finger nails.


d) Abundant scalp hair.
351- In a woman with a favorable cervix and an estimated fetat weight of 3850 g, what is the
management at 42 weeks' gestation?
a) Labor induction.
b) fetal surveillance started.
c) Amniocentesis performed for lung maturation studies.
d) Cesarean delivery scheduled.
352- In a woman with an unfavorable cervix and an estimated fetalweight of 3800 B, which is the most
appropriate management at 42 weeks' gestation?
a) Sonography to recalculate pregnancy dates.
b) Cesarean delivery.
c) Hospitalization and bed rest.
d) Cervical ripening, then labor induction.
353- The first step in the assessment of the postterm gestation is:
a) Ultrasound examination.
b) Determination of the true length of gestation.
c) Measurement of fetal heart rate (FHR).
d) Determination of amniotic fluid volume.
e) Contraction stress test.
354- After having identified a fetus at risk in prolonged pregnancy, management shoutd consist of:
a) Amniocentesis for maturity studies.
b) Delivery regardless of the status of the cervix.
c) Fetal sampling of scalp pH.
d) Measurement of human chorionic somatomammotropin.
e) Repeat antepaftum studies in I week.
355- Which of the following IS NOT included in the biophysical profile for the risk assessment of a
postterm fetus?
a) Fetal breathing.
b) Amniotic fluid volume.
c) Fetal tone.
d) Contraction stress test (CST).
e) Fetal rnotion.
356- How is symmetrical growth restriction characterized?
a) Reduction in head size.
b) Reduction in body size.
c) Reduction in both body size and head size.
d) Reduction in body and femur length.
357- A small-for-gestational age neonate is the one whose birth weight falls below the following
percentile for its gestational age?
a) 5'n.
b) l oth.
c) l5'h.
d) 2o'h.
358- Which of the following placentalabnormalities IS NOT associated with growth restriction?
a) Circumvallate placenta.
b) Placenta previa.
c) Acute abruption.
d) Velamentous cord insertion.
WhiteKnightLove

e) Battledore placenta.
359- Very low birth weight neonates are susceptible to:
a) Cerebral palsy.
b) Periventricular leukomalacia.

-41-
Freely you have received; freely give.

c) Seizures.
d) AII of the above.
360- Which of the following ultrasound measurements is the most reliable index of fetal weight?
a) Biparietal diameter.
b) Abdominal circumference.
c) Femur length.
d) Intrathoracic ratio,
361- Which sonographic measurement in a growth-restricted fetus correlates best with significant
perinatal mortality?
a) Biparietal diameter.
b) Abdominal circunrference.
c)
Femur length.
d)
Oligohydramnios.
362- Which of the following IS NOT a risk factor for macrosomia?
a) Diabetes,
b) Female fetus.
c) Matemal obesity.
d) Gestational age > 42 weeks.
363- As regards fetal weight more than 4.5 kg all are true EXCEPT:
a) Is almost always due to poorly controlled diabetes.
b) It increases the risk ofshoulder dystocia.
c) Is a contra-indication to vaginal delivery ofa baby presenting by the breech.
d) Another large baby is likely in a subsequent pregnancy.
e) It is associated with increased risk of PPH.
364- Amnionic fluid index is derived by adding the vertical depth measurements calculated by
ultrasound from which of the following?
a) The deepest pockets ofthe four uterine quadrants.
b) The smallest pockets of the four uterine quadrants.
c) The deepest pockets ofthe right and left uterine halves.
d) The smallest pockets of the right and left uterine halves.
365- Hydramnios is diagnosed by an amnionic fluid index greater than which of the following:
a) l6 cm.
b) 20 cm.
c) 24 cm.
d) 28 cm.
366- Oligohydramnios may be associated with the following EXCEPT:
a) Postdate pregnancy.
b) Chronic placental insuffi ciency.
c) Esophageal atresia,
d) Renal agenesis in the fetus.
e) Rupture of membranes.
367- Causes of oligohydramnios include the followings W.EEI:
a) Postdate pregnancy.
b) Chronic placental insufficiency.
c) Rh isoimmunization.
d) Renal agenesis in the fetus.
e) Rupture of membranes.
368- Complications of polyhydramnios include all the followings EXCEPT:
a) Preterm labor and malpresentations.
WhiteKnightLove

b) Preterm premature rupture of membranes.


c) Antepartum hemorrhage.
d) Postpartum hemorrhage,
e) Renal agenesis in the fetus.

-42-
'It is more blessed to give than to receive.

369- which congenital fstal malformations that Do Nor induce polyhydramnios:


a) Duodenal atresia.
b) Anencephaly.
c) Omphalocele.
d) Renal agenesis.
e) Open spina bifida.
370- What is the MOST COMMON cause of chronic potyhydramnios?
a) Fetal anomalies.
b) Maternal diabetes.
c) Multifetal gestation,
d) Idiopathic.
371- The CORRECT statement for pregnant patient known to be Rhesus sensitized:
a) The fetus must be affected if the father is heterozygous for the Rh +ve blood group.
b) Ultrasound detection of fetal scalp edema does indicate inevitable fetal death.
c) Monitoring of rnaternal Rhesus antibody concentration is of no clinical value.
d) Group O Rh negative blood can be transfused into the fetal circulation through its umbilical artery.
e) Transfused blood for the fetus is matched to the father's blood group.
372- The CORRECT statement regarding prophylaxis against Rhesus sensitization:
a) Anti-D is not required if the mother is group O and the baby group B.
b) The Kleihauer test is not accurate and no longer performed.
c) Prophylaxis is unnecessary in ectopic pregnancy.
d) Routine provision of anti-D during pregnancy is recommended at 20 weeks' gestation.
e) Postnatal anti-D is not necessarily ifthe neonate is Rh negative.
373- The CORRECT statement for Coombs' test:
a) The direct test detects maternal IgM on fetal cells.
b) Is positive in the baby with jaundice due to spherocytosis.
c) The indirect test is used to detect antibodies in the maternal serum during pregnancy.
d) Is used in the investigation of thrombocytopenia.
374- The CORRECT statemnent regarding Kleihauer Test:
a) It may be used to confirrn the presence of Rhesus antibodies.
b) It should be performed routinely at 28 and 36 weeks in the woman who is Rhesus negative.
c) It is no longer required after delivery in the Rhesus negative woman.
d) It is based on the relative resistance of fetal haemoglobin to denaturation using ultra-violet light.
e) It is most beneficial if excess fetomaternal hernorrhage is suspected.
375- Rh negative mother married to Rh positive father, the followings are true statement EXCEPT:
a) First baby usually passed unaffected.
b) Anti D immunoglobulin must be taken after abortion, ectopic pregnancy, vesicular mole, chorionic
villus sampling.
c) Anti D imnrunoglobulin is best given within the first week after delivery.
d) She may get Rh negative baby.
e) She may be sensitized during her first pregnancy due to weak placental barrier.
376- Which of the following maternal antibodies DOES NOT cause erythroblastosis fetalis in the fetus?
a) Anti-C.
b) Anti-E.
c) Anti-D.
d) Anti-Lewis.
377- Which of the following characteristics and complications of pregnancy DOES NOT increase the
risk of Rh sensitization?
a) Type of delivery.
WhiteKnightLove

b) Vaginal bleeding.
c) Preeclampsia.
d) Maternal age,
e) Manual removal of placenta.

-43-
Freely you have received; freely give.

378- Possible episodes causing sensitization of Rh negative mother include all the followings EXCEPT:
a) Spontaneous or elective aboftion.
b) Previous transfusion with Rh negative blood,
c) Previous delivery of Rh positive fetus.
d) Antepartum hemorrhage.
e) Manual removal of placenta.
379- Rh immune globulin IS NOT recommended in the following situation:
a) At 20 weeks' gestation.
b) Postpartum within 72 hours.
c) After spontaneous aborlion or ectopic.
d) 300 ug after full term delivery.
e) 100 ug if pregnancy terminated before 13 weeks.
380- The severity of fetal affection in Rh sensitized woman can be detected by all the followings EXCEPT:
a) Umbilical cord blood sampling (PUBS).
b) Indirect Coombs' test.
c) Ultrasound.
d) Detailed history.
e) Amniocentesis.
381- At 32 weeks' gestation amniocentesis is performed to Rh negative isoimmunized patient (having
antiD titer of 1:128) the AOD is placed high in zone II on chart. Which of the following measures
should be done:
a) Perform an immediate intrauterine transfusion.
b) Prepare for pregnancy termination.
c) Repeat maternal serum antibody titer.
d) Repeat U/S examination and amniocentesis after I week.
e) Perforrn RH typing of her husband.
382- A 38-year-old G3Pl comes to see you for her first prenatal visit at l0 weeks' gestational age. She
had a previous term vaginal delivery rvithout any complications and one spontaneous complete
abortion at 7 weeks' gestation 4 years previously. You detect fetal heart tones at this visit, and The
uterine size is consistent with dates. You also draw her prenatal labs at this visit and tell her to
follow up in 4 weeks for a return visit. Two weeks later, the results of the patients prenatal labs
come back. Her blood type is A RH- negative, with an anti-D antibody titer of l:4. What is the
most appropriate next step in the management of this patient?
a) Schedule an amniocentesis for amniotic fluid bilirubin at l6 weeks.
b) Repeat the antibody titer after 4 weeks.
c) Repeat the antibody titer at 28 weeks' gestation.
d) Schedule PUBS (percutaneous umbilical blood sampling) to determine fetal hematocrit at 20 weeks.
e) Schedule PUBS as soon as possible to determine fetal blood type.
383- An unsensitized Rh-negative woman in her second pregnancy is seen in her thirty-sixth week. She
complains of edema in her legs and some tingling in her left hand. The most appropriate action at
this time is
a) Analysis of the husband's blood type.
b) Intramuscular Rh (anti-D) immune globulin.
c) Assessment for possible preeclampsia.
d) Rh antibody titer.
e) Amniocentesis.
384- An ultrasound examination of a 30-year-old woman reveals the followlng findings, all of which are
suggestive of erythroblastosis fetalis EXCEPT:
a) Bowing of fetal femur.
WhiteKnightLove

b) Fetal ascites,
c) Fetal cardiornegaly.
d) Thickening of the placenta.
e) Fetal head measurement that is large for gestational age.

-44-
'It is more blessed to give than to receive.

385- In RH sensitized mother the percutaneous umbilical cord blood sampling (PUBS) used to detect
all the following EXCEPT:
a) Level of antibodies using indirect coomb test.
b) Fetal blood group and RH.
c) Fetal hemoglobin and hematocrit level.
d) Bilirubin level.
e) Reticulocytic count.
386- Detection of maternal antibodies in the fetal blood that had been attached to fetal cells is
accomplished by:
a) Indirect Coombs' test.
b) Direct Coombs' test.
c) Radioimmunoassay.
d) Rosette test.
e) Enzyme-linked antiglobulin test.
387- A routine urine analysis of a primigravida showed l-2 pus cells/[IPF, but the bacteriat colony
countwasmorethanl00,000/ml.Asregardsthiscondition,the@statementis:
a) It is more common in non-pregnant than pregnant women.
b) Diagnosis depends on cornplex of signs.
c) Follow up only is satisfactory initial management.
d) Enterobacter species is the most common causative organism.
e) Furlher urine culture is needed for follow up.
388- In asymptomatic bacteruria, the INCORRECT statement is:
a) If left untreated,it progresses to acute pyelonephritis in approximately 25%o of pregnant wome.
b) Asymptomatic bacteruria is associated with premature labor,
c) Enterobacter species is the most common causative organism.
d) Further urine culture is needed for follow up after treatment.
e) Asyrnptomatic bacteruria is associated with low birth weight
389- Asymptomatic bacterui'ia is diagnosed when the colony count in urine exceeds:
a) 1,000 organisms per milliliter.
b) 10,000 organisms per milliliter.
c) 100,000 organisms per milliliter.
d) 1,000,000 organisms per rnilliliter.
390- Management of asymptomatic bacteruria includes:
a) Expectant management.
b) lnduction of labor.
c) Antibiotics.
d) Diuretics.
e) Intravenous hydration.
391- Which of the following factors DOES NOT contribute to etiology of acute urinary tract infection
during pregnancy, delivery, and the puerperium?
a) Compression of the ureter by the large uterus at the pelvic brim.
b) Increased ureteral tone and increased peristalsis.
c) Asymptomatic bacteruria.
d) Decreased bladder sensitivity after epidural anesthesia.
e) Bladder catheterization following delivery.
392- Which of the following IS NOT a true statement about urinary tract infections during pregnancy?
a) Asymptomatic bacteruria in pregnancy needs to be treated.
b) Pregnancy does not increases the risk for development of asymptomatic bacteruria.
c) Acute systemic pyelonephritis resulting from asymptomatic bacteruria is associated with premature
WhiteKnightLove

Iabor and delivery.


d) Few women who are not bacteriuric at their first prenatal visit develop asymptomatic bacteruria.
e) The incidence of asymptomatic bacteruria is higher in pregnant women with a low socioeconomic
status and increased parity and age.

-45-
Freely you have received; freely give.

393- Oliguria is diagnosed when urine volume is:


a) Less than 800-1600 rnl / 24 hours.
b) Less than 400-500 rnl / 24 hours.
c) Less than I 00-400 ml / 24 hours.
d) Less than 50-100 ml / 24 hours.
394- The followings are known to cause acute renal failure EXCEPT:
a) HELLP syndrome.
b) Sickle cell trait.
c) Septicemia.
d) Concealed anteparlum hemorrhage,
e) Incornpatible blood transfusion.
395- Obstetrical causes of acute renal failure include the followings EXCEPT:
a) Septic aborlion.
b) Eclampsia.
c) HELLP syndrome.
d) Placenta circumvallate.
e) Abruptio placentae.
396- The CORRECT statement for anemia in pregnancy:
a) It is defined as hemoglobin of 9 g/dl or less.
b) It may be caused by hook worm infestation.
c) 50 mg oral elemental iron i day is adequate treatrnent for iron deficiency anemia.
d) Megaloblastic anemia is usually due to vitamin B l2 deficiency.
e) It is less common with multiple pregnancy than with single pregnancy.
397- WHO defines anemia with pregnancy as follow:
a) Hemoglobin level below 1l gldL.
b) MCV value below 80 fL.
c) Hematocrit value less than 25%.
d) MCHC below 33%.
e) Reticulocytic count less than 2010.
398- Anemia in pregnancy IS NOT related to:
a) Iron deficiency.
b) Chronic pyelonephritis.
c) Folic acid deficiency.
d) Multiple pregnancy.
e) Amniotic fluid volume,
399- In iron deficiency anemia in pregnancy, the following is true:
a) Mean corpuscular hemoglobin content and mean corpuscular concentration are both low.
b) Mean corpuscular volume is raised.
c) Blood transfusion is indicated if HB level fall below 9 gldl.
d) There is usually a chronic blood loss causing the anemia.
e) There is an increased risk of preeclampsia.
400- Folic acid deficiency during pregnancy is associated with the following EXCEPT:
a) Hypersegmented neutrophils.
b) Megaloblastic erythropoiesis.
c) Increased rate of systemic infections,
d) Megaloblastic anemia.
e) Abruptio placentae.
401- In the metabolism of iron, the following statements are correct EXCEPT:
a) Absorption occurs mainly in the duodenum and upper jejunum.
WhiteKnightLove

b) Absorption is increased when stores are depleted.


c) Traces ofcopper are essential for hemopoiesis.
d) Transferrin is increased in pregnancy.
e) Ferric salts are more readily absorbed than ferrous salts.

-46-
'It is more blessed to give than to receive.

402' A pregnant IYoman with the sickle cell trait is at risk for an increased incidence of which of the
following?
a) Perinatal morlality.
b) Low-birth weight infants.
c) Pregnancy-induced hyperlension.
d) Urinary tract infection.
e) Spontaneous abortion.
403- A primigravida at26 weeks' gestation and her CBC showing HB of 9,4 gldL; MCV of 77fL and
TIBC saturation < l5%. What is the INCORRECT statement regarding her managementy:
a) The recommended oral dose of elemental iron is 150 - 300 mg/day.
b) Oral iron therapy may cause constipation.
c) Parentral iron therapy might induce teratogenic effects.
d) Adding folic acid to the iron formula is better than iron formula alone,
e) Cyanocobalamin is essential for normal erythropoiesis.
404- The CORRECT statement for the management of iron deficiency anemia with pregnancy:
a) The recommended oral dose of elemental iron is 150 - 300 mg/day.
b) Oral therapy raises the Hb level 2 glweek.
c) Total dose infusion of iron is contraindicated.
d) Main complication of parentral iron therapy is its teratogenic effect.
e) Adding calcium preparation to iron formula is better than iron formula alone.
405- The CORRECT statement in the management of iron deficiency anemia:
a) Diarrhea is a common side effect of oral iron therapy,
b) There is no advantage of intramuscular iron therapy over oral therapy.
c) 50 mg of iron taken orally each day is adequate.
d) Total dose infusion of iron is contraindicated,
e) Packed RBCs transfusion is a rapid method for correction of the anemia (emergency treatment).
406- Which of the following is the MOS'[ SIGNIFICANT SIGN of heart disease in pregnancy?
a) Lower extremity edema.
b) Systolic murrnur,
c) Increased respiratory effort.
d) Arrhythmia.
e) Dyspnea.
407- Pregnancy should be strongly discouraged in women who have which of the following:
a) Atrial septal defect.
b) Ventricular septal defect.
c) Patent ductus ateriosus.
d) Eisenmenger's syndrome.
e) None ofthe above.
408- The INCORRECT statement in the presence of bacterial endocarditis or lts sequelae:
a) Antibiotic prophylaxis is necessary for all women in labor who have had previous bacterial
endocard itis,
b) Antibiotic prophylaxis is necessary for all women in labor who have undergone previous
reconstructive cardiac surgery.
c) Antibiotic prophylaxis is necessary for all
women in labor with prosthetic heart valves.
d) In obstetric practice bacterial endocarditis
is usually caused by fecal streptococci.
e) A single intramuscular injection of 80 mg gentamicin at the onset of labor does not provide adequate
antibacterial cover.
409- A class II cardiac patient who is 38 weeks' pregnant presents to the hospital in labor with dyspnea
on exertion and chest rales. Proper management of this pregnant patient include all of the
WhiteKnightLove

following EXCEPT
a) Epidural anesthesia.
b) Diuretics.
c) Digitalis.

-47-
Freely you have received; freely give.

d) Oxygen therapy.
e) Cesarean delivery.
410- There is NO evidence of immunological basis for the following pregnancy related phenomena: :
a) Recurrent fetal losses.
b) Ectopic pregnancy.
c) Neonatal jaundice.
d) Pre-eclampsia.
e) Transient neonatal thyrotoxicosis.
4l l- The CORRECT statement at treatment of thyrotoxicosis during pregnancy:
a) Propylthiouracil and methimazole are principle medication used.
b) Subtotal thyroidectomy is usually indicated,
c) Radioactive iodine could be used in certain cases.
d) Beta-blocking agents is are not useful.
e) Propylthiouracil do not cross the placenta barrier.
412- Which complication is increased in pregnancy affected by hypothyroidism?
a) Abortion.
b) Fetal macrosomia.
c) Polyhydramnios may occur,
d) Gestational diabetes mellitus.
e) DIC.
413- The complications of untreated hypothyroidism during pregnancy include:
a) Preeclampsia.
b) Heart failure.
c) Placental abruption.
d) Allof the above.
414- Effects of hypothyroidism on pregnancy DO NOT include increased risk of:
a) Spontaneous aboftion.
b) Abruptio placentae.
c) Preeclampsia.
d) Neonatal weight of > 4,5 kg,
e) Hean failure.
415- With respect to thyrotoxicosis in pregnancy:
a) Thyroid enlargement indicates inadequate treatment.
b) Mild thyrotoxicosis cannot be distinguished clinically from normal pregnancy.
c) Subtotal thyroidectomy is acceptable treatment in the second trimester.
d) Neonatal hyperthyroidism does not occur ifthe mother has been euthyroid.
e) Breastfeeding is contraindicated in women taking propylthiouracil.
416- The adverse pregnancy outcomes in antiphospholipid antibody syndrome may include the
followings EXCEPT:
a) Recurrent fetal losses.
b) Early onset pre-eclampsia.
c) Placental abruption.
d) Polyhydramnios.
e) Arterial and venous thrombosis.
417- The following is NOT related to antiphospholipid antibody syndrome:
a) Venous thrombosis.
b) Arlerial thrombosis.
c) Intrauterine fetal death.
d) Congenital malformations.
WhiteKnightLove

418- The INCORRECT statement regarding toxoplasmosis in a pregnant woman:


a) It can be acquired by eating infected raw meat.
b) It can be acquired by contact with cat feces.
c) lnfection in early pregnancy may lead to abortion,

-48-
'It is more blessed to give than to receive.

d) Acute infection is diagnosed by the detection maternal IgG.


e) Fetal affection may occur through placental transfer.
419- which of the following is a common reservoir for toxoplasmosis?
a) Infected school-age children.
b) Urine ofinfected pregnant patients.
c) Contarninated seafood.
d) Excreta ofinfected neonates.
e) Cats and dogs,
420- Heparin therapy in thromboembolic diseases:
a) Osteoporosis is more common with low molecular weight heparin (LMWH) treatment.
b) Thrombocytopenia does not occur except after I week of heparin treatment.
c) Overdose is the rnajor risk of heparin treatment.
d) Long term heparin treatment often results in thrombocytopenia.
e) Unfractionated heparin (UFH) does not cross the placenta while low molecular weight heparin do
cross.
421- During treatment with heparin during pregnancy, the following should be malntained at a level of
1.5-2.5 times baseline value:
a) Bleeding tirne.
b) Clotting time.
c) Prothrombin time.
d) Parlial thromboplastin time.
e) Partial prothrombin time.
422- As regard warfarin therapy in thromboembolic diseases:
a) It crosses the placenta and may has harrnfuI effects on the fetus.
b) Breast feeding is a contraindication during warfarin administration.
c) It is usually stafted after complete stoppage of heparin treatment.
d) Its dose is controlled by measurements of partial thromboplastin time (PTT).
e) It is the best treatment ciuring labor.
423- The COMMONEST cause of maternal jaundice during pregnancy is:
a) Acute fatty liver.
b) Intrahepatic cholestasis of pregnancy.
c) Viral hepatitis.
d) Hemolytic uremic syndrome.
e) Physiological.
424- A 33-year-old woman at l0 weeks presents for her first prenatal examination. Routine labs are
drawn and her hepatitis B surface antigen is positive. Liver function tests are normal and her
hepatitis B core and surface antibody tests are negative. Which of the following is the best way to
prevent neonatal infection?
a) Provide immune globulin to the mother.
b) Provide hepatitis B vaccine to the mother.
c) Perform a cesarean delivery at term.
d) Provide hepatitis B vaccine to the neonate.
e) Provide immune globulin and the hepatitis B vaccine to the neonate.
425- Which of the following is the LEAST LIKELY fetaUneonatal sequelae of herpes simplex virus?
a) Transplacental infection.
b) Encephalitis.
c) Disseminated neonatal infection.
d) Conjunctivitis.
e) Gastrointestinal infection
WhiteKnightLove

-49-
Freely you have received; freely give.

426- A 33-year-old woman with a microadenoma of the pituitary gland becomes pregnant. When she
reaches 28 weehs' gestation, she complains of headaches and visual disturbances. Which of the
following is the BEST THERAPY?
a) Craniotomy and pituitary resection,
b) Tamoxifen therapy.
c) Oral brornocriptine therapy.
d) Expectant management.
e) Lumbar puncture.
427- The following complications could frequently affect the elderly primigravida W.EEf:
a) Gestational diabetes mellitus.
b) Pre-eclampsia.
c) Precipitate labor.
d) Vesicular rnole.
e) Placental abruption.
428- Cigarette smoking has lQf been linked to which of the following?
a) Intraepithelial neoplasia ofthe cervix.
b) Accidental hemorrhage,
c) Preeclampsia.
d) Preterm premature rupture of membranes.
e) Preterm labor.

Direction: Each set of matching questions in this section consists of a list of numbered items followed
by several Iettered options. For each numbered item, select the ONE best lettered option that is most
closely associated with it. Each lettercd heading may be selected once, more than once, or not at all.
For each of the following clinical presentations numbered (429-433), select the most suitable line of
management lettered (a-e).
EACH LETTERED HEADING MAY BE USED ONCE, MORE THAN ONCE, OR NOT AT ALL.

429- Patients with placenta previa with severe bleeding and I a) Conservative treatment.
preterm fetus. I b) Allowance or trial of vaginal
430- Patient with placenta previa centralis and at 37 weeks' I delivery,
gestation. I c) Elective cesarean section.
431- Patientwith mild placental abruption, no fetal distress I d) Emergencycesareansection.
and at 33 weeks' gestation, I e) Cesarean hysterectomy,
432- Patient with seyere accidental hemorrhage,
intrauterine fetal death and cervix is 8 cm dilated.
433- Patient with placental edge at 3 cm from the cervical
internal os, presenting in active labor.

For each of the following descriptions numbered (434-440), select the antihypertensive drug lettered
(a-e).
EACH LETTERED HEADING MAY BE USED ONCE, MORE THAN ONCE, OR NOT AT ALL.

434- Acts on the cr receptors in the brain stem. a) Nifedipine.


435- Causes tachyphylaxis if given orally. b) Methyldopa.
436- Produces fetal renal failure. c) Atenolol.
437- Drug ofchoice in acute severe hypertension. d) Hydralazine.
438- Causes lupus-like syndrome. e)
WhiteKnightLove

Captopril.
439- Is given as tocolytic as well.
440- Mav cause fetal heart block.

-50-
'It is more blessed to give than to receive.

For cach of the following clinical pictures numbered (441-447), select the class of classificfation of
diabetes mellitus with pregnancy lettered (a-f).
EACH LETTERED HEADING MAY BE USED ONCE, MORE THAN ONCE, OR NOT AT ALL.

441- Pregnant woman who is 2l years old and had known diabetes for a) Class A.
12
years. b) Class B.
442- Pregnant woman who is 35 years old and had known diabetes for 5 years. c) Class C.
443- Pregnant woman who is 2l years old and had developed diabetes at 29 d) Class D.
weeks' gestation. e) Class R.
444- Pregnant woman who is l8 years old and had known diabetes for 3 years. f) Non of the above.
445- Pregnant woman who is 21 years old and had renal transplantation.
446- Pregnant woman who is 23 years old and had proliferative retinopathy.
447- Pregnant woman who is 27 years old and had nephropat

Questions 448-451
For each description that follows numbered (448-451), select the lettered option (a-e) with which it is
most likely to be associated.
Each lettered choice may be used once, more than once, or not at all.
448- Associated with the Arias-Stella phenomenon. a) Placental polyps.
449- Premature placental separation. b) Placenta previa.
450- Associated with maternal hypertension. c) Abruptio placentae.
451- Usually accompanied by thickened placental villi, hemorrhage into d) Ectopic pregnancy.
the decidua basalis, and evidence of necrosis in tissue near the e) Spontaneous abortion.
bleeding.

WhiteKnightLove

-5 1-
Freely you have received; freely give.

ABNORMAL LABOR

Direction: Select the ONE BEST lettered answer or completion in each question.

452- The CORRECT statement regarding occipitoposterior position:


a) The perineum is distended by the suboccipitofrontal diameter during the delivery of direct occipito
posterior.
b) Deep transverse arrest is best managed by vacuum extraction.
c) The progress oflabor depends on the degree ofhead extension.
d) It is more common in the android pelvis.
e) Only few cases deliver spontaneously by long internal rotation as occipito anterior.
453- The CORRECT statement regarding occipitoposterior position:
a) During delivery of direct occipitoposterior, the perineum is distended by the occipitofrontal
diameter.
b) Deep transverse arrest is best managed by vacuum extraction,
c) The progress oflabor is not related to the degree ofhead flexion.
d) Android pelvis is a rare cause.
e) During labor, 1/3 ofthe cases deliver spontaneously as occipito anterior.
454- The COMMONEST outcome in OP position is:
a) Persistent OP.
b) Deep transverse Arrest,
c) Rotate to OA position.
d) Direct OP.
455- Management of OP position in a primigravida rvith normal pelvis and who just started the 2nd
stage of labor:
a) Immediate C.S.
b) Observe for up to two hours for rotation.
c) Immediate forceps delivery.
d) Immediate ventouse delivery.
456- Management of OP position in the 1st stage of labor may include all of the following EXCEPT:
a) Artificial rupture of the membranes.
b) Oxytocin augmentation.
c) Observation by partogram.
d) Manual rotation of the head.
457- Persistent OP is common in:
a) Flat pelvis.
b) Gynecoid pelvis.
c) Android pelvis.
d) Anthropoid pelvis.
458- Primary face presentation is common in all EXCEPT:
a) Anencephaly.
b) Cystic hygroma.
c) Hydrocephalus.
d) Dolicocephaly.
459- Allof the following may be a method for delivery in face presentation EXCEPT:
a) Observe for spontaneous vaginal delivery.
WhiteKnightLove

b) Forceps delivery.
c) Ventouse delivery.
d) Cesarean section.

-52-
'It is more blessed to give than to receive.

460- A 2l-year-old primiparous patient arrives in labor with poor prenatal care, her tast visit bcing 8
rveels ago. She is 4l weel<s by dates, and statcs shc rupturcd membranes approximately 12 hours
ago, On examination, estimated rveight is 3.9 kg. She has thick meconium. Her cervix is 3 cm,
dilated, and the presenting part is at - 2 station. The prcscnting part is a facc. The fetal heart
tones are excellent, she is not contracting. What is the best management?
a) Oxytocin induction and prophylaxis against group B streptococci.
b) Misoprostol 25 ug, group B streptoccus prophylaxis.
c) Expectant management with group B streptococcus prophylaxis.
d) Epidural anesthesia to relax the pelvis, group B streptococcus prophylaxis, and then oxytocin
e) Cesarean section.
461- Thcre is no mechanism for vaginal delivery rvith brotv prcscntation as the engaging diameter is:
a) Mento bregmatic.
b) Mento vertical.
c) Mento bregmatic and bitemporal diameters.
d) Mento vertical and bitemporal diameters.
462- The following statements regarding face presentation are correct EXCEPT:
a) It occurs once in 300 cases.
b) Mild degree of cephalopelvic disproportion is the commenest cause of secondary fac presentation.
c) The engaging diameter equals the suboccipitobregamatic dianreter in length.
d) Secondary face is common due to extension of occipito-posterior leading to mento-anierior position.
e) The commonest cause of face presentation is hydrocephalus.
463- The following statements, regarding brcech prcsentation, are correct EXCEPT:
a) Uterine septum can cause breech presentation.
b) The corrmonest cause is hydrocephalus.
c) The commonest cause of neonataldeath after vaginaldelivery is intracranial hernorrhage.
d) A safe method of delivery of the after-coming head is by forceps.
e) Prolapse ofthe cord has good prognosis rvith breech presentation compared to cephalic presentation.
464- The follorving conditions may predisposc to a brcech presentation EXCEPT:
a) Hydrocephalus.
b) Android pelvis.
c) Placenta previa.
d) Prematurity.
e) Septate uterus.
465- The condition that DOES NOT predispose to brccch presentation:
a) Hydrocephalus.
b) Succenturiate placenta.
c) Placenta previa.
d) Prematurity.
e) Septate uterus.
466- The type of breech in which both thighs arc flcxed on thc abdomcn and the legs upon the thighs is:
a) Complete.
b) Incomplete.
c) Frank.
d) Double footling.
e) Single footling.
467- A 27-year-old- un-booked primigravida prcsents in labor. Shc has not had any antcnatal care and
it was estimated that she is at term. Abdominal palpation rvas not conclusive for determining the
fetal disposition. Vaginal examination rcvcalcd that the forcwater is bulging and intact, the cervix
is 3 cm dilated but no fetal parts rvas possiblc to be felt. Emcrgency ultrasound revealed that the
WhiteKnightLove

fetal head is at the fundus, the fetal spinc is parallel to tlrc thc nrotherrs spine. The fetal knees and
hips are flcxed. Both arms are flexcd at thc clbow. What is the fctal prcsentation?
a) Frank breech.
b) lncomplete breech.

-s3-
Freely you have received; freely give.

c) Complete breech.
d) Vertex.
e) Unstable.
468- Regarding breech presentation:
a) More common in preterm than at term pregnancy.
b) Cornplete breech is the commonest type.
c) Is common with anencephaly,
d) All cases are delivered by C.S.
469- Indications for elective cesarean section in breech presentation include all the following EXCEPT:
a) Knee presentation.
b) Footling presentation.
c) Frank breech.
d) Extended head.
470- Regarding external cephalic version all are true EXCEPT:
a) Anti-D should be given to Rhesus negative unsensitized mothers.
b) Is best done at thirly weeks.
c) May cause premature labor.
d) May cause ruptured uterus.
471- Complications of cxternal cephalic version are all the follorving EXCEPT:
a) Positive Kleihauer test.
b) Fetal bradycardia.
c) Placental abruption.
d) Prernature rupture of the membranes.
e) Amniotic fluid ernbolisrn.
472- During external cephalic version, all the following statements are true EXCEPT:
a) General anesthesia is required.
b) FHS are checked.
c) Trendelenberg position is recommended.
d) The vulval area is left exposed.
e) The breech is pushed toward the head.
473- Breech extraction is done in all of the following EXCEPT:
a) Delivery of the 2nd twin.
b) Breech delivery during C.S.
c) Breech with cord prolapse and fully dilated cervix.
d) Breech with mild degree contracted pelvis.
474- Which DOES NOT increase thc failure rate of external cephalic version in breech presentation?
a) Polyhydramnios.
b) Ritodrine infusion.
c) Short cord.
d) Hydrocephalus.
e) Frank breech,
475- The following is a cause of transverse lie:
a) Placental abruption.
b) Cervical dystocia.
c) Postterm pregnancy.
d) Crandmultiparity.
e) Gestational diabetes.
476- The CORRECT statement [or transverse lie at 36 weeks.
a) It is commonly due to a bicornuate uterus.
WhiteKnightLove

b) It is an indication for induction oflabor.


c) Ifitpersistsduringthefirststageoflabor,itshouldbetreatedwithinternalversion.
d) Particularly occurs in the multipara.
e) It should be managed by immediate CS.

-54-
'It is more blessed to give than to receive.

471- Management of neglected shoulder is:


a) Internal podalic version and breech extraction.
b) ECV and forceps vaginal delivery.
c) ECV and ventouse vaginal delivery.
d) Upper segment cesarean section.
478- The CORRECT statement for cord Prolapse.
a) In breech presentation, it is most likely with frank breech.
b) It occurs only after ROM.
c) It should be managed by ventouse extraction.
d) In first pregnancy, it grounds for elective Caesarean section in the subsequent pregnancy.
479- Cord presentation is:
a) Descent of the umbilical cord below the presenting part after ROM.
b) Descent of the umbilical cord below the presenting part before ROM.
c) Presence of the umbilical cord beside the presenting part.
d) Presence of the urnbilical cord above the presenting part.
480- Management of complex presentation during the first stage of labor is:
a) Expectant follow up.
b) Cesarean section.
c) Manual reposition and ventouse delivery.
d) Manual reposition and forceps delivery.
481- Regarding twins all are true EXCEPT:
a) The incidence increased in recent decades.
b) Has a perinatal mortality rate higher than that of singleton pregnancy.
c) Is more common with ovulation stimulating drugs,
d) Is associated with hyperemesis.
e) Is associated with normal levels of alphafetoptotein.
482- Regarding twins, all are true EXCEPT:
a) Dizygous type is more comrnon than monozygotic type.
b) Incidence is higher with increasing maternal age.
c) Is associated with an increased risk of congenital malformations.
d) Should be managed in hospital starting 28 weeks.
e) There is increased risk of preeclampsia.
483- Regarding twins, all are true @[:
a) Monozygotic twins are at higher risk of IUFD.
b) When single fetal demise occurs early, it results in fetus papyraceus.
c) Twins should be scanned to look for growth retardation.
d) Singte umbilical artery is more common in dizygotic than monozygotic twin.
484- Twin gestation is associated with:
a) Decreased incidence of polyhydramnios.
b) Increased incidence of prolonged pregnancy.
c) Increased incidence of pregnancy induced hypertension.
d) Decreased incidence of postparlum hemorrhage.
e) Decreased incidence ofplacenta previa.
485- Indication of elective C.S. in fwin pregnancy include allSlQP'f:
a) Weight less than 2 kg.
b) First twin vertex.
c) Previous myomectomy.
d) Single amniotic cavity.
e) Conjoined twins.
WhiteKnightLove

486- For twin infants to be monozygotic, which of the following characteristics must be present:
a) Identical sex.
b) A single amniotic sac.
c) ldentical birth weight.
-55-
Freely you have received; freely give.

d) A single placenta.
e) Absence of chorion between the two amnions,
487- The CORRECT statement in multiple pregnancy:
a) The incidence of spontaneous twin pregnancy is I:80,
b) Perinatal monality rate in triplet pregnancy is similar to that in twin pregnancy.
c) Incidence of congenital fetal malformation in twin pregnancy is similar to that in single pregnancy.
d) Prophylactic cervical cerclage is recommended against preterm labor.
e) The outcome of triplet is not related to fetal weight at delivery.
488- Regarding trvin pregnancy, all the followings arc true EXCEPT:
a) Monozygotic trvins usually have a single placenta.
b) Dizygotic twins have a familial trait.
c) Monozygotic twin rates are influenced by paternal factors.
d) Pregnancy-induced hypertension is more common in twin pregnancy.
e) Cephalic-cephalic twin presentation is the most common presentation.
489- The following statements regarding trvin pregnancy are correct EXCEPT:
a) Cephalic-cephalic twin presentation is the most common presentation.
b) Incidence of trvinning as diagnosed by early US is sirnilar to its incidence at bir1h.
c) Multifetal pregnancy could be suspected if a pregnant lady had an early threatened abortion.
d) Special measures at antenatal care are needed for a pregnant lady with multifetal pregnancy.
e) Examination of the placenta is important in determination of fetal zygozity.
490- The CORRECT statement regarding twin pregnancy:
a) Positive history in the father's family is more important than positive history in the mother's family.
b) Diamniotic dichorionic twin occurs if division occurs after chorionic differentiation.
c) Conjoined twins results from early cleavage of embryo in first 3 days of conception.
d) Diamniotic monochorionic twin occurs if cleavage of the ernbryo occurs after amnion differentiation.
e) Diamniotic monochorionic twin occurs if the division occurs from day 4 to day 8.
491- True statements about the twin-to-twin transfusion syndrome include which of the following?
a) The donor twin develops hydramnios more often than does the recipient twin.
b) Gross differences may be observed between donor and recipient placentas.
c) The donor twin usually suffers from a hemolytic anemia.
d) The donor twin is more likely to develop widespread thromboses.
e) The donor twin often develops polycythemia.
492- In management of multifetal pregnancy, the following is true:
a) Twin pregnancy is considered as a high risk pregnancy.
b) The lower the fetal weight, the safer the vaginal route of delivery.
c) The larger the fetal number, the more small the fetal weight and the safer vaginal route of delivery.
d) Mono-amniotic twins are associated have less perinatal mortality than diamniotic twins.
e) If the first twin is presenting by the breech internal podalic version is indicated.
493- Naegele's pelvis has the following congenital malformation:
a) Arrested development of both ala of sacrurn.
b) Arrested development of one ala of the sacrum.
c) Arrested development of the sacrum.
d) Separation of both pubic bones.
e) Absence ofthe coccyx.
494- Which of the following DOES NOT affect the pelvic capacity:
a) Poliornyelitis.
b) Hip joint disease.
c) Fracture femur.
d) Osteornalacia.
WhiteKnightLove

e) Diabetes mellitus.
495- Contraindication to trial of labor in contracted pelvis:
a) Elderly primigravida.
b) Vertex presentation.

-56-
'It is more blessed to give than to receive.

c) l" degree cephalo-pelvic disproportion.


d) Gynecoid pelvis,
e) Uterine inertia.
496- Which IS NOT a possible complication of contractcd pclvis cluring prcgnancy:
a) Pyelonephritis.
b) Non engagement of the presenting part.
c) Malpresentation.
d) Spontaneous aboftion.
e) Incarceration of a retroverted uterus.
497- Ccphalopelvic disproportion in thc absencc of gross pelvic abnormality can be diagnosed by:
a) Ultrasound.
b) A maternal stature of <155 cm.
c) Trial of labor.
d) X-ray pelvimetry,
e) Pelvic examination.
498- Effccts of contractcd pclvis on thc l'ctus DOES NOT includc:
a) Caput succedaneum.
b) Over moulding of the fetal head,
c) Intra cranial hemorrhage.
d) Fetal malformation.
e) Fetal death.
499- Contraindications of trial of Iabor include thc following W.EEI:
a) Non vertex presentation.
b) Mild degree of cephalopelvic disproportion.
c) Previous two C.S.
d) Elderly primigravida.
e) Heart disease with pregnancy.
500- The follorving findings are suggestive of contracted pclvis during cxamination EXCEPT:
a) Kyphosis.
b) Non-engagement ofthe head.
c) Limping gait.
d) Height 170 cm.
e) Pendulous abdomen in primigravida.
501- Management of slroulder dystocia DOES NOT include:
a) McRoberts maneuver.
b) Cenerous episiotomy.
c) Sustained fundal pressure.
d) Shoulder girdle rotation into one of the oblique diameters of the pelvis,
e) Symphysiotomy.
502- Regarding shoulder dystocia, rvhich is CORRECT?
a) It is not related to maternal diabetes mellitus.
b) Arrest occurs at pelvic inlet.
c) Oligohydramnios is a predisposing condition.
d) Most cases can be resolved by fundal pressure.
e) Facial palsy is a possible complication.
503- The INCORRECT statement regarding shoulder dystocia:
a) More likely to occur in diabetic patients.
b) Anencephaly fetuses have higher risks than normal fetuses.
c) It affects about 0.15-1.7% of allvaginaldeliveries.
WhiteKnightLove

d) Shoulder dystocia never occurs in infants less than 4 kg.


e) The "Turtle Sign" is characteristic.

-57-
Freely you have received; freely give.

504- The first thing to do in shoulder dystocia:


a) Symphysiotomy.
b) Fundat pressure.
c) McRoberts maneuver.
d) Twist the posterior shoulder 180'.
e) Zavanelli maneuver.
505- When managing a shoulder dystocia, McRoberts maneuver
a) Relieves cord compression.
b) Is dangerous in the second stage of labor.
c) Is a rapid technique for increasing the available posterior pelvic diameters.
d) Increases the chance ofchanging an OP presentation to an OA presentation.
506- Precipitate Iabor and delivery may lead to the following complications p$pff:
a) Fracture fetal skull.
b) Amniotic fluid embolism.
c) Vaginal and perineal tears.
d) Constriction ring in the uterus.
e) Atonic postpartum hemorrhage.
507- The following IS NOT associated with precipitate labor:
a) Arnniotic fluid embolism.
b) Intrapartum hemorrhage.
c) Increased perinatal morlality and morbidity.
d) Chorioamnionitis.
e) Genital prolapse.
508- Management of pregnancy with previous precipitatc labor is:
a) Hospitalization all through next pregnancy.
b) Controlled delivery in the next pregnancy by induction of labor once at term.
c) Elective C.S. in the next pregnancy.
d) Tocolytics all through 3rd trimester in the next pregnancy.
509- The CORRECT statement for contraction ring is that it:
a) Occurs at any ofthe three stages of labor.
b) Is always located between the upper and the lower uterine segments.
c) Rises up as labor proceeds.
d) Is felt and seen abdominally
e) Is associated with fetal and maternal distress.
510- The CORRECT statemcnt for pathological retraction ring is that it:
a) Doesn't change in position during progress of labor.
b) Is felt vaginally only and not per abdomen.
c) Occurs during prolonged 2nd stage.
d) Occurs at any level ofthe uterus.
e) Allows feeling the fetal parts,
5l l- The INCORRECT statement for the pathological retraction ring:
a) It is a complication of obstructed labor.
b) It is also called Bandl ring.
c) lt can be felt and seen abdorninally.
d) It is situated between the upper uterine segment and the lower uterine segment.
e) Its abdominal site is constant and does not change with labor progression.
512- All the following are true about obstructed labor EXCEPT:
a) The primigravid uterus responds by hypotonic inertia.
b) The rnultigravid uterus responds by excessive contraction.
WhiteKnightLove

c) It may end by intrauterine fetal death.


d) Obstructed labor may be an acute event that occur in few minutes.
e) Rupture of the lower uterine segment may occur.

-58-
'It is more blessed to give than to receive.

513- Prolonged labor, frequcnt strong uterine contractions and carly premature rupture membranes,
then THE MOST LIKELY diagnosis is:
a) Obstructed labor.
b) tuFD.
c) Ruptured uterus.
d) Uterine inertia and atony.
e) Placental separation (abruption placentae),
514- Cervical causes of obstructed labor may include all of the follorving EXCEPT::-
a) Cervical dystocia.
b) Cervical ectropion.
c) Cervical stenosis.
d) Cervical scar.
e) Cervical cancer,
515- Fetal causes of obstructed labor may bc duc to all of the follorving EXCEPT:
a) Down syndrome.
b) Shoulder presentation.
c) Macrosomia.
d) Brow presentation.
e) Conjoined twins.
516- The INCORRECT statement for the pathological rctraction ring:
a) It occurs rvith prolonged second stage oflabor.
b) It is also called Bandl ring.
c) The upper segment becomes tender and tonically retracted.
d) It occurs in between the upper uterine segment and the lower uterine segment.
e) It can be relieved with antispasrnodics.
517- The INCORRECT statement for obstructed labor:
a) Late cases manifest by impending rupture of the uterus,
b) The vagina is dry, hot and edematous in established obstructed labor,
c) The incidence is stable and cannot be reduced by antenatal care.
d) Occiptoposterior position ofthe fetal head is a possible etiological factor.
e) Necrotic vesicovaginal fistula is a possible late sequel.
518- A 39-year-old wife was admitted to the labor ward. She was complaining of sudden onset of lower
abdominal pain about 3 hours previously. The duration of her gestation was 32 weeks by dates
and thc height of the uterine fundus rvas compatible rvith this. She had had four uncomplicated
previous pregnancies. On admission, her temperature rvas 36.6oC, her pulse ratc 92 beats / minute
and her blood pressure 150/90 mmHg. Abdominal cxamination revealed a cephalic prcsentation,
three-fifths palpable. The uterus was tender and palpable contractions were felt every l-2
minutes. Vaginal examination revealed a cervix 0.5 cm dilatcd, but still 2 cm long and firm.
Cardiotocography rvas commenced, and the fetal hcart rate showed a baseline tachycardia of 170
beats/minute with loss of the baseline variability and late decelerations. Repeat cervical
assessmcnt after onc hour revealed no changc in dilatation or effacement. Which abnormality in
this patient is associated rvith thc clevated blood pressure?
a) Obstructed labor.
b) Placental abruption.
c) Prolonged labor.
d) Cervicaldystocia.
e) None ofthe above.
519- Uterine rupture is more common in multiparous rvomen due to increased incidencc of all the
following EXCEPT:
WhiteKnightLove

a) Malpresentation.
b) Intra-uterine grorvth restricted fetus.
c) Pendulous abdomen.
d) Osteomalacia.

-59-
Freely you have received; freely give.

e) Uterine rvall weakness.


520- Rupture of the uterus is morc common in pregnancies following upper segmcnt cesarcan section
(USCS) than following lorver segmcnt cesarean section (LSCS). The explanation EE_M
include:
a) Improper cooptation of the edges of the upper segment incision.
b) Infection is more common in the lower segment.
c) The upper segment is much thicker than the lower segment.
d) The upper segment is active during puerperium.
e) Placental implantation is more common at the upper segment than the lower segment.
521- Rupture of the uterus is NOT associatcd with:
a) Fetal distress.
b) Hematuria.
c) Blood-stained liquor.
d) Uterine hyperlonia.
e) Increased maternal mortality.
522- The CORRECT statement regarding rupturc uterus:
a) More common in nulliparous than multiparous women.
b) More common with malpresentations.
c) Is classified into mild, moderate and severe.
d) Previous uterine scar must rupture in consequence to vaginal delivery.
e) Increases the fetal moftality but not the maternal mortality,
523- Prophylactic measures against rupture of the uterus include all the following EXCEPT:
a) Once cesarean section always cesarean section.
b) Proper performance of instrumental deliveries,
c) Good antenatal care.
d) Special attention to scar ofprevious uterine operation.
e) Proper cervical examination.
524- The following statements regarding rupture of the uterus are correct EXCEPT:
a) It may occur during manual separation of the placenta.
b) During labor, it may occur due to extension of old cervical tear.
c) Rupture during pregnancy is almost always due to external trauma.
d) Rupture during pregnancy presents with anteparlum hemorrhage.
e) More common in multiparous than primiparous women.
525- The following statements regarding rupture of the uterus are correct EXCEPT:
a) It may occur during manual separation of the placenta.
b) In its incomplete type, the uterine cavity is continuous with the peritoneal cavity.
c) It can occur in primiparous women due to inappropriate use of oxytocin during labor.
d) Its complete type is more common in the upper uterine segment.
e) Its incidence varies according to the standard ofantenatal care.
526- The clinical picture of ruptured uterus in labor following obstructed labor @[@include:
a) The patient feeling of obstruction is replaced by feeling of giving way.
b) The patient becomes shocked.
c) Vaginal examination reveals receding freely mobile presenting part.
d) Continuous increase ofthe strong uterine contractions.
e) Vaginal bleeding.
527- After rupture of the uterus following obstructed labor, the follorving may occur EXCEPT:
a) Hypotension.
b) Increased uterine contractions.
c) Hernaturia.
WhiteKnightLove

d) Vaginal bleeding.
e) Fetal distress.

-60-
'It is more blessed to give than to receive.

528- The follorving statements regarding rupture of upper cesarean section scar are correct
EXCEpT:
a) The incidence is2-4%.
b) The scar is weak because of irnproper coaptation of the edges.
c) Upper segment scar is overstretched by subsequent pregnancies,
d) Upper segrnent of the uterus is passive during puerperium.
e) Infection is more common in the upper segment scar than the lower segment scar.
529- complete uterine rupture is characterizedby the foilowing EXCEpT:
a) Massive hemorrhage.
b) All layers of the uterus are involved.
c) The fetus is extruded outside the uterus.
d) The visceral peritoneum may be intact.
e) The fetus is almost always dead.
530- Which of the following IS NOT a complication of uterine rupturc:
a) Maternal shock.
b) Renal failure.
c) Acute uterine inversion.
d) Paralytic ileus.
e) Fetal loss.
53I- Lines of treatment of ruptured utcrus arc variablc, but it DOES NOT inctude:
a) Immediate laparotorny through a midline abdorninal incision.
b) Repair ofthe uterine tear and bilateral internal iliac artery ligation.
c) Repair of the tear and bilateral tubal Iigation.
d) Supravaginal hysterectorny.
e) Antishock measures then conservative follow up.
532- All of the follorving are lines of management of uterine rupture in a patient 38 years old EXCEpT:
a) Total abdominal hysterectomy and bilateral salpingoophorectomy (TAH & BSO).
b) Antishock rrreasures.
c) Repair of the ruptured site.
d) Internal iliac artery ligation.
e) Total abdorninal hysterectomy.
533- The following IS NOT a risk factor for primary postpartum hemorrhage:
a) Maternal anemia,
b) Intrauterine growth retardation.
c) History of a previous postpartum hemorrhage.
d) Uterine fibroids.
e) Mismanagement of 3'd stage of labor.
534- Primary postpartum hemorrhage occurs more commonly in the following conditions EXCEPT:
a) Uterine inertia.
b) History of a previous postpartum hemorrhage.
c) Fibroid uterus.
d) Premature Iabor.
e) Misrnanagement of the 3'd stage of labor.
535- Uterine atony after cesarean section is treated by the following EXCEPT:
a) Uterine massage.
b) Intrarnyometrial syntocinon injection.
c) Intravenous infusion of methergine.
d) Intramyometrial injection of prostaglandin F2q.
e) Bilateral hypogastric artery Iigation.
536- A 34-year-old vvoman is noted to havc significant uterinc bleeding after a vaginal delivery
WhiteKnightLove

complicated by placenta abruption. She is noted to be bleeding from multiplc vcnipuncture sites.
Which of the followings is the BEST therapy?
a) hnmediate hysterectomy.
b) Packing ofthe uterus.

-61-
Freely you have received; freely give.

c) Hypogastric artery ligation.


d) Ligation of utero-ovarian ligaments.
e) Correction of Coagulopathy.
537- The CORRECT statement for primary postpartum hemorrhage:
a) It may occur at any time in the first week after delivery.
b) It may occur with retained placental tissue.
c) The commonest cause is coagulation failure.
d) It is less common with polyhydramnios.
e) It is commonly due to uterine inversion.
538- The initial procedure in atonic postpartum hemorrhage:
a) Antiprostaglandins.
b) Manual massage of the uterus.
c) Bilateral uterine artery ligation.
d) B-lynch sutures.
e) Embolization of pelvic vessels.
539- The late sequelae of the obstetric cervical tears arc all EXCEPT:
a) Recurrent cervical tears in subsequent pregnancies.
b) Eversion and ectropion ofthe cervix.
c) Incompetent cervical os.
d) Retroversion ofthe uterus.
e) Chronic cervicitis.
540- The CORRECT statement for fourth degree perineal tears:
a) The vulva, vagina, external anal sphincter are the only involved structures.
b) Evident intraamniotic injection (chorioamnionitis) is a leading cause,
c) There are two dirnples at the sides of the anus (external and sphincter),
d) Anal tone is preserved ifthe patient constricts herself(voluntarily).
e) If discovered within the first week after delivery, surgical repair is to be done immediately.
541- A woman delivers a 4.5 l<g infant rvith a midline episiotomy and suffers a third-degree tear.
Inspection shows that which of the following structures is intact?
a) Anal sphincter.
b) Perineal body.
c) Perineal rnuscles.
d) Posterior vaginal wall.
e) Rectal rrrucosa.
542- The following statements regarding secondary postpartum hemorrhage are correct EXCEPT:
a) It is excessive blood loss from the genital tract starting 24 hours after delivery and up to 6 weeks
thereafter.
b) Retained products of the placenta are the commonest cause.
c) It is frequently associated with uterine infection,
d) It does not need any surgical treatment.
e) Choriocarcinoma is suspected if bleeding is recurrent.
543- Thc CORRECT statement regarding manual removal of thc placenta:
a) It is perfonned using a pudendal block as analgesia.
b) It is an indication for giving prophylactic antibiotics.
c) It must be followed by uterovaginal pack for 24 hours.
d) It is usually done by piece-meal extraction of placenta.
e) It should be performed if placenta failed to separate within l0 minutes.
544- Manual removal of the placenta:
a) Is performed using a pudendal block as analgesia.
WhiteKnightLove

b) Has been superseded by the use ofthe suction curette.


c) ls usually done by piece-mealextraction of placenta.
d) Should be perforrned if placenta failed to separate within l0 minutes.
e) Is an indication for prophylactic antibiotics.

-62-
'It is more blessed to give than to receive.

545- As regards placenta percreta the INCORRECT statement is:


a) It may be associated with placenta previa.
b) It should be managed by removal of as much placenta as possible.
c) It is associated with a previous cesarean section.
d) It may necessitate hysterectomy.
e) It commonly involves invasion through to the serosal coat of the uterus.
546- Eight minutes after a normal delivery under pudendal anesthesia, the patient has not completed
the third stage of labor. The uterus is globular and firm; no bleeding is evident. What should you
do?
a) Pull steadily but with greater traction on the cord.
b) Perform Crede's maneuver.
c) Augment the contractions with low-dose oxytocin.
d) Manually remove the placenta.
e) Gently massage the uterus and wait.
547- The INCORRECT statement for paravaginal hcmatomas:
a) They rnay follow forceps d livery,
b) They usually resolve spontaneously.
c) Surgical intervention is always necessary.
d) They frequently have no identifiable source.
e) They may cause postpartum collapse.
548- As regards acute uterine inversion, the INCORRECT statement is:
a) It only occurs with a relaxed uterus.
b) It is usually caused by applying fundal pressure.
c) It is managed by irnmediate removal of placenta before reposition of the uterus.
d) It can be managed by increasing the hydrostatic pressure in the vagina.
e) Maintaining sufficient intravenous lines is essential.
549- Immediate therapy for a completely inverted uterus after delivery includes:
a) Maintaining sufficient intravenous ecbolics.
b) Infusing oxytocin.
c) Intravenous fluids and administration of nitroglycerine.
d) Emergency laparotorny.
e) Emergency vaginal hysterectomy.
550- Postpartum collapse without vaginal bleeding may be due to any of the following EXCEPT:
a) A ruptured uterus.
b) Acute uterine inversion.
c) Amniotic fluid embolism.
d) A paravaginal hematoma.
e) Uterine subinvolution.
551- The CORRECT statement regarding amniotic fluid embolism (AFE):
a) AFE carries a higher imrnediate rnorlality rate than pulmonary thromboembolism.
b) AFE occurs more often after rupture of membranes than in a patient with intact membranes.
c) Autopsy is of no value in retrospective diagrrosis of AFE.
d) Normal pregnancy induces a hypocoagulable blood state.
e) The commonest cause of AFE is premature rupture of membranes.
552- Regarding amniotic fluid embolism, all thc following are correct EXCEPT:
a) It causes cyanosis.
b) It is a complication peculiar to the puerperium.
c) It rnay be associated with accidental lremorrhage.
d) It is complicated by disseminated intravascular coagulopathy.
WhiteKnightLove

e) It occurs more common in rnultifetal pregnancy than in single pregnancy.


553- Coagulation failure IS NOT a major complication of thc following:
a) Amniotic fluid embolus.
b) Abruptio placenta.

-63-
Freely you have received; freely give.

c) Placenta previa.
d) Cram-negative septicemia.
e) HELLP syndrome.
554- The following findings are found in consumptive coagulopathy EXCEPT:
a) Increased prothrornbin time.
b) Increased partial thromboplastin time.
c) Increased antithrombin III consumption.
d) Increased plasma fibrinogen.
e) Increased fibrin degradation products.
555- All may be acceptable line of management of consumptive coagulopathy EXCEPT:
a) Heparin injection to stop furlher coagulation.
b) Treatment of the cause (septic anemia - hemorrhage).
c) Replacernent of the clotting factor.
d) Administration of E-amino caproaic acid to block fibrinolysis.
e) Intramuscular methotrexate I mg/kg.
556- The CORRECT statement regarding amniotic fluid embolism:
a) The ideal care is I00 % effective in preventing maternal mortality.
b) Admission to the intensive care unit is mandatory.
c) Carries better prognosis when amniotic fluid is meconium stained.
d) Definitive diagnosis is made 3-6 weeks after delivery.
e) It has no fetal deleterious effects.
557- The INCORRECT statement, about disseminated intravascular coagulopathy (DIC) is:
a) Fibrin degradation products (FDP) are increased in cases of consumption coagulopathy.
b) Fresh blood should never be given in a case with DIC.
c) Evidence of bleeding with presence of predisposing factor is the I't sign of occurrence of DIC.
d) DIC should be anticipated & blood tests should be performed for early diagnosis.
e) Excessive blood transfusion may be a cause.
558- The MOST COMMON cause of consumptive coagulopathy in pregnancy is:
a) Intrauterine fetal death.
b) Placenta previa.
c) Sepsis.
d) Missed aborlion.
e) Placental abruption.
559- Causes of acute abdomen during pregnancy include the following EXCEPT:
a) Placenta abruption.
b) Complicated fibroid.
c) Ruptured tubal pregnancy.
d) Complicated ovarian cyst.
e) Placenta previa.
560- Causes of acute abdomen with pregnancy include the following EXCEPT:
a) Acute pyelonephritis.
b) Acute appendicitis.
c) Complicated fibroid.
d) Acute polyhydrarnnios.
e) Preterm rupture of fetal membranes.
561- The INCORRECT statement, about acute appendicitis with pregnancy is:
a) It is the most common surgical complication rvith pregnancy.
b) Ctassical signs are often absent.
c) Antibiotics are usually indicated.
WhiteKnightLove

d) It may be complicated by preterm labor.


e) Maternal mortality may occur in 50% because of surgical delay.

-64-
'It is more blessed to give than to receive.

562- The INCORRECT statement for acutc appendicitis with pregnancy:


a) It is the most coffuron surgical complication with pregnancy,
b) Incidence about l/1500.
c) Differential diagnosis includes ectopic pregnancy and accidental hemorrhage.
d) Laparotomy is indicated regardless gestational age.
e) Right lower quadrant pain and Iocal rebound tenderness are usually present.
563- Which of the following statements concerning appendicitis in pregnancy is true?
a) Diagnosis is similar to that in the nonpregnant patient.
b) The maternal death rate is highest in the first trimester.
c) Surgical treatment should be delayed until the dlagnosis is firmly established.
d) The incidence is unchanged by pregnancy.
e) The rate of fetal loss is about 50%.
564- The INCORRECT statement about fibroid uterus with prcgnancy:
a) It is the commonest gynecological tumor with pregnancy,
b) It is commonly complicated with fatty degeneration in pregnancy.
c) It increases the risk ofabortion and preterm labor.
d) It increases the risk ofpost partum hemorrhage.
e) It increases the risk of malpresentation.
565- As regards uterine fibroid with pregnancy, the INCORRECT statement is:
a) It is the comrnonest gynecological tumor with pregnancy.
b) It irrcreases risk of abortion and preterm bir1h.
c) It may associate with post partum hernorrhage.
d) The tumor increases in size during pregnancy.
e) Myomectomy is preferably done during cesarean section.
566- Regarding uterine leiomyoma during pregnancy, the CORRECT statement is:
a) Subserous fibroids carries more complications than submucous fibroids.
b) May cause pregnancy induced hypertension.
c) Liable for red degeneration (necrobiosis).
d) It is best treated by rnyomectomy during CS.
e) Once patient delivered, there is no risk of increased puerperal complications.
567- The INCORRECT statement about ovarian swellings during pregnancy is:
a) Most of them are functional.
b) Most of thern disappear by l4 weeks.
c) Teratoma accounts for 25Yo of ovarian tumors with pregnancy.
d) Serous cystadenoma is more common than mucinous cystadenoma.
e) Most ovarian swellings with pregnancy are more than 5 cm diameter.
568- The best course of action for a complex l2-cm adnexal mass noted at l8 weeks'gestation is:
a) Review after 4 weeks.
b) Laparotorny postponed until after delivery.
c) Immediate laparotomy.
d) Sonographically directed aspiration.
e) Laparoscopy and aspiration.
569- Management of marked atypical cells in a cervical smear at l2 rveeks pregnancy p[Qfinclude:
a) Colposcopy,
b) Treatment of associated infection.
c) A repeat cervical srlear.
d) Termination of pregnancy.
e) Directed cervical biopsy.
WhiteKnightLove

-65-
Freely you have received; freely give.

570- The INCORRECT statement for cancer cervix with pregnancy:


a) Pregnancy does not affect the growth of carcinotna in situ,
b) The comrnonest presentation in pregnancy is pain.
c) Cone biopsy in pregnancy is associated with increased risk of abortion and preterm labor.
d) If detected before 30 weeks' gestation hysterectomy is done followed by external radiation.
e) If detected after 30 weeks'gestation treatment is delayed till35 weeks.
571- The INCORRECT statement for retroverted gravid uterus:
a) Spontaneous correction usually occurs by l2 weeks.
b) It accounts for 50% ofall gravid uteri.
c) Impacted retroverted gravid uterus may cause acute abdomen.
d) hnpacted retroverted gravid uterus may cause abortion.
e) Impacted retrovefied gravid is common in contracted pelvis.

Direction: Each set of matching questions in this section consists of a list of numbered items followed
by several lettered options. For each numbered item, select the ONE best lettered option that is most
closely associated with it. Each lettered heading may be selected once, more than once, or not at all.
For each of the following positions or the presentations numbered (572-576), select the approximate
incidence lettered (a-e).
EACH LETTERED HEADING MAY BE USED ONCE, MORE THAN ONCE, OR NOT AT ALL.

572- Shoulder presentation a 0.003%.


573- Occipitoposterior position b 0.03%.
574- Breech presentation 03%.
575- Face presentation d 3,5%,
576- Brow presentation e) 20%.

For each clinical situation described numbered (577-579), choose the appropriate type of breech
position Iettered (a-e).
EACH LETTERED HEADING MAY BE USED ONCE, MORE THAN ONCE, OR NOT AT ALL.
577- Lower extremitiesareflexedatthehipsandextendedattheknees la) Completebreech.
with the feet lying close to the fetal head. I b) Incomplete breech.
578- Lower extrernities are flexed at the hips, and one or both knees are I c) Frank breech.
flexecl. I d) Single footling breech.
579- The most common breech presentation near term. I e) Double footling breech.

For each case presented below numbered (580-582), select the diagnosis lettered (a-e) that best describes the
tient's clinical condition. Each lettered heading may be used once, more than once, or not at all.
580- A lyoman presents to the Iabor room complaining of painful contractions I a) False labor.
that occur every 2 minutes. She is 2 cm dilated. Two hours later, she I b) Hypertonic uterine
continues to complain of frequent painful contractions, but she is still only I dysfunction.
2 cm clilated. I c) Hypotonic uterine
581- A woman presents to the tabor room with cervix 3 crn dilated and I dysfunction.
contractions every 5-7 minutes. Two hours later she is having contractions I d) Active phase of labor.
every 3 minutes and the ceryix is 6 cm dilated. One hour later the cervix I e) Latent phase of labor.
was 8 cm dilated.
582- A primigravida presents to the labor room with contractions 8-12 minutes
apart, each contraction lasts for 20 second and they are associated with
lower abdominal discomfort. The cervix was 2 cm long and closed. After
WhiteKnightLove

sedation, the contractions continued at a rate of once every 15 minutes


without cervical cha nges.

-66-
'It is more blessed to give than to receive.

For each clinical presentation listed belorv numbered (583-585), select the most likety cause of the
excessive bleeding lettered (a-e) that would be most appropriate.
EACH LETTERED HEADING MAY BE SELECTED ONCE, MORE THAN ONCE OR NOT AT
ALL
583- A 36-year-old woman has just delivered vaginally by forceps her six I a) Cervical laceration.
infant that weighted 4,4 kg, The labor was prolonged and augmented I b) Atonic uterus.
with oxytocin. She is bleeding heavily despite the use of oxytocics, a I c) Uterine rupture.
rvell-contracted uterus, and no evidence ofvaginal or cervical tears. I d) Retention of a
584- A 3l-year-old woman in the recovery room bleeding heavily after I succenturiatelobe,
having vaginaldelivery of twins. I e) Thrombocytopenia.
585- Follorving a spontaneous vaginal delivery, a Z4-year -old rvoman
continues to bleed despite the usc of oxytocin. The uterus appears to
contract well but then relaxes with increased bleedi

WhiteKnightLove

-67-
Freely you have received; freely give.

NORMAL AND ABNORMAL PUERPERIUM

Direction: Select the ONE BEST lettered answer or completion in each question.

586- Constipation is common in early puerperuim due to:


a) Dehydration and low oral intake of fluids and food.
b) Reflex constipation from painful episiotomy.
c) Atony of intestinal, abdominal & perineal muscles.
d) All of the above.
587- Sudden suppression of lochia:
a) Can occur normally.
b) Occurs in subinvolution.
c) May indicate severe infection.
d) None ofthe above.
588- The fundal level of the uterus one week after delivery is:
a) Above the level of the umbilicus.
b) Slightly below the level of the umbilicus.
c) Midway between umbilicus and symphysis pubis.
d) The uterus is again a pelvic organ.
589- Values of proper postnantal exercise include:
a) Decreases respiratory and vascular complications.
b) Decreases genital prolapse.
c) Decreases stress urinary incontinence.
d) AIlof the above.
590- During puerperium, incomplete urinary bladder emptying is common due to:
a) Laxity of abdominal and urinary bladder muscles.
b) Uretheral compression by edema.
c) Recumbent position.
d) All of the above,
591- The following statements about postpartum lactation are true EXCEPT:
a) Breast engorgement may occur during the first postpartum day.
b) Menstruation may resume by 6 to 8 weeks in most nonlactating women.
c) Ovulation may occur within 6 weeks in lactating women.
d) Only colostrum is produced at the time of delivery.
e) Oxytocin is not crucial for the production of milk.
592- The following hormone is responsible for the contraction of the myoepithelial cells of the breast:
a) Oxytocin.
b) Prolactin.
c) Progesterone.
d) Placental lactogen.
e) Estrogen.
593- The INCORRECT statement regarding colostrum:
a) lt contains antibodies to E. coli.
b) It has more fat than mature human milk.
c) It has less carbohydrate than cow milk.
d) It contains immunoglobulins.
WhiteKnightLove

e) It contains steroid hormones.


594- The following statement about birth control after delivery is CORRECT:
a) It is not irnportant until after the first menses.
b) It is not necessary in a woman who is nursing.

-68-
'It is more blessed to give than to receive.

c) It should begin within 3 weeks in a non-lactating woman.


d) It is not necessary for 3 months after a cesarean section.
e) Combined oral pills can be used.
595- Maternal mortality refers to the number of maternal deaths that occur as the result of the
reproductive process per:
a) 1000 births.
b) 10.000 births.
c) 100.000 births.
d) 10,000 Iive births.
e) 100.000 live births.
596- During the first postpartum week, the uterus loses:
a) l0 percent of its immediate postparlum weight.
b) 25 percent of its immediate postpaftum weight.
c) 50 percent of its immediate postpartum weight.
d) 75 percent of its irnmediate postpartum weight
e) 90 percent of its immediate postpartum weight.
597- After belivery, it is normal for hemorrhagic lochia to last up to:
a) 2 days.
b) 5 days.
c) 8 days.
d) I I days.
e) l4 days.
598- Which of the following is the INCORRECT statement regarding postpartum mastitis:
a) The rnost common pathogenic organism is staphylococcus aureus.
b) It usually occurs within the first postpartum week.
c) The offending organism could be cultured from breast rnilk.
d) The mother was probably a chronic carrier of the organism prior to breast feeding.
e) There is no contraindication to the continuance ofbreast feeding.
599- Each of the following is a risk factor for postpartum endomyometritis EXCEPT:
a) Numerous vaginal exarninations.
b) Bacterial vaginosis.
c) Internal uterine pressure monitors.
d) Prolonged rupyure of membranes,
e) Induced labor.
600- A 22-year-old woman who underwent cesarean delivcry has persistent fever of 38.9"C, despite
antibiotic therapy (ampicillin, gentamycin, and clindamycin). The urinalysis, wound, breasts, and
uterine fundus are normal on examination. A CT scan of the pelvis is suggestive of septic pelvic
thrombophlebitis. Which of the following is the best therapy for this condition?
a) Continued triple antibiotic therapy.
b) Discontinue antibiotic therapy and initiate intravenous heparin,
c) Continue antibiotic therapy and begin intravenous heparin.
d) Surgical embolectomy,
e) Streptokinase therapy.
601- A 30-year-old G1 Pl who underwent a cesarean section 3 days previously has a fever of 40'C. The
wound is indurated and erythematous. Which of the following is the best management?
a) Initiation of intravenous ampicillin.
b) Initiation of intravenous heparin.
c) Corticosteroids therapy.
d) Placement of a wann compress on the wound.
WhiteKnightLove

e) Wound drainage.

-69-
Freely you have received; freely give.

THE FETUS AND THE NEWBORN

Direction: Select the ONE BEST lettered answer or completion in each question.

602- Neonatal morbidity and mortality are most strongly influenced by which of the following?
a) Birth weight.
b) Gestational age.
c) Level ofneonatal nursery care.
d) Maternal medical condition.
603- Which of the following factors IS NOT associated with an increased risk of perinatal morbidity?
a) Low socioeconomic status.
b) Low maternal age (iess than 20 years old).
c) Heavy cigarette smoking.
d) Alcohol abuse.
e) Physical exercise.
604- Shortly after vaginal delivery of an infant in an occiput posterior position, a vaguely demarcated
edematous area over the midline of the skull was noted. This observed lesion is MOST LIKELY to
be:
a) A cephalhematoma.
b) A skull fracture.
c) Caput succedaneum.
d) A subduralhematoma.
e) Subgaleal hematoma.
605- The INCORRECT statement, as regards cephalhematoma, is:
a) Is hemorrhage under the periosteum ofskull bone.
b) Instrumental delivery is the main predisposing factor.
c) The swelling disappear within 24 hours after delivery.
d) Skull fractures may be seen in some of cases.
e) Hyperbilirubinemia are common sequalae in large cephalhematoma.
606- Suggestive criteria of IUFD include the following EXCEPT:
a) Milk secretion from the breast.
b) Vaginal dark brown discharge.
c) Pregnancy test becomes negative within 24 hours.
d) Spalding's sign by plain X-ray.
e) Hypofibrinogenemia is a serious complication.
607- Which IS NOT a sign of hyaline membrane disease (RDS):
a) Increased respiratory rate.
b) Grunting respiration.
c) Chest wall retraction during inspiration.
d) Retraction ofthe subsoctal area.
e) Jaundice.
608- The INCORRECT statement, as regards respiratory distress syndrome, is:
a) Is common in neonates of diabetic mothers.
b) Corticosteroid before 35 weeks ofpregnancy has a preventive role.
c) In diabetic patient, detection of phosphatidylglycerol in amniotic fluid is the most specific &
sensitive test in detection of lung maturity.
WhiteKnightLove

d) Management of RDS may be conrplicated by retrolental fibroplasia.


e) RDS starts to present from second day after delivery.

-70-
'It is more blessed to give than to receive.

609- The mechanism by which betamethasonc reduccs respiratory distress syndrome is by increasing:
a) Increased cytokine production.
b) Increased prostaglandin production.
c) Increased surfactant production (Phosphatidylglycerol production).
d) Increased alveolar growth.
e) Increased latency period.
610- Recognized features of Turner's syndrome include the following EXCEPT:
a) Coarctation of the aorta.
b) Elevated serum gonadotrophin levels (hypergonadotrophic hypogonadism).
c) Increased carrying angle (cubitus vulgus).
d) The incidence rises with advanced maternal age.
e) Neonatal edema of the feet.
6l l-
As regards c feto-protein, the following statements are correct EXCEPT :
a) The concentration is reduced in the maternal serum in Down's syndrome.
b) lt is produced by the yolk sac.
c) It constitutes a part ofthe triple test.
d) The highest concentration in the maternal serum at about l6 week's gestation.
e) The concentration in fetal serum and amniotic fluid is different,
612- A woman who is l6 weeks pregnant has a maternal serum a-fetoprotein (MSAFP) level of 2.8
multiple of the mean (MOM). Which of the following conditions IS NOT an explanation of this
abnormalfinding?
a) Anencephaly.
b) Down syndrorne.
c) Duodenal atresia.
d) Omphalocele.
e) Twins.
613- The following statements regarding screening for Down's Syndrome are correct EXCEPT:
a) Biochemical screening has been shown to improve detection rates.
b) Chorionic villus sarnpling (CVS) is one component of the triple test.
c) Biochernical screening is rnost strongly dependent on rnaternal serum o, feto-protein (MSAFP).
d) A booking ultrasound examination is required prior to biochernical screening,
e) Parental karyotype analysis is indicated if there is a family history of Down's syndrome.
614- A 20-year-old primigravida, who is 24 weeks' pregnant, expresses concern about the normality of
her fetus after learning that a close friend has just delivered an infant with hydrocephalus. Details
about hydrocephalus that should be included in her counseling include all the following
EXCEPT?
a) May be due to congenital infection with toxoplasmosis.
b) May be due to congenital infection with cytomegalovirus.
c) Commonly associated with other congenital abnormalities.
d) Can be cured in almost all case by intrauterine placement of shunts.
e) Can not be identified in the first trimester of pregnancy.
615- Causes of neonataljaundice include the following EXCEPT:
a) Polycythemia.
b) Beta thalasemia.
c) Congenital biliary atresia.
d) Toxoplasmosis.
e) Excessive oxytocin adrninistration during labor,
616- The INCORRECT statement, as regards physiological neonatal jaundice is:
a) Almost 50% of all newborns have visible jaundice in the first week of life.
WhiteKnightLove

b) Thejaundice appears in the third day after delivery.


c) In preterm newborns there is a delay in the disappearance ofjaundice.
d) Conjugated bilirubin is less than unconjugated,
e) Total bilirubin usually exceeds 20 mg/dl.

-7 l-
Freely you have received; freely give.

617- The antibiotic that, if given to the near-term pregnant woman, may result in significant neonatat
hyperbilirubinemia is:
a) Penicillin.
b) Cephalosporin.
c) Clindamycin.
d) Azithromycin.
e) Sulfonamides.
618- Neonatal respiratory distress syndrome is due to:
a) Increased lung fluid.
b) Decreased lung fluid.
c) Deficiency ofthe Iung surfactant.
d) Increased surfactant.
619- A low one-minute Apgar score helps identification of:
a) Distressed newborn who needs resuscitation.
b) Newborn with binh trauma.
c) Newborn destined to develop neurological problems.
d) Newborn likely to develop cerebral palsy,
e) Newborn likely to develop RDS.
620- Which of the following IS NOT a component of the Apgar score?
a) Heart rate.
b) Respiratory effort.
c) Color of the skin.
d) Amniotic fluid consistency.
621- What does the Apgar score at five minutes represent?
a) Likelihood ofcerebral palsy.
b) Index ofeffectiveness ofresuscitation and neonatal prognosis.
c) Early neonatal maturity.
d) Incidence ofbinh asphyxia.
622- Which of the following neurological deficits is most clearly related to perinatal asphyxia?
a) Mental retardation.
b) Epilepsy.
c) Hypotonia.
d) Cerebral palsy.
623- What percentage of newborns among general population has a structural congenital anomaly?
a) <l%.
b) 2-s%.
c) 7-8%.
d) t0%.
624- What is the MOST COMMON chromosomalabnormality in early spontaneous abortions?
a) 45 X.
b) 47, XXY.
c) 47, XXX.
d) Trisomy.
625- Which of the following IS NOT a characteristic finding in newborns with Down syndrome?
a) Large head.
b) Flattened occiput.
c) Upslanting palpebral fissures.
d) Small, flat nasal bridge.
626- Failure of development of the vault of the skull and the underlying cerebral hemispheres lead to:
WhiteKnightLove

a) Exencephaly.
b) Anencephaly.
c) Encephalocele,
d) Iniencephaly.

-72-
'It is more blessed to give than to receive.

627- Which of the following ultrasound findings is most strongly associated with Down syndrome?
a) Enlarged cardiac ventricles.
b) Increased fetal muscular tone.
c) Thickened nuchal fold.
d) Verlebrat anomalies.
628- Which of the following women have an increased risk of birth defects?
a) Epileptic women on carbamazepine.
b) Epileptic women on hydantoin.
c) Untreated epileptic women.
d) All of the above.
629- ln utero exposure to diethylstilbestrol causes which of the follorving?
a) Male infertility.
b) Testicular cancer.
c) Uterine malfonnations.
d) Vaginal adenocarcinoma.
630- There is a clear dose-dependent relation betrvcen tobacco smoking and which of the following?
a) Decreases in intelligence scores.
b) Fetal growth restriction.
c) Postterm pregnancy.
d) Severity of pregnancy-inducedhypertension.
631- Which of the following antibiotics has the least access to the fetus when given to the mother?
a) Peniciltin.
b) Erythromycin.
c) Cephalosporins.
d) Tetracyclines.
632- Which has been proved to be most effective, when given to the mother in premature labor, to
reduce neonatal intraventricular hemorrhage?
a) Vitamin A.
b) Vitamin E.
c) Phenobarbital.
d) Corlicosteroids.
633- What fetal heart rate pattern is suggestive of severe fetal anemia?
a) Sinusoidal pattern.
b) Lost beat-to-beat variability.
c) Repetitive late decelerations.
d) Early deceleration pattern.
e) Acceleration pattern.
634- What type of blood is utilized for the initial exchange transfusion in the anemic newborn?
a) O, D-negative.
b) O, D-positive.
c) Maternal blood type, D-positive.
d) AB, D-negative.
635- Onc dose of 300pg of anti D-immunoglobulin will protect the mother against a fetomaternal
hemorrhage of approximately how much fetal blood?
a) 5 mL.
b) 30 mL.
c) 90 mL.
d) 150 mL.
WhiteKnightLove

-73-
Freely you have received; freely give.

636- In the otherwlse uncompllcated tcrm ncwborn, whot h thc unconJugrted bltlrubln tevel above
which kernicterus ls likely to develop?
a) 3 meldL.
b) 7 mg/dL.
c) 15 to 16 mg/dL,
d) 20mg/dL.
18 to
637- Wlth physlologlcrl Jaundlce, what ls the merlmum level thrt scrum blllrubin may rerchu?
a) 2neldL,
b) 5 me/dL,
e) l0 mg/dL.
d) 18 mddL.
538' What ls the MQSI.,$QMMQN caure of neonatrl lntracranlal hemorrhage?
a) Preterm birth.
b) Birth trauma.
c) lnfectious causes.
d) Incorrect modication.
639. Whlch k the focal swelllng of the rcalp from odema overlylng the perlorteum?
a) Cephalohematoma.
b) Caput succedaneum.
c) Periosteal hematoma.
d) Caput perlosteum.
640. Tortlcollis ls morc commonly assoclated with whlch of the followlng dellvery modec?
a) Spontaneous vertex.
b) Breech extraction.
c) Forceps.
d) Cesarean.

Dlrcstlon: Each sct of matchlng questlons ln thls soctlon conslsts of a llrt of numbered ltems followed
by revernl lettered optlons. For each numbered ltem, rclect the ONE bcst lettered optlon that ls most
clorely ossoclated wlth it. Each lettercd headlng moy bc gelected oncq more than once' or not at all.
For eech of the following drugr used ln pregnancy numbcrcd (641-645), select the presumptlve etfcct
on the fetur lettered (a-l).
EAEH LETTERED HEADING MAY BE USED ONCE, MORE THAN ONCq OR NOT AT ALL

t, mphenlcol a) Ototoxicity.
64?' Sulphonamldes b) Bone affection,
6{3. Cerbamazeplne c) Kernicterus.
644. Tctracycllne d) Cardiotoxicity.
645. Gentamlcln e) Grey baby syndrome.
f) Neural tube dofeet.
WhiteKnightLove

'74'
'It is more blessed to give than to receive.

OPERATIVE OBSTETRICS

Direction: Select the ONE BEST lettered answer or completion in each question.

646- Indications of induction of labor includc the followings EXCEPT:


a) Pre-eclampsia.
b) PROM at 38 weeks' gestation.
c) Fetal demise.
d) Post term pregnancy.
e) Placenta previa with mild bleeding, no labor pains at 30 weeks.
647- Side effects of oxytocin administration @ NOT include:
a) Hypotension.
b) Hypernatremia.
c) Fetal distress.
d) Uterine rupture.
e) Amniotic fluid ernbolism.
648- Contraindication of augmentation of labor by oxytocin is:
a) Occipitoposterior.
b) Hypotonic ineftia.
c) Previous rupture uterus.
d) Rupture fetal membranes.
649- The CORRECT statement for oxytocin:
a) Is a steroid hormone.
b) The uterine sensitivity to it declines towards the end of pregnancy.
c) Has some anti-diuretic action.
d) Is synthesized in the posterior lobe ofthe pituitary gland.
e) Secretion is stimulated by alcohol.
650- Uterine stimulants include all of the following EXCEPT:
a) Oxytocin.
b) Ritodrine.
c) Ergometrine.
d) Prostaglandins.
651- Contraindications of ergot alkaloids include all of the following EXCEPT:
a) Postpartum hemorrhage.
b) During pregnancy before delivery ofthe fetus.
c) Severe cardiac disease.
d) Eclampsia.
652- Side effccts of prostaglandins given in late pregnancy include all of the following EXCEPT:
a) Nausea, vomiting and diarrhea.
b) Bronchospasm.
c) Neonatal hyperbilirubinemia.
d) Fever and headache.
653- Oxytocin for induction of labor may be associated with all of the follorving EXCEPT:
a) Fetal distress.
b) Fluid overload.
c) Uterine rupture.
WhiteKnightLove

d) Hyperrension.
e) Neonataljaundice.

-75-
Freely you have received; freely give.

654- Thc following IS Nor an indication for induction of tabor:


a) Prelabor rupture of the membranes at full term pregnancy.
b) Chorioamnionitis.
c) Anencephalic fetus.
d) Maternal heart failure,
e) Post tenn pregnancy.
655- All of the following items constitute Bishop score @:
a) Cervical consistency.
b) Station ofthe presenting part.
c) Pelvic capacity.
d) Cervical position.
e) Degree of effacement.
656- The following are correct about Bishop score EXCEpT:
a) It is used to estimate the possible success of induction of labor.
b) Assessment is done during vaginal examination.
c) It is the basic rnethod to estimate ripening of the cervix.
d) The relevant factors include condition of the fetal membranes.
e) A score more than "8" indicates high possibility for successful induction of labor.
657- The following about misoprostol is true EXCEPT:
a) It is formed of prostaglandin El.
b) The vaginal route is the only route of administration.
c) It may be complicated by fetal distress.
d) Uterine hyperstimulation is more likely than with prostaglandin 82.
e) The induction-delivery interval is shorter than with prostaglandin E2.
658- The CORRECT statement concerning amniotomy:
a) It slows down the progress of labor.
b) It can be used only when the patient is in labor.
c) It decreases the risk ofcord prolapse.
d) It inhibits the release of prostaglandins.
e) It increases the risk ofintrauterine infection.
659- All the following are complications of induction of labor at term EXCEpT:
a) Increased incidence of chorioamnionitis.
b) Increased incidence offetal distress.
c) Increased incidence of neonatal RDS.
d) Increased incidence ofprolonged labor.
e) Increased rate ofcesarean section.
660- Concerning cervical ripening, the following is true EXCEpT:
a) It is a complex process resulting in physical softening of the cervix,
b) It can be spontaneous or aftificial.
c) It is done under the effect of prostaglandins El, F,2,F2o..
d) Usually starls spontaneously at 34 weeks' gestation,
e) It occurs through dissolution of collagen bundles and increase in submucosal water content.
661- Advantages of prophylactic episiotomy include:
a) Less incidence ofdyspareunia.
b) Reduction in duration ofsecond stage oflabor.
c) Decreased blood Ioss.
d) Reduction ofsubsequent pelvic congestion.
e) Avoidance of perineal discomfort during puerperium.
WhiteKnightLove

-76-
'It is more blessed to give than to receive.

662- Advantages of a median episiotomy inctude thc follorvings EXCEpT:


a) Increased area of vaginal outlet to facilitate delivery.
b) Less blood loss compared to mediolateraltechnique.
c) Avoidance of major perineal lacerations.
d) Decreased risk ofinjury to the anal sphincter and mucosa.
e) Greater ease of repair compared to mediolateral technique.
663- The INCORRECT statement for mediolateral episiotomy:
a) More difficult to repair than rnedian episiotomy.
b) Faulty healing is more common than median episiotomy.
c) Dyspareunia is more common following mediolateral than median episiotomy.
d) Extension to the anal sphincter is more common than with median episiotomy.
e) Blood loss is more than following rnedian episiotomy.
664- Advantages of prophylactic episiotomy include all of the following EXCEPT:
a) Surgical repair is better than that after perineal tear.
b) Reduction of duration of first stage of labor.
c) Decreased trauma to pelvic floor.
d) Avoid damage to rectal sphincter.
665- As regards episiotomy:
a) Mediolateral episiotomies are easier to repair than median episiotomies.
b) Dysparunia is not a significant complication.
c) Median episiotomies should be avoided in patients with short perineum.
d) Most deliveries are performed with episiotorny,
666- The following statements regarding instrumental vaginal delivery are correct EXCEPT:
a) It is common in laboring women with epidural anesthesia.
b) With the vacuum extractor, it is too slow to be useful when rapid delivery is required.
c) It may be avoided by the appropriate use ofsyntocinon in the second stage oftabor.
d) It leads to third degree perineal tears more frequently with forceps than with the vacuum extractor.
e) The vacuum extractor is associated with more maternal trauma than the obstetric forceps.
667- The following IS NOT a prerequisite for forccps application:
a) Head must be engaged,
b) Fetus must present either by the vertex or the by the face and the chin posterior,
c) Cervix must be completely dilated.
d) Membraes must be ruptured,
e) Urinary bladder must be evacuated.
668- The CORRECT statement regarding obstetric forceps:
a) Should be used only to assist delivery in cephalic presentation.
b) Should be used before engagement ofthe fetal head.
c) Should not be used with preterm labor.
d) Can be used to assist delivery ofthe fetal head during cesarean section.
e) May be used in face presentation when the chin is directed to towards the sacrum (mentoposterior).
669- The INCORRECT statement regarding traction with the forceps:
a) Traction should be intermittent.
b) Episiotomy is essential,
c) Delivery of the fetal should be slow.
d) It is preferable to apply traction during uterine contractions to avoid excessive pressure.
e) The fetal head should be extended.
670- Kielland's forceps is characterized by one of the following:
a) lt is the best for after coming head of the breech.
b) The blades are shorter and heavier than the mid-forceps.
WhiteKnightLove

c) It can be applied on asynclitic head.


d) It has a deep pelvic curve.
e) It has a german lock.

-77-
Freely you have received; freely give.

671- Which of the following is CORRECT rcgarding application of the obstetric forceps:
a) Cephalic application means application to the biparietal diameter in line with the submento-vertical
diameter.
b) Pelvic application is more safe for the fetus.
c) Cephalic application can cause rnore trauma to the maternal pelvic joints.
d) Cephalo-pelvic application is possible in case of left occiptoanterior position.
e) All of the above.
672- All the following are indications for the use of obstetric forceps EXCEPT:
a) Prophylactic outlet forceps to prevent head compression and perineal injury.
b) Rigid perineum.
c) Maternal heart disease,
d) Prophylactic for the aftercoming head of breech.
e) Fetal macrosomia.
673- All the following are correct about the technique of obstetric forceps EXCEPT:
a) Anesthesia must be used.
b) The right blade is applied first followed by the left blade.
c) The traction is done during uterine contractions.
d) The traction must be in the direction of the pelvic axis.
e) Episiotorny must be done.
674- One of the following is INCORRECT concerning obstetric forceps delivery:
a) It can be used for contracted pelvis at the outlet.
b) Piper forceps is used for delivery of aftercoming head of breech.
c) Kielland's forceps is used for occipto posterior positions of the head.
d) Wrigley forceps is used for low forceps operations.
e) The cervix must be fully dilated.
675- All of the following are true about the obstetric forceps EXCEPT:
a) lt has a traction action.
b) Cephalopelvic application is the safest application.
c) Low forceps is applied for station *2 or more.
d) Mid-forceps is applied for unengaged head.
e) Can be applied for a malrotated head.
676- All of the following can be a cause of failed forceps EXCEPT:
a) Contracted pelvis.
b) Uterine inertia.
c) Short umbilical cord.
d) Cephalopelvic disproportion.
e) Contraction ring.
677- Contraindication of vacuum extractor in assisted vaginal delivery include:
a) Fetal distress in the second stage.
b) Borderline cephalopelvic disproporlion.
c) Shortening the second stage in hypertensive mothers.
d) Suspected fetal coagulopathy.
e) Occiptoposterior postion.
678- All of the following are contraindications to the use of the ventouse EXCEPT:
a) Face presentation.
b) Pretenn labor.
c) Suspected fetal coagulopathy.
d) lntrauterine fetal death.
e) 9 cm dilatation of the cervix.
WhiteKnightLove

-78-
'It is more blessed to give than to receive.

679- Conccrning thc ventouse, rvhich of thc follorving is true:


a) The cervix must be fully dilated.
b) [t needs general or regional anaesthesia.
c) It is more traurnatic to the rnaternal tissues as cornpared to the forceps.
d) It can be applied for a non-vertex presentation.
e) The maximum vacculrr to be applied is - 0.8 kglcm2.
680- All the following are possible complications due to ventouse delivery EXCEPT:
a) Vaginal lacerations.
b) Cervical lacerations,
c) Respiratory distress syndrorne.
d) Subgaleal haemorrhage,
e) Neonataljaundice.
681- A patient rvho is in labor and has one previous lower segment cesarean section (LSCS):
a) Should be delivered irnrnediately by repeat lower segment caesarean section.
b) Epidural anesthetics are absolutely contraindicated.
c) Should be delivered by repeat LSCS if the previous surgery was complicated by uterine infection.
d) Should not have intravenous syntocinon to augment labor.
682- The following statements regarding cesarean section (C.S.) are true EXCEPT:
a) It is safer than vaginal delivery for the breech presentation.
b) Associated infection is rnarkedly reduced by the use of prophylactic antibiotics.
c) Elective C,S. under regional anesthesia is as safe for the mother as normal delivery.
d) lt is commonly performed for prolonged labor (dystocia).
e) The lower segment one is safer than the upper segment one.
683- A classical ccsarean section:
a) Should be done in cases oftransverse Iie.
b) Is performed when large fibroids occupy the upper segment of the uterus.
c) Is perforrned in rnost cases of placenta previa.
d) Is perforrned through a transverse incision in the upper segment of the uterus.
e) The scar is liable to rupture at the third trimester of subsequent gestation.
684- Indications of a cesarean section include all of the following EXCEPT:
a) Failed forceps delivery.
b) Previous cesarean section.
c) Fetal distress.
d) Chronic cervical tear.
e) Cord prolapse.
685- Causes for increased cesarean section (C.S.) rate include the following EXCEPT:
a) Dystocia.
b) Breech presentation.
c) Fetal distress.
d) Intra-uterine fetal death.
e) Repeat C.S,
686- The CORRECT statement regarding the advantages of the upper segment cesarean section over
the lower segment cesarean section:
a) Less possibility of primary bleeding.
b) Less possibility of paralytic ileus.
c) Safer in the presence ofpelvic adhesions.
d) Lower incidence ofsubsequent uterine rupture.
e) Lower possibility of infection.
WhiteKnightLove

-79-
Freely you have received; freely give.

687- The INCORRECT statement regarding preoperative preparation for cesarean section:
a) The bladder must be full to identify the lower segment during surgery.
b) Intravenous fluids to keep normal hydration.
c) Skin area is prepared by antiseptic wash.
d) Blood transfusion is prepared for high risk pregnancies.
e) Confirrn no recent oral intake.
688- Primigravida in labor with a fully dilated cervix for 2 hours, head station -l with moulding and
diffuse caput is best delivered by:
a) Cesarean section.
b) Forceps delivery.
c) Vacuum extraction.
d) Internal podalic version then breech extraction.
e) Fundal cornpression with deep episiotomy.
689- Regarding anesthesia for cesarean section the following statements are correct EXCEPT:
a) Local anesthesia is used in cefiain conditions.
b) Maternal position during the procedure is very irnportant.
c) Anesthetic complications are irnportant causes for maternal mortality.
d) Postoperative aspiration is a common cause of maternal morlality.
e) Epidural anesthesia is associated with high risk of postoperative aspiration.
690- The INCORRECT statement regarding postoperative care after uncomplicated cesarean section:
a) Oral fluids can be started 12 hours after the operation.
b) Should monitor urine output.
c) Confinn uterine contraction by abdorninal palpation.
d) Early ambulation is encouraged.
e) Breathing exercises and leg exercises are to be stafted only afterthe puerperium.
691- Criteria to allow vaginal birth after cesarean section (C.S.) include the following:
a) The previous C.S was because of contracted pelvis.
b) The previous C.S was a c,lassic one (upper segment).
c) Oversized fetus in the current pregnancy.
d) Breech presentation in the current pregnancy.
e) The post partum period following the cesarean section was uneventful.
692- Indications of cesarean hysterectomy DO NOT include:
a) Uterine atony not responding to conservative treatlnent.
b) Placenta accrete.
c) Uterine rupture that cannot be repaired.
d) Multiple fibroid uterus.
e) Bicornuate uterus.
693- The following are true about classical cesarean section EXCEPT:
a) It is associated with increased blood loss.
b) It is done through a vertical incision in the lower uterine segment.
c)It is indicated in previous successful repair ofvesico-vaginal fistula.
d) It is better for rapid errtry to the uterus.
e) It is associated lvith increased risk ofsubsequent uterine rupture.
694- All the following are maternal indications for cesarean section EXCEPT:
a) Severe rnaternal hypertension.
b) lnvasive cervical carcinoma.
c) N4aternal cerebral aneurysrx.
d) Maternal heaft failure.
e) Previous myolnectomy operation.
WhiteKnightLove

-80-
'It is more blessed to give than to receive.

The following is the MOST COMMON fetal complication of elective cesarean section:
a) Fracture ofthe skull.
b) Fracture ofthe fernur.
c) Intracranial haemorrhage.
d) Iatrogenic RDS.
e) Neonatal sepsis.
Emergency cesarean hysterectomy is mostly needed in the following condition:
a) Grand multiparous patient.
b) crN.
c) Placenta circumvallate.
d) Placenta increta.
e) Red infarction ofthe placenta.

WhiteKnightLove

-81-
Freely you have received; freely give.

OBSTETRICAL MCQs ANSWERS

NORMAL PREGNANCY AND NORMAL LABOR


I E 30 C 59 C 88 D tt7 A 146 C 174 B
2 A 31 D 60 B 89 A ll8 D t47 A 175 c
3 C 32 B 61 D 90 E ll9 E 148 C 176 D
4 D 33 D 62 D 9t D 120 C 149 E 177 E
5 B 34 D 63 D 92 C t2l D 150 E 178 A
6 B 35 B 64 D 93 A 122 C 15r A 179 C
7 B 36 B 65 B 94 E 123 E 152 C 180 D
8 B 37 C 66 C 95 E 124 B 153 A l8l B
9 A 38 D 67 D 96 D t2s A 154 C 182 C
0 D 39 A 68 B 97 E 126 C 155 C 183 A
I A 40 C 69 C 98 E 127 D 156 D 184 C
7 A 4t C 70 A 99 D 128 C 157 A 185 A
3 E 42 B 71 E 100 B 129 B 1s8 E 186 D
4 D 43 D 72 E 101 B 130 C 159 C 187 C
5 B 44 D 73 E 102 D l3r C 160 D 188 A
6 D 45 A 74 B 103 C t32 B t6l D 189 E
7 D 46 D 75 B t04 B 133 D 162 D 190 B
8 D 47 C 76 C 105 B 134 D 163 C 191 G
9 B 48 A 77 D 106 B 135 B 164 B 192 F
20 D 49 C 78 C 107 B 136 C r6s B 193 D
2l D 50 C 79 B r08 A 137 B 166 D 194 C
22 E 5I D 80 D 109 B 138 C 167 A r95 B
23 A <, E 8l E ll0 D t39 B 168 A 196 C
24 E 53 B 82 E 111 B 140 D t69 B t97 D
25 E 54 B 83 C tt2 D t4t C 170 D 198 B
26 C 55 C 84 D 113 C 142 C 771 B 199 H
27 B 56 D 85 A Lt4 B 143 D 172 E 200 B
28 C 57 C 86 A Ir5 D t44 B 173 E 201 E
29 D 58 B 87 E 116 B 145 A

WhiteKnightLove

-82-
'It is more blessed to give than to receive.

OBSTETRICAL AND NON.OBSTETRICAL COMPLICATIONS


DURING PREGNANCY
202 E 238 B 274 C 310 B 346 C 382 B 417 D
203 D 239 A 275 A 311 B 347 C 383 C 418 D
204 E 240 A 276 C 312 D 348 C 384 A 419 E
205 A 241 A 277 C 3r3 E 349 D 385 A 420 C
206 B 242 A 278 B 3t4 A 3s0 A 386 B 421 D
207 D 243 A 279 B 315 E 351 A 387 E 422 A
208 E 244 B 280 A 316 B 352 D 388 C 423 C
209 E 245 C 281 C 317 A 353 B 389 C 424 E
2r0 C 246 D 282 C 318 D 354 B 390 C 425 A
2tt E 247 D 283 D 319 D 3s5 D 391 B 426 C
212 D 248 C 284 B 320 B 3s6 C 392 B 427 C
213 C 249 C 285 C 321 B 357 B 393 B 428 C
214 C 2s0 B 286 D 322 D 358 C 394 B 429 D
215 B 251 D 287 E 323 D 3s9 D 39s D 430 C
216 E 252 E 288 A 324 A 360 B 396 B 431 A
217 C 2s3 C 289 E 325 C 361 D 397 "A 432 B
218 B 254 E 290 A 326 A 362 B 398 E 433 B
219 D 2s5 C 291 C 327 A 363 A 399 A 434 B
220 C 256 C 292 A 328 A 364 A 400 c 43s D
221 E 257 D 293 D 329 C 365 B 401 E 436 E
11) D 258 A 294 B 330 B 366 C 402 D 437 D
223 A 259 D 29s B 331 B 367 c 403 c 438 D
224 A 260 C 296 D 332 D 368 E 404 A 439 A
225 B 261 A 297 A 333 B 369 D 405 E 440 C
226 B 262 A 298 B 334 B 370 D 406 D 441 D
227 B 263 E 299 c 335 D 371 D 407 D 442 B
228 D 264 A 300 D 336 D 372 E 408 D 443 A
229 E 265 B 301 B 337 C 373 C 409 E 444 C
230 E 266 A 302 A 338 A 374 E 410 B 445 F
231 A 267 B 303 C 339 C 375 C 4tl A 446 E
232 A 268 E 304 D 340 E 376 D 412 A 447 F
233 C 269 B 305 E 341 B 377 D 4t3 D 448 D
234 D 270 A 306 c 342 A 378 B 414 D 449 C
23s E 27t C 307 D 343 B 379 A 415 B 450 C
236 C 11'' D 308 C 344 A 380 B 416 D 451 E
237 D 273 C 309 C 345 B 38r D

WhiteKnightLove

-83 -
Freely you have received; freely give.

ABNORMAL LABOR
452 D 472 A 491 B 510 C 529 D s48 C 567 E
453 A 473 D 492 A 5ll E 530 C 549 C s68 C
454 C 474 B 493 B 512 D 531 E 550 E 569 D
455 B 475 D 494 E s13 A 532 A 551 A 570 B
456 D 476 D 495 A 514 B 533 B 552 B 571 B
457 C 477 D 496 D 515 A s34 D 553 C 572 C
458 C 478 B 497 C 516 E 535 B 554 D 573 E
459 C 479 B 498 D 577 C 536 E 555 E 574 D
460 E 480 A 499 B 518 B 537 B 556 B 575 C
46r B 481 E 500 D 519 B 538 B 557 B 576 B
462 E 482 D 501 C 520 B 539 D 5s8 E 577 C
463 B 483 D 502 B 521 D 540 C 559 E 578 A
464 B 484 C 503 D 522 B 54r E 560 E 579 C
465 B 485 B 504 C s23 A 542 D 561 E 580 B
466 A 486 A 50s C s24 C 543 B 562 E 581 D
467 C 487 A s06 D 525 B 544 E s63 c 582 A
468 A 488 C 507 D 526 D 545 B 564 B s83 C
469 C 489 B 508 B 5?7 B s46 E 505 E 584 B
470 B 490 E 509 A 528 D 547 C 566 C 585 D
41t E

NORMAL AND ABNORMAL PUERPERIUM


s86 D 589 D 592 A 594 C s96 C 598 D 600 C
587 C s90 D 593 B 59s E 597 E s99 E 60r E
588 C 59t A

TI{E FETUS AND THE NEWBORN


602 B 609 C 616 E 622 D 628 D 634 A 640 B
603 E 6r0 D 617 E 623 B 629 C 635 B 641 E
604 C 611 D 618 C 624 D 630 B 636 D 642 C
60s C 612 B 619 A 625 A 63t B 637 C 643 F
606 C 613 B 620 D 626 B 632 D 638 A 644 B
607 E 614 D 621 B 627 C 633 A 639 B 645 A
608 E 615 B

OPERATIVE OBSTETRICS
646 E 654 D 662 D 669 E 676 B 683 E 690 E
641 B 655 C 663 D 670 C 677 D 684 D 69t E
648 C 6s6 D 664 B 671 C 678 E 68s D 692 E
649 C 657 B 665 C 672 E 679 E 686 C 693 B
650 B 6s8 E 666 E 673 B 680 C 687 A 694 D
6s1 A 6s9 C 667 B 674 A 681 C 688 A 695 D
652 C 660 D 668 D 675 D 682 C 689 E 696 D
653 D 66t B
WhiteKnightLove

-84-
'It is more blessed to give than to receive.

GYNECOLOGY

APPLIED BASIC SCIENCES

Direction: Select the ONE BEST lettered ans\ryer or completion in each question.

697- The vestibule of the vulva is:


a) An area.
b) A fossa.
c) A cavity.
d) A tissue.
e) An opening.
698- Which DOES NOT normally open at thc vestibule?
a) The vagina.
b) The external urethral meatus.
c) The anal canal.
d) The right Bartholin's duct.
e) The left Bartholin's duct.
699- The normal distance between the urethral orifice and the clitoris:
a) No distance as they are very close to each other.
b) 0.5 cm.
c) 2-3 crn,
d) 5 cm.
e) No fixed relationship,
700- The following IS NOT a component of the normal vaginal fluid:
a) Cervical Mucus.
b) Desquamated epithelial cells.
c) Vaginal wall transudate.
d) Vaginal wall secretions.
e) Doderlein's bacilli.
701- The shape of the hymenal opening in normal individuals include the following EXCEPT:
a) Annular.
b) Crescentric.
c) Septate.
d) Cribriform,
e) Segmoid.
102- The vulval blood supply is via the:
a) Internal pudendal artery.
b) Vaginal artery,
c) Azygos artery,
d) Superior vesical artery.
WhiteKnightLove

e) Middle rectal aftery.

-85-
Freely you have received; freely give.

703- Nerve supply of the vulva include the following EXCEPT:


a) The pudendal nerve supplies sensory and motor fibers.
b) Labial branches of the ilioinguinal nerve.
c) Genital branch of genito-femoral nerve.
d) Perineal branch ofthe lateral cutaneous nerve ofthe thigh.
e) The pre-sacral nerve fibers.
704- The nerve supply of the vulva and perineum is derived from the following nerve:
a) Pudendal nerve.
b) Femoral branch of the genito-femoral nerve.
c) Anterior cutaneous nerve of the thigh.
d) Obturator nerve.
705- Which nerue pQ[S supply the vulva:
a) Pudendal nerve.
b) Genital branch of the genito-femoral nerve.
c) Lateral cutaneous nerve ofthe thigh.
d) Obturator nerve.
706- Lymphatic drainage of the vulva ppf include:
a) The femoral lymph nodes.
b) The inguinal lyrnph nodes.
c) The external iliac lymph nodes.
d) The pre-sacral lymph nodes.
e) the gland of Cloquet.
707- A 70-year-old woman presents forevaluation of a pruritic lesion on thevulva. Examination shows
a rvhite, friable lesion on the right labia majora that is 3 cm in diameter. No othersuspicious areas
are noted, Biopsy of the Iesion confirms squamous cell carcinoma. In this patient, lymphatic
drainagc characteristically would be FIRST to the:
a) External iliac lyrnph nodes.
b) Superficial inguinal lymph nodes.
c) Deep femoral lymph nodes.
d) Periaonic nodes.
708- As regards the vagina, all the following are correct EXCEPT:
a) The lymphatics of upper two thirds of vagina drain with the cervix.
b) It relates posteriorly to the recturn in its middle third.
c) The anterior wall is longer than the posterior.
d) It relates anteriorly to the urethra in its lowest third.
e) It is supplied in part by the uterine artery.
709- In the vagina
a) The lining is a transitional epithelium.
b) The karyopyknotic index is highest just before menstruation.
c) The pH rises during menstruation.
d) The pH may be norrnally 7.0.
e) Vaginal fluid is secreted by glands subepithelially.
710- Tall columnar cells cover the epithelial lining of the following:
a) The cervical canal.
b) The vaginal wall.
c) The labia rninora.
d) The surface epitheliurn of the ovaries.
e) The surface epitheliurn of the vestibule.
WhiteKnightLove

-86-
'It is more blessed to give than to receive.

7l I- The IXCQRRECT statcment regarding thc utcrus,:


a) The ratio of the Iength of uterine body to the length of the cervix is 2:l in adult.
b) Anteflexion is maintained mairrly by the tone of the uterosacral ligament.
c) The round liganrent is attached to the anterior surface just below the cornu.
d) The uterine artery passes above the ureter to reach the uterus.
e) The uterus receives blood supply from the ovarian artery.
712- The lorver uterine segment during normal pregnancy develops from:
a) The isthrnus of the uterus.
b) The fundus ofthe uterus.
c) The cervical canal,
d) The squamo-columnar junction.
e) The porliovaginalis.
713- The INCORRECT statement about the lower part of the utcrus:
a) The squarno-colurnnar junction in norrnal adults is not always at the external os,
b) The uterine aftery passes inferior to the ureter
c) A parous external os is slit-shaped.
d) Peritoneurn passes to cover the upper part ofthe posterior wall ofthe vagina.
e) The isthrnus is a part between the cervix and body of the uterus.
714- The INCORRECT statement concerning the cervix:
a) It undergoes cyclic changes during the menstrual cycle.
b) The epithelial lining of the cervical canal is tall columnar partially ciliated.
c) The ratio of the length of the cervix to the length of the uterine body in adults is l:2.
d) lt produces a thick scanty discharge at ovulation.
e) The surface epithelium of the portiovaginalis is similar to the vagina.
715- Factors maintaining thc uterus in its normal position include the following EXCEPT:
a) Ligaments of the uterus.
b) The anteverled anteflexed position ofthe uterus.
c) The pelvic floor muscles.
d) The peritoneal reflections.
e) The ovarian ligarnent.
716- The main supports of the uterus DO NOT include:
a) Cardinal (Mackenrodt's) ligarnents.
b) Uterosacral ligaments.
c) Levator ani muscle.
d) Pubocervical ligaments.
e) Round ligaments.
717- The normal supports of uterus DO NOT include:
a) The ovarian ligarnents.
b) The transverse cervical (cardinal) ligaments.
c) The uterosacral ligaments.
d) The pubo-cervical ligaments.
e) The Ievator ani rnuscles.
718- All the follorvings about the round ligamcnt of the uterus are correct EXCEPT:
a) It raises a ridge on the anterior (inferior) layer of the broad Iigament,
b) It passes into the inguinal canal.
c) lt contains the cremaster rnuscles.
d) lt is derived from the gubernaculum.
e) It is continuous with the ovarian ligarnent at the cornu of the uterus.
719- All thc follorving statements are correct about the pelvic ligaments ffpff;
WhiteKnightLove

a) Uterosacral ligament extends frorn the posterolateral part of the supravaginal portion of the cervix
and posterior vaginal fornix to the front of the sacrum.
b) The ovarian ligarnent is one of true ligaments of the uterus.
c) The round ligarnent contains lymphatic vessels.

-87-
Freely you have received; freely give.

d) The ureter penetrates the cardinal ligament.


e) The cardinal ligaments extend from the lateral aspect of the cervix and upper vagina to the white line
of the lateral pelvic wall.
720- All the following statements are correct about the pelvic ligaments and fascia ffifl:
a) The cardinal Iigarnents arise from the inferior pubic rami.
b) Peritoneuln covers the superior portion of the post vaginal wall.
c) The suspensory ligament of the ovary is superiorto the round ligarnent.
d) The round ligament contains lyrnphatic vessels.
e) The ureter penetrates the cardinal ligarnent.
721- The follorving statement is CORRECT about the ovarian ligaments:
a) They contain ureters.
b) They contain ovarian arteries,
c) They are attached laterally to pelvic wall.
d) They Iie anterior to the broad ligament.
e) They are homologous to paft of the gubernaculurn testis in the male.
722- The body of the uterus drains to the following nodes EXCEPT:
a) Sacral group of lymph nodes.
b) Superficial inguinal group of lyrnph nodes.
c) Obturator group of lyrnph nodes.
d) Fernoral group of lymph nodes.
e) External iliac group of lyrnph nodes.
723- The body of the uterus DOES NOT drain to the following group of lymph nodes:
a) The sacral group of lymph nodes.
b) The para-aoftic group of lyrnph nodes,
c) The obturator group of lyrnph nodes,
d) The femoral group of lyrnph nodes.
e) The internal iliac group of lyrnph nodes.
724- The lymphatic drainage of the cervix DO NOT reach the following group of lymph nodes:
a) The gland of Cloquet.
b) The internal iliac lymph nodes.
c) The para-cervical lymph nodes.
d) The pre-sacral lymph nodes.
e) The Obturator lymph nodes.
725- The lymphatic drainage from the uterine cervix is through the following lymph nodes ffpff:
a) External iliac group of lymph nodes.
b) Obturator group of lymph nodes.
c) Pre-sacral group of lymph nodes.
d) Internal iliac group of lymph nodes.
e) Superficial inguinal group of lymph nodes.
726- A,50-year-old !1,oman is diagnosed with cervical cancer. Which lymph node group would be the
first involved in metastatic spread of this disease beyond the cervix and uterus?
a) Common iliac nodes.
b) Parametrial nodes.
c) External iliac nodes.
d) Paracervical or ureteral nodes.
e) Para-aortic nodes.
e) lnternal iliac rrodes.
727- The parasympathetic nerves DO NOT inncrvate the following structure:
a) The cervix.
WhiteKnightLove

b) The lower uterine segment.


c) The uterine fundus.
d) The uterosacral ligaments,
e) The cardinal ligaments.

-88,
'It is more blessed to give than to receive.

728- The INCORRECT statement as regards the Fallopian tube:


a) It possesses a ciliated lining.
b) It is actively rnotile.
c) lt consists of four portions.
d) It is covered totally by peritoneum.
e) The ampulla is the widest part with thin muscle wall.
729- Each fallopian tube consists of the following parts EXCEpT:
a) The isthmus.
b) Interstitial part.
c) The infundibulum.
d) The ampulla.
e) The cornu.
730- In the development of the ovaries, all the following are correct EXCEpT:
a) The structures in the ovaries have three different origins.
b) lit involves migration of the gerrn cells.
c) It involves migration of the ovaries.
d) The homologous structure in the rnale is the testis,
e) Differentiation into ovary needs XX even in presence of Y chromosome.
731- The CORRECT statement as regards the ovary:
a) It lies posterior to the broad Iigament.
b) It is covered by peritoneurn.
c) It receives blood supply from a branch from the internal iliac artery.
d) Venous blood is drained directly into inferior vena cava.
e) It has lymphatic drainage to the internal iliac lymph nodes.
732- The INCORRECT statement concerning the ovary:
a) It is derived from the paramesonephros.
b) It is attached to the uterine cornu by the ovarian ligarnent.
c) lt Iies posterior to the board ligarnent.
d) Its lymphatic drainage is to the para-aorlic lymph nodes.
e) It is anached to the posterior surface of the broad ligament by the mesovarium.
733- The INCORRECT statement regarding the right ovarian artery:
a) It is a branch of the abdominal aorta just below the renal artery.
b) It runs in the infundibulo-pelvic fold.
c) At the pelvic brim, the artery crosses the external iliac vessels.
d) It supplies branches to the upper paft ofthe vagina.
e) It anastonroses with the right uterine artery.
734- The following vessels are branches of the internal iliac artery EXCEPT:
a) Uterine artery.
b) Obliterated urnbilical artery.
c) Pudendal aftery.
d) Superior rectal aftery.
e) Vaginal artery.
735- Which IS NOT derived from the internal iliac artery:
a) The uterine artery.
b) The vaginal artery.
c) The middle vesical artery.
d) The rniddle hemorroidal artery.
e) The ovarian artery.
WhiteKnightLove

-89-
Freely you have received; freely give.

736- Which lS NOT a branch of the anterior division of the internat iliac artery:
a) Obturator artery.
b) Internal pudendal aftery.
c) Superior vesical aftery.
d) Inferior gluteal.
e) Ilio-lumbar artery.
737- ln the female pelvis, the peritoneum DOES NOT cover:
a) The upper portion of the anterior wall of the uterus.
b) The whole of the posterior wall of the uterus.
c) The fallopian tube partially.
d) The surface of the ovary.
e) The upper half of the posterior wall of the vagina,
738- The ureteric canal:
a) It lies in the uterosacral ligaments.
b) It lies in the base of the pubocervical ligaments,
c) lt penetrates the base ofthe bladder.
d) It penetrates the Mackenrodt's Iigaments.
e) lt is in close approximity to the round Iigaments.
739- The INCORRECT statement regarding the ureter in the adult female is that it:
a) Crosses under the uterine artery.
b) ls rnesodermal in origin.
c) Crosses under the genitor-femoral nerve.
d) Crosses under the ovarian vessels.
e) Crosses over the bifurcation of cornmon iliac artery.
740- The urogenital diaphragm is pierced by thc:
a) Recturn.
b) Vaginal fornices.
c) Ureters,
d) Urethra.
e) Obturator nerve.
741- Structures arising from Wolffian remnants DO NOT include:
a) The epoophoron.
b) Gaftner duct.
c) The paraoophoron.
d) The ureters.
e) The round ligament.
742- Which of the following are derived from gubernaculum in female:
a) Ovarian ligament and round ligament.
b) Mesovarium and ovarian ligament.
c) Ovarian ligament and uterosacral ligarnent.
d) Round ligament and broad ligament.
e) Mackenrodt's ligament and ovarian ligament.
743- All the following are homologous organs in the male and female, respectively EXCEPT:
a) Prostate and Skene's duct.
b) Scrotum and labium minor.
c) Cowper's gland and Bartholin's gland.
d) Corpus spongiosum and vestibular bulb.
e) Gubernaculums of testis and round ligament of ovary.
WhiteKnightLove

-90-
'It is more blessed to give than to receive.

744- All the following are vestigial remnants EXCEPT:


a) Kobelt tubules.
b) Epoophoron,
--'c) Tubal fimbria.
d) Paraoophrorn.
e) Gartner duct.
745- The urogenital sinus gives the followings EXCEPT:
a) Urinary bladder.
b) Skene ducts.
c) Lower part ofthe vagina.
d) The ureteric bud.
e) Bartholin glands.
746- The permanent kidney originates from:
a) Pronephros.
b) Mesonephros.
c) Metanephros.
d) Pro and metanephros.
e) Urogenital sinus.
747- The INCORRECT statement as regards the imperforate hymen:
a) It may Iead to hematocolpos.
b) It is usually identified before puberty.
c) It may present by acute retention of urine.
d) lt rnay be presented with pelviabdominal mass.
e) Generally requires surgery.
748- The patient with imperforate hymen might present with the following EXCEPT:
a) Primary amenorrhea.
b) Cyclic menstrual rnolimina.
c) Acute retention of urine.
d) Abdominal mass,
e) Hypomenorrhea.
749- The clinical findings of imperforate hymen include the following EXCEPT:
a) Hypomenorrhe.
b) Cryptomenorrhea.
c) Hematometra.
d) Pelvi-abdominal swelling.
e) Retention of urine.
750- Imperforate hymen DOES NOT result in:
a) Hernatocolps,
b) Hernatosalpinx.
c) Hematometra.
d) Pelvic endometriosis.
e) Hypoplastic uterus.
751- The INCORRECT statement regarding the pelvi-abdominal swelling in cases of imperforate
hymen:
a) lt is a distended vagina full of menstrual fluid.
b) It is a distended uterine cavity full of menstrual fluid.
c) lt is soft and cystic.
d) Clinically it may resemble an ovarian cyst.
e) Could result in acute retention of urine.
WhiteKnightLove

-91-
Freely you have received; freely give.

152- Congenital absence of the vagina is most commonly associated with:


a) Absent secondary sexual characteristics,
b) Absent uterus.
c) Exposure to diethyl stilbosterol in utero.
d) Turner syndrome,
e) Lnperforate anus.
753- Bicornuate uterus might predispose to the following EXCEPT:
a) Recurrent preterm labor.
b) Primary amenorrhea.
c) Recurrent oblique lie.
d) Retention of the placenta after delivery,
e) Menorrhagia.
754- Pregnancy complications that can occur due to bicornuate uterus include the followings
EXCEPT:
a) Operative delivery.
b) Oblique lie of the fetus.
c) Premature labor.
d) Twin pregnancy,
e) Habitual abortion.
755- A 30-year-old G0P0, asking for a second opinion after being diagnosed to have a bicornuate
uterus. She and her husband are planning to start a family and want to know how this diagnosis
would affect the future reproductive life. Which is the MOST LIKELY complication?
a) Infertility requiring surgical intervention.
b) Recurrent miscarriage.
c) Preterm rupture of membranes.
d) Breech presentation when becorne pregnant.
e) Bicornuate uterus should have no effect on the reproductive life.
756- While evaluating a 3O-year-old woman for infertility, you diagnose a bicornuate uterus. You
explain that additional testing is necessary because of the woman's increased risk of congenital
anomalies in which organ system?
a) Skeletal.
b) Hernatopoietic.
c) Central nervous.
d) Urinary,
e) Tracheoesophageal.
757- Possible clinical consequences of congenital malformations of the female genital system include all
of the followings EXCEPT:
a) Habitual abortion.
b) Congenital fetal malformations.
c) Ectopic pregnancy.
d) Oblique lie of the fetus.
e) Expulsion of intrauterine devices.
758- The presence of a uterus in a phenotypically normal male is due to:
a) Lack of rntillerian inhibiting factor.
b) Lack of testosterone.
c) Increased levels ofestrogens.
d) 46, XX karyotype.
e) Presence of ovarian tissue early in embryonic developrnent.
WhiteKnightLove

-92-
'It is more blessed to give than to receive.

759- Anomalies of the following system are commonly associated with miillerian deformities:
a) Cardiovascular system.
b) Urinary system.
c) Gastrointestinal systern.
d) Musculoskeletal system.
e) Respiratory system.
160- A l7-year-old girl undergoes a laparotomy forappendicitis. On laparotomy, there is noted to be a
torted gangrenous ovarian cyst, for which she undergoes a unilateral oophorectomy. Further
exploration reveals a complete absence of the uterus and both fallopian tubes. The renal tract
appears normal. Vaginal examination reveals a blind pouch of 3 cm length. FSH, LH and serum
estradiol levels are normal. She is about to get married. What is the NEXT STEP in her
management?
a) Karyotyping.
b) Psychosexual counseling.
c) Vaginoplasty.
d) Utriculoplasty.
e) Removal ofthe other gonad.
761- The COMMONEST cause of congenital obstructive anomaly of the lower genital tract:
a) Irnperforate hymen.
b) Low transverse vaginal septum.
c) Vaginal aplasia.
d) Complete vaginal hypoplasia.
e) Partial vaginal hypoplasia.
762- The follorving investigations are !Q rccommended in case of uterus didelphys:
a) Hysterosalpingography.
b) Pelvic ultrasound examination.
c) [ntravenous pylography.
d) Colposcopy.
e) Hysteroscopy.
763- The INCORRECT statement regarding cystic vestigial remnants:
a) Cornmonly they are small and not significant.
b) They are filled with translucent fluid.
c) They rnay be large enough to fill the broad ligament.
d) Differential diagnosis includes ectopic pregnancy.
e) Differential diagnosis includes benign ovarian cyst.
764- Possible consequences of congenital malformations of the female genital system DOES NOT
involve:
a) Prirnary arnenorrhea.
b) Secondary arnenorrhea.
c) Menorrhagia.
d) Hypornenorrhea.
e) Dysmenorrhea.
765- Body anomalies possibly associated with congenital anomalies of the genital system DO NOT
include:
a) Pelvic kidney.
b) Absent one kidney.
c) Double ureter.
d) Vestibular anus.
e) Diverticulosis.
WhiteKnightLove

-93-
Freely you have received; freely give.

766- Complications of congenitalanomalies of thc genitalsystem during pregnancy DO NOT include:


a) Abortion.
b) Premature labor.
c) Preeclampsia.
d) Malpresentations.
e) Placental insufficiency.
767- Complications of congenitalanomalies of the genitalsystem during laborppf include:
a) Failure of dilatation of the cervix.
b) Soft tissue obstruction.
c) Postpartum hemorrhage.
d) Contracted pelvis.
e) Retained placenta.

Direction: Each set of matching questions in this section consists of a Iist of numbered items followed
by several lettered options. For each numbered item, selcct the ONE best lettered option that is most
closely associated with it. Each lettered heading rnay be selected once, more than once, or not at all.

Match each of the following vessaels numbered (768-772), with the corresponding supply or drainage
lettered (a-h).
EACH LETTERED HEADING MAY BE USED ONCE, MORE THAN ONCE, OR NOT AT ALL.

768- Uterine vein I a) Anterior division of internal iliac artery.


769- Right ovarian vein I b) Posterior division of internal iliac artery.
770- Left ovarian vein I c) External artery.
771- Uterineartery I d) Common iliac aftery.
772- Ovarian artery | e) Aofta.
l) lnferior vena cava.
g) Internal iliac vein,
h) Renal vein.

For each organ numbered (773-777), choose the best dimension listed below lettered (a-e).
EACH LETTERED HEADING MAY BE USED ONCE, MORE THAN ONCE, OR NOT AT ALL.

773- Length ofthe urethra a) 3 cm.


774- Length of the clitoris b) 4 cm.
775- Length of the posterior vaginal wall c) 3 mm.
776- Length of the nulliparous uterus d) 7cm
777- Diameter of the nulliparous internal cervical os e) 10.5 cm

For each of the following organ numbered (778-782), choose the epithelium that is lining it lettered (a-
d).
EACH LETTERED HEADING MAY BE USED ONCE, MORE THAN ONCE, OR NOT AT ALL.

778- Dome of the urinary bladder a) Stratified squamous.


779- External urethral meatus b) Pseudo-stratified.
780- Endocervix c) Transitional.
781- Adult vagina d) Columnar.
782- Endometrium
WhiteKnightLove

-94-
'It is more blessed to give than to receive.

For each of the following structures in female numbered (783-787), choose the homologous
corresponding structures in male lettered (a-e).
EACH LETTERED HEADING MAY BE USED ONCE, MORE THAN ONCE, OR NOT AT ALL.

783- Ovary a) Testis.


784- Clitoris b) Ventral aspect ofthe penis.
785- Ovarian ligament c) Prinis.
786- Labia majora d) Scrotum.
787- Labia minora e) Gubernaculum testis.
f) Cremastric muscle.
Glans penis.

WhiteKnightLove

-95-
Freely you have received; freely give.

PATIENT EVALUATION

788- The INCORRECT statement regarding Schiller iodine test:


a) Normally the epithelium of the ectocervix stain brown.
b) Nonnally the epitheliurn of the vaginal wall stain brown.
c) Areas of colurnnar metaplasia do not stain brown.
d) Areas that do not stain brown are definitely malignant tissues.
e) lt is complementary but not competitor to colposcopy.
789- The tissue that stain brown with Schiller iodine test is:
a) Immature epitheliurn.
b) Scar tissue.
c) Metaplastic tissue.
d) Normal tissue.
e) Neoplastic tissue.
790- Colposcpic examination IS NOT useful in the following conditions:
a) Suspected malignancy of the cervix.
b) To document rnalignancy of the endometrium.
c) Follow up ofcases treated for pre-malignant cervical lesions.
d) Follow up after treatment of malignant lesions of the cervix.
e) Directed biopsy of cervical lesions.
791- The INCORRECT statement regarding exfoliative cytology to diagnose cervical neoplasia:
a) The aim is to sample the surface cells of the cervical transfonnation zone.
b) The sarnpling device must cover 360" of the cervix.
c) Fixation must take place immediately.
d) Aspiration of cells frorn the posterior fornix pool is less useful than using an Ayre spatula to scrape
the cervical surface.
e) will be detected in70% of cases.
The presence of endometrial cancer
792- A 55-year-old postmenopausal woman complains of vaginal burning and dyspareunia. She has a
thin, waterT discharge. She has not been on estrogen replacement therapy. The use of a lubricant
has not helped. Which of the following characteristics of the vagina IS NOT included atthat stage
of life:
a) A thinned out epithelium.
b) Superficial cells at vaginal smear.
c) Decreased glycogen at the vaginal cells.
d) An alkaline pH of the discharge.
e) Loss ofthe vaginal rugae (ridges).
793- Transvaginal approach of the ultrasound is less valuable then abdominal ultrasound in the
following condition:
a) Evaluation of the endometrial thickness.
b) Evaluation ofthe ovarian tissues.
c) Evaluation ofthe cul-de-sac.
d) Evaluation of multiple fibroid uterus.
e) Assessment of early pregnancy.
794- Hysterosalpingography can visualize the following EXCEPT:
a) The uterine cavity.
b) The lumen of the fallopian tube.
WhiteKnightLove

c) The spillof the dye into the pelvic peritoneal cavity.


d) The Graafian follicles of the ovaries.
e) The cornu ofthe uterus.

-96-
'It is more blessed to give than to receive.

795- Contraindications of hysterosalpingography DO NOT include:


a) Suspected blockage ofthe fallopian tubes.
b) Suspected acute pelvic inflammatory disease.
c) Suspected intrauterine pregnancy.
d) Premenstrual phase.
e) Active bleeding from the uterus.
796- lndications of hysterosalpingography DO NOT include:
a) Primary infertility.
b) Secondary infertility.
c) Detection of endometrial polyp.
d) Detection of septate uterus.
e) Detection of benign ovarian turnors.
797- Complications of hysterosalpingography DO NOT include:
a) Iatrogenic endornetriosis.
b) Neurogenic shock.
c) Ascending infection.
d) Endometrial hyperplasia.
e) Flaring of silent subclinical infection.
798- The INCORRECT statement regarding vaginal cytology:
a) It helps to study the hormonal pattern.
b) It can detect subclinical infection.
c) lt may detect malignant endometrialcells.
d) The normal cyclical pattern consists of four phases.
e) The estogenic phase shows mature superficial cells.
799- Ultrasound !f[Qf useful in the following condition:
a) Suspected ectopic pregnancy.
b) Suspected missed abortion.
c) Pelvic swelling.
d) Diagnosis of rectocele,
e) Diagnosis of polycystic ovarian disease.
800- The INCORRECT statement regarding endometrial biopsy:
a) It could be collected without general anesthesia.
b) It could be collected without dilatation of the cervical canal.
c) It is useful in infertility work-up.
d) It is the best technique for the diagnosis of endometriosis.
e) It is useful in the diagnosis ofdysfunctional uterine bleeding.

WhiteKnightLove

-97-
1
Freely you have received; freely give.

BENIGN CONDITIONS OF REPRODUCTIVE ORGANS


AND MENSTRUAL DISORDERS

Direction: Select the ONE BEST lettered answer or completion in each question.

801- The CORRECT statement for adhesions between the two labia minora (labial adhesions):
a) Acquired adhesions are more common than congenital adhesions.
b) Congenital adhesions present clinically rnainly during neonatal period.
c) Local estrogen cream is a predisposing factor.
d) Clinical presentation is uncommon after menopause.
e) Congenital labial adhesions are never associated with other genital malformations,
802- The TNCORRECT statement regarding labialadhesions:
a) Acquired cases are lrore common than congenital cases.
b) Acquired Iabial adhesions are more cornmon during reproductive age than during menopause,
c) Congenital cases could be due to intrauterine virilizing agent.
d) Differential diagnosis includes congenital adrenogenital syndrome.
e) Estrogen cream could be used in the management of acquired cases.
803- The COMMONEST cystic swelling of the vulva:
a) Endometrioma.
b) Inclusion derrnoid cyst.
c) Baftholin cyst.
d) Hydrocele ofthe canal ofnuck,
e) Hidradenor-na.
804- All the following about Bartholin glands are correct EXCEPT:
a) They are mesonephric in origin,
b) They can commonly be infected by E coli.
c)Excision of the cyst when infected is usually contraindicated.
d) They are situated at the posterior parls of labia majora.
e) They ahould be marsupilized when acutely infected.
805- The CORRECT statement regarding the Bartholin cyst:
a) It is the comrnonest cyst in the vulva.
b) True cystic swelling is present in anterior part of labium major.
c) The rnain cause is obstruction of Bartholin gland duct by a benign tumor.
d) They should be excised ifthe patient is above 30 years.
e) The main symptom is throbbing pain.
806- Bartholin abscess present with variety of symptoms that DO NOT include:
a) Impairment of ability to walk.
b) Dyspareunia.
c) Local throbbing pain.
d) Purulent discharge.
e) Pruritus vulva.
807- Management of Bartholin abscess DOES NOT include:
a) Incision and drainage.
b) Marsupialization.
c) Excision ofchronic abscess
d) Saucerization operation.
WhiteKnightLove

e) Wood catheter.

-98-
'It is more blessed to give than to receive.

808- A Z4-year-old woman G0 married for 5 months presents with a non-tender cystic mass in her
right vulva that causes some discomfort when walking and during coitus. The mass was at the
posterior part of labium major and was about 2.0 X 2.0 cm dimensions. What is the MOST
APPROPRIATE initial decision?
a) Marsupialization.
b) Administration of antibiotics.
c) Surgical excision.
d) Incision and drainage.
e) Observation.
809- A 65-year-old patient presents with a non-tender solid mass in the right vulva that causes some
discomfort when walking and during coitus and that is consistent with a diagnosis of swelling of
the Bartholin gland. What is the MOST APPROPRIATE procedure?
a) Marsupialization.
b) Adrninistration of antibiotics.
c) Surgical excision.
d) Incision and drainage.
e) Observation.
810- The INCORRECT statement regarding pruritus vulvae:
a) It is rarely to be idiopathic.
b) It might be iatrogenic.
c) It might be associated with malignant disease,
d) It might be associated with sexual frustration,
e) It might be associated with pediculosis.
-
8t l In cases of pruritus vulvae:
a) Radical vulvectomy is indicated.
b) Hydrocorlisone is contraindicated.
c) Recurrent symptoms are uncommon.
d) Cornmon after menopause.
e) Antihistarninics are ineffective for treatment,
812- The COMMONEST white lesion of the vulva:
a) Squarnous cell hyperplasia.
b) Lichen sclerosis.
c) Carcinoma of the vulva.
d) Paget disease ofthe vulva.
e) Lichen planus.
813- Which character IS NOT applicable to vulval ulcer due to squamous cell carcinoma:
a) Raised everted edges.
b) Sloughing surface.
c) There rnay be fibrosis.
d) Indurated base.
e) May be infected.
814- The INCORRECT statement regarding epithelial disorders of the vulva:
a) They may invade the basement membrane of the epithelium.
b) The changes may affect only portion of the whole thickness of the epithelium.
c) There is abnormal growth and maturation of the epithelial cells.
d) Lichen sclerosis is one of the epithelial disorders of the vulva.
e) Lichen planus is one of the epithelial disorders of the vulva.
815- The INCORRECT statement regarding lichen sclerosis:
a) Spontaneous cure never occurs.
WhiteKnightLove

b) Farnilial tendency could be detected.


c) Chronic mechanical irritation is a known etiological factor.
d) Associated autoimmune disease might be detected.
e) Pruritus is the most common symptom.

-99-
Freely you have received; freely give.

816- The INCORRECT statement regarding lichen scterosis:


a) The naked eye appearance is not characteristic.
b) The epithelium is inactive and thin.
c) The lesion never extends to the perineum or the anal area.
d) There may be contracture of the introitus.
e) The lesion does not affect the vagina.
817- The INCORRECT statcmcnt rcgarding lichcn sclcrosis:
a) Biopsy is always needed.
b) There may be an obvious underlying etiology.
c) Local sodium bicarbonate I oh may be useful.
d) Extra heat might improve the symptoms.
e) Superpotent steroid therapy is a common effective therapy.
818- The INCORRECT statement regarding pruritus vulvae:
a) Pre malignant lesions are not of the causes of pruritus vulvae.
b) Malnutrition & vitamins deficiencies may lead to it.
c) Pruritus vulvae resistant to treatment should be subjected to biopsy.
d) Estrogen can be used for treatment.
e) Surgical treatment includes circum-vulval incision, or simple vulvectomy.
819- As regards cervical ectopy (erosion), the following is CORRECT:
a) It is an ulcer ofthe ectocervix.
b) It is treated by cone biopsy in pregnant females.
c) Cauterization should be done as hrst line oftherapy,
d) Pap smear is advisable before management.
e) Commonly causes pain, dyspareunia & low back pain.
820- The INCORRECT statement regarding cervical ectropion:
a) It can be differentiated from cervical lesion by presence ofcervical tear.
b) It may associate incompetent cervix.
c) It is treated by trachelorraphy ifcervical tear is present.
d) Endo-cervix appears due to ulceration.
e) Active treatment is not always necessary.
821- Cervical ectopy is defined as:
a) Replacement of the stratified squamous epithelium of the portiovaginalis by columnar epithelium.
b) Replacement of the of the columnar epithelium of the portiovaginalis by stratified squamous
epithelium.
c) Area of denuded epithelium.
d) Columnar epithelium covering over the stratified squamous epithelium.
e) Physiological ulcer.
822- Causes of cervical ectopy DO NOT include:
a) Pregnancy.
b) Combined oral contraceptive pill.
c) Estrogen replacement therapy.
d) Acute cervisitis.
e) Carcinoma of the cervix
823- Chronic noninfectious cervical ectopy might present clinically with:
a) Pelvic pain.
b) Mucoid vaginal discharge.
c) Dyspareunia.
d) Low back pain.
e) Spasmodic dysmenorrheal.
WhiteKnightLove

-100-
'It is more blessed to give than to receive.

824- Which IS NOT valuable in the management of cervical ectopy?


a) Cervico-vaginal smear.
b) Colposcopy.
c) Endometrial biopsy.
d) Directed biopsy.
e) Papanicolaou smear.
825- Types of cerryical polyp DO NOT include :
a) Mucous polyp.
b) Infectious polyp due to chlamydia infection.
c) Infectious polyp due to tuberculosis infection
d) Fibroid polyp.
e) Malignant polyp.
826- Cervcial ectropion might present with all of the following EXCEPT:
a) Mucoid vaginal discharge.
b) Infertility.
c) Recurrent abortion.
d) Menorrhagia,
e) Premature labor
827- Ulcer of the cervix might be due to all of the following EXCEPT:
a) Herpes simplex infection.
b) Malignancy.
c) Syphilis.
d) Chlamydia trachomatis.
e) Tuberculosis.
828- The MOST COMMON benign mass of the ceryix and endocerrix is:
a) Polyp.
b) Leiomyoma.
c) Nabothian cyst.
d) Cervical hood.
e) Gartner's duct cyst.
829- Regarding cervical polyps all the following are correct EXCEPT:
a) They may cause post coital bleeding.
b) The most common type is mucous polyp.
c) Usually, they arise from the cervical canal.
d) They are usually pre cancerous.
e) They should be subjected to histopathological examination.
830- A 37-year-old woman complains of postcoital bleeding. The LEAST LIKELY cause of her
bleeding would be cervical
a) Polyps.
b) Ectropion.
c) Carcinoma.
d) Nabothian cyst.
e) Infection.
831- All the follorving about uterine fibroids are corrcct EXCEPT that thcy:
a) Occur in over 30% of women of reproductive age.
b) Can undergo sarcomatous change in 5%o ofcases.
c) Are associated with rnenorrhagia.
d) Can cause polycythemia.
e) Can cause pressure manifestations in the pelvis.
WhiteKnightLove

-l 0t-
Freely you have received; freely give.

832- All the following about uterinc fibroid are correct EXCEpT:
a) It is estrogen dependent.
b) It gets larger during treatment with progestogens.
c) It is usually asymptomatic.
d) It shrinks in response to treatment with LHRH agonists.
e) It may be treated conservatively.
833- All the following about uterine fibroids are correct EXCEpT that:
a) They originate from smooth muscle.
b) They are usually submucous.
c) They contain muscle and connective tissues.
d) They are usually rnultiple.
e) They are uncorlrlon in the cervix.
834- AII the following statements about uterine fibroid are correct EXCEpT:
a) They are usually rrultiple.
b) They are rnore common in negroids than among white race.
c) They present lnost commonly before the age of twenty years.
d) They are estrogen dependent.
e) They shrink after the menopause.
835- The following are risk factors to develop uterine myomata EXCEpT:
a) Low parity.
b) Late menarche,
c) Negroid race.
d) Positive farnily history of the disease.
e) Associated endometriosis.
836- The MOST COMMON site of uterine myoma is:
a) Subserous.
b) Interstitial.
c) Submucous.
d) Cornual.
e) Cervical.
837- The COMMONEST symptom of intramural fibroids:
a) Intermenstrual bleeding.
b) Postcoital bleeding.
c) Postmenopausal bleeding.
d) Deep dyspareunia.
e) Menorrhagia.
838- The COMMONEST secondary change in uterine fibroids is:
a) Fatty degeneration.
b) Myxomatous degeneration.
c) Hyaline degeneration.
d) Cystic degeneration.
e) Calcification.
839- Sarcomatous change in uterine fibroids occurs in:
a) 0.05%,
b) 0.s%.
c) 1.5%.
d) s%.
e) t5%.
WhiteKnightLove

-102-
'It is more blessed to give than to receive.

Fibroids DO NOT lead to:


a) Menorrhagia.
b) Polycythernia.
c) Intestinal obstruction.
d) Acute urinary retention.
e) lnfertility.
841- Epidemiological studies of fibroid uterus indicate:
a) It is more common in multiparas than nullipara.
b) It is uncornmon after the age of 35 years.
c) It is more common in white race than in negroid race race.
d) It is the comrnonest tumor of the female genital system.
e) It is rrore comrron in the fertile than in the non-fertile women.
The fibroid is clinically NOT felt soft in the following condition:
a) Pregnancy.
b) Cystic changes.
c) Malignancy.
d) Infection.
e) Episodes ofvaginal bleeding.
843- Possible complications of fibroid uterus DO NOT include:
a) Secondary arnenorrhea.
b) lnfertility.
c) Postparturn hernorrhage.
d) Sarcomatous changes.
e) Pressure on the urethra.
844- Which type of degeneration of fibroid is accompanied with acute abdominal pain:
a) Hyaline degeneration.
b) Fatty degeneration.
c) Red degeneration.
d) Calcification.
e) Myxomatous changes.
845- Fibroid uterus may be associated rvith the following EXCEPT:
a) Precocious pubefty.
b) Menorrhagia.
c) Metrorrhagia.
d) Postmenopausal bleeding.
e) Frequency of micturition.
846- Causes of symmetrically enlargcd uterus DO NOT include:
a) Norrnal pregnancy.
b) Pyometra.
c) Adenomyosis.
d)Subinvolution ofthe uterus.
e)Solitary subserous fibroid uterus.
847- Operative treatment of fibroid uterus is essential EXCEPT in the following condition:
a) Rapidly growing fibroid.
b) Cervical fibroid 4x4 cm.
c) Pedunculated fibrois.
d) Subserous fibroid 4x4 cnr.
e) Submucous fibroid 4x4 cm.
WhiteKnightLove

-r03-
Freely you have received; freely give.

848- Operative treatment of fibroid uterus is essential in the following condition:


a) Red degeneration of fibroid.
b) After menopause,
c) Asymptornatic subserous fi broid.
d) Large cervical fibroid.
e) Small subserous fibroid.
849- Medical treatment of fibroid might include all the following EXCEPT:
a) Tamoxifen.
b) Danazol.
c) LH-RH analogues.
d) Recombinant FSH,
850- Indications of hysterectomy for fibroid uterus DO NOT include:
a) Single subserous fibroids.
b) Severe bleeding during myomectomy,
c) Recurrent fibroid uterus after myomectomy.
d) Complaining postmenopausal women.
e) Suspected sarcomatous changes.
851- Precautions to minimize bleeding during myomcctomy DO NOT include:
a) Preoperative LH-RH analogue.
b) Tirning of the operation in the premenstrual phase.
c) Single rather than rnultiple incisions.
d) Midline rather than Iateral incisions.
e) Preoperative antibiotics.
852- The etiology of endometriosis relates to the following EXCEPT:
a) A transformation of celornic epithelium.
b) Direct invasion through the uterine serosa into the pelvic cavity.
c) Retrograde rnenstrual flow.
d) Vascular transpoll of endometrial fi'agments.
e) Lymphatic transpoft of endometrial fragments.
853- Possible mechanisms for the development of cndometriosis DO NOT include:
a) Hematogenous spread.
b) Lyrnphatic spread.
c) latrogenic dissemenation.
d) Viral infection.
e) Genetic predisposition.
854- The following physical findings are consistent with a diagnosis of endometriosis EXCEPT:
a) Fixed retroversion ofthe uterus.
b) Tender pelvic masses.
c) Adnexal enlargement.
d) Cul-de-sac nodules.
e) Omental nodules.
855- During Laparoscopy, a women was found to have powder-burn lesions on the serosat surfaces of
the peritoneum and a large ovarian cyst that contains a thick, viscous darl<-brown fluid. These
findings characteristically occur in women who are:
a) In the lower socio-economic group.
b) Perirnenopausal.
c) Nulliparous.
d) Using IUCD.
e) Using Oral contraceptives.
WhiteKnightLove

-r04-
'It is more blessed to give than to receive.

856- A woman with symptomatic endometriosis is lil<ely to have the following complaints EXCEPT:
a) Dyspareunia.
b) Mucopurulent vaginal discharge.
c) Painful defecation.
d) Severe dysmenorrhea.
e) Infertility,
857- The major evidences of adenomyosis is/are:
a) Adenxal mass.
b) Menorrhagia and dysmenorrhea.
c) Urinary frequency.
d) Dysmenorrhea and infertility.
e) Pressure symptoms.
858- The Follorving are useful in the treatment of endometriosis EXCEPT:
a) Human chorionic gonadotrophin.
b) Oophorectomy.
c) Hysterectorny.
d) Oral contraceptive pills.
e) Danazol,
859- Danazol treatment for cndometriosis is associated with all the following EXCEPT:
a) Acne.
b) Weight gain.
c) Hot flashes.
d) Mucoid vaginal discharge.
e) Decreased breast size.
860- Endometriosis treated with prolonged estrogen and progesterone combination therapy exhibits
which of the following histological characteristics?
a) Marked ederna.
b) Decidual-like reaction.
c) Glandular hypertrophy.
d) lnfl amrnatory infi ltrate.
e) Cyclic changes.
861- The INCORRECT statement in pelvic endometriosis:
a) Treatment by danazol may produce hirsutism.
b) The tubes are uncomrnonly blocked.
c) There is a close association with the unruptured follicle syndrome.
d) The amount of pelvic pain is not related to the extent of the disease.
e) Clomiphene citrate should be used to induce ovulation.
862- Characteristic symptoms of endometriosis @ NOT include:
a) Dysrnenorrhea.
b) Deep dyspareunia.
c) Infenility.
d) Pelvic pain.
e) Premenstrual tension.
863- The following is the essential step in diagnosis and staging of pelvic endometriosis:
a) Laparoscopy.
b) cA-r25,
c) Biopsy forrn the suspicious nodules.
d) Ultrasonography.
e) Hysteroscopy.
WhiteKnightLove

-l0s-
Freely you have received; freely give.

864- The INCORRECT statement regarding endomctriosis:


a) lt is estrogen-dependant disease.
b) It is progestogen-dependant disease.
c) It is peculiar to reproductive years oflife.
d) Immunologic alterations explain some of the etiological factors.
e) There is genetic predisposition.
865- The INCORRECT statement regarding clinical presentation of pelvic endometriosis:
a) It could be asymptomatic.
b) Dysrnenorrhea is a common symptorn.
c) It could result in deep dysparenia.
d) lnfertility might be the presenting symptom.
e) Vesico-vaginal fistula is a possible complication.
866- Theories that explain histiogenesis of endometriosis DO NOT include:
a) Transporl theory.
b) Metastasis theory.
c) Celornic metaplasia.
d) Induction theory.
e) Infectious theory.
867- The INCORRECT statement regarding endometriosis:
a) It is an estrogen-dependant disease.
b) There is evidence to support immunological etiological factors.
c) Wonren with prirnary relatives with endometriosis have a greater risk of having the disease.
d) There is no evidence of spread with lymphatics,
e) Endometriosis is nrost often at the dependent portions of the pelvis.
868- The INCORRECT statement regarding endometriosis:
a) It rray be asynrptomatic.
b) It may be responsible for chronic acyclic pelvic pain.
c) The only presenting symptom might be infertility.
d) It might be associated with luteal insufficiency.
e) The process of ovulation is not disturbed in patients with endometriosis,
869- The INCORRECT statement regarding endometriosis:
a) Clinical picture is not diaguostic.
b) The tumor marker CA 125 nright be elevated.
c) Laparoscopic picture is helpful and biopsy is essential.
d) Chocolate fluid represents inspissated old blood.
e) Classification system of the endometriosis is not related to the type of adenxal adhesions.
870- A 36-year-old nulligravid woman complaining of painful menstrual periods and failure to achieve
a pregnancy after I year of unprotected intercourse. She wonders if she could have endometriosis.
Positive finding of which of the following is the MOST VALUABLE to reach a finaldiagnosis?
a) Nodularity of the uterosacral ligarnents.
b) Ovarian enlargement.
c) Fixed retroversion ofthe uterus.
d) Laparoscopic visualization of irnplants.
e) Dyspareunia.
871- Side effects of progestins in treatment of endometriosis DO NOT include:
a) Depression.
b) Nausea.
c) Weight gain.
d) Secondary amenorrhea.
WhiteKnightLove

e) Breakthrough bleeding,

-r06-
'It is more blessed to give than to receive.

872- The INCORRECT statement regarding danazol in the treatment of endometriosis:


a) The aim is production of amenorrhea.
b) The dose is fixed at 400 mg/day.
c) The drug is successful in relieving the symptoms.
d) It has androgenic side effects.
e) It produces atrophy of the endometrium,
873- Thc following is true about danazol EXCEPT:
a) lt is a synthetic compouhd.
b) I!has an androgenic properties.
c) It has an anabolic effects.
d) It has an antigonadotrophic effects.
e) The main indication is in the treatment of dysfunctional uterine bleeding.
874- The INCORRECT statement regarding conservative surgical treatment of endometriosis:
a) It involves restoration of the normal anatomy whenever possible.
b) The aim is to get rid of all implants.
c) Operative laparoscopy is suitable in most of the cases.
d) Endometriotic lesions could be dealt with by coagulation.
e) Preoperative progestins increase chances ofsuccessful surgery.
875- Non-neoplastic masses of the ovary include all the following EXCEPT:
a) Theca-lutein cysts.
b) Pregnancy luteomas,
c) Endometriotic cysts.
d) Corpus luteum cysts.
e) Dermoid cyst.
876- A benign ovarian tumor has one of the following gross features:
a) Excrescences on the surface.
b) Peritoneal implants.
c) Intra-cystic papillae.
d) Free mobility.
e) Capsule rupture.
877- The complications of bcnign ovarian neoplasms include thc following EXCEPT:
a) Torsion.
b) Infection.
c) Malignant change.
d) Rupture.
e) Pulmonary embolism.
878- All the following about dermoid cyst are true EXCEPT:
a) It is a germ cell tumor.
b) It is the commonest ovarian tumor during pregnancy.
c) Malignant change occurs in about 1.5%.
d) It is usually detected bilaterally.
e) It usually causes harm when ruptures.
879- Treatment of benign ovarian neoplasms may include the following EXCEPT:
a) Debulking.
b) Ovarian cystectomy.
c) Unilateral oophorectomy.
d) Hysterectomy plus unilateral salpingo-oophorectomy.
e) Pan hysterectomy.
WhiteKnightLove

-107-
Freely you have received; freely give.

880- Functional ovarian masses include:


a) Paraovarian cyst.
b) Endometriomas.
c) Luteomas.
d) Dennoid cysts.
e) Fibromas.
881- Regarding Brenner tumor, allthe following are correct EXCEPT:
a) It usually develops in patients over 40 years.
b) It is usually unilateral.
c) It rarely turns malignant.
d) It may secrete androgen.
e) It is usually found in the wall of mucinous cystadenoma.
882- Benign cystic teratoma (dermoid cyst):
a) It represents 50% ofthe ovarian neoplasms.
b) It is the commonest ovarian tumor during pregnancy.
c) Malignant change occurs in about 80%.
d) Choriocarcinoma is the comrnonest nralignant change.
e) Rarely causes harm to the patient when ruptures.
883- Clinical picture of benign ovarian swelling DOES NOT include:
a) Central abdorninal location of the swelling.
b) Local tenderness.
c) Smooth surface.
d) Well defined edges.
e) Dullness on percussion.
884- Pressure symptoms of large benign ovarian srvelling DO NOT include:
a) Frequency of rnicturition.
b) Indigestion.
c) Vomiting.
d) Edema of the lower limbs.
e) Dyspnea.
885- Complications of benign ovarian neoplasms DO NOT include:
a) Malignant transformation.
b) Torsion.
c) Hyaline degeneration.
d) Hemorrhage.
e) Infection.
886- Surgical removal of benign ovarian tumors is essential if any of the following is diagnosed
EXCEPT:
a) Mal ignant transfonnation.
b) Torsion.
c) Incarceration.
d) Vaginal bleeding.
e) Rupture.
887- Options for surgical techniques in case of benign ovarian tumor DO NOT include:
a) Vaginal hysterectomy and retnoval of both adenxae.
b) Abdorninal hysterectonty and retnoval of the diseased adnexa.
c) Abdominal hysterectotny and removal of both adenxae.
d) Ovariectomy.
e) Ovarian cystectomy.
WhiteKnightLove

-108-
'It is more blessed to give than to receive.

888-Causes of pelviabdominal swelling DO NOT include:


a) Ascites.
b) Fibroid uteus.
c) Overfilled bladder,
d) Pregnancy.
e) Tubal pregnancy.
889- Risk factors of endometrial hyperplasia DO NOT include:
a) Anovulatory disorders.
b) Chronic endometritis.
c) Obesity.
d) Tarnoxifen,
e) Unopposed excessive estrogen.
890- Endometrial hyperplasia could be expected in the following conditions:
a) Endodennal sinus turnor.
b) Cystic teratoma.
c) Polycystic ovary disease.
d) Serloli-Leydig cell turnor.
e) Dysgerminoma.
891- The MOST LIKELY functional ovarian tumor in a 54-year-old woman with endometrial
hyperplasia:
a) Benign cystic teratoma.
b) Lipid cell tumor.
c) Serloli-Leydig tumor.
d) Mucinous cystadenocarcinoma.
e) Granulosa-theca cell tumor.
892- Comptex endometrial hyperplasia without atypia in a woman aged 4l years could be treated with
the following EXCEPT:
a) Progestins.
b) Estrogens.
c) Progestins plus estrogens.
d) Oral contraceptive pills.
e) Hysterectorny.
893- Forms of endometrial hyperplasia pp include:
a) Sirnple hyperplasia with atypia.
b) Simple hyperplasia without atypia.
c) Cystic glandular hyperplasia.
d) Adenomyoma with atypia.
e) Adenomatous hyperplasia without atypia.
894- Which IS NOT valuable in the diagnosis of endometrial hyperplasia:
a) Endometrial biopsy.
b) Transvaginal ultrasound.
c) Abdorninal ultrasound.
d) Hysteroscopy.
e) E,ndornetrial smear.
895- Possible lines of treatment of endometrial hyperplasia ppl include:
a) Endometrial curettage.
b) Cyclical progestogen.
c) Vaginal hysterectomy
d) Cyclical estrogen.
WhiteKnightLove

e) Abdorninal hysterectorny.

-109-
Freely you have received; freely give.

896- Histopathological criteria of arias-stella reaction DO NOT include:


a) Cellular enlargement.
b) Hyperchromatosis.
c) Pleomorphism.
d) Mitotic activity.
e) Aneuploidy of the epithelial cells.
897- Arias-stella reaction may be found in association with the following EXCEPT:
a) Normal pregnancy.
b) Ectopic pregnancy.
c) Endornetriosis.
d) Pelvic inflarnmatory disease.
e) Abortion.
898- Causes of intrauterine adhesions DO NOT include:
a) Fibroid uterus.
b) Previous endometrial curettage.
c) Previous rnanual removal of the placenta.
d) Previous septic aborlion.
e) Endometrial tuberculosis.
899- Clinical criteria of Ashernran syndrome DO NOT include:
a) Secondary amenorrhea.
b) Hyponrenorrhea.
c) Inferlility.
d) Abortion.
e) Cryptomenorrhea.
900- The INCORRECT statement for the genitourinary listula:
a) Congenital fistula is the most common etiological cause in the current practice.
b) Pelvic tubeculosis might be an etiological factor.
c) Stage IV carcinoma of the cervix rnight involve vesicovaginal fistula.
d) Vesicovaginal fistula is the most common type,
e) The incidence of the fistula in a community reflects the standard of the obstetric practice.
901- The INCORRECT statement regarding the vesicovaginal fistula:
a) It might present in association with secondary amenorrhea.
b) It might be associated with phosphatic deposits on the vulval skin.
c) It might present with stress incontinence.
d) Incontinence is by day and by night.
e) lt rnay follow complicated cesarean section.
902- The INCORRECT statement regarding thc gcnitourinary fistulae that it might be associated
with:
a) Perineal tears.
b) Vaginal scars.
c) Lichen planus of the vulval skin.
d) Sheehan's syndrome.
e) Cervical atresia.
903- The INCORRECT statement regarding necrotic vesicovaginal fistula:
a) It results frorn prolonged labor.
b) It results from pressure ofthe fetal head on the bladder tissue against symphysis pubis.
c) It results from ischemic necrosis of the bladder wall.
d) Urinary incontinence appears immediately after vaginal delivery.
e) Sim's position is classic for examination to visualize the fistulous opening.
WhiteKnightLove

-l l0-
'It is more blessed to give than to receive.

904- The INCORRECTstatement regarding postoperative care after surgical repair of the
vesicovaginal fistula:
a) Urinary antiseptics have to be given for all cases.
b) lt is not necessary to keep the patient in bed.
c) A urinary catheter should be inserted every four hours.
d) Bowel habit is encouraged to be normal.
e) Postoperative pyrexia should be taken seriously.
905- Which is a usefultest in vesicovaginalfistula:
a) Fern test.
b) Papanicolaou test.
c) Schiller iodine test.
d) Click test.
e) Cystometry.
906- The INCORRECT statement regarding ureterovaginal fistula:
a) It might happen during cornplicated vaginal surgery in third degree uterine prolapse.
b) It might happen during surgery for broad ligament tumors,
c) Obstetric causes are the rnost comrnon cause.
d) lnjury of the ureter may be at the level of the pelvic brim.
e) Surgical treatment involves laparotomy.
907- Rectovaginal fistula might result from all the following EXCEPT:
a) Advanced pelvic rnalignancy.
b) Herpes vulvitis.
c) Complicated vaginal delivery.
d) Genital tuberculosis.
e) Irradiation to the pelvis.
908- The INCORRECT statement for primary dysmenorrhea:
a) There is no obvious pelvic pathology.
b) It appears forthe first tirne usually l-2 years after menarche,
c) The pain usually starts 2 days before the menstrual flow.
d) The pain is usually colicky and in the suprapubic region.
e) There is no rebound tenderness in the suprapubic region.
909- Primary dysmenorrhea:
a) Is associated with post-menarchal anovulatory cycles.
b) Ofa severe degree occurs in up to l5% ofteenage girls.
c) Will respond to treatment with NSAIDs or COCP in 30% of cases.
d) Is associated with chronic PID,
e) None ofthe above,
910- An lS-year-old consults you for evaluation of disabling pain with her menstrua! periods. The pain
has been present since menarche and is accompanied by nausea and headache. History is
otherwise unrenrarkable, and pelvic examination is normal. You diagnose primary dysmenorrhea
and recommend initial treatment with which of the following?
a) Ergot derivatives.
b) Anti prostaglandins.
c) Gonadotropin-releasing hortnone (GnRH) analogues.
d) Danazol.
e) Codeine.
9l l- The INCORRECT statement for secondary dysmenorrhea is that it:
a) Never occurs with anovulatory cycles.
b) May occur with bicornuate uterus.
WhiteKnightLove

c) May occur with pelvic inflammatory disease.


d) May occur with uterine polyps.
e) May occur with pelvic congestion syndrome,

-r I l-
Freely you have received; freely give.

912- A l9-year-old woman comes to a physician for evaluation of sharp pain thad occurs in her lower
abdomen for 2-3 days every month since her menses began at l4 years of age. Approximately 2
weeks after she experiences this pain, she has her menses. The MOST PROBABLE etiology for
her pain is:
a) Endometriosis.
b) Dysmenorrhea.
c) Pelvic infection.
d) Mittelschmerz.
e) Ectopic pregnancy.
913- A 35-year-old lvoman complains of constant, deep, pelvic pain. It vyorsens during menstruation,
sexual intercourse, and bowel movements. Her LMP was I week ago. Vital signs are blood
pressure ll0l70 mmHg; pulse 80 bpm; and temperature 37.zoc. Abdominal examination elicits
bilateral lower quadrant tenderness without rebound. Pelvic examination demonstrates a tender
6-cm left adnexal mass and fixation of the uterus and uterosacral ligaments, Laboratory data are
hematocrit 4C7o; white blood count 7000/mL; and negative serum pregnancy test. Transvaginal
sonography shows a 6-cm echogenic left adnexal mass. The uterus and right adnexa are normal.
Which of the following is the MOST LIKELY diagnosis?
a) Follicular cyst.
b) Adnexal torsion.
c) Benign cystic teratorna.
d) Leiornyornata.
e) Endornetrioma.
914- A 34-year-old lvoman seeks consultation for menorrhagia and dysmenorrhea, which have
developed during the past year. Her pelvic examination reveals an 8-week-size uterus with no
evidence of an adnexal mass. A pregnancy test is negative, and the hematocrit is 31.27". Which of
the follorving is the FIRST DIAGNOSTIC test to perform?
a) Ultrasound.
b) Hysterosalpingography (HSG).
c) Magnetic resonance imaging of the pelvis.
d) Endornetrial biopsy.
e) Computed tomography scanning.
915- A 32-year-old Pl woman presents with complaints of severe cramps with her menses. The pain is
so bad that it keeps her out of work 3-4 days cach month. Which of the following is the MOST
UNLIKELY to be the cause?
a) Asherman's syndrotne.
b) Endornetriosis.
c) Adenomyosis.
d) Cervical stenosis.
e) Uterine fibroids.
916- A 39-year-old woman, gravida3, para3, complains of severe, progressive secondary dysmenorrhea
and menorrhagia. Pelvic examination demonstrates a tender, diffusely enlarged uterus with no
adncxal tenderness. Results of endometrial biopsy are normal. This patient MOST LIKELY has:
a) Endonretriosis.
b) Endometritis.
c) Adenornyosis.
d) Uterine sarcorrra.
e) Leiornyorna.
WhiteKnightLove

-tt2-
'It is more blessed to give than to receive.

917- The abnormal pattern of bleeding that appears in conjunction with a short follicular phase is
called:
a) Menorrhagia.
b) Menometrorrhagia.
c) Polymenorrhea.
d) Anovulatory bleeding.
e) Metrorrhagia.
918- The INCORRECT statement for metropathia hemorrhagica is:
a) The endometrium is thickened because of hyperplasia.
b) There may be endometrial polypoidal reaction'
c) The condition is usually ovular.
d) Endometrium may show swiss cheese appearance'
e) Menstrual pattern shows short period of amenorrhea followed by bleeding'
919- The INCORRECT statement for dysfunctional uterine bleeding is association with:
a) Metropathia hemorrhagica.
b) lncreased amount of estrogen and decreased amount of FSH'
c) lncreased levels of PG E2 in the endometriurn'
d) Functiorral ovarian cYst.
e) Estrogens inhibiting the arachidonic acid cascade.
920- The INCORRECT statement regarding cyclical menorrhagia at the age of 46 years:
a) It is likely to be cured by an oral progestogen.
b) Serious underlying pathology is unlikely.
c) Hysterectomy may be needed if medical treatment fails'
d) Dilatation and curettage is always needed at early stages in the management'
e) Chance ofthe cycles to be anovular is 570.
g2l- A l5-year-old girl presented with heavy vaginal bteeding over the last two months' all the
following are correct EXCEPT:
a) Coagulation disorder should be excluded.
b) Ovulatory dysfunction is a common cause.
c) Plegnancy complications should be excluded.
d) The comtnonest hematological disorder is hemophilia'
e) Evaluation of the patient must include abdominal ultrasonography.
922- Dysfunctional uterine bleeding (DUB) is frequently associated with:
a) Endometrial PolYPs.
b) Anovulation.
c) Cervical polYPs.
d) Uterine fibroids.
e) von Willebrand disease
923- Treatment of rlysfunctional uterine blcecling include the following EXCEPT:
a) Progestin theraPY.
b) Estrogen theraPY.
c) Danazol.
d) Endometrial ablation'
e) Brornoci'iPtine.
924- postmenopausal bleeding is a common presentation of all the following WI::
a) Cervical ectroPion'
b) Carcinorrra of the endometrium'
c) Atrophic vaginitis.
d) Carcinorna of the cervix'
WhiteKnightLove

e) Sarcotna of the uterus.

-l l3-
Freely you have received; freely give.

925- In women rvith postmenopausal bleeding, it is best to perform endometrial sampling if the
endometrium on transvaginal ultrasound is thicher than:
a) I rnm.
b) 2 rnm.
c) 5 mm.
d) 7 mm.
e) l0 mm.
926- A 60-year-old woman had unexpected slight blood on her underwear. It was bright red without clots
and there was no pain or discomfort. The bleeding recurred twice in similar fashion. Her last period
was at the age of 48 years and she has had no other intervening bleeding episodes. She had hot
flushes and night srveats around the time of her menopause. She had normal cervical smear 4
months previously. She had two vaginal deliveries then used intrauterine devices for contraception.
She did not use hormone-replacement therapy. She is slightly overweighed. Abdominal examination
is normal, The vulva, vagina and cervix appear normal for her age. The uterus was felt small and
anteverted on clinical palpation. There was no palpable adenxal masses. Transvaginal ultrasound
scan showed the endometrium is l2 mm in thickness. What is the NEXT STEP in management?
a) Cyclical honnonal therapy.
b) Estrogen replacement therapy.
c) Hysterectomy.
d) Estirnation of tlre turnor markers.
e) Endornetrial biopsy.
927- A single obese woman, aged 55 years, began to have scant irregular bleeding seven years after
menopause. The hymen was intact and rigid. There was fresh thin blood-tinged discharge per
vagina. Rectal examination revealed no abnormality. The NEXT PROCEDURE should be:
a) A therapeutic trial ofestrogen.
b) Observation.
c) Vaginal and cervical smears at regular intervals.
d) Speculurn examination under analgesia.
e) Diagnostic curettage and cervical biopsy.
928- The CORRECT statement regarding postmenopausal bleeding:
a) Hysteroscopy is a useful diagnostic tool.
b) The most colnmon cause of bleeding is endometrial cancer,
c) Sonographic measurement of endometrial thickness is not useful.
d) Benign causes ofbleeding are rare.
e) Colposcopy is not useful.

For each case history that follows numbered (929-932), select the type of menstrual bleeding lettered
(a-e) rvith rvhich it mst likely is associated.
EACH LETTERED CHOICE MAY BE USED ONCE, MORE THAN ONCE, OR NOT AT ALL.
929- A Z4-year-old nulligravid lvoman is being evaluated for infertility. I a) Progesterone
Her cycles are irregular, lasting for 30 to 90 daysl menstrual flow is I breakthrough bleeding.
usually heavy. She also has increasing hirsutism and obesity. I b) Progesterone
930- A 55-year-old lvoman, three year postmenopause, is bothered by hot I withdrawalbleeding.
flashes. Her cloctor orders conjugated estrogen (Premarin), 1.35 mg I c) Estrogen breakthrough
daily for 2l of 28 da1's. On the fourth month of Premarin therapy, the I bleeding.
woman has heavy menstrual flow from the 25th to the 28th day. I d) Estrogen withdrawal
93 l- An lS-year-olcl \voman complains of severc dysmenorrhea. She has I bleeding.
rcgular periocls cvery 28 days, rvith light flow on the first day, heavy I e) None of the above.
flow on thesecond and third days, and lightspottingon the fourth day.
WhiteKnightLove

932- A 33-year-old woman is paced 0n one of thc newer low-dose birth-


control pills. Two weeks later, having tal<en nine of the birth-control
pills, she calls your office because she has begun to have menstrual
flow.
-1 l4-
'It is more blessed to give than to receive.

For each patient described below numbered (933-936), select the medication or testing lettered (a-e )
suited to correct or diagnose the dysfunctional bleeding.
EACH NUMBERED CHOICE MAY BE USED ONCE, MORE THAN ONCE, OR NOT AT ALL.
933- A 24-year-old sexually active woman presents with irregular cycles I a) Clomiphene citrate
at intervals anywhere from 30 to 90 days. Menstrual flow is usually | (Clomid).
heavy, and she has noted increasing hirsutism. I b) Oral contraceptives,
934- A 52-year-old obese, hypertensive woman presents with abnormal I c) Antiprostaglandin.
bleeding. I d) Endometrial sampling.
935- A lS-year-old girl, who is not sexually active, complains of I e) None of the above.
excessively heavy menstrual flow ryith associated dysmenorrhea.
936- A 32-year-old infertility patient with intermenstrual spotting had an
endometrial biopsy that was interpreted as revealing an inadequate
luteal nhase.

For each patient described below numbered (937-938), select the medication or testing lettered (a-e)
suited to corrcct or diagnose the dysfunctional blceding. EACH NUMBERED CHOICE MAY BE
USED ONCE, MORE THAN ONCE, OR NOT AT ALL.
937- A lS-year-old girl complains of excessively heavy menstrual a) Clorniphene citrate (Clomid).
flow rvith associated dysmenorrhea. b) Oral contraceptives.
938- A 32-year-old infertility patient with intermenstrual spotting c) Antiprostaglandin.
had an endometrial biopsl, that was interpreted as revealing an d) Endometrial sampling.
inadequate luteal phase. e) None ofthe above.

WhiteKnightLove

-115-
Freely you have received; freely give.

THE BREAST IN GYNECOLOGICAL PRACTICE

Direction: Select the ONE BEST lettered answer or completion in each question.

939- Which IS NOT suitable for treatment for mastalgia:


a) Surgery.
b) Bromocriptine.
c) Danazol.
d) Gammalenolenic acid (GLA).
e) Antiprostaglandins.
940- Which is INCORRECT statement for nipple discharge:
a) lt is common complaint.
b) Galactorrhea is usually unilateral.
c) It is usually physiologic.
d) Duct ectasia is a benign cause for the discharge.
e) Mamrnography should be a part of the evaluation with bloody discharge.
941- Causes of breast pain DO NOT include:
a) Fibrocystic changes ofthe breast.
b) Pectoral rnuscle exeftion.
c) Cyclical rnastalgia.
d) Danazol,
e) Malignant lesion of the breast.
942- Causes of nipple discharge DO NOT include:
a) Tranquilizers.
b) Galactorrhea.
c) Malignant lesion of the breast.
d) Fibroadenoma of the breast.
e) Intraductal papillorra.
943- Risk factors of breast cancer DO NOT include:
a) Genetic predisposition.
b) Early menarchae,
c) Late menopause.
d) Multiparity.
e) Longevity.
944- Clinical findings of breast mass that indicate early breast carcinoma DO NOT include:
a) Cystic changes in the mass.
b) No tenderness of the mass.
c) Firm to hard consistency of the mass.
d) No pain related to the mass.
e) Ill-defined edges of the mass.
945- Breast self examination (BSE) involves all the following EXCEPT:
a) It is advised to be done once per month to all women above 20 years of age,
b) The best time is postmenstrual.
c) The aim is detection of the smallest lesions.
d) It is to be carried out in more than one position to demonstrate lesions embedded within the fat,
e) Marnmography is performed only if there is positive findings on BSE.
WhiteKnightLove

-l l6-
'It is more blessed to give than to receive.

GYNE COLOGICAL INFE CTIONS

Direction: Select the ONE BEST lettered answer or completion in each question.

946- The natural defense of the vagina to infection includes all of the following EXCEPT:
a) The acidic vaginal pH.
b) The presence ol Doderlein bacilli.
c) The physical apposition of the pudendal clelt and the vaginal walls.
d) The bacteristatic secretions of vaginal glands.
e) The vaginal stratified squalnous epitheliurn.
917- The following statenrents about vaginal discharge are correct EXCEpT:
a) Leucorrhea means excesstve atnount ofnonnal vaginal discharge.
b) Columnar epithel iurn of the cervix contributes to the normal vaginal discharge.
c) Atrophic vaginitis is relatively colnmon in postmenopausal women.
d) Estrogen therapy increases the atnount ofvaginal discharge.
e) The alrount ofvaginal discharge increases only during infection.
948- Which is CORRECT about vaginal discharge:
a) Cervical ectopy might induce vaginal discharge.
b) Normally, the vagina is dry.
c) Atrophic vaginitis is uncommon in postmenopausal women.
d) Progesterone causes a proliferation of the vaginal epitheliurn.
e) Excessive vaginal discharge must indicate local infection.
949- The MOST COMMON site of infection of the vulva is:
a) The clitoris.
b) The vestibular bulbs.
c) The labia rnajora.
d) The Bartholin glands.
e) The Hyrnen.
950- Persistent vaginal dischargc in a pediatric patient should prompt a search for which of the
following?
a) Foreign body.
b) Pinwonns.
c) Illicit drug use.
d) Vaginal lacerations.
e) Ectopic ureter.
95I- All the following are causes of vaginal discharge in a prepubertal girl EXCEPT:
a) Ovarian dysgerminoma.
b) Sarcoma botryoides.
c) Enterobius vermicularis.
d) Foreign body.
e) Candidiasis.
952- The INCORRECT statement for vaginal candidal infcction:
a) Pregnancy is a predisposing cause for active infection.
b) Vaginal pH is usually acidic.
c) Vaginal discharge tends to cling to the vaginal walls.
d) Metronidozole is an effective therapy.
WhiteKnightLove

e) Vaginal miconazole is an effective therapy.


953- The following about candidalvaginal infection is INCORRECT:
a) The infection rate rises with pregnancy.
b) Vaginal pH is usually alkaline.

-t17-
Freely you have received; freely give.

c) Vulval itching may occur.


d) Vaginal isoconazole or miconazole are effective.
e) Diagnosis is usually made without need for culture.
954- The INCORRECT statement for fungal vulvovaginitis:
a) Asymptornatic carriers are rare.
b) Most of cases are due to candida albicans strains.
c) Pregnancy is a risk factor.
d) Debility is a risk factor.
e) Cytotoxic medications are risk factor.
955- The INCORRECT statement for vaginal discharge of fungal vulvovaginitis:
a) Thick, sernisolid.
b) White.
c) Soapy.
d) Clings to the vaginal wall.
e) Frothy.
956- The INCORRECT statement for fungalvulvovaginitis:
a) Vulval itching tends to increase premenstrual.
b) There rnay be dyspareunia.
c) The vulval skin may show edema with the redness,
d) The infection does not cross the genitocrural skin folds.
e) White patches rnay be studded on the vaginal walls.
957- The INCORRECT statement for fungalvulvovaginitis:
a) Recurrence after syrnptornatic irnprovement is a rare possibility.
b) There may be genitointestinal reservoir.
c) Reinfection from the sarne partner is a possibility.
d) Reinfection from the patient underwear is a possibility.
e) Long terrn course with oral antifungals may be needed.
958- The following statements about trichomonas vaginalis are correct EXCEPT:
a) Trichomonas vaginalis (TV) may be a sexually transrnitted disease.
b) Most patients, having TV have symptoms.
c) The disease rnay cause vulval itching.
d) Unlike bacterial vaginosis, metronidazole is an ineffective therapy.
e) It rnay cause a strawberry appearance of the cervix.
959- The INCORRECT statement for trichomnas vaginalis:
a) Asyrnptor-natic carriers are rare.
b) The main method of transmission is at sexual intercourse.
c) Decreased vaginal acidity is a predisposing factor.
d) Symptorns exacerbate after nrenstrual flow.
e) It rray be a cause of postcoital bleeding.
960- The INCORRECT statement for the vaginal discharge with trichomnas vaginalis:
a) Malodorus fishy.
b) Profuse.
c) Foamy.
d) Greenish yellow.
e) Clings to the vaginal wall.
961- The INCORRECT statement for trichomnas vaginalis:
a) The vulva may be affected with erythema and edema.
b) The vaginal wall is reddish with peticheal hemorrhages.
c) The cervix is never affected.
WhiteKnightLove

d) The discharge is abundant and frothy.


e) Introduction of the speculum into the vagina may be painful.

-l l8-
'It is more blessed to give than to receive.

962- All the following about bacterial vaginosis are correct EXCEPT:
a) The discharge is a gray homogenous rvith malodor.
b) The discharge does not usually cause considerable itching.
c) Vaginal pH is decreased,
d) Cure may be achieved by oral rnetronidazole.
e) The organism is a bacterium.
963- Thc following about bacterial vaginosis is INCORRECT:
a) The discharge is scanty and usually odorless.
b) The discharge usually does not cause considerable itching.
c) Cure can be achieved by oral metronidazole.
d) It is nonspecific type of bacterial infection.
e) Clue cells are characteristic.
964- Bacterial vaginosis IS NOT associated with:
a) Positive "Whiff test",
b) "Clue cells".
c) Positive response to metronidazole.
d) Vaginal pH of rnore than 4.5.
e) lvlarked vaginal inflamrnation.
965- As rcgards bacterial vaginosis, all thc follorving are corrcct EXCEPT:
a) lt is the commonest cause of vaginal infection.
b) The vaginal pH is usually below 4.5.
c) It rrray present with a fishy smell discharge.
d) Its fishy smell may increase after intercourse.
e) lt can be diagnosed by clue cells on Grarn staining of vaginaldischarge.
966- Risk factors of bacterial vaginosis DO NOT include:
a) Conorrhea.
b) Increased alkalinity oFthe vaginal discharge.
c) Poor liygiene,
d) lncreased lactobacillus.
e) Sexual intercourse.
967- Cluc cells arc:
a) Epithelial cells ,rvith intracellular bacteria.
b) Epithetial cells with extracellular bacteria.
c) Leukocytes rviLh intracellular bacteria.
d) L,eukocytes rvith extracellular bactena.
e) Epithelial cells rvith intracellular leukocy'tes.
968- The INCORRECT statcment for the vaginal discharge witlr bacterial vaginosis:
a) It is white crearn.v.
b) It is hornogenous.
c) lt has an arnine malodrous smell.
d) It alr.r,ays cause intense itching.
e) It does not clirrg to the vaginal wall,
969- Thc bacterial infection that IS NOI sexually-transmitted:
a) Neisseria gonorrhea.
b) Cardnerella vaginalis.
c) Group B streptococci.
d) Treponerna pal lidurn.
e) Trichomonas vagirralis.
970- As regards Chlamydial inl'ection, tlre following statcments are correct EXCEPT:
a) The organisrns are obligatory intracellular organistns.
WhiteKnightLove

b) It rnay cause sterile pyuria.


c) Chlamydia and gonorrhea are the main causes of PID.
d) The husband should be treated.

- u9-
Freely you have received; freely give.

e) Azithrornycin2 g in single dose is an effective treatment.


971- As rcgard Chlamydial infection, the following statements are correct EXCEPT that is:
a) Is usually symptomatic.
b) ls commonly a cause of vaginal discharge,
c) May be treated with either erythromycin or tetracyclin.
d) Can cause neonatal pneurnonia,
e) Can cause sterile pyuria.
972- The following about Chlamydia trachomatis are correct EXCEPT that it:
a) Can cause neonatal pneumonia.
b) Can cause sterile pyuria.
c) Can be cultured frorn vaginal discharge.
d) Is a colrmon cause of pelvic inflarnmatory disease.
e) May be treated with either erythromycin or aminoglycosides.
973- The follorving about Mycoplasma are correct EXCEPT:
a) It uray cause tnale infertility.
b) lt is the srnallest self-replicating; pleornorphic bacteria.
c) It can be successfully treated with penicillin.
d) lt can cause bacterial vaginosis.
e) It rnay cause pelvic inflarnrnatory disease.
974- The follorving about human papilloma virus (HPV) infection are correct EXCEPT:
a) The disease is the most common viral STD.
b) It rnay lead to CIN and cervical cancer.
c) It rnay be warty or flat condyloma.
d) It can be treated by trichloroacetic acid.
e) It is not usually associated with other STDs.
975- The CORRECT statentent for human papilloma virus:
a) It is a rare type of STD.
b) It is classified into subtypes according to types ofantibodies.
c) It has a questionable role in development of CIN.
d) Koilocytic atypia is the main feature in the cytological study.
e) The most effective anti-viral is acyclovir.
976- Which of thc follorving primary treatment is most appropriate for a patient with extensive vulval
Iesions diagnosed to be condyloma accuminatum (venereal warts):
a) Application of podophyllin.
b) 5-fluorouracil.
c) Morcellation.
d) Sirnple vulveclorny.
e) Local exicision.
917- Thc CORRECT statement for herpes simplex:
a) Type I can invade the genital tract.
b) In prirnary fortns, the leading complaint is agonizing pain.
c) The recurrent attacks tend to be less severe and have generalized manifestations more than the
primary attack.
d) During primary infection, vaginaldelivery may result in neonatalherpes in 100%.
e) Long-term suppression rvith acyclovir increases the number ofrecurrent attacks.
978- Which charactcr IS NOT applicable to vulval ulcer due to herpes zoster:
a) Unilateral.
b) Painful.
c) Superficial
WhiteKnightLove

d) Rolled edges.
e) lnflarnrned.

-t20-
'It is more blessed to give than to receive.

919- A 32-year-old woman complains of a vulval fishy odor and a vaginal discharge. The speculum
examination reveals an erythematous vagina and punctuations of the cervix. Which is the MOST
LIKELY diagnosis?
a) Candidal vaginitis.
b) Trichomona[ vaginitis.
c) Bacterial vaginosis.
d) Human papilloma virus.
e) Herpes simplex virus.
980- A 29-year-old woman G0 is complaining of a vaginaldischarge for the past 2 weeks. She describes
the discharge as thin in consistency and of a grayish white color. She has also noticed a slight
smelly vaginal odor that seems to have started with the appearance of the discharge. She denies
any vaginal or vulval pruritus or burning. She had no intercourse during the past year. She is cur-
rently on no medications. On physical exam, the vulva appears normal and the cervix is not
inflammed. There is a copious thin whitish discharge in the vaginal vault. Wet preparation of the
vaginal fluid showed absence of WBCs and stippling of epithelial cells. What is the MOST
LIKELY diagnosis?
a) Candidiasis.
b) Bacterial vaginosis.
c) Trichornoniasis.
d) Physiologic discharge.
e) Chlarnydia,
981- Which confirms the diagnosis of PID?
a) White blood cell count.
b) Laparoscopy.
c) Culdocentesis.
d) Cervical Gram stain.
e) Pelvic ultrasound.
982- Risk factors in the developrnent of acute salpingitis include all the following EXCEPT:
a) An age of l5 to 24 years.
b) Oral contraceptives.
c) Presence ofan intrauterine device.
d) Multiple sex partners.
e) Previous gonorrhea or salpingitis.
983- Acute gonorrhea might present with all the following EXCEPT:
a) Lower abdonrinal pain.
b) Dyspareunia.
c) Urinary tract infection.
d) Vulval ulcer.
e) Mucopurulent vaginal discharge.
984- A 23-year-old woman presents to the physician's office complaining of a mucopurulent vaginal
discharge, lower abdominal pain and a fever which began towards the end of her menstrual
period. Which scxually transmitted disease (STD) is shc MOST LIKELY to have?
a) Gardnerella vaginalis.
b) Chlarnydia trachomatis.
c) Neisseria gonorrhoeae.
d) Chancroid.
e) Lymphogranuloma venereum.
WhiteKnightLove

-121-
Freely you have received; freely give.

985- A 27-year-old woman complains of vaginal discharge. On speculum examination, she has a mucoid
yellowish discharge and her cervix appears erythematous. On bimanual exam, she has cervical
motion tenderness, no uterine tenderness, and no adnexal tenderness. Her temperature is 36.7oC,
WBC 8.4x103/mL, and the rest of the vital signs and laboratory resutts are within normal limits.
The MOST LIKELY diagnosis is:
a) Vaginitis.
b) Cervicitis.
c) Endomyometritis.
d) Pelvic inflarnmatory disease.
e) Tubo-ovarian abscess.
986- A 23-year-old woman with right-sided lower abdominal pain and chills is seen in the emergency
department. The pain began 3 days ago and is associated with vaginal discharge. Her LMP was 5
days ago. She uses an intrauterine dcvice for contraception and had coitus I week ago. There is no
history of nausea, vomiting, or diarrhea. Her vital signs are blood pressure 120/80 mmHg; pulse
100 bpm; and temperature 38.6oC. Abdominal examination shows bilateral lower quadrant
guarding ryith rebound tenderness on the right side. Pelvic examination shows pus at the cervical
os and a tender 6-cm right adnexal mass. Laboratory data are hematocrit 387o; white blood count
25,000/mL; and negative serum pregnancy test. Transvaginal sonography shows a 6-cm complex
right adnexal mass. The uterus and left adnexa are normal. Which of the following is the MOST
LIKELY diagnosis?
a) Appendicitis.
b) Adnexal torsion
c) Pyosalpinx.
d) Hydrosalpinx.
e) Endornetritis.
987- The single-dose parenteral drug of choice used in uncomplicated gonorrhea is:
a) Ofloxacin 400 mg.
b) Spectinornycin 500 tng.
c) Cefixiure 400 rng.
d) Ceftriaxone 250 mg.
e) Ciprofloxacin 500 mg.
988- Regarding syphilis, all the following are correct EXCEPT:
a) The organism is diagnosed by dark field illumination.
b) The classic finding in primary syphilis is a hard chancre.
c) Secondary syphilis is not associated with a rash over the hands and feet.
d) Gurnrnas are found in late syphilis.
e) Syphilis is produced by a spirochete.
989- Which character IS NOT applicable to vulval ulcer due to primary chancre:
a) It rnay be rounded.
b) It rnay be serpiginous.
c) The edges are sharp.
d) lt is painful.
e) The base is hard.
990- The COMMONEST site to be affected by genital tuberculosis is:
a) The ovaries.
b) The fallopian tubes.
c) The uterus.
d) The cervix.
e) The vagina.
WhiteKnightLove

991- The following about pelvic tuberculosis is $RECT:


a) It is most often bovine.
b) It is usually a printary lesion.
c) It is usually associated with regular menstruation.

-122-
'It is more blessed to give than to receive.

d) It rnostly involves the fallopian tubes.


e) It is decreasing in incidence.
992- The INCORRECT statement for tuberculosis of the genital tract:
a) The tubes are the commonest site of affection.
b) Infertility rnay be the only symptom.
c) Infection is usually sexually transmitted.
d) The treatment is mainly medical.
e) lt may be responsible for menstrual disorders.
993- A 33-year-old infertile patient complains of pelvic pain and 3 month-secondarT amenorrhea.
Physical examination demonstrates a tender pelvic mass. At laparoscopy findings included dense
pclvic adhesions, segmental dilatation of the fallopian tubes, and everted fimbria. Which is the
MOST PROBABLE diagnosis do these findings suggest?
a) Pelvic endometriosis.
b) Chronic non-specific pelvic inflarnmatory disease.
c) Pelvic tuberculosis.
d) Gonorrheal salpingitis.
e) Salpingitis isthmica nodosa.
994- The CORRECT statement for genital tuberculosis:
a) The most affected site is the ovaries.
b) The main route of transrnission is through sexual intercourse.
c) Pelvic masses are usually the leading clinical sign.
d) Negative tuberculin test excludes the presence ofthe disease.
e) Hysterectomy and bilateral salpingo-oophorectomy is the main therapeutic option.

Direction: Each set of matching questions in this section consists of a list of numbered items followed
by several lettered options. For each numbered item, select the ONE best lettered option that is most
closely associated with it. Each lettered heading may be selected once, more than once, or not at all.

For each of the following conditions numbered (995-999), choose the the appropriate pH lettered (a-b).
EACH LETTERED HEADING MAY BE USED ONCE, MORE THAN ONCE, OR NOT AT ALL.

995- Cervicalsecretions I a) Generally acidic.


996- Semen I b) Generally alkaline.
997- Urine
998- Menstruation
999- Normal vasinal DH

For each ofthe item numbered (1000-1004), choose the effect lettered (a-c).
EACH LETTERED HEADING MAY BE USED ONCE, MORE THAN ONCE, OR NOT AT ALL.

1000- Menopause a) Turns pH of the vagina more acidic.


001- Lactobacilli b) Turns pH of the vagina more alkaline.
002- Semen c) Keeps pH ofthe vagina constant.
003- Menstruation
004- Estroeen thera
WhiteKnightLove

-123-
Freely you have received; freely give.

For each of the following genital infections numbered (1005-1009), choose the MOST appropriate
chemotherapeutic agent lettered (a-g).
EACH LETTERED HEADING MAY BE USED ONCE, MORE THAN ONCE, OR NOT AT ALL.

005- Neisseria gonorrheae (Uncomplicated) a) Azithromycin.


006- Chlamydia trachomatis b) Itraconazle.
007- Gardnerella vaginalis c) Penicillin C.
008- Candida albicans d) Tetracycline.
009- Trichomonas vagina lis e) Metronidazole.

For each of the following genital infections numbered (I010-1015), choose the the diagnostic modality
lettrered (a-h).
EACH LETTERED HEADING MAY BE USED ONCE, MORE THAN ONCE, OR NOT AT ALL.

0l 0- Chlamydia infection a) Culture on Thayer Martin medium.


0l I- Candidal infection b) Culture on Dorset egg medium.
012- Gonorrhea infection c) Culture on Sabouraud medium,
0I3- Gardenerella vaginalis d) Culture on aerobic blood agar.
()I4. HPV e) Micro-immunofl uorescent test.
015. HIV l) Western blot.
g) PCR and Southern blot.
h) Whifftest.

For each of the descriptions of vaginitis numbered (1016-1018), select the therapy that would be most
appropriate lettered (a-e).
EACH LETTERED HEADING MAY BE SELECTED ONCE, MORE THAN ONCE OR NOT AT
ALL
016-A woman states that she has been on ampicillin for I week because of a la) Metronidazole.
urinary tract infection. Upon completing the antibiotics, she noted a thick, lb) Estrogencream.
white vaginal discharge with severe vulval itching. I c) Imidazole antifungal
017- A patient states that she has a malodorous discharge and intense itching. She I agent.
adds that her partner also has a slight discharge. Pelvic examination reveals I d) Vinegar douche.
"strawberry spots" on the cervix. I e) Sulfonamide vaginal
018- A patient complains of a watery, malodorous discharge with very little I cream.
itching or burning. A wet mount preparation in saline of the vaginal
secretions reveals clue cells.

WhiteKnightLove

-124-
'It is more blessed to give than to receive.

UROGYNECOLOGY AND GENITAL DISPLACEMENT

Direction: Select the ONE BEST lettered answer or completion in each question.

l0l9- The CORRECT statement for uterovaginal prolapse:


a) Syrnptoms are often appears after cesarean delivery.
b) tt may cause intestinal obstruction if there is a large rectocele.
c) The cervix is often elongated.
d) ln its acute stage, it is very painful condition.
e) It is more common in black races.
1020- Etiological factors of uterine prolapse include:
a) Nulliparity.
b) Bed rest without postnatal exercises at puerperium.
c) Crede maneuver for delivery of the placenta,
d) First degree perineal tears.
e) Acquired myopathies.
l02l- Factors that are $l predisposing to the development of uterovaginal prolapse include:
a) Congenital weakness of the supporting ligaments.
b) Postmenopausal atrophy.
c) Injury during childbirth.
d) Subrnucous uterine polyps.
e) Ovarian tumors.
1022- Genital prolapse is associated with the following EXCEPT:
a) Chronic constipation.
b) Prolonged second stage of labor.
c) Unrepaired
hidden perineal tear.
d) Inrproperty
applied forceps delivery.
e) The use of HRT at the menopause.
1023- Factors important to the development of genital prolapse include the following EXCEPT:
a) Poor tissue strength.
b) Residence at a high flat.
c) Chronic straining at bowel movements.
d) Menopause.
e) Childbirth trauma.
1024- The following statements regarding genitalprolapse are correct EXCEPT:
a) Dyschasia and piles may develop in association with genital prolapse.
b) Stress incontinence is a common urinary tract symptorn associated with prolapse.
c) Coital difficulties can be cotlrlon.
d) Menopause predisposes to genital prolapse.
e) Uterine prolapse is the commonest type of prolapse.
1025- The INCORRECT statement regarding genital prolapse:
a) Postnatal physiotherapy has a role in prophylaxis.
b) The use of intrauterine contraceptive device may predispose to uterine prolapse.
c) Pessary has a lirnited role in the treatment of prolapse in the current practice.
d) Abdorninal cervicopexy operation is among the surgical operations of choice especially if fertility is
to be maintained.
WhiteKnightLove

e) Recurrence ofprolapse occurs in 5 to l0%o ofcases after surgery.


1026- The INCORRECT statement regarding second degree uterine prolapse:
a) It is diagnosed when the cervix protrudes through the vulval orifice.
b) It is also known as complete procidentia.
-125-
Freely you have received; freely give.

c) It causes sacral backache.


d) It may precede vaginal wall prolapse.
e) It rnay be associated with rnenorrhagia.
1027- As regards cystocele the followings are true EXCEPT:
a) It is a prolapse of the bladder in upper part of the anterior vaginal wall.
b) Menopause is a risk factor for the development of cystocle.
c) It is the main cause of stress incontinence of urine.
d) It rnay lead to urinary tract infection.
e) It is uncommon in nulliparous women.
1028- The INCORRECT statement for cystocele:
a) It rnay be associated with stress incontinence.
b) Contains the base ofthe urinary bladder.
c) At the early stages, it might present with frequency of micturation.
d) It is treated by anterior colporrhaphy operation,
e) It is best treated with a ring pessary.
1029- The INCORRECT statement regarding rectal examination in the gynecological practice is that it
is:
a) Useful in the diagnosis of enterocele.
b) Useful in suspected pelvic abscess,
c) Indicated in the assessment of carcinorna of the cervix.
d) Not to be preformed as a routine in the gynecological practice.
e) Not useful in assessment of ectopic pregnancy.
1030- As regards entcrocele, the following is CORRECT:
a) lt is a pro)apse of the rectum.
b) It rnay occur following colposuspension.
c) Sigrnoidoscopy is used for diagnosis.
d) It may resolve spontaneously.
e) It is a colrrmon cause of stress incontinence.
l03l- The INCORRECT statement regarding enterocele:
a) It usually contains recturn.
b) It is lined by peritoneum.
c) It frequently contains srnall bowel.
d) It may develop as a long-tenn cornplication of vaginal hysterectomy.
e) Barium enerrra is not useful for diagnosis.
I032- An enterocelc is BEST characterized by which of the following statements?
a) It is not a true hernia.
b) It is a herniation ofthe bladder floor into the vagina.
c) It is a prolapse ofthe uterus and vaginal wall outside the body,
d) It is a herniation of the rectal and vaginal wall into the vagina.
e) It is a protrusion of the pelvic peritoneal sac and vaginal wall into the vagina.
1033- As regards anterior colporrhaphy, all the following are correct EXCEPT:
a) It rnay cause transient postoperative retention of urine.
b) It is the treatment of anterior vaginal wall prolapse.
c) It can be cornbined with vaginal hysterectomy.
d) It is better postponed in the presence ofurge incontinence.
e) It should be postponed until cornpletion of childbearing.
1034- The CORRECT statement about retroversion of the uterus, is that it:
a) Occurs in20Yo of normal women.
b) Is a colruron cause of infertility.
WhiteKnightLove

c) Should be corrected with a Hodge pessary in early pregnancy.


d) May be corrected by Fothergill operation.
e) Is caused by heavy lifting.

-126-
'It is more blessed to give than to receive.

1035- The INCORRECT statement about bacl<ache due to gynccological causes is that it:
a) May be caused by a fixed retrovefted uterus.
b) Is usually felt in the lumbar area.
c) May be due to uterine prolapse.
d) May be due to chronic pelvic infection.
e) May be caused by endometriosis.
1036- The Manchester repair DOES NOT includes:
a) Arnputation of the cervix.
b) Posterior colpoperineorrhaphy.
c) Anterior colponhaphy.
d) Dilation of the cervical canal.
e) Colposuspension.
1037- The most important suspensory mechanism of the urethra is:
a) Anterior pubourethral ligaments.
b) Intennediate pubourethral ligaments.
c) Posterior pubourethral ligarnents.
d) White line.
e) Pubocervical fascia.
1038- All the following statements about genuinc stress incontinence in the female are correct EXCEPT:
a) [t occurs transiently during pregnancy.
b) It is more common in parous women.
c) It can be corrected surgically.
d) Urodynamic studies are usually normal.
e) It is usually associated with utero-vaginal prolapse.
1039- The following IS NOT related to the etiology of overactive bladder:
a) Cystitis.
b) Urinary stones.
c) Cerebrovascular disease,
d) Multiple sclerosis.
e) Cornbined contraceptive pills,
1040- The follorving are treatment modalities for overactive bladder EXCEpT:
a) Srnooth rnuscle relaxants.
b) Parasympathomimetics.
c) Tricyclic antidepressants.
d) Anticholinergics.
e) Behavioral therapy.
l04l- Ove ractive bladder gets benefit from the following EXCEPT:
a) Suburethral sling operation.
b) Srnooth muscle relaxants.
c) Anticholenergic agents.
d) Bladder drill,
e) Pelvic floor muscle exercises.
1042- True incontinence occurs in all the following EXCEPT:
a) Vesicovaginal fistula.
b) Bilateral ureterovaginal fistula.
c) Unilateral ureterovaginal fistula.
d) Vesicocervicovaginal fistula.
e) Vesicourethrovaginal fi stula.
1043- Utcrovesical fistula usually presents with:
WhiteKnightLove

a) Amenorrhea.
b) Menuria.
c) Dysnrenorrhea.
d) Terrninal hematuria.

-127-
Freely you have received; freely give.

e) True incontinence.
1044- In the treatment of gcnuine stress incontinence (CSI) (urodynamic stress incontinence), the
following is INCORRECT:
a) Preoperative urethral pressure profilometry can predict the success of surgical treatment.
b) Physiotherapy will improve the tone of the urethral ligaments.
c) Patients with co-existent detrusor instability could have surgical treatment.
d) Vaginal surgery for stress incontinence carries a high failure rate.
e) The standard operation is the Burch operation.
1045- The INCORRECT statement in the treatment of urinary stress incontinence:
a) Non-surgical management is useless.
b) Reconstruction and elevation ofthe urethrovaginal angle cures the condition.
c) Co-existent detrusor overactivity should be treated before surgery for stress incontinence.
d) Abdonrinal approach for surgery is more successful than vaginal approach.
e) Burch operation (colposuspension) is carried out through an abdominal incision.
1046- The CORRECT statement for urethral caruncle is that it:
a) ls covered usually by transitional epithelium,
b) Is usually symptomatic.
c) May clinically resemble urethrocele.
d) Usually originates from the anterior wall of the urethra.
e) Occurs in children mainly.
1047- Continence of urine depends on the following:
a) Positive urethral closure pressure.
b) Negative urethral closure pressure.
c) Reversal ofthe controlled intraurethral pressure.
d) Negative intraperitoneal pressure.
e) Relaxed detrusor muscle.
10,t8- The normal pressure inside the urethra IS NOT related to:
a) The urethral tnucosal seal.
b) The submucosal cushion of veins.
c) The urethral coat of intrinsic smooth and striated muscles.
d) The pubocervical fascia.
e) The action of progesterone on the epithetlium of the urethral iumen.
1049- Causes of dl,suria include all of the following EXCEPT':
a) Urethritis, cystitis.
b) Stress incontinence.
c) Local trautna to the urethra.
d) Chemical irritation.
e) Menopausal atrophic changes.
1050- False incontinence DOES NOT indicate the following:
a) There is no incontinence ofurine.
b) Overflow incontinence.
c) Chronic urinary retention with bladder overdistension.
d) Urine dribbling out the urethra frorn the distended bladder.
e) Possible organic nervous disorders as diabetic neuropathy.
l05l- Stress incontinence IS NOT common in:
a) Nulliparous.
b) Multiparous.
c) Pregnancy.
ci) State of physical activity.
WhiteKnightLove

e) Chronic chest inf'ection.


1052- Thc first stcp for thc detection ofstress incontinence:
a) Urodynamic study.
b) Clinical examination to demonstrate the incontinence.

-128-
'It is more blessed to give than to receive.

c) Intravenous pylography.
d) Midstrearn urine analysis.
e) Urethrocystoscopy.
1053- A 65-year-old woman complains of leakage of urine. Which of the following is THE MOST
COMMON cause of this condition in such patient?
a) Anatornic stress urinary incontinence.
b) Urethral diverticulum,
c) Overflow incontinence.
d) Unstable bladder.
e) Fistula.
1054- Non-surgical conservative management of urinary stress incontinence DOES NOT include:
a) Weight loss for obese patients.
b) Cessation of srnoking.
c) Kegle's exercises.
d) Vaginal cones.
e) Urinary antiseptics.

Direction: Each set of matching questions in this section consists of a list of numbered items followed
by several lettered options. For each numbered item, select the ONE best lettered option that is most
closely associated with it. Each lettered heading may be selected once, more than once, or not at all.

For each ofthe follorving diseases (1055-1059), choose the most suitable operation lettered (a-f).
EACH LETTER.ED HEADING MAY BE USED ONCE, MORE THAN ONCE, OR NOT AT ALL.

055- Second degree uterine descent in 32-year-old woman I a) Sacrospinous colpopexy.


rvants to preserve her potential fertility. I b) Anterior colponhaphy.
056- Second degree uterinc descent in 75-year-old-woman. I c) Burch colposuspension.
057- Stress (urodynamic) incontinence. I d) Vaginal hysterectomy with vault
058- Vault prolapse. I suspension.
059- Cystocele with midlinc defect. I e) Fothergill's operation.
f) Sacral cervicopexy (Sling operation).

For each of the following disease intity numbered (1060-1064), choose the common presentation
lettered (a-f).
EACH LETTERED HEADING MAY BE USED ONCE, MORE THAN ONCE, OR NOT AT ALL.

060- Vesicovaginal fistula I a) True urinary incontinence.


061- Uterovesical fistula I b) Menuria.
062- Left ureterovaginal fistula I c) Urge incontinence.
063- Genuine stress incontinence I d) Parlial urinary incontinence.
064- Overactive bladder I e) Overflow urinary incontinence.
l) Urinary incontinence at sexual penetration.
For each of the follorving operation numbered (1065-1069), choose the disease entity for which the
operation is indicated lettered (a-d).
EACH LETTERED HEADING MAY BE USED ONCE, MORE THAN ONCE, OR NOT AT ALL.

065- Sims dedoublement operation a) Urodynarnic stress incontinence.


WhiteKnightLove

066- Burch colposuspension b) Overactive bladder.


067- Marshal Marchetti Krantz operation c) Vesicovaginal fistula.
068- Clam cystoplasty d) Vault vesical fistula.
069- Latzko operation (partial colpocleisis)

-129-
Freely you have received; freely give.

GYNECOLOGICAL ONCOLOGY

Direction: Select the ONE BEST lettered answer or completion in each question.

1070- The INCORRECT statement regarding vulval intraepithelial neoplasia:


a) Paget disease is considered a variant of VIN.
b) VINI can not be visible macroscopically.
c) Diagnosis of VIN is best archived by cytology and colposcopy.
d) VIN can be treated by CO2 laser to a depth of 3 mm.
e) VIN can be treated by wide local excision to a depth of 5 mrn.
l07l- The INCORRECT statement regarding invasive cancer of the vulva are correct:
a) It is usually squamous in type.
b) Generally, prognosis is better than with cancer ovary.
c) It could be associated with other genital malignancies.
d) Local vulvectomy is not a line of treatment.
e) Pain is an early symptom.
1072- Differential diagnosis of vulval cancer DO NOT include:
a) Granuloma inguinale.
b) Syphilis.
c) Herpes simplex type II infection.
d) Barlholin cyst.
e) Condylorna accurninata.
1073- The COMMONEST vulval malignancy is:
a) Adenocarcinoma.
b) Squarnous cell carcinoma.
c) Basal cell carcinorna.
d) Melanosarcoma.
e) Leiornyosarcoma.
1074- Carcinoma of the vulva:
a) Usually ulcerates even ifnot advanced.
b) Is usually histologically anaplastic.
c) Spreads initially to iliac nodes via vaginal lymphatics.
d) Seldorn involves lynrph nodes at the time of presentation.
e) ls equal Iy amenable to treatment by surgery and radiotherapy.
1075- Risk factor for cancer vulva is:
a) HIV infection.
b) Vulval dystrophy.
c) Lichen sclerosis,
d) Herpes sirnplex.
e) Bartholin abscess.
1076- Treatment of cancer of the vulva includes all the following EXCEPT:
a) Radical excision of the vulva.
b) Dissection of fernoral triangle,
c) Unilateral or bilateral inguinal lyrnphadenectomy.
d) Pre-operative radiation.
e) Postoperative methotrexate.
WhiteKnightLove

1077- Malignant melanoma represents what percent of the whole cancer vulva occurring?
a) l%.
b) 3%.
c) 5%.

-130-
'It is more blessed to give than to receive.

d) t0%.
e) 15%.
1078- Regarding cervical intraepithelial neoplasia (CIN), all the following are correct EXCEPT:
a) Transformation zone is the most susceptible location for development.
b) It is usually unifocal.
c) Abnormal vascular pattem seen by colposcopy include punctuation and mosaicism.
d) Total hysterectomy is a line of management.
e) Large loop excision of transformation zone is a common line of management.
1079- Definitive final diagnosis of cervical intraepithelial neoplasia III (CIN III) is made by:
a) Cervical smear.
b) Colposcopy.
c) Histopathology.
d) Tumor markers.
e) Schiller iodine stain.
1080- High risk factors of cervical intraepithelial neoplasia (CIN) include the following EXCEPT:
a) HPV types 6 and I 1.
b) Hrv.
c) Smoking.
d) First sexual intercourse before 18 years age.
e) Multiple sexual partners,
1081- Pattern of CIN lesions DO NOT include:
a) Regular surface contour.
b) Stain brown with Lugol iodine.
c) A marked acetowhite appearance.
d) Coarse epithelial punctuations.
e) Multisector involvement of the transformation zone.
1082- The CORRECT statement for CIN III lesions extending into the cervical canal is that they are:
a) Often invasive.
b) Best treated by radiotherapy.
c) Best treated by total hysterectomy.
d) Safely treated by directed biopsy.
e) Safely treated by large loop excision ofthe transformation zone.
1083- A 40-year-old woman with hypertrophic, clinically suspicious cervix and a negative smear, the
BEST NEXT STEP is:
a) Previous electrodiathermy is of little relevance to the follow up,
b) Post-coital bleeding is not a relevant symptom.
c) The negative smear is reassuring.
d) Outpatient punch biopsy is not a satisfactory technique.
e) Cone biopsy is needed under general anesthesia.
1084- In diagnosis of CIN the following are true EXCEPT:
a) Pap smear is best method for screening.
b) High grade squamous intraepithelial lesions include CIN II and III.
c) Low grade squamous intraepithelial lesions include CIN I and flat condyloma.
d) Any abnormal Pap smear is an indication for immediate cervical biopsy.
e) Abnormal Pap smear is an indication for colposcopic examination and directed biopsy.
1085- A 39-year-old, para 3, presents with an abnormal Pap smear, On colposcopy, abnormal vfscular
pattern was seen including punctuation and mosaicism. The definitive final diagnosis of her
condition is made by:
a) Histopathology.
WhiteKnightLove

b) Tumor markers.
c) Schiller iodine stain.
d) Any ofthe above.
e) None of the above.

-l3r-
Freely you have received; freely give.

1086- In assessing the lristopathological type of cervical intraepitheliat neoplasia (CIN), the fotlowing
factors are tahen into account EXCEPT:
a) Numbers of mitotic figures.
b) The nuclear-cytoplasmic ratio.
c) Epithelial differentiation.
d) Crypt involvement is not a serious finding.
e) The presence of human papillorna virus (HPV).
1087- The following statements are true of Iarge loop excision of the cervical transformation zone
(LLETZ) EXCEPT:
a) lt is used even when a CO2 laser is available,
b) It requires anesthesia.
c) It is a cheap and effective way of rernoving the cervical transformation zone.
d) Secondary hernorrhage is a rare complication.
e) Cervical stenosis may occur.
1088- The CORRECT statement regarding cancer cervix:
a) Adenocarcinoma occurs in about 30%o ofcases.
b) It is the most colnrrron cancer arnong women.
c) It is rrrore colrmon in smokers.
d) It does not spread by direct infiltration.
e) It is more common in nulliparous women.
1089- The INCORRECT statement regarding carcinoma of the cervix:
a) AdenosquarTrous and tumors of mixed histological type rnostly arise from transition zone epithelium.
b) Ninety per cent are pure squarnous cell tumors.
c) Mixed adenosquamous tumors are associated with poorer survival rates than pure adenocarcinomas.
d) The presence of vascular space permeation is a prognostic indicator independent of lymph node
status.
e) The risk of lyrnph node metastases in women with rnicroinvasive disease is l0%.
1090- The INCORRECT statement regarding FIGO staging of cancer cervix:
a) Stage IIla denotes involvement of the lower l/3 of the vagina.
b) Stage IB denotes clinically evident cancer with possible lymph node involvement.
c) Cystoscopy is essential for proper staging.
d) Rectal examination is useful in the staging systern.
e) It is a surgical staging system.
l09l- The CORRECT statement regarding stage II carcinoma of the cervix:
a) The upper third of the vagina may be involved.
b) The tumor is fixed to the lateral pelvic wall.
c) It does not involve extension into the body ofthe uterus.
d) A five-year survival rate of 80% can be expected.
e) The growth is confined to the cervix.
1092- Clinical evidences of cervical cancer may include all of the following EXCEPT:
a) Perinrenopausal bleeding.
b) Urernia.
c) Postmenopausal bleeding.
d) Gastric upset.
e) Sevele pelvic pain.
1093- An intravenous pyelogram (lVP) shows hydronephrosis in the workup of a patient with cervical
cancer othcrwise confined to a cervix of normal sizc. This indicates which one of the following
sta ges ?

a) Microinvasive stage.
WhiteKnightLove

b) t.
c) ll.
d) rrr.
e) IVb.

-132-
'It is more blessed to give than to receive.

1094- A 5l-year-old \ryoman is diagnosed with invasive cervical carcinoma by cone biopsy Pelvic
examination and rectovaginal examination reveals the parametrium to be free of disease, but the
upper portion of thc vagina is involved with tumor. Intravenous pyelography (lVP) and
sigmoidoscopy are negative, but a computed tomography (CT) scan of the abdomen and pelvis
shows grossly enlarged pelvic and periaortic nodes.'I'his patient is classified at which of the
following stages?
a) IIa.
b) ilb.
c) Illa.
d) ilrb.
e) lV.
1095- The INCOITRECT statement regarding the prognosis of cervical cancer:
a) Recurrence occurs in35% of cases.
b) The majority of recurrence occurs after 3 years of treatment.
c) The 5 year- survival for stage Il is 50%.
d) Surgical management of local recurrence following radical surgery is disappointing,
e) In stages IB & II A, there is little difference between results of surgery & radiotherapy.
1096- ThelNCORRECTstatementregardingpatientwithstagelbcarcinomaofthecervixundergoesa
radical hysterectomy and pelvic lymphadenectomy:
a) Prophylactic heparin is mandatory.
b) Irradiation is an alternative comparable therapeutic modality.
c) Ureteric fistulae are usually due to intra-operative surgical trauma,
d) Significant long-term bladder dysfunction is conrmon.
e) Pelvic lyrnphocyst fornratiorr is characteristically a late complication.
1097- Microinvasion of carcinoma of the cervix involves a depth below the base of the epithelium of no
more than:
a) I mm.
b) 2 mrn.
c) 3 rnrn,
d) 4 mrn.
e) 5 rnrn.
1098- Carcinoma in situ of the cervix precedes invasive for an avcrage period oft
a) I year.
b) 2 years.
c) 5 years,
d) l0 years.
e) I 5 years.
I 099- Invasive canccr of the cervix:
a) Worldwide, it is the commonest malignant tumor in women.
b) It occurs most cornrnonly in women under the age of 40 years.
c) It is usually of squarnous type,
d) lt occurs less commonly in smokers.
e) It is uncommon in developing countries.
ll00- ThecommonpresentationsofcarcinomaofthecervixincludethefollowingsEXCEPT:
a) An asymptomatic abnorrnal smear.
b) Pelvic pain.
c) Vaginal dlscharge.
d) Postcoital bleeding.
e) Intermenstrual bleeding.
WhiteKnightLove

-133-
Freely you have received; freely give.

I l0l- The follorving statements concerning endometrial cancer are correct EXCEPT:
a) lnvolvernent of the cervix occurs in stage ll disease.
b) Myornetrial invasion of a depth more than 50% puts the case into stage II.
c) Initial treatrnent of stage I disease is by total abdorrinal hysterectomy and bilateral salpingo-
oophorectomy.
d) Postoperative radiotherapy is rarely required in stage IA disease.
e) Five-year survival is90%o in stage l.
I 102- Which IS NOT a risk factor for the development of endometrial cancer:
a) Obesity.
b) Tamoxifen use.
c) Srnoking.
d) Polycystic ovarian disease and amenorrhea,
e) Heredity.
1103- The risk factors for the development of adenocarcinoma of the endometrium DO NOT include:
a) Lifelong obesity.
b) Multiparity.
c) Chronic anovulation.
d) Late menopause.
e) Tamoxifen.
I104- The INCORRECT statement for endometrial carcinoma:
a) 5% of cases occur in wolren below the age of 40 years.
b) Obesity is a recognized risk factor.
c) 80% of women present with postmenopausal bleeding.
d) Universal screening is not advisable.
e) Treatment is essentially by radiotherapy,
I105- The CORRECT statement regarding endometrial canccr:
a) lt constitules 5o/o of all gynecological malignancies.
b) The cornrronest type is adenosquamous.
c) Pelvic nodes are involved in 5% of poorly differentiated cases.
d) Papillary serous histopathological type has a better prognosis than adenocarcinoma,
e) lt has a greater tendency to metastasize if it involves the lower uterus than the fundus.
I106- Incidence of the following is increased with tamoxifen use EXCEPT:
a) Endometrial polyp.
b) Twin pregnancy.
c) Endometrial adenocarcinotna.
d) Endometrial hyperplasia.
e) Uterine fibroids.
I107- Endometrial carcinoma:
a) Typically presents with interrnenstrual bleeding.
b) ls a sequel to prenatal estrogen treannent.
c) ls associated rvith use of the COCP f,or more than l0 years.
d) May involve the para-aorlic nodes without pelvic node involvelnent.
ll08- Womcn are at high risk for endometrial carcinoma if they have one or more of the following
characteristics EXCEPT:
a) Hyperlension.
b) Diabetes.
c) Smoking.
d) Obesity.
e) Farnilial history of endometrial carcinoma.
WhiteKnightLove

I109- The WRONG statcmcnt, about endometrial carcinoma is that it is:


a) Seen lnost corrlronly in the 65- to 75-year age group.
b) Very rare in wornen under the age of 40 years.
c) More aggressive in postmenopausal wolnen.

-134-
'It is more blessed to give than to receive.

d) Frequentll, seen irr cases ol untreated Turner's syndronre.


e) Related to pol;'cystic ova;r' svndrotne.
I I l0- All the follorving statenrents rcgarding cndomctrial carcinonra are true EXCEPT:
a) 80% ofcases present \\,ith postnrenopausal bleeding.
b) It nray irrclude areas of benign squalxous epitheliurn.
c) It rnay include areas of malignant squamous epithelium.
d) It is the comrnonest tumor of the endornetriurn.
e) Treatrnent is essentially by radiotherapy,
III l- Endometrial cancer with vaginal metastasis is staged:
a) II b.
b) Ill a.
c) lll b.
d) III c.
e) lV a.
I I l2- In endomctrial carcinoma, progestogen therapy is useful in the following circumstances
EXCEPT:
a) Well-diff'erentiated endometrial carcinoma.
b) Steroid receptor negative endonietrial carcinoma.
c) As an adjLrvant to surgery.
d) Recurrent endometrial carcinorna.
e) In the treatment of pulrnorrary rnetastases.
1ll3- Radiotherapy technique that IS NOT used in the managemcnt of endometrial carcinoma:
a) Preoperative radiotherapy.
b) Postoperative adjuvant therapy.
c) As prirnary treatrnent when surgey is contrainidicated.
d) Pelvic side wall recurrence.
e) Vaginal vault recurrence after hysterectomy.
I I l4- The definitive diagnosis of leiomyosarcoma of the uterus requires at least how many mitotic cells
per high power field (HPF)?
a) l.
b) 4.
c) 10.
d) r6.
e) 20.
II l5- Which of the follorving statements about ovarian tumors in premenarcheal girls is CORRECT?
a) The malignancy rate is low.
b) The majority are of celornic epithelial origin.
c) Ovarian neoplasias are more colnmon before than after puberty.
d) Most ovarian enlargernent in the newborn is functional cysts.
e) Gernr turnors are a frequent cause of precocious puberty.
I I l6- A risk factor for canccr ovary is:
a) The use ofprogesterone only injectable contraception.
b) The use oforal contraceptive pills.
c) Farnily history of cancer colon.
d) Serous ovarian cyst.
e) Multiparity.
II l7- Clinical prcsentation is usually at Stagc III of the disease for the following type of malignancy:
a) Adenocarcinorna of the endonretriurn.
b) Squaurous cell carcinorna of the cervix.
WhiteKnightLove

c) Ovarian rnalignancy.
d) Squamous cell carcinoma of the vulva.
e) Malignant diseases of the vagina.

- 135-
Freely you have received; freely give.

1ll8- The following findings make a diagnosis of malignancy more likely in a woman with an oyarian
tumor EXCEPT:
a) Farnily history of ovarian cancer.
b) Previous use ofthe contraceptive pills.
c) The presence of ascites.
d) Bilaterality of the tumor.
e) Prolonged use ofovulation inducing drugs.
I I l9- The risk factors for the development of ovarian malignancy DO NOT inctude:
a) Positive farnily history.
b) Advancing age.
c) Chronic anovulation.
d) Nulliparity,
1120- The following criteria are in favor of diagnosis of ovarian cancer during laparotomy EXCEPT:
a) Huge ovarian cyst,
b) Extra cystic papillae.
c) Areas of hemorrhage and necrosis.
d) Heterogeneous consistency.
e) Limited mobility.
l12l- The follorving criteria at Iaparotomy for ovarian cysts are in favor of malignancy EXCEPT:
a) Huge size of the ovarian cyst.
b) Papillae on the outer surface.
c) Areas of hemorrhage and necrosis.
d) Nodularity of the olnentum.
e) Peritoneal adhesions.
ll22- The follorving ovarian tumor is always malignant:
a) Myxorna peritonei.
b) Endodenral sinus tumor.
c) Solid teratoma,
d) Granulosa cell tumors.
e) Brenner tumors.
ll23- The major mode of spread of ovarian neoplasms is by:
a) Ovarian veins.
b) Ovarian lymphatics.
c) Pelvic lyrnphatics.
d) Local extension.
e) Peritoneal seeding.
ll24- The follorving is an epithelial ovarian tumor EXCEPT:
a) Endor.netrioid adenocarcinorna.
b) Transitional cell turnor.
c) Brenner's tumor.
d) Thecoma.
e) Mucinous cystadenocarcinoma.
I125- The INCORRECT statemcnt regarding CA 125:
a) Its level is elevated in endometriosis.
b) Elevated levels are rnore specific to rnucinous ovarian rnalignancy than serous ovarian rnalignancy.
c) Elevated levels in rnalignancy are related to the bulk of tumor tissue.
d) Its level rnay be elevated during normal pregnancy.
e) It is useful in follow up to detect recurrence of ovarian malignancy.
ll26- All the following statements about carcinoma of the ovary are correct EXCEPT:
WhiteKnightLove

a) Cancers of epithelial origin comprise 90% of total primary ovarian malignancies.


b) Borderline tulnors may lnetastasize but do not invade adjacent tissue.
c) Transcelorxic rnetastases are a coftlrron finding at laparotomy.
d) HRT lnay reasonably be prescribed after surgery of ovarian cancer.

-136-
'It is more blessed to give than to receive.

e) It is predisposed to by prolonged use ofthe oral contraceptive pill.


I 127- As regards the spread of epithelial ovarian cancers, all the following are correct EXCEPT:
a) Spread via the blood stream occurs early in the disease,
b) Spread to para-aortic lyrnph nodes puts the case at stage IIL
c) Spread to the underside of the diaphragm is common.
d) Spread around the peritoneal cavity has usually occurred by the time of diagnosis.
e) Spread to the onrenturn frequently occurs.
I128- Important prognostic factors concerning ovarian epithelial carcinoma include the following
EXCEPT:
a) Volume of the tumor.
b) Affection of ovarian stroma.
c) Extent of the tumor,
d) Histologicaldifferentiation of the tumor.
e) Presence ofascites.
ll29- TheMOSTCOMMONprimarysite foratypical secondariesmetastasizingtotheovaryis:
a) Stornach.
b) Lung.
c) Colon.
d) Breast.
e) Esophagus.
I 130- Elevated o fetoprotein is used for detection of:
a) Cancer vulva.
b) Cancer cervix.
c) Endodennal sinus tumor.
d) Serous cystadenocarcinoma.
e) Mucinous cystadenocarcinoma.
I I31- Tumor markers are most reliable for:
a) Serous cystadenocarcinoma of ovary.
b) Granulosa cell tumor.
c) Derrnoid cysts.
d) Cancer cervix.
e) Cancer vulva,
I132- Tumor marker CA-125 is most_reliable for:
a) Epithelial ovarian carcinoma.
b) Cranulosa cel I tumor.
c) Endoderrnal sinus tuuror.
d) Cancer cervix.
e) Cancer vulva.
I133- The ovarian tumor that is BEST responds to radiotherapy is:
a) Serous cystadenocarcinoma.
b) Mucinous cystadenocarcinorna.
c) Transitional adenocarcinoma.
d) Dysgerminorna.
e) Granulosa cell tuuror.
I134- Thc INCORRECT statement for ovarian cancer:
a) Advanced disease is seen in rnore than 50Yo of cases.
b) For all stages, the five-year survival is less than 30%.
c) Survival is related to the residual tumor present postoperatively.
d) The prevelance ofthe disease drops after the age of65 years.
WhiteKnightLove

e) Combined oral contraceptive pill for more than 5 years is not a risk factor.

-137-
Freely you have received; freely give.

1135- The INCORRECT statement regarding ovarian canccr:


a) Advanced disease is seen in more than 50% of cases;
b) Survival is not related to the residual tumor present postoperatively.
c) For all stages, the five-year survival is less than 30%.
d) Positive family history is a significant risk factor.
e) History of breast cancer increases the risk for development of ovarian cancer.
I136- Cancer ovary staging depends mainly upon:
a) Surgical exploration.
b) U/S examination.
c) Tumor markers.
d) MRI examination.
e) Clinical examination.
I137- Cancer ovary commonly presents by:
a) Menorrhagia.
b) Metrorrhagia.
c) Gastrointestinal upset.
d) Urinary retention,
e) Change in voice.
I138- The INCORRECT statement for dysgerminoma is that it:
a) Is commoner in younger than older rvomen.
b) Frequently presents with abdominal pain.
c) Is common in patients with testicular feminization syndrome.
d) Is radiosensitive.
e) Is bilateral in 50% ofcases.
I 139- Thc INCORRECT statement for dysgerminoma of ovary is that:
a) It metastasizes only by lymphatics.
b) The survival rate is high with proper treatment.
c) lt is radio-sensitive.
d) It may produce alphafetoprotein (AFP).
e) It is bilateral in l0oh of cases.
ll40- The CORRECT statement regarding fibroma of the ovary is that it:
a) Commonly presents as genital bleeding.
b) Frequently becomes malignant.
c) Is usually cystic.
d) Often contains teeth.
e) May be associated with a hydrothorax.
1l4t- A 45-year-old woman is found to have an 8-cm adnexal mass, ascites, and bilateral pleural
effusions. The level of the serum tumor markers are within normal. Which of the following is the
MOST LIKELY diagnosis?
a) Endometrioma.
b) Benign teratoma.
c) Papillary serous cystadenoma.
d) Ovarian fibroma.
e) Theca lutein cysts.
I142- Which tumor of the ovary is commonly hormonc secreting?
a) Serous cystadenomas.
b) Granulosa cell tumors.
c) Transitional cell tumors.
d) Clear cell adenocarcinoma.
WhiteKnightLove

e) Mature cystic teratomas.

-138-
'It is more blessed to give than to receive.

I143- The ovarian tumor that IS NOT associated rvith Meig syndrome:
a) Teratoma.
b) Thecoma.
c) Granulosa cell tumors.
d) Brenner tumor.
e) Fibrorna.
1144- Chemotherapeutic agents used to treat epithelial ovarian cancers:
a) Are more effective after cytoreductive surgery.
b) Are best given continually for six months.
c) Are best comined with radiotherapy for most of the cases.
d) Usually result in a cure.
e) Should be used for all stages ofthe disease.
ll45- A PG presents, while pregnant at 16 wks, with an oyarian cyst measured 18X15 cm that was
discovered accidentally during routine ultrasound. Before surgery to remove this cyst, she should
be counseled that:
a) This is rnostly a germ cell tumor.
b) Malignant change is a possibility.
c) Rupture of the cyst carries the risk of peritonitis.
d) Conservative follow up is not an option.
e) All of the above.
1146- A 67-year-old woman who had been in good health until approximately 6 months previously,
prcscnts rvith increasing abdominal discomfort, persistent abdominal fullness. Menopause was at
thc age of 55 years and she has no past history of disorders of pregnancy or gynecologic diseases.
The patient is married and has 2 daughters. Examination of the abdomen shorvs obvious
abdominal distention. A fluid wavc is present, rvith shifting dullness. No masses are appreciated.
Recto-vaginal examination demonstrates a largc, firm mass in the right adnexa and nodularity in
the cul-dc-sac. The limits of the mass cannot be defined. A computed tomography (CT) scan
revcals a 12-cm mass of heterogeneous texture in the right ovary and a large number of ascites is
present. What is the MOST AIIBQPRIATE next step?
a) Laparoscopy.
b) Estimation of tumor rnarkers.
c) MRI.
d) Vaginal smear for cytology.
e) Ultrasound examination.
lL47- A,63-year-old lvoman has bloating associated with tightening of her clothing around her abdomen.
She recently has developed dyspepsia and has lost 6 kg unintentionally. She is short of breath.
Abdominal percussion causes a wavelike movement of fluid around a central tympanitic area.
Pelvic examination demonstrates a fixed, irregular nodular adnexal mass with cul-de-sac
nodularity. A chest radiogram shorvs bilateral pleural effusion. Which of the following is the
MOST LIKELY diagnosis?
a) Cervical carcinoma.
b) Ovarian carcinoma.
c) Uterine leiomyoma.
d) Uterine sarcoma.
e) Endometrial carcinoma.
I148- A tumor marl<er that is useful in screening for epithelial cancer:
a) CA 19.9.
b) cAr25.
c) Alkaline phosphatase.
WhiteKnightLove

d) Alpha feto-protein.
e) LDH.

-139-
Freely you have received; freely give.

Direction: Each sct of matching questions in this scction consists of a list of numberecl itcms follou,cd
by sevcral lettercd options. For each numbercd itcm, select the ONE bcst lettered option that is nrost
closely associated rvith it. Each lettered heading may be selected once, more than once, or not at all.

For each of the following genital malignancy numbered (1149-1153), choose thc mean age of
occu rrence lettered (a-d).
EACH LETTERED TIEADING MAY BE USED ONCE, MORE THAN OIYCE, OR NOT AT ALL.

149- Endometrial cancer a) 4-5 years.


150- Sarcoma botryoids b) I 0-20 years.
151- Ccrvical cancer c) 45-50 years.
I152- Vulval canccr d) 57-65 years.
153- Dysgerminoma

For cach of the follotving malignant tumor numbcrcd (1154-1159), choose the main typc of sprcad
lettercd (a-d).
BACH LETTERED HEADING MAY BE USED ONCE, MORE THAN ONCE, OR NOT AT ALL.

154- Serous cystadenocarcinoma of the ovary | a) Tumor is locally malignant.


155- Dysgerminoma of the ovary b) Lymphatic.
156- Malignant melanoma of the vulva c) Hematogenous.
157- Basal cell carcinoma of the vulva d) Transcelomic.
I158- Squamous cell carcinoma of the vulva
159- Squamous cell carcinoma of the cervix

For cach of the follorving findings numbered (1160-1164), choosc the least stage of the tumor lettcred
(a-h).
EACH LETTERED HEADING MAY BE USED ONCE, MORE THAN ONCE, OR NOT AT ALL.

160- Tumor size of 5 cm which is confined to the cervix. a) Stage I A.


l6l- Tumor of size 3 cm which extends to the pelvic wall. b) Stage I Bl.
162- Tumor size of 4 cm with multiple hepatic secondaries. c) Stage I 82.
163- Tumor size of 4 cm with bilateral hydronephrosis. d) Stage IIA.
164- Tumor size of 2 cm with affection of 2 cm of the vaginal e) Stage IIB.
vault. f) Stage III.
g) Stage IVA.
h) Stage IVB.

WhiteKnightLove

-140-
'It is more blessed to give than to receive.

REPRODUCTIVE ENDOCRINOLOGY AND INFERTILITY

Direction: Select the ONE BEST lettered answer or cornpletion in each question.

I165- The follorving is correct about gonadotrophin releasing hormone (GnRH) analogues:
a) They are given continuously in vitro fertilization cycles (lVF) to stimulate follicular groMh.
b) They may cause endometrial hyperplasia.
c) They can be given intramuscular.
d) Their long tenn use increases bone density.
e) Progesterone back-up is better used with prolonged administration.
1166- The INCORRECT statement as regards gonadotrophin releasing hormone (GnRH):
a) It can be used for induction of ovulation in patients rvith hypogonadotrophic hypogonadism.
b) It is indicated in treatment of infertility associated rvith Kallman's syndrome.
c) It induces a state of hyperestogenism.
d) It has ferv multiple pregnancy rates cornpared with human menopausal gonadotrophin therapy
(HMG),
e) The risk of ovarian hyperstirnulation is absent.
ll67- The INCORRECT statement regarding ovarian cycle:
a) At puberty, each ovary contains two million primordial follicles.
b) It is usually characterized with a dorninant follicle at ovulation measuring 20mm.
c) It conrmences with each primordial follicle having an oocyte arrested in prophase of meiosis.
d) ln absence ofpregnancy, corpus luteurn degenerates to a corpus albicans.
e) The LH surge is essential for the process ofovulation.
ll68- Thc INCORRECT statement regarding the corpus luteum in the menstrualcycle is that it:
a) Rernains active for 14 days.
b) Secretes progesterone.
c) Is maintained by gonadotrophin.
d) Secretes estrogen.
e) Secretes pregnanediol.
ll69- The CORRECT statement regarding the first half of the reproductivc cycle:
a) Some ovarian follicles degenerate.
b) Meiotic figures are seen in the endometrium.
c) The corpus luteum begins to degenerate.
d) Serum progesterone levels are high.
e) The endometrium is rich in glycogen.
I170- As regards the anterior pituitary, all the following are correct EXCEPT:
a) It develops from the alimentary tract,
b) Dopamine reduces prolactin release from the pituitary.
c) It is controlled by releasing factors produced in the hypothalamus.
d) In short feedback loop: gonadotrophins reduce GnRH release
e) It lies above the optic chiasma.
I l7l- As regards pituitary follicle stimulating hormone, all the follorving are corrcct EXCEPT that it:
a) Is a glycoprotein,
b) Is excreted in increased amount at menopause.
c) Stimulates spermatogenesis in male.
d) With LH stirnulate oestrogen secretion and ovulation.
WhiteKnightLove

e) Is identical with hurnan chorionic gonadotrophin.

-l4t-
Freely you have received; freely give.

I172- Folliclc-stimulating hormone is elaborated by:


a) Chrornophobe cells of the adenohypophysis.
b) Basophilic cells of the adenohypophysis.
c) Acidophilic cells of the adenohypophysis.
a) Follicular granulosa cells.
b) Follicular cumulus cells.
I173- As regards follicle stimulating hormone, allthe following are correct EXCEPT:
a) It stimulates spermatogenesis.
b) Its plasma concentration is high in Klinefelter syndrome.
c) It stirnulates ovarian estrogen production.
d) lt is secreted by basophilic cells ofthe adenohypophysis.
e) It prevents regression of the corpus luteum.
ll74- As regards luteinizing hormone, all the following are true EXCEPT:
a) Its surge coincides rvith the second peak ofFSH just before ovulation.
b) lt is the stimulus for ovulation.
c) It stimulates the synthesis of testosterone in the male.
d) Its release is stirnulated by thyrotrophin releasing hormone.
e) It is a water soluble glycoprotein.
1175- The INCORRECT staternent regarding luteinizing hormone:
a) It stimulates androgen production in males.
b) It stirnulates luteinztion ofthe granulose cells.
c) Its plasma concentrations are decreased in ovarian failure.
d) It is a glycoprotein.
e) It is released in pulsatile fashion.
1176- The hormone primarily responsible for production of sex steroids iS:
a) FSH.
b) LH.
c) ACTH.
d)'fhyroid-stirnulating hormone (TSH).
e) Prolactin.
I177- As regards luteinizing hormone the followings are true EXCEPT:
a) It stirnulates androgen production.
b) Its plasrna concentration is increased in pregnancy.
c) Its plasrna concentrations are increased in ovarian failure.
d) Its surge coincides with the second peak of FSH just before ovulation.
e) It stimulates progesterone synthesis in the corpus luteum.
I178- The functions of mid-cycle LH surge include the following EXCEPT that it:
a) Enhances thecal cell androgen production,
b) Luteinizes granulosa cells.
c) [nitiates resumption of meiosis.
d) Facilitates oocyte expulsion.
e) Produces luteolysis.
ll79- According to the two cell theory, the site of production of follicular estrogen is:
a) Theca cells.
b) Stromal cells.
c) Granulosa cells.
d) Endometriurn.
e) Luteinized granulose cells.
I180- As regards ovulation in thc human, all the following are true EXCEPT that it is:
WhiteKnightLove

a) Associated with a surge of luteinizing hormone.


b) Characteristically followed by the development of secretory endometrium.
c) Associated with an increase in motility of the fallopian tube.
d) Associated with a sustained raise in basal body temperature.

-142-
'It is more blessed to give than to receive.

e) Followed by a rise in urinary pregnantriol.


I l8l- As regards ovulation in the human, all the follotving rrc truc EXCEPT:
a) It is associated with a surge of luteinising hornrone.
b) It is followed by secretory endometrium.
c) The dominant follicle at ovulation measures about 20mm,
d) It is associated with a sustained fall in basal body temperature.
e) LH surge coincides with the2ed peak of FSH just before ovulation.
1182- Thc main form of circulating estrogen in ndult human femalc during reproductive !ife:
a) Estrone.
b) Estradiol.
c) Estriol.
d) Ethinyl estradiol.
e) Sex hormone-binding globulin (SHBG).
I183- As regards estrogen, all the following arc true EXCEPT:
a) Estradiol is the rnajor hormone in the human,
b) tt is produced in the corpus luteunr.
c) It inhibits follicle stimulating honnone secretion by the anterior pituitary,
d) It is the dominant gonadal hormone at puberty.
e) It is responsible for secretory changes in the endometrium.
I184- As regards estrogen, all thc followings are true EXCEPT:
a) It is rnainly secreted by the ovary as estrone.
b) It is produced in the corpus luteum.
c) It inhibits follicle stirnulating hormone secretion by the anterior pituitary.
d) It has feed back mechanism on the pituitary-hypothalamus.
e) It is responsible for proliferative changes in the endometrium,
1185- As regards the corpus luteum, all the follorving are true EXCEPT that it:
a) Remains active for 2 weeks.
b) Secretes progesterone.
c) Is maintained by LH.
d) Secretes estrogen.
e) Secretes pregnandiol.
1186- The follorving normal finding on the hventy-third day of a normal twenty-eight day menstrual
cycle provides strong evidence that ovulation has occurred:
a) Cervical rrrucus with positive arborization when dried on a glass slide.
b) Subnuclear vacuolation apparent on endometrium biopsy.
c) High cornification index in vaginal cytology preparations.
d) High level of total estrogens in a twenty-four hour specimen of urine.
e) Serurn progesteron more than 10 nglml.
1187- The average amount of blood lost during a normal menstrual cycle is
a) l0-15 mL.
b) 25-60 mL.
c) 85-120 mL.
d) 150-200 mL,
e) 250-300 mL.
1188- Characteristics of fertile phase of the menstrual cycle include all the following W.EEI:
a) Peak levels ofestrogen.
b) Clear, abundant cervical mucus.
c) Ovulation,
d) Production of progesterone.
WhiteKnightLove

e) Sensation of vulval wetting.

-143-
Freely you have received; freely give.

ll89- The estrogenic phase of the menstrual cycle is characterized on vaginal cytology by the
predominance of:
a) Mature superficial cells.
b) lntermediate cells.
c) Basophilic blue stained cells,
d) Cells with curled edges.
e) Small rounded cells.
I190- The following hormonal changes occur after menopause EXCEPT:
a) Estrogen levels decrease drarnatically.
b) There is a relative increase in testosterone level.
c) The main source ofprogesterone production is from the adrenal glands.
d) Estrone is the main estrogen and results from peripheral conversion ofandrostenedione.
e) Gonadotrophins levels show no changes.
1l9l- The following arc hormonal changes associated rvith menopause_ffi[ff:
a) I FSH.
b) J estradiol.
c) J progesterone.
d) J SHBG.
e) JLH.
I192- Premature menopause could be due to:
a) Autoimrnune disorders as thyroditis,
b) Use of cornbined oral contraceptive pill for long time.
c) Benign ovarian tumor.
d) Klinefelter syndrorne.
e) Testicular ferninization syndrome,
I193- Menopause could be diagnosed by:
a) High FSH in any age group,
b) Absent menstruation for 3 months near the median age of the menopause.
c) Absent rnenstruation for l2 months near the median age of th: menopause.
d) Hot flushes & Iack of sleep.
e) Abrupt stoppage of rnenstruation for 3 months at the age of 50 years.
ll94- The TRUE statement as regard the menopause is:
a) The average age in Egypt is 55 years.
b) Is earlier in smokers.
c) Plasma cholesterol and triglyceride levels fall.
d) lf betow 40 requires ovarian biopsy for confirmation.
e) It is not influenced by vaginal hysterectomy.
I195- The major estrogenic substance, in postmenopausal women, is:
a) Estradiol.
b) Estrone.
c) Estriol.
d) Exogenous estrogens,
e) Inhibin.
I 196- As regards unopposed estrogen therapy for the postmenopausal rvoman, all the follorving are true
EXCEPT that it:
a) Improves the urethral syndrome.
b) Decreases urinary calcium excretion.
c) Causes increased incidence of endometrial carcinoma.
d) Causes increased breast cancer risk.
WhiteKnightLove

e) Causes hypertension.

-144-
'It is more blessed to give than to receive.

I197- Contraindications to postmenopausal estrogcn replaccment therapy include the following


BXCEPT:
a) Recent deep vein thrombosis.
b) Acute liver disease.
c) High serum triglycerides.
d) Premenopausal mastectonry for breast cancer.
e) Well controlled diabetes mellitus.
ll98- Absolute contraindications for hormone rcplaccment therapy (HRT) in menopausal women @
NOT include:
a) Active liver disease.
b) Systemic lupus erythernatosis.
c) Unexplained vaginal bleeding.
d) Recent history of thrombo-embolic attack.
e) Recent myocardial infarction.
1199- Hormonal replacement therapy might be associated with all the following EXCEPT:
a) Disappearance ofthe flushes.
b) Decrease the risk of endometrial carcinoma.
c) Prevents furlher bone loss.
d) Increase the relative risk for breast cancer.
e) Increase thromboernbolic disease.
1200- Complications that could be attributed to post-menopausal hormonal changes include the
following EXCEPT:
a) Procidentia.
b) Coronary heaft diseases.
c) Cancer breast.
d) Stress urinary incontinence.
e) Depression.
l20l- Risk factors for postmenopausal ostcoporosis DO NOT include:
a) Cigarette smoking.
b) Obesity.
c) Long term glucocorlicoid therapy,
d) Premature ovarian failure.
e) Heparin administration.
1202- ln normal puberty, all the following are correct EXCEPT:
a) The first few menstrual cycles are anovulatory,
b) Axillary hair growth occurs before menstrual period.
c) There are changes in the vaginal epithelium.
d) Estrogens produce the growth spurt and then closure oflong bone epiphyses.
e) Pubic hair growth is the first sign of puberty.
1203- Which is a normal feature at puberty:
a) Pubic hair growth is the first sign.
b) The first few menstrual cycles are anovulatory.
c) Axillary hair growth occurs after the first menstrual period.
d) There is decreased acidity of the vaginal environment.
e) There is cessation of growth of the long bones.
1204- As regards the four stages of puberty, what is the MOST COMMON chronological order of thcse
stages at normal puberty:
a) Gonadarche, thelarche, pubarche, menarche.
b) Gonadarche, menarche, thelarche, pubarche.
WhiteKnightLove

c) Thelarche, gonadarche, pubarche, tnenarche.


d) Thelarche, gonadarche, menarche, pubarche.
e) Thelarche, pubarche, gonadarche, menarche.

-145-
Freely you have received; freely give.

1205- All the following about precocious puberty is true EXCEpT:


a) It may involve menarche before the age of 8 years.
b) False precocious puberty means that conception could not occur.
c) Constitutional factor is rare.
d) It may be due to serious tumors.
e) It may be associated with contrasexual development.
1206- All the following about precocious puberfy is true EXCEPT:
a) It is usually constitutional.
b) It may be associated with long bone fractures.
c) Pubic hair is first sign.
d) It can be delayed by GnRH analogues.
e) It results in adult short stature.
1207- Causes of delayed puberty include all of the followings EXCEPT:
a) Hypothyroidism.
b) Pitutary adenoma.
c) Extensive physical training as athletes,
d)
Malnutrition.
e)
Functioning ovarian cysts.
1208- Concerning hirsutism, all the follorving are true_@ff:
a) Adrenal enzyme deficiency can present in adult women.
b) The oral contraceptive pill can be used to treat hirsutisrn.
c) Cyproterone acetate should be given in a contraceptive regimen.
d) Dexamethasone is an alternative anti-androgen.
e) Circulating testosterone is not always elevated in lrirsute women.
1209- A 34-year-old patient with three children and using intrauterine device for contraception, presents
with irregular menstrual cycles at intervals anyrvhere from 33 to 90 days. Menstrual flow is
usually heavy, and she has noted increasing hirsutism. The mcdication MOST SUITABLE for
her is:
a) Clomiphene citrate (Clomid).
b) Cornbined oral contraceptives.
c) Antiprostaglandin.
d) Endometrial sampling.
e) None ofthe above.
1210- A 24-year-old woman complains of bothersome hirsutism and skipping periods, she does not have
evidence of voice changes, hair loss, or clitoromegaly. The pelvic examination does not reveal
adnexal masses. The serum DHEA-S, testosterone, and l7-hydroxyprogesterone levels are
normal. The LH to FSH ratio is 2:1. Which of the following is the MOST LIKELY diagnosis?
a) Polycystic ovarian syndrome.
b) Familial hirsutism.
c) Ovarian tumor.
d) Adrenaltumor,
e) Cushing syndronre.
l2ll- As regards prolactin and hyperprolactinemia, all the following are true-ffipff:
a) Prolactin inhibits ovarian steroidogenesis.
b) Bromocriptine is the drug of choice in hyperprolactinemia.
c) Prolactin is secreted from the anterior pituitary by the lactotrophs.
d) Micro-adenoma may undergo spontaneous resolution.
e) Bromocriptine has teratogenic effect rvhen given to pregnant women.
l2l2- Increased circulating prolactin concentration is associated with all the followings EXCEPT:
WhiteKnightLove

a) Stress.
b) Hypothyroidism,
c) Anorexia nervosa.
d) Acromegaly.

-t46-
'It is more blessed to give than to receive.

e) Pregnancy,
l2l3- Hyperprolactinemia may be associatcd with all the follorving EXCEPT:
a) Chronic renal failure.
b) Hypothyroidisrn.
c) Pituitary adenoma.
d) Methyl dopa therapy.
e) Adrenogenital syndrome.
1214- As regards bromocriptine, allthe following are truc EXCEPT that it:
a) Is a dopamine agonist.
b) Inhibits prolactin secretion.
c) Can be administered vaginally.
d) Is less potent than lisuride in treatment of hyperprolactinemia,
e) Can cause hypertension.
l2l5- Amenorrhea could be due to the followings EXCEPT:
a) Combined oral contraceptive.
b) Depot medroxy progesterone acetate.
c) GnRH analogues.
d) Danazol.
e) [ntrauterine contraceptive device.
l2l6- The follorving diagnosis is likely in a 30-ycar-old woman rvith secondary amenorrhea and low
gonadotrophin levels:
a) Premature ovarian failure.
b) Resistant ovary syndrorne.
c) Sheehan's syndrorne.
d) Asherman's syndrome.
e) Post-pill arnenorrhea.
l2l7- ln amenorrhea, all the following are true EXCEPT:
a) In Turner syndrome there is failure of menstruation,
b) Kalhnan syndrome is associated with amenorrhea and anosmia.
c) Sheehan syndrorne is associated with amenorrhea.
d) Ashernran syndrorne is associated with retinitis pigmentosa.
e) In Fitz Hugh Curtis, there is perihepatitis.
l2l8- Initial investigations for primary amenorrhea D$_include:
a) Serum estrone.
b) Pelvic ultrasound.
c) Serum prolactin.
d) SkullX-ray.
e) Chromosome analysis.
1219- Cryptomenorrhea may present with variety of clinical problems. These NEVER include:
a) Acute urinary retention.
b) Hematocolpos.
c) Lower abdorninal pain.
d) Premenstrual spotting.
e) Cyclic pain.
1220- True statements conccrning anorexia nervosa include the following EXCEPT:
a) It is seen predominantly in females, rarely in males.
b) Most affected patients have an obsessive -compulsive personality.
c) Mean 24-hour concentration of cortisol is twice normal.
d) Thyroid hormones are in the norrnal range.
WhiteKnightLove

e) It is occasionally fatal.

-t47 -
Freely you have received; freely give.

l22l- The INCORRECT statement for patients with testicular feminization:


a) Chromosomal pattern is 46, Xy,
b) They have no uterus.
c) They have well developed breasts.
d) They have normal pubic and normal axillary hair.
e) They have primary amenorrhea.
1222- All the following are correct about Turner syndrome EXeEpT:
a) It is associated with gonadal failure,
b) There is failure ofthe secondary sex characters.
c) Total number of chromosomes is 45.
d) Estrogen replacement therapy is useful.
e) Ovaries may be polycystic.
1223- Turner syndrome is assiociated with:
a) Decreased incidence of ovarian carcinoma.
b) Increased risk ofhypertension in adulthood.
c) Increased incidence of Diabetes.
d) Increased incidence of Deafness.
e) All of the above.
1224- The following is characteristic of Sheehan syndrome:
a) Profuse lactation.
b) Arnenorrhea.
c) Hyperthyroidism.
d) Renal insufficiency.
e) Cushingoid faces.
1225- An l8-year-old patient presents to you for evaluation because she has not yet stafted her period.
On physical examination, she is 170 cm in tall. She has minimal breast development and no
axillary or pubic hair. On pelvic examination, she has a normally developed vagina. A cervix is
visible. Thc uterus is palpable, as are normal ovaries, Which of the following is the BEST NEXT
STEP in the evaluation of this patient?
a) Draw her blood for a karyotype.
b) Test her sense of smell.
c) Draw her blood for TSH, FSH, and LH levels.
d) Order an MRI of the brain to evaluate the pituitary gland.
e) Prescribe a progesterone challenge to see if she will have a withdrawal bleed.
1226- A 26-year-old nulliparous lvoman presents with a history of lower abdominal pain and vaginal
bleeding that started 5 days later than her expected menses. She is suffering of primary infertility
for 4 years. On examination, her temperature is 37.6oC, her uterus is top normal size and tender,
and she has right adnexal fullness. Before starting work up, which is the LEAST LIKELY
d iagnosis?
a) Ectopic pregnancy.
b) Pelvic inflammatory disease (PID).
c) Adenomyosis.
d) Appendicitis.
e) Threatened abortion.
1227- The general incidence of infertility is:
a) 5%.
b) l0- rs%.
c) 25%.
d) 30%.
WhiteKnightLove

e) 35%.

-148-
'It is more blessed to give than to receive.

1228- According to WHO (world health organization) parameters the percentage of normal morphology
of normal semen analysis is:
a) >-30%.
b) <30%.
c) >-50%.
d) >70%.
e) >90%.
1229- According to WHO parameters, evidences of normal semen analysis include:
a) A sperm count of l0 rnillion per mL.
b) A volume of 1 ml-.
c) Motility within 60 rninutes of ejaculation is more than 50%.
d) Normal sperm rnorphology in 600/o of sperms
e) A positive mixed agglutination reaction.
1230- The follorving is INeQBRECI as regards spermatozoa:
a) They contain 23 chromosomes.
b) They are produced at a faster rate when testicular temperature is raised.
c) They require testosterone for normal development.
d) They require follicle stimulating hormone (FSH) for normal development.
e) They are produced from spermatogonia in approximately 70 days.
l23l- Normal human seminal fluid:
a) Coagulates in vitro.
b) Contains sucrose.
c) Has a pH of4-5.
d) May contain up to 70-80% of rnorphologically abnorrnal spermatozoa.
e) Originates mainly in the testes.
1232- According to WHO parameters, unaccepted values for a normal semen analysis is:
a) A sperm count of 30 rnillion per mL.
b) Progressive pattern of rnotility within 60 minutes of ejaculation is l5%.
c) A volume of 2.5 rnl.
d) 20% abnormal forms,
e) A negative mixed agglutination reaction.
1233- The following statements as regards normal cervical mucus at the time of ovulation, are true
EXCEPTI
a) Amount of the mucus is so profuse that it may be noticeable as a vaginal discharge.
b) Spinnbarkeit test is l0 crn or more.
c) It contains low amount of crystals of sodium and potassium chloride.
d) It shows positive ferning test,
e) It becomes more alkaline.
1234- The character of normal cervical mucus at the time of ovulation:
a) Highly viscous and turbid.
b) Spinnbarkeit less than 3 cm.
c) Contains low amount of crystals of sodium and potassium chloride.
d) Positive ferning test.
e) Acidic.
1235- The following statements are correct as regards serum progesterone measurement in work up of
infertility EXCEPT:
a) The value shows biphasic pattern of throughout the normal ovulatory cycle.
b) Estimation is at the mid-luteal phase of the cycle,
c) It can be used for assessment ofthe luteal phase defect.
WhiteKnightLove

d) It cab be used to diagnose polycystic ovarian disease.


e) It has a value in the diagnosis of anovulation.

-t49-
Freely you have received; freely give.

1236- A 34-year-old infertilc woman is noted to have evidence of blocked fallopian tubes by
hysterosapingogram. Which of the following is THE BEST NEXT STEp for this patient?
a) Short term trial ofFSH therapy.
b) Clomiphene citrate therapy.
c) Laparoscopy.
d) Intrauterine insemination,
e) IVF cycle.
1237- The following are advantages of laparoscopy in the diagnosis of infertiliS_EXC.EP'I:
a) Good evaluation of tubal factor
b) Diagnosis of endometriosis.
c) Diagnosis of Asherman syndrome.
d) Good visualization of pelvic adhesions.
e) Direct visualization of corpus luteum.
1238- Hysteroscopy is important for diagnosis of the following lesions EXCEPT:
a) Intrauterine adhesions.
b) Submucous fibroids.
c) Peritubal adhesions.
d) Cornual block.
e) Septate uterus.
1239- A 26-year-old nulligravida lvoman has regular menses every 28 days. She presented with primary
infertilitl,for 3 years. The semen analysis is normal. The patient had a postcoital test revealing
motile sperm and stretchy watery cervical mucus. She has been treated for chlamydial infection in
the past. Which of the following is the MOST LIKELY etiology of her infertility?
a) Peritoneal factor.
b) Male factor.
c) Cervical factor.
d) Uterine and tubal factor.
e) Ovulatory factor.
1240- The CORRECT statement, about in vitro fertilization (IVF), is:
a) Down regulation of the pituitary gland is achieved by the use of human menopausal gonadotrophins
(HMG).
b) IVF is indicated for treattrent of luteal phase defect in inferlility,
c) IVF is indicated for treatment of unexplained infertility.
d) Oocyte retrieval is performed 28 hours after human chorionic gonadotropin (hCG) injection.
e) Success rates are increased with increasing age.
1241- Polycystic ovarian disease is characterized by all of the following EXCEPT:
a) Increased LH level.
b) Increase in androgen levels.
c) Decrease in estrogen levels.
d) Oligomenorrhea.
e) Android obesity.
1242- Concerning polycystic ovary syndrome, all the following are true EXCEPT:
a) Most women are obese.
b) It is familial.
c) Ferlility is reduced by the oral contraceptive pills.
d) It is characterized by a thick walled capsule and a normal number of prirnary and secondary follicles.
e) lt is excluded by a normal serum luteinising hormone level.
1243- A 28-year-old woman presents with oligohypomenorrhea, facial hairsutism, and transvaginal
ultrasounrl showing multiple subcortical small cysts 4-6 mm in both ovaries. Laboratory findings
WhiteKnightLove

would shorv which of the following?


a) Decreased LH level.
b) Increase in androgens level.
c) Decrease in estrogen level.

-150-
'It is more blessed to give than to receive.

d) Decreased circulating glucocorticoids.


e) Decreased fasting serum insulin.
1244- Suggestions of anovulation includc the following EXCEPT:
a) Flat cells with pyknotic (small, dark) nuclei on vaginal smears.
b) Disturbed menstrual cycles.
c) Serial ultrasound scans show failure offollicle rupture.
d) Secretory changes detected at histological examination of the endometrium.
' e) Monophasic basal body tenrperature chart.
1245- Facts about clomiphene citrate include the following EXCEPT:
a) It has antiestrogenic properlies.
b) It may be starled with a 5O-mg dose daily.
c) It is useful in primary ovarian failure.
d) It may cause cervical mucus hostility.
e) Multiple pregnancy may be a side effect of its use.
1246- Facts about clomiphene citrate include the following EXCEPT:
a) It has both weak estrogenic activity and anti-estrogenic actions.
b) It may be started rvith a 50-rng dose daily.
c) It is useful in primary ovarian failure.
d) It may cause cervical mucus hostility,
e) Multiple pregnancy may be a side effect of its use.
1247- Which of the following conditions is suitablc for clomiphene citrate stimulation?
a) Polycystic ovarian syndrorne.
b) Resistant ovary syndrome.
c)
Kallman's syndrome.
d) Ashennan's syndlorne.
e) Conadal dysgenesis.
1248- ln severe ovarian hyperstimulation syndrome (OHSS), all the follorving are true EXCEPT:
a) Pleural effusion may occur.
b) Intravascular clotting can be a complication.
c) Hypoproteinemia may develop.
d) Surgical managernent is indicated in most of the cases.
e) Paracentesis may be of benefit.
1249- The INCORRECT statement regarding infertility:
a) The general incidence of infertility is 10-15%.
b) The normal conception rate (following normal unprotected sexual life) after one month is 15-20%.
c) According to WHO, the normal sperm concentration <10 million/ml.
d) Infertility is diagnosed after unprotected sexual life for one year.
e) More than one cause of inferlility could be found in one couple,
1250- When investigating an infertile couple:
a) Routine hysteroscopy should be carried out,
b) Laparoscopy and dye hydrotubation is more infonrative of uterine cavity abnormality than
hysterosalpingography.
c) Acute pelvic infection is an absolute contraindication to HSG.
d) Urinary LH detection using a commercially available test kit is diagnostic of ovulation.
e) A routine cervicalsmearshould be included in the investigation of infertility.
l25l- Evidence of anovulation include:
a) Biphasic basal body temperature char1.
b) Disturbed menstrual cycles.
c) Rise in serum progesterone in the second halfofthe cycle.
WhiteKnightLove

d) Secretory changes detected at histological examination of the endometrium.


e) LH surge.

- 151-
Freely you have received; freely give.

1252- Evidence of anovulation include:


a) Biphasic basal body temperature chart.
b) Disturbed rnenstrual cycles.
c) Serum progesterone level in 2l't day of cycle is more than l0 ng%.
d) Secretory endometrium at the second halfofthe cycle.
e) Vaginal cytology at 2nd half of the cycle: basophilic blue stained cells with curled edges.
1253- The INCORRECT statement regarding clomiphene citrate:
a) It has antiestrogenic effects.
b) Side effects include multiple pregnancy.
c) Ovarian hyperstimulation syndrome is rare.
d) It is a steroidal agent.
e) It is a synthetic product.
1254- The CORRECT statement regarding clomiphene citrate used in the treatment of infertility is that
it:
a) Increases the incidence ofbreast carcinoma.
b) Is not associated with ovarian cyst fonnation.
c) Increases the risk of fetal malformations.
d) May causes vasomotor symptoms as a side effect.
e) Is prescribed in a starling dose of 500 mg daily for five days / month.
1255- Drugs of antiestrogenic effects DO NOT include:
a) Tarnoxifen.
b) Letrozole.
c) Clomiphene citrate.
d) Cyclofenil,
e) Bromocriptirre,
1256- The CORRECT statement for polycystic ovarian disease:
a) Laparoscopy is essential for aqccurate diagnosis.
b) First line treatment is wedge resection of the ovaries.
c) It is associated with hypoestrogenism"
d) Anovulatory women with PCOD are at decreased risk of endometrial cancer.
e) LH is increased and FSH is normal or decreased.
1257- The INCORRECT statement regarding human menopausal gonadotrophin (HMG):
a) lt can be extracted fronr postmenopausal urine.
b) It can be given intramuscular or intravenous.
c) Ovarian hyperstinrulation could occur.
d) Human chorionic gonadotrophins (hCG) may be used to triggerovulation after HMG.
e) Effect can be monitored by serial ultrasound folliculometry.
1258- The INCORRECT statement in case of luteinized unruptured follicle:
a) Progesterone is secreted normally.
b) Menstrual cycles are usually regular.
c) Corpus luteum is formed,
d) Induction of ovulation is a useful modality for treatment,
e) Patient can benefit from human chorionic gonadotrophin (hCG) injections given in high doses at
mid-luteal phase.
WhiteKnightLove

-152-
'It is more blessed to give than to receive.

Direction: Each set of matching questions in this section consists of a list of numbered items followed
by several lettered options. For each numbered item, select the ONE best lettered option that is most
closely associated with it. Each lettered heading may be selected once, more than once, or not at all.

For each of the following evaluations numbered (1259-1263), select the most appropriate day of a
normal 28-day menstrual cycle lettered (a-f) for the evaluation to be performed.
EACH LETTERED HEADING IVIAY BE USED ONCE, MORE THAN ONCE, OR NOT AT ALL.

259- Endometrial biopsy for evaluation of infertility a) Day l.


260- Post-coital test b) Day 3.
26 l- Hysterosa lpingogram c) Day 8.
262- Determination of serum progesterone level to document ovulation d) Day 14.
263- Gonadotrophin evaluation e) Day 21,
t) Day 26.

For each of the follorving descriptions numbered (1264-1266), select the most appropriate item lettered
(a-e).
EACH LETTERED HEADING MAY BE USED ONCE, MORE THAN ONCE, OR NOT AT ALL.

264- Non-estrogenic non-hormonal a) Clomiphene citrate,


265- Oral antiestrogen b) Bromocriptine.
266- Ovulatory agent for hypogonadotrophic hypogonadism c) Gonadotropin-releasing hormone.
d) Dexamethasone.
e) Non ofthe above.

WhiteKnightLove

-153-
Freely you have received; freely give.

FAMILY PLANNING

Direction: Select the ONE BEST lettered ans'wer or completion in each question.

1267- Which of the following listings correctly ranks contraceptivc methods in terms of decreasing
effectiveness:
a) Oral contraceptives, IUCD, spennicides, diaphragm, rhythm.
b) Oral contraceptives, diaphragm, IUCD, sperrnicides, rhythm.
c) IUCD, oraI contraceptives, diaphragm, spermicides, rhythnr.
d) Rhythm, oral contraceptives, IUCD, diaphragm, spermicides.
e) Oral contraceptives, IUCD, diaphragm, spermicides, rhythrn.
1268- Identification of the fertile phase could be done by the followings_ffi[ff ::
a) The calendar method.
b) The basal body temperature method (BBT).
c) The cervical rrucus method (Billing method).
d) The symptom-thermal method.
e) Measurement of serum progesterone.
1269- The lactational amenorrhoea method (LAM) has the followings advantages EXCEPT:
a) Universally available to all breastfeeding women.
b) Effectiveness is 98% in well selected women.
c) Protection begins imrnediately after delivery.
d) Provide health benefits to the infant.
e) Duration of method is unlimited.
1270- The CORRECT statement for hormonal contraception:
a) Ethinyt Estradiol (EE) is the estrogen present in the combined oral contraceptive pills (COCS).
b) Progestins are synthetic compounds that mimic the structure of natural progesterone.
c) Failure rate is about l/HWY with COCS and 5/HWY with POPs.
d) Cornbined oral contraceptive pills increases the risk of anemia.
e) History of hepatic adenoma is not a contraindication of COC pills.
1271- The absolute contraindication of combined oral contraceptive pills is:
a) Deep venous thrombosis.
b) Heavy smoking.
c) Sickle cell anemia.
d) Essential hyperlension controlled by medications.
e) Diabetes rnellitus controlled by diet.
1272- Which IS NOT a contraindication to using combined oral contraceptives:
a) Pulmonary ernbolus.
b) Porphyria.
c) Sickle-cell disease.
d) Migrane preceded by focal aura.
e) Depression.
1273- Advantages of OCs include the following EXCEPT:
a) Very effective, failure rate is about 0.I/HWY with COCs, and 1-2IHWY with POPs.
b) Reversible with rapid retum to fertility once stopped.
c) No action needed at the time of sexual intercourse.
d) They have beneficial health effects other than contraception.
WhiteKnightLove

e) Offer protection against sexually transmitted infections (STDs).

-154-
'It is more blessed to give than to receive.

1274- The use of combined oral contraceptives reduces the risl< of the following @:
a) Ectopic pregnancy.
b) Hepatic adenoma.
c) Salpingitis.
d) Ovarian cancer.
e) Endometria[ cancer.
1275- The followings are reduced with combined oral contraceptive use Sl@:
a) Breast milk production.
b) Cervical cancer.
c) Salpingitis.
d) Epithelial ovarian cancer.
e) Endometrial cancer.
1276- Non-contraceptive health benefits of COCs include the follorving EXCEPT that they:
a) Lnprove the rhythm of the menstrual flow.
b) Decrease the arnount of the menstrual flow.
c) Are used to improve endometriosis.
d) Prevent functional ovarian cysts.
e) Reduce risk ofcervical cancer.
1277- Non-contraceptive health benefits of COCs include the following EXCEPT that they:
a) Reduce the risk of endometrial hyperplasia and endometrial and ovarian carcinoma.
b) Reduce risk of pelvic inflammatory disease.
c) Reduce risk ofbenign breast disease.
d) Reduce risk of rnood changes.
e) Lnprove acne and hirsutisrn.
1278- AII the following are health benefits of combined oralcontraceptive pills EXCEPT:
a) Endometria[ carcinorna protection.
b) Protection against surface ovarian tumors.
c) Treatment of benign breast lesions.
d) Protection against cervical cancer.
e) Decrease amount of menstrual flow.
1279- Side effects of OCs include the following EXCEPT:
a) Nausea.
b) Breast tenderness,
c) Headaches.
d) Breakthrough bleeding.
e) Excessive nrenstrual loss.
1280- The side effects of combincd oral contraceptive pills include the following EXCEPT:
a) Nausea.
b) Dizziness.
c) Vaginal discharge.
d) Menorrhagia.
e) Weight gain.
l28l- According to the WHO medical Eligibility Criteria, absolute contraindications (Grade 4
conditions) of OCs include the following EXCEPT:
a) History of DVT, pulmonary embolus, cerebral hemorrhage, or coronary at1ery disease.
b) Markedly impaired liver function.
c) Estrogen-dependant malignant tumors: carcinoma of the breast and uterus.
d) History of cholestasis during pregnancy,
e) Diabetes or history of gestational diabetes.
WhiteKnightLove

1282- As regareil the relative contraindications of OCs, all the follorving are true @:
a) Heavy smoking and age more than 35 years.
b) Family history of hyperlipidemia or myocardial infraction in a parent.
c) Undiagnosed genital bleeding.

-155-
Freely you have received; freely give.

d) Sickle cell disease or sickle C disease.


e) History of cholestasis during pregnancy.
1283- Which of the following is an absolute contraindication for oral progesterone only contraceptive
use:
a) Cigarette sr-noking.
b) Depression.
c) Gall bladder stones.
d) Mild hyperlension.
e) Unexplained uterine bleeding.
1284- As regards oral progesterone only contraceptive, all the following statements are true_ffip,ff:
a) 60% of women using it will ovulate.
b) Ideally, it should be taken just before bedtime.
c) The pearl index is higher in the older reproductive age group.
d) lt is associated with a lower risk of an ectopic pregnancy than for a non-user.
e) It is at Ieast as effective as the combined contraceptive pill.
1285- Regarding the progesterone only pill, the CORRECT statement is:
a) Anovulation is the main mechanism of contraceptive action.
b) Ideally, it should be taken exactly just before meals.
c) Contraceptive efficacy is higher among women in the older reproductive age group.
d) It is not suitable for breast feeding mothers.
e) It is at Ieast as elfective as the cornbined contraceptive pill,
1286- The follorving statements regarding progestin only oral contraceptive preparations are correct
EXCEPT:
a) They are suitable methods of contraception for breast feeding women.
b) Woman needs to take them at the same tirne each day.
c) They are more reliable as the patient's age increases.
d) They are contraindicated in smokers over the age of40.
e) They are Iess reliable than the combined oral contraceptive pill in preventing pregnancy.
1287- The following statements regarding progestin only injectable contraceptives are correct
EXCEPT:
a) They have no effect on lactation and can be used by breastfeeding women,
b) The lnechanism of their action is rnainly by alteration of cervical mucus.
c) Return to fertility after discontinuation is usually delayed.
d) Young user could have increased an increased risk for developing osteoporosis later in life.
e) DMPA (Depo-medroxyprogesterone acetate) is taken every 2 month.
1288- Which is the INCORRECT statement regarding progestin only injectables as contraceptives:
a) They have no effect on lactation and can be used by breastfeeding women.
b) The mechanism of their action is mainly by suppression of ovulation.
c) Return to fertility after discontinuation is usually delayed,
d) Young user could have increased an increased risk for developing osteoporosis later in life.
e) DMPA (Depo-rnedroxyprogesterone acetate) is taken every 3 months.
1289- The INCORRECT statement regarding progestin only injectables:
a) DMPA (Depot-medroxyprogesterone acetate) is taken every month.
b) Mechanisrn of action is mainly by making cervica mucus not permeable to sperms.
c) They have no effect on lactation and can be r-rsed by breastfeeding women.
d) Return to fertility after discontinuation is usually delayed.
e) Young user could have increased an increased risk for developing osteoporosis later in life.
1290- As regared progestin-only injectables, all the following are true EXCEPT:
a) Menstrual irregularities are more common than for OCs.
WhiteKnightLove

b) Contraceptive effect and side effects cannot be stopped imrnediately.


c) Return to fertility after discontinuation is usually delayed.
d) Weight gain (rnore cornmon for progestin-only injectables).
e) Offer protection against sexually transmitted infections.

-156-
'It is more blessed to give than to receive.

l29l- Advantages of implanon include the following EXCEPT:


a) Effectiveness: 99% effective in preventing pregnancy.
b) No action needs to be taken at the time of intercourse.
c) It is long acting, but reversible and rapidly restoring fertility.
d) It contains no estrogen, so can be used by women who are breastfeeding.
e) Offer protection against sexually transmitted infections.
1292- The CORRECT statement regarding norplant:
a) The main mechanism of action is on the ovulatory function.
b) It contains estrogen.
c) It is composed of the progestin, levonorgestrel.
d) It is irreversible contraceptive.
e) Nonserious side effects are rare.
1293- Disadvantages of Norplant include the followings EXCEPT:
a) It cannot be initiated and discontinued without a provider's help.
b) A minor surgical procedure is required for both insertion and removal.
c) It provides no protection against STIs, including HIV.
d) Side effects as bleeding irregularities are common.
e) Effectiveness: 90% effective in preventing pregnancy.
1294- All of the following statements with regard to the copper IUCD are correct EXCEPT:
a) Modern copper IUCDs are clinically effective and safe for at least five years.
b) It reduces the number of sperm reaching the fallopian tube and their capacity to fertilize the egg.
c) It is associated with a slight increased risk of PID.
d) It is contraindicated in women with irregular uterine bleeding,
e) Levonorgestrel releasing devices are associated with menorrhagia.
1295- All of the following statements with regard to the copper IUCD are correct EXCEPT:
a) Modern copper IUCDs are clinically effective and safe for at least five years.
b) They do not affect steroidogenesis.
c) No systernic side effects.
d) It is contraindicated in recurrent pelvic inflammatory disease,
e) Levonorgestrel releasing devices are associated with menorrhagia.
1296- Mechanism of action of copper IUCD includes:
a) Tubal block.
b) Inhibition of ovulation.
c) Increased tubal motility.
d) Cervical mucus hostile to the spenns.
e) Mechanical inflammatory reaction of the endometrium.
1297- The INCORRECT statement regarding thc IUCDs:
a) Copper IUCDs are effective for at Ieast five years.
b) IUCDs reduce the capacity of sperms to fertilize the egg.
c) The risk of pelvic inflarnmatory disease is slightly increased,
d) They are contraindicated in women with subseptate uterus.
e) Levonorgestrel releasing devices are associated with a significant increase in blood loss during
menstruation.
1298- The INCORRECT statement regarding intrauterine devices:
a) They neither affect ovulation nor steroidogenesis,
b) Not suitable for lactating mothers.
c) Perforation ofthe uterus is rare.
d) When intrauterine pregnancy occurs on top of IUCD the risk of spontaneous abortion increases.
e) IfapregnancyoccursinIUCDuser,theriskofitbeingectopicishigherthaninwomenusingother
WhiteKnightLove

contraceptive methods.

-157-
Freely you have received; freely give.

1299- Advantages of IUD include the follorving EXCEPT:


a) Safe and highly effective (failure rate l-2/HWY).
b) Easy to use, requiring no action at the tirne of intercourse.
c) Long-acting for many years.
d) Easily reversible, with retum to fertility very soon after removal.
e) Can be recommended for women at risk of sexually transmitted infections (STIs).
1300- As regard the hormone releasing intrauterine systems the followings are true EXCEPT:
a) The physical characters of the cervicalmucus changes and become impermeable to sperms.
b) It causes partial suppression ofovulation.
c) It reduces the duration and quantity ofmenstrual bleeding and pain.
d) Provides protection from pelvic inflammatory disease.
e) The hormone releasing intrauterine system is not more expensive than other IUDs.
I30l- At eight weeks' gestation, a woman has an intrauterine device (IUCD) and the strings are visible
at the external os. The BEST management is:
a) Antibiotics as prophylactic measure against septic abortion.
b) Progesterone as prophylactic measure against aborlion.
c) Dilation and curettage.
d) Removal of IUCD.
e) Bed rest and observation.
1302- All these methods could be recommended to be used during lactation EXCEPT:
a) Depot provera.
b) MiniPills. *:
c) Combined contraceptive pills,
d) rucD.
e) Male condom.
1303- Advantages of barrier methods DO NOT include:
a) They have a role in preventing sexually transmitted diseases (STDs).
b) They have no systemic side effects.
c) Easy to initiate and discontinue.
d) hnrnediate return to fertility after stopping its use.
e) Efficient to the same extent as the oral contraceptive pill.
1304- Disadvantages of barrier methods include the following EXCEPT:
a) They are not as effective as other modern methods.
b) Sorne couples find difficulty to use them consistently and correctly.
c) Some rnethods require paftner's participation.
d) They do not intenupt the natural phases ofsexual activity.
e) They need proper storage in order to maintain the quality of the products.
1305- The following organism is responsible for toxic shock syndrome:
a) Staph aureus.
b) Staph epidermidis,
c) Strept Pyogenes.
d) Group B streptococci.
e) Staph Toxi.
1306- Regarding emergency contraception, all the following are correct EXCEPT:
a) When using estrogens, they usually have side effects,
b) When using the IUCD, it must be inserted the "morning after".
c) It is contraception used after intercourse but prior to implantation.
d) When used estrogens must be taken within 72 hours of intercourse.
e) Established contraindications to the oral contraceptive pill and the intrauterine contraceptive device
WhiteKnightLove

still apply.

-158-
'It is more blessed to give than to receive.

1307- The INCORRECT statement regarding emergency contraception:


a) It is that contraceptive method used after intercourse but prior to the implantation of the fertilized
ovum,
b) Estrogenic preparations must be taken within 72 hours of intercourse.
c) Effective estrogenic preparations usually have significant side effects.
d) When using the IUCD, it must be inserted the "morning after".
e) Mifepristone is highly effective.
1308- Which method does protect against sexually transmitted diseases:
a) Male condom.
b) Coitus interruptus.
c) Depoprovera.
d) Combined contraceptive pills.
e) IUCD.

Direction: Each set of matching questions in this section consists of a list of numbered items followed
by several lettered options. For each numbered item, select the ONE best lettered option that is most
closely associated with it. Each lettered heading may be selected once, more than once, or not at all.

For each of the following condition numbered (1309-1313), choose the contraceptive method lettered
(a-f) with which it is most likely to be associated.
EACH LETTERED HEADING MAY BE USED ONCE, MORE THAN ONCE, OR NOT AT ALL.

309- Toxic shocksyndrome I a) Combined oral contraceptives.


310- Cholelithiasis I b) Progestin-only pill.
3ll- Actinomyces Israeli I c) Condoms.
312- Highest risk of contraceptive failure I d) Cervical cap.
313- Dysmenorrhea I e) Intrauterine contraceptive device (IUCD).
f) Coitus i

For each of the following case numbered (1314-1317), choose the LEAST appropriate contraceptive
method lettercd (a-f).
EACH LETTERED HEADING MAY BE USED ONCE, MORE THAN ONCE, OR NOT AT ALL.

314- A l7-year-old woman with a history of ectopic I a) Combined oral contraceptives.


pregnancy I b) Progestin-only pill.
315- A 2S-year-old lvoman who is nursing I c) Levonorgestrel implant.
316- A 38-year-old woman who smokes 40 cigarettcs daily I d) Condoms.
317- A 37-year-old \voman with a large cystocele I e) Diaphragm.
f) Intrauterine contraceptive device
(rucD).
WhiteKnightLove

-159-
Freely you have received; freely give.

GYNECOLOGICAL MCQs ANSWERS

APPLIED BASIC SCIENCES


697 A 710 A 723 D 736 E 749 A 762 D 775 E
698 C 7tt B 724 A 737 D 750 E 763 D 776 D
699 C 7t2 A 725 E 738 D 751 B 764, B 777 C
700 D 713 B 726 D 739 C 752 B 765 E 778 C
70r E 7t4 D 727 C 740 D 753 B 766 C 779 A
702 A 715 E 728 D 741 E 754 D 767 D 780 D
703 E 716 E 729 E 742 A 755 D 768 G 78r A
704 A 717 A 730 E 743 B 756 D 769 F 182 D
705 D 718 C 731 A 744 C 757 B 770 H 783 A
706 D 719 B 732 A 745 D 758 A 777 A 784 C
707 B 720 A 733 D 746 C 759 B 772 E 785 E
708 C 721 E 734 D 747 B 760 C 773 B 786 D
709 C 11' D 735 E '148 E 761 A -174 A 787 B

PATIENT EVALUATION
788 D 790 B 792 B 794 D 796 E 798 D 8001 o
789 D 791 E 793 D 795 A 797 D 799 D

BBNIGN CONDITIONS OF REPRODUCTIVE ORGANS


AND MENSTRUAL DISORDERS
80r A 821 A 84r D 861 E 881 D 90r C 920 D
802 B 822 E 842 E 862 E 882 B 902 C 921 D
803 C 823 B 843 A 863 A 883 B 903 D 922 B
804 A 824 C 844 C 864 B 884 C 904 C 923 E
805 A 825 B 845 A 865 E 88s C 905 D 924 A
806 E 826 D 846 E 866 E 886 D 906 C 925 C
807 D 827 D 847 D 867 D 887 A 907 B 926 E
808 A 828 A 848 D 868 E 888 E 908 C 927 E
809 C 829 D 849 D 869 E 889 B 909 B 928 A
810 A 830 D 850 A 870 D 890 C 910 B 929 C
8lr D 831 B 851 B 871 D 891 E 9tt A 930 D
812 B 832 B 8s2 B 872 B 892 B 912 D 931 B
813 C 833 B 8s3 D 873 E 893 D 9r3 E 932 A
814 A 834 C 854 E 874 B 894 E 914 A 933 B
815 A 83s B 855 C 875 E 895 D 915 A 934 D
816 C 836 B 8s6 B 876 D 896 E 916 C 935 C
817 D 837 E 857 B 877 E 897 D 917 C 936 A
818 A 838 C 8s8 A 878 D 898 A 918 C 937 C
819 D 839 B 859 D 879 A 899 E 919 E 938 A
WhiteKnightLove

820 D 840 C 860 B 880 C 900 A

-160-
'It is more blessed to give than to receive.

THE BREAST IN GYNECOLOGICAL PRACTICE

GYNECOLO GICAL INFECTIONS

946 D 957 A 968 D 979 B 989 D 999 A 1009 E


947 E 958 D 969 C 980 B 990 B 1000 B 1010 E
948 A 959 A 970 E 981 B 99r D 1001 C r0l1 C
949 D 960 E 97r A 982 B 992 C 1002 B r0t2 A
9s0 A 961 C 972 E 983 D 993 C I 003 B 1013 H
951 A 962 C 973 C 984 C 994 D 1004 A t0t4 G
952 D 963 A 974 E 98s B 99s B I 005 C 1015 F
953 B 964 E 975 D 986 C 996 B l 006 A l0l6 C
954 A 965 B 976 A 981 D 997 A 1007 E 1017 A
955 E 966 D 9'.77 B 988 C 998 B 1008 B 1018 A
956 D 967 B 978 D

UROGYNECOLOGY AND GENITAL DISPLACEMENT


l0l9 C 1027 C I 035 B 1042 C 1049 B 1056 D 1063 F
1020 C 1028 E I 036 E 1043 B 1050 A 1057 C 1064 C
l02l E t029 E 1037 C 1044 B I 051 A I 0s8 A 106s C
1022 E 1030 B 1038 D 1045 A 1052 B I059 B 1066 A
1023 B l03l A 1039 E 1046 A 1053 D 1060 A ta67 A
1024 E 1032 E 1040 B 1047 A r054 E I 061 B I068 B
t02s B 1033 E 1041 A 1048 E r 055 F 1062 D 1069 D
1026 B 1034 A

GYNECOLOGICAL ONCOLOGY
1070 C I 084 D l 098 D ll 2 B I 26 E 1 139 D I 52 D
1071 E I 085 A 1099 C ll 3 A I 27 A I 140 E I 53 B
1072 D 1 086 E l 100 B ll 4 C I 28 B tt41 D I 54 D
I 073 B I 087 D I l0l B 11 5 D I 29 A tr42 B I 55 B
r074 A 1088 C lt02 C 1l 6 C 1 30 C I 143 A I 50 C
1075 B 1089 E I 103 B lt 7 C 3l A tt44 A I 57 A
1076 E I 090 E 1104 E ll 8 B I 32 A I 145 E I 58 B
1077 C 1091 A 1105 E 1l 9 C 33 D 1146 B I 59 B
1078 B t092 D 1 106 E tt20 A 34 D tt47 B I 60 C
1079 C l 093 D r 107 D 1727 A 35 B I 148 B I 6t F
1080 A 1091 A I 108 C rt22 B I 36 A tt49 D I 62 H
l08l B 1095 B r 109 D tt23 E I 3l C I150 A I 63 F
1082 C 1096 A 1110 E rt24 D I 38 E I 151 C I 64 D
1083 E 1097 C Illt C tt25 B
WhiteKnightLove

-l6r-
Freely you have received; freely give.

REPRODUCTIVE ENDOCRINOLOGY AND INFERTILITY


1I65 C I 180 E I 195 B L2t0 A 1225 B 1239 D 1253 D
1 166 C I 181 D I 196 E tztt E 1226 C 1240 C 1254 D
lt67 A 1 182 B lt97 E t2t2 C 1227 B t24l C 1255 E
I 168 E I 183 E 1198 B t2t3 E 1228 A 1242 E 1256 E
I r69 A 1 184 A tt99 B tzt4 E 1229 C 1243 B 1257 B
I 170 E 1 18s E 1200 C 1215 E 1230 B 1244 D 1258 E
tt7t E l 186 E 1201 B t2t6 C t23l A 1245 C t2s9 F
tt72 B 1 187 B 1202 E t2t7 D 1232 B 1246 c 1260 D
1173 E 1188 D 1203 B t2t8 A 1233 C 1247 A t26t C
lt14 D I 189 A 1204 A t2t9 D 1234 D 1248 D t262 E
tt75 C I 190 E 1205 C 1220 C t235 D 1249 c 1263 B
tt76 B I 191 E 1206 C 1221 D 1236 C 1250 C 1264 B
tt17 B tt92 A 1201 E 1222 E 1237 C t25t B 1265 A
tt78 E 1 193 C 1208 D 1223 E 1238 C 1252 B t266 E
tt79 C tt94 B 1209 B t224 B

FAMILY PLANNING
1267 E 1275 B 1283 E 1290 E 1297 E 304 D l31l E
1268 E 7276 E 1284 E t29l E 1298 B 305 A t3t2 F
t269 E 7277 D 1285 C 1292 C t299 E 306 B 1313 E
1270 A 1278 D 1286 D 1293 E 1300 E 307 D 1314 F
t27t A 1279 E 1287 E 1294 E t30l D 308 A 1315 A
1272 E 1280 D 1288 B 1295 E 1302 C 309 D 1316 A
1273 E 1281 E 1289 A 1296 E 1303 E 310 A L3t7 E
1274 B 1282 E

WhiteKnightLove

-162-
'It is more blessed to give than to receive.

WhiteKnightLove
Freely you have received; freely give.

OBSTETRICS

NORMAL PREGNANCY

llyq,plogy of Concepti on
Maternal Physiology During Pregnancy
Diagnosis of Pregnancy
Antenatal Care

Write short essav/notes on:


I - Morphology of the placenta at term.
2- Structures of the mature villus of the placenta
3- Functions of the placenta.
4- The umbilical cord.
5- Liquor arnnii.
6- The cardiovascular adaptation during pregnancy.
7- Diagnosis of pregnancy in the first trimester (Symptoms, Signs and investigations).
8- Diagnosis of 8 weeks pregnant woman.
9- The subjective symptoms of early pregnancy.
10-Pregnancy tests.
I 1- Antenatal care (objectives and components).
l2- Uses of U/S in obstetrics.
I 3- B iophysical profi le.

Enumerate:
l- The possible sources of arnniotic fluid.
2- Sure signs ofpregnancy.
3- Uterine signs of pregnancy in the first trimester (five signs).
4- Imporlant causes for proteinuria during pregnancy (four causes).
5- Routine laboratory tests to be done at the first antenatal visit.
6- Parameters of fetal biophysicalprofile.
7- Indications of anrniocentesis.
8- Causes of oversized uterus.
9- Risk factors in pregnancy that define it as high risk?
WhiteKnightLove

-1 63-
'It is more blessed to give than to receive.

NORMAL TABOR

Aoolied Basic Sciences


Manaqement of Labor
Uterine Stimulants
Analgesia and Anesthesia in Labor

Write short essay/notes on:


l- Anatomy and diameters of the female pelvic inlet.
2- The diameters of the inlet in normal gynecoid pelvis,
3- The pelvic outlet.
4- Signs of placental separation.
5- Methods of placental separation.
6- Management of third stage of labor.
7- Management of third stage of uncomplicated labor.
8- Apgar score.
9- Criteria of intrapaftum fetaldistress
10- Management of a newborn with one minute Apgar score.
ll-Caputsuccedaneum.
l2- Indications & techniques and complications of episiotomy.
l3-Uses of ecbolics in 3rd stage of labor.
l4- Uterine stimulants (oxytocin, prostaglandins & ergometrine).
15- Oxytocin in obstetrics.
16-Methods, dosage & contraindications of ergometrine use in the 3rd stage of labor.
l7- Signs of labor.
l8-The principles of active management of labor.
19- Diagnosis & rnanagement of the I st stage of normal labor.
20- The partogram.
2l- Onset of labor at term pregnancy.
22- Diagnosis of the onset of I st stage of labor.
23- Uterine contractions during labor? How would you monitor these contractions and what is
the significance of this monitoring?
24-The four main types of female pelvis. Table is recommended.
25- Causes of non-engagement of fetal head at term. How you would investigate for each cause?

Enumerate
l - Important diameters in the female pelvis related to the mechanism of labor.
2- What are causes prolonging the 2nd stage in labor in vertex presentation?
3- Ecbolic drugs used to induce aborlion and stimulate uterine action during labor and to avoid
atonic PPH after labor (mention dosage).
WhiteKnightLove

-164-
Freely you have received; freely give.

OBSTERICAL COMPTICAIONS DURING PREGNANCY

Aborlion
Ectopic Pregnancy
Gestational Trophoblastic Diseases (GTD)
Antepartum Hemorrhase (APH)
Hypertensive Disorders During Pregnancy'
Rh Iso-lmmunization Durins Presnancy
Pre-Term Labor
Premature Rupture of the Membranes GROM)
Post-Term Pregnancy
Polyhydramnios
Olisohydramnios
Fetal Growth and its Disorders

Write short essav/notes on:


I - Types & management of abortion.
2- Immunological factors in abortion.
3- Diagnosis and treatment of threatened abortion.
4- Inevitableabortion.
5- Missed abortion
6- Septic abortion
7- Recurrent abortion. Investigations, etiology and management.
8- Cervical incompetence.
9- Evaluation of cervical incompetence as a cause of habitual abortion.
l0- How you would investigate habitual abortion in the second trimester of pregnancy?
I 1- Differential diagnosis of bleeding in early pregnancy.
l2- Differentia[ diagnosis and management of l st trimester abortion,
13- Differential diagnosis between threatened abortion and ectopic pregnancy.
l4- Diagnosis, differential diagnosis and treatment of acutely disturbed tubal pregnancy.
15- Symptoms & signs of ectopic pregnancy
1 6- Fate of tubal ectopic pregnancy.

l7- DifferentiaI diagnosis of types of tuhal pregnancy


18- Update diagnosis and management of ectopic pregnancy.
l9- Modern treatment of ectopic pregnancy.
20- Differential diagnosis of acutely disturbed ectopic pregnancy.
2l- Clinical picture of molar pregnancy
22- Diagnosis & complications of molar pregnancy
23- Diagnosis & treatment of gestational trophoblastic disease
24- How do you manage a patient pregnant 34 weeks complaining of bleeding from genital tract
25- Treatment of antepartum haemorrhage
26- Cornplications of accidental hemorrhage.
27-Differential diagnosis between placenta previa and accidental hemorrhage.
28- Classification of placenta previa.
29- Diagnosis of placenta previa.
WhiteKnightLove

30- Management of placenta praevia


3l-Vasaprevia.
32- Diagnosis of preeclampsia.

-165-
'It is more blessed to give than to receive.

33- Symptoms & signs of pr-eclampsia.


34- Complications of preeclampsia.
35- Management of pre-eclampsia.
36- Differential diagnosis & management of pre-eclampsia.
37- Stages of eclamptic fits.
38-The eclamptic fit and its treatment.
39- Management of eclarnpsia.
40- Domestic (at home) management of a case of eclampsia.
4l- Indications to induce labor prematurely in a case of preeclamptic toxemia
42- Indications & methods of pregnancy termination in pregnancy induced hypertension
43- Management of non sensitized and sensitized Rh negative primigravid woman.
44- Causes of preterm labor.
45- Treatment of cases with established diagnosis of preterm labor?
46- Management of a case of premature rupture of membranes during pregnancy.
47- Pharmacokinetics of tocolytics.
48- Post maturity.
49- Polyhydramnios.
50- Macrosomia (etiology, diagnosis, complications and management).
5l - Causes and diagnosis of intrauterine growth retardation (f UGR).
52- Management of intrauterine growth retardation.

Enumerate:
l- Clinicaltypes of spontaneous abortion.
2- Criteria of diagnosis of a case with threatened abortion.
3- Causes ofrecurrent abortion.
4- Risk factors for ectopic pregnancy.
5- Possible outcomes of tubal pregnancy.
6- Varieties of clinical presentation of tubal pregnancy.
7- Causes of antepartum hemorrhage.
8- Risk factors of accidental hemorrhage.
9- Maternal and fetal cornplications of accidental hemorrhage.
l0- Criteria of severity of preeclampsia.
I I - Cornplications of preeclampsia.
l2- Complications of eclampsia.
l3-Bad prognostic signs of eclampsia.
14- Differential diagnosis of eclamptic fits
l5- Causes of preterrn labor.
l6-Complications of premature preterm rupture of fetal membranes.
l7- Etiotogical factors of polyhydramnios
I 8- Cornplications associated with polyhydramnios.
I 9- Causes of oligohydramnios.
20- Complications of oligohydramnios
2l - EtiologicaI factors of intra-uterine growth restriction.
WhiteKnightLove

-166-
Freely you have received; freely give.

NON.OBSTERICAT COMPLICAIONS DURING PREGNANCY

Minor Disturbances Durins Presnancv


Diabetes Mellitus Durine Presnancy
Gastrointestinal Disorders During Pregnancy
Intrahepatic Cholestasis of Presnancy
Renal and Urinary Tract Disorders During Pregnancy
Anemia During Pregnancy
Cardiac Diseases During Pregnancy
Thyroid Diseases During Pregnancy
Respiratory Disorders Durins Presnancy
Thromboembolic Diseases During Presnancy
AbdominaI Pain Durins Presnancv
Common Sureical Conditions in Presnancv
Druss and Medications Durins Presnancv

Write short essay/notes on:


1- Vomiting in early pregnancy.
2- Infant of diabetic mother, clinical features and liable complications.
3- Cornplications of diabetic pregnancy.
4- Management of hyperemesis gravidarum.
5- Iron deficiency anemia with pregnancy.
6- Treatment of iron deficiency anemia during pregnancy.
7- Differential diagnosis of acute abdomen during pregnancy

Enumerate:
l- Reasons to explain the statement" PREGNANCY IS DIABETOGENIC" (4 reasons).
2- Stigmata for potential D.M.
3- Causes of hyperemesis gravidarum.
4- Fetal and neonatal complications of diabetic pregnancy.
5- Predisposing causes of acute pyelonephritis during pregnancy.
6- The cardiac conditions that contraindicate pregnancy & necessitate termination.
7- Causes of acute abdominal pain during third trimester of pregnancy.
8- Causes of abdominaI pain during pregnancy,

WhiteKnightLove

-167-
'It is more blessed to give than to receive.

ABNORMAL TABOR

Occipito-Posterior Position
Breech Presentation
Face Presentation
Brow Presentation
Shoulder Presentation (Transverse and Oblique Lie)
Cord Presentation and Cord Prolapse
Complex Presentation
Multiple Pregnancy
Abnormal Uterine Action
Contracted Pelvis and Cephalopelvic Disproportion
Shoulder Dystocia
Soft Tissue Obstruction
Obstructed Labor
Obstetric Trauma
Prirnary Postparlurn Hemorrhage
Secondary Postpartum Hemorrhage
Subinvolution of the Uterus
Retained Placenta
Acute Inversion of the Uterus
Consumptive Coasulopathv in Obstetrics
Amniotic Fluid Embolism
Shock in Obstetrics

Write short essav/notes on:


l- Etiology, diagnosis and management of occipto-posterior position of the fetus in labor.
2- The mechanism of labor in occipitoposterior presentation of the fetus?
3- Causes of breech presentation.
4- Diagnosis of breech presentation.
5- Managenrent of a case with breech presentation.
6- Aftercoming head in breech delivery.
7- Delivery of aftercorning head in breech presentation.
8- Retained aftercoming head of breech.
9- After coming head of breech nrechanism, management (cornplicated & uncomplicated),
10-Mechauism of labor, management & conrplications of breech presentation.
I I- Causes of arrest of breech presentation on the pelvic outlet.
12- Breech presentation with extended legs.
l3- Complications of vaginal breech delivery.
14-Fetal morbidity & mortality in breech delivery.
15- Diagnosis and management of face presentation.
16- Etiology, clinical picture and treatment of shoulder presentation.
l7- Managernent of cord prolapse.
l8- Etiology, diagnosis, management & cornplication of rnultifetal pregnancy.
l9- Conrplications of rnultiple pregnancy.
WhiteKnightLove

20- Management of retained second twin.


21- Deep transverse arrest of the head.
22-Contracted outlet.

-168-
Freely you have received; freely give.

23- Management of labor in patient with contracted outlet.


24- Diagnosis and treatment of cephalopelvic disproportion.
25-Macrosomia: definition, risk factors, complications, diagnosis & treatment.
26- Hydrops fetalis,
27-Cervical dystocia.
28- Management of cervical dystocia.
29- How can you manage an ovarian cyst detected at 8 weeks, at 24 wks & 34 weeks of
pregnancy and if detected irnmediately after delivery.
30- Causes of ruptured uterus
3 l- Signs and syrnptoms of impending rupture of uterus?
32- Causes of perineal tear during labor.
33- Diagnosis & management of complete perineal tear immediately after labor,
34- Complete perineal tear.
35- Complications of 3rd stage of labor.
36- Causes, management of postpartum haemorrhage. Outline your treatment
37- Etiological factors and treatment of primary postpartum haemorrhage.
38- Diagnosis & treatnrent of atonic PPH page
39- Traumatic PPH
40- Retained placenta,
41- Retained separated placenta.
42- Causes and treatment of secondary postpartum hemorrhage.

Enumerate:
l- Four procedures you will follow to deliver a primigravida with deep transverse arrest of the
head into the pelvis with fully dilated cervix. Mention the best and state why?
2- Etiological factors of occipito-posterior position of the fetal head.
3- Factors that hinder forward rotation of the head in occiptoposterior position.
4- Etiological factors of breech presentation.
5- Methods for delivery of each of the following parts of breech: buttocks, shoulders, legs,
head.
6- Etiological factors of transverse lie.
7- Complications of twin pregnancy.
8- Causes of macrosomia.
9- Causes prolonging the second stage of labor in vertex presentation.
l0- Complications of prolonged labor.
I l- Causes of arrest of the head into the pelvis during Iabor?
12- Classification of the abnonnal uterine action.
I 3- Classification of cephalo-pelvic disproportion & name the mode of delivery in each degree.

14- Causes of obstructed labor.


l5- Complications of obstructed labor.
l6- Complications of ruptured uteus.
l7-Causes of primary postpartum hemorrhage.
I 8- Causes of secondary post paftum haemorrhage.
l9- Mention causes of retained seprated placenta.
20- Causes of shock in obstetrics.
WhiteKnightLove

-169-
'It is more blessed to give than to receive.

NORMAT AND ABNORMAT PUERPERIUM

Puerperal Pyrexia
Pueroeral Sepsis
Breast Disorders During Puerperium
Maternal Moftality

Write short essav/notes on:


l - Puerperal Pyrexia.
2- Causes of puerperaI pyrexia (five methods)
3- How would you investigate pyrexia after labor.
4- Diagnosis & treatment of pueiperalsepsis

Enumerate:
l- Causes for puerperal pyrexia related to the process of delivery (mention 4).
2- Causes of puerperal pyrexia.
3- Four predisposing factors for puerperal infection.
4- Causes that hinder the internal os to close and the uterus to become a pelvic organ after
delivery, State when does the internal os close after delivery? & when does the uterus
become a pelvic organ after normal labor. Mention

THE FETUS AND THE NEWBO

Fetal Birth Iniuries


Perinatal Mortality
Fetal Distress
Neonatal Resoiratorv Distress and Neonatal Resuscitation
Coneenital Fetal Anomalies
Neonatal Jaundice
Fetal Therapy

Write short essay/notes on:


I- How to improve the maternal mortality rate,
2- How to improve neonatal outcome.
3- Cephalohematoma.
4- Still birth rate & neonatal mortality rate & perinatal mortality rate.
5- Causes, diagnosis and treatment of intrauterine fetal demise.
6- Diagnosis of intrauterine fetal death.
7- Radiological finding of intrauterine fetal death.
8- Neonatal asphyxia.
9- Treatment of asphp<ia livida.
l0- Respiratory distress syndrome of the newborn.
WhiteKnightLove

I I - Causes ofjaundice in the newborn within the I st week of delivery.

-170-
Freely you have received; freely give.

Enumerate:
l- of maternal mortality in Egypt versus developed countries (in order of frequency).
Causes
2- Fetal birth injuries.
3- Causes of intrauterine fetal dernise.
4- Enumerate methods of diagnosis of intrauterine fetaldeath.
5- Serologicaltests carried for a case of repeated IUFD in late pregnancy.
6- Causes of fetal distress.
7- Causes of neonatal convulsions shortly after delivery.
8- Four common congenitalfetalmalformations (in order of frequency).
9- Causes of elevated maternal serum alpha fetoprotein.
l0-Possible malformations inflicted to the fetus of a mothsr who contacted German measles
during the I st trimester of pregnancy.

OPERATIVE OBSTETRICS

Induction of Labor
Instrumental Assisted Vaeinal Deliverv in Modern Obstetrics
Cesarean Section

Write short essay/notes on:


I - Induction of labor
2- Methods for induction of labor
3- Indications for forceps delivery.
4- Complications of forceps delivery.
5- Failed forceps.
6- Indications & complications of vacuum extraction.
7- Indications of cesarean section.

Enumerate:
l- Prerequisits of applications of the obstetrical forceps.
2- Postoperative and the long-term complications of C.S.
3- Indications of cesarean hysterectomy?

WhiteKnightLove

-t7 t-
'It is more blessed to give than to receive.

GYNECOLOGY

ANATOMY AND EMBRYOTOGY

Anatomv of the Female GenitalTract


Development of the Urosenital System
ital Malformations of the Female Uroeenital S

Write short essav/notes on:


l- Lymphatic drainage of the vulva.
2- Anatomy of Bartholin gland
3- Anatomy of the vagina.
4- Anatomy & function of pelvic floor muscles.
5- Anatomy of levator ani muscle.
6- Development of the ovary.
7- Embryology of the ovary.
8- Septate uterus.
9- Imperforate hymen.
I 0- Clinical presentation of imperforate hymen.

Enumerate:
l - The lyrnphatic drainage of the cervix.
2- The masses that could be felt in Douglas pouch. Define Douglas pouch.
3- The anatomical structures between the 2 layers of the broad ligament.
4- Cornplications of fernale genital mutilation.

WhiteKnightLove

-172-
Freely you have received; freely give.

REPRODUCTIVE ENDOCRINOLOGY

Physio loey of Fernale Reproduction


Physioloey of Menstrual Cvcle
Normal and Abnormal Puberty
The Menopause and Hormonal Replacement Therapy
Hyperandroginism
Disorders of Ovulation:
- Anovulation
- Polycystic ovarian disease (PCOD)
- Luteal Phase Defect
- Luteinized unruptured follicle
Hyperprolactinemia
Hormone and Anti-Hormone Administration in Gynecoloeical Practice
Intersex
Female Sexual Dysfunctions

Write short essav/notes on:


l- Symptoms, signs of ovulation.
2- Development of secondary sexual characters at puberty.
3- Types and TTT ofprecocious puberty
4- Management of normal menopause.
5- Osteoporosis definition, diagnosis & treatment.
6- Causes of hirsutism.
7- Causes of anovulation
8- Diagnosis of anovulation.
9- Induction of ovulation.
l0-The long term complications and treatment of polycystic ovarian disease.
I l - Polycystic ovarian syndrome.
I 2- Hyperpro lactinemia.

I 3- Management of hyperprolactinemia.

l4- Estrogen uses in gynecology.


I 5- Prolactin.
I 6- Conadotropin releasing hormones.

Enumerate:
I- Causes of anovulation.
2- Causes of hyperprolactinemia.
3- Causes of disordered puberty,
4- Predisposing conditions for luteal phase defect.
WhiteKnightLove

-173-
'It is more blessed to give than to receive.

GYNECOLOGICAL DIAGNOSIS

Clinical oach of the gynecological ent


Diasnostic and investisations

Write short essay/notes on:


l- Pap smear
2- Vaginal smear.
3- Hysterosalpingography.

Enumerate:
I- Indications and prerequisites for Pap smear.
2- Contraindications and complications of hysterosalpingography.

MENSTRUAL DISORDERS

Premenstrual Syndrome (PMS)


Dvsmenorrhea
Menstrual Headache
Amenorrhea
Abnormal Uterine B leeding

Write short essay/notes on:


I - Spasrnodic dysmenorrhea.
2- Secondary (congestive) dysmenorrhea.
3- Causes and management of amenorrhea.
4- & treatment).
Secondary amenonrrhea (causes, investigations
5- How to proceed with investigation of a case of Zry amenorrhea.
6- Cryptomenorrhea.
7- Diagnosis of Turner's syndrome.
8- Differential diagnosis & management of menorrhagia.
9- Management of 1ry amenorrhea.
l0- Dysfunctional uterine bleeding (types and management).
I 1- Causes of perimenopausal bleeding.
l2-Definition, etiology, pathology, investigation & management of postmenopausal bleeding.
l3- How would you investigate a case of postmenopausal bleeding
14- Contact bleeding.
l5- Differential Diagnosis of post coital bleeding

Enumerate:
l- Methods of pain control in primary (spasmodic) dysmenonhea (mention five).
WhiteKnightLove

2- Classification of functional hypothalamic amenorrhea.

-174-
Freely you have received; freely give.

GENITAL DISPLACEMENT

Genital Prola
Retroversion - Retroflexion of the Uterus
Inversion of the Uterus

Write short essav/notes on:


I - Supports ofthe uterus'
2- Types of genital ProlaPse.
3- The etiology &/or prevention of genitalprolapse
4- Clinical picture of fernale genitalprolapse
5- Treatment of genitalProlaPse'
6- Uterine/utero-vaginaI prolapse.
7- Symptoms of uterine/genital prolapse.
8- Causes and treatment of uterine prolapse.
9- Types and cornplications of uterovaginal prolapse'
10-Treatment of 2nd degree uterine prolapse.

Enumerate:
1- Types of female genital prolapse.
2- Types and treatment of female genitalprolapse'
3- The degrees and etiology of genital prolapse.

INFERTILITY

Write short essav/notes on:


l- Fernale causes of infertilitY.
2- Causes and investigations of uterine factor in infertility.
3- Causes and diagnosis of tubal factor in infertility'
4- Tubal factor in infertilitY.
5- Diagnosis/detection of ovulation'
6- Write short notes on immunological factors in infertility.
j- How can you investigate and treat a case of secondary infertility in a patient 30
years old

who delivered once 5 Years ago.


8- Treatment of bilateraltubal block.
9- Investigations of a couple with lry &Zry infertitity'
10-The use of ultrasound imagining in the investigation of infertility.

Enumerate:
1- Causes of cervical factor in infertility.
2- Causes of tubal and peritoneal factor in infertility'
WhiteKnightLove

-175-
'It is more blessed to give than to receive.

GENITAT TRACT INFECTIONS

Lower Genital Tract Infections


Upper Genital Tract Infections
Sexuallv Transmitted Diseases
Tuberculosis of the Female Genital Tract
Schistsomiasis of the Female Genital Tract

Write short essav/notes on:


l- Give Differential diagnosis of vaginal discharge.
2- Causes ofexcessive vaginal discharge
3- Etiology of leucorrhea.
4- Management of leucorrhea.
5- Itchy vaginal discharge in a woman in the child bearing period (diagnosis and treatment).
6- Vulvovaginitis of children.
7- Microbial vulvovaginitis during childbearing period.
8- Candidal vulvovaginitis (rnoniliasis),
9- Diagnosis and ttt of rnonilial and trichornonal vaginitis,
I 0- Fungal vulvovaginitis.

I l- Trichornonas vaginitis.
l2- Bacterial vaginosis (Gardenerella vaginalis). Mention its complications
l3- Diagnosis and treatment of pelvic inflammatory disease.
14- Diagnosis and treatment of acute pelvic inflammatory disease.
15- Etiology of actue salpingitis
16- Diagnosis and treatment of chronic pelvic inflammatory disease.
l7- Pathological types of chronic salpingitis.
18- Diagnosis & treatment of gonoccocal infection of primary sites.
19-Complications and treatment of genitalchlamydial infection in female genitaltract.
20- Chlamydial infection in the female genital tract.
21- Diagnostic features of female genital tuberculosis.

Enumerate:
l- Causes of vaginaI discharge.
2- Causative organisms of the acute pelvic inflammatory disease (PID).
3- Regimens for antibiotic administration for cases of acute pelvic inflammatory disease,

TRAUMA TO THE FEMATE GENITAL TRACT

Write short essay/notes on:


l- Causes of urinary fistula in women.
2-
WhiteKnightLove

Etiology, diagnosis and treatment of vesico-vaginal fistula.


3- Complete perinealtear.
4- OId complete perinealtear.

-176-
Freely you have received; freely give.

GYNECOLOCICAT UROLOGY

Write short essay/notes on:


1- Clinical picture of true urinary incontinence.
2- Investigations of true urinary incontinence.

Enumerate:
l- Types of urinary incontinence.
2- Causes of urinary incontinence in female. (five causes)

BENIGN GYNECOLOGY DISEASES

Benign Diseases of the Vulva and Vagina


Benisn Diseases of the Cervix
Benisn Diseases of the Uterus
Benien Ovarian Neoplasm

Write short essav/notes on:


1- Bartholinitis,
2- Pruritis vulva.
3- Pathology of vulvar ulcers.
4- Differential diagnosis of suspicious cervical ulcer.
5- Cervical erosions.
6- Differential diagnosis of uterine polyps.
7- Symptoms of uterine fibroid tumors.
8- Diagnosis, complications and management of uterine leiomyoma.
9- Complications of fibroid uterus.
l0- Secondary changes of leiomyoma (uterine fibroid).
1 I - Diagnosis of pelvic endometriosis.

I 2- Treatment of pelvic endometriosis.

I 3- Non surgical treatment of endometriosis.


l4- Chocolate cyst of the ovary.
l5- Clinical picture and complications of simple cysts ovarian tumors.
l6- Complications and treatment of ovarian cysts.
I 7- Possible complications of medium sized cysts ovarian swelling?

l8- Signs and symptoms of twisted ovarian cyst?


l9- Dermoid cyst (benign cystic teratoma) of the ovary.

Enumerate:
l- Causes of endometrial polyps.
2- Complications of fibroid uterus.
WhiteKnightLove

-177-
'It is more blessed to give than to receive.

GYNECOLOGICAL ONCOTOGY

Epithelial Abnorrnalities of the Genital Tract


Carcinoma of the Vulva
Carcinoma of the Vagina
Invasive Carcinoma of the Cervix
Gestational Trophoblastic Tumors
Invasive Carcinoma of the Body of the Uterus
Ovarian Malignancy
Radiotherapy
Chemotherapy
Management of Advanced Malignancy

Write short essay/notes on:


l- Discuss diagnosis of preclinical carcinoma of the genital tract.
2- Methods of early diagnosis of genital malignancy.
3- Cervical intraepithelial neoplasia (risk factors, prophylaxis, clinical picture, diagnosis and
treatment).
4- Screening methods for carcinoma of cervix.
5- Early detection & management of preclinical cervical cancer. '

6- Diagnosis of early cancer cervix.


7- Early diagnosis of cervical carcinoma.
8- The clinical presentations of cancer cervix?
9- Differential diagnosis of cervical carcinoma.
l0- Ir4anagenrent of carcinoma in situ of cervix
1 l- Definition of stage I cancer cervix.
l2- Clinical picture & management of cancer cervix
l3- Endometrial hyperplasia (causes, types, complications and rnanagement).
l4- The significance of endometrial hyperplasia? Name the pathological varieties.
l5- Early diagnosis and management of endometrial carcinoma.
16- Etiology, diagnosis and management of endometrial carcinoma.
I 7- Treatment of choriocarcinoma.
18- Diagnosis of ovarian tumors.
l9- Clinical features of malignant changes in ovarian tumors.
20- Complications and treatment of ovarian tumors.
21-Clinical picture, diagnosis and management of ovarian carcinoma.
22-Outline the treatment of this case if the ovarian tumor proves to be:
A-Benign B-Malignan .

Enumerate:
l- Methods of early detection of genital rnalignancy. (five methods)
2- Classification of ovarian germ cell tumours.
3- Diagnostic features of cervical intraepithelial neoplasia.
WhiteKnightLove

-178-
Freely you have received; freely give.

FAMILY PLANNING

Write short essay/notes on:


l- Natural methods of contraception.
2- Immunological factors in contraception.
3- Contraception during lactation.
4- Hormonal contraception (advantages and disadvantages).
5- Contraindications of hormone contraceptives.
6- Types & complications of hormonalcontraception
7- Oral contraceptive pills.
8- Side effects of contraceptive pills.
9- Long acting hormonal contraceptive methods.
I 0- Injectable contraceptive.
I I - What is the inffauterine contraceptive device (IUCD) and its complications.
l2-Types and complications of IUCD.
l3- Contraindications to intrauterine contraceptive device (IUCD) use.
l4- Missed IUCD.
15-Management of missed IUCD
16- Bioactive IUD.
l7- Post coital contraception.
l8- Contraception for newly married woman
I 9- Postpartum contraception.

20-Evaluation of the common methods of contraception that can he used for multipara 35 years
old, who delivered 5 living children? She is clinically healthy.

Enumerate:
l- Types of hormonal contraception & their advantages & disadvantages.
2- Side effects of the combined oralcontraceptive pills.
3- Advantages & disadvantages of subdermal implants
4- Indications, contraindications and complications of IUD
5- Side effects of IUCD
6- The available rnethods for emergency contraception.

SURGICAT OPERATIONS AND PROCEDURES


IN THE GYNECOTOGICAT PRACTICE

Write short essav/notes on:


l- Indications of D& C.
2- Complications of the operation of D&C.
3- Management of uterine perforation during D&C operation.
4- Laparoscopy.
5- Indications of dilatation & curettage.
WhiteKnightLove

6- Complications of D&C operation.


7- Causes of acute abdomen in gynecology.

-179-
'It is more blessed to give than to receive.

INSTRUMENTS

WhiteKnightLove
Freely you have received; freely give.

GYNECOLOGICAL INSTRUMETNS

1. VAGINAL SPECULAE
(l) Cusco's bivalve speculum (Fig. 1):
Types:
l. Fenestrated
2. Not fenestrated.
Indications:
a. During routine clinical exanrination to expose the cervix and vagina while the patient in
the I ithotonry position.
b. During hysterosalpingography, inserlion of I.U.C.D., cauterization or local treatment for
the cervix.

Fig. 1: - Fenestrated Cusco's vaginal speculum Cusco's vaginal speculum

(2) Sims' speculum (Fig. 2):


. It has 2 blades ofdifferent sizes connected together by a bar.
o It is used to expose the anterior vaginal wall in cases of vesico-vaginal fistula, while the
patient lies in Sirns' position. Also used during operative repair of the fistula.

Fig. 2: Sim's speculunr


WhiteKnightLove

-l 80-
'It is more blessed to give than to receive.

(3) Auvard's self-retaining vaginal speculum (Fig. 3):


.It has a grooved handle [for drainage of the blood during the vaginal operations], a heavy
metal ball [either detachable or fixed] and in sorne types 2 holes on each side [for fixation by
towelclips to prevent fallingl.
. It is applied on the posterior vaginal wall of the patient under anesthesia in the lithotomy
position during the most of the vaginal operations.

Fig. 3: Auvard's vaginaI speculunr

2. THE UTERINE SOUND


(Fig. a)
. It is made of pliable rnetal and its handle is corrugated on one side, which is directed towards the
bent at the end. This enables us to know the direction of the tip by inspecting the corrugations on
the handle.
a Marks [in cm or inch] are present on its straight part,
a After clinical exantination and exposure of the cervix; the sound-with its handle loosely held
between the thumb and index fingers-is gently introduced in the cervix.
WhiteKnightLove

Fig.4: The uterine sound

-l8l-
Freely you have received; freely give.

Indications:
1- Determination of the direction of the uterus (AVF or RVF).
2- Measuring the length of the cervix and that of the whole uterus. The first resistance felt is at the
internal os and the second resistance is atthe fundus ofthe uterus.
3- Differentiation between corporeal and cervical polypi.
4- Differentiation between fibroid polyp and chronic inversion of the uterus.
5- Diagnosis of septate uterus or intrauterine foreign body (using X-ray in lateral position).
6- Diagnosis of the presence of I.U.C.D. inside the uterus (using X-ray in lateral position).
7- Testing friability in cancer cervix: the sound is used here as a probe.
8- Diagnosis of vesico-vagina[fistula. (a click is elicited against a nretalcatheter).
Complications:
I - Perforation ofthe uterus.

2- Introduction of infection.
3- Disturbance of pregnancy if already existing.

3. THE CERVICAL DILATORS

HEGAR'S DILATOR (Fig. 5)


Types:
a- Single ended.
b- Double ended.
- It has a uniform thickness and the diameter in millimeters which is written on the handle.
- The tip of the single ended dilators is curved in the direction of the flattened surface of the
handle,
- Two successive sizes are represented in each of the double ended dilators e.g.5,6 and 9,10
etc.

Fig. 5: Hegar's dilator

Indications for cervical dilatation: (SEE APPENDIX 2)


l- Preliminary to curettage [Diagnostic or therapeutic or evacuation of the uterus].
2- Preliminary to operations on the cervix as:
-Operations of the cervix.
-Fothegill, s operation
-Trachelorrhaphy.
WhiteKnightLove

-Cauterization.
3- Prior to insufflation or hysterosalpingography in patients with stenosed cervix.
4- Preliminary to introduction of radium in the uterine cavity.

-182-
'It is more blessed to give than to receive.

5- Drainage of pyometra or hematometra (dilatation only).


6- Treatment of spasmodic dysnrenorrhea if conservative means fail (dilatation only).
- 25%, permanent cure.
- 29%.....,,,.no improvement.
- 50%. variable periods of improvement.
N.B.: Retrograde dilatation of the cervical canal is done through the opened uterine cavity at
laparotomy for myomectomy operation or elective C,S.

Complications of dilatation of the cervical canal: (SEE APPENDIX 2)


1) Perforation ofthe uterus.
- It is more liable to occur if the uterus is softened by pregnancy or friable as in cancer cervix or
corpus or if pyometra or septic abortion is present.
- Undue force or introduction of the dilator in a wrong direction may perforate the uterus.
- The cervix rnay be rarely perforated (false passage).
2) Cervical laceration:
- It is either in the form of tears or splitting of the cervix or rarely it extends laterally to involve
the uterine vessels with sever hemorrhage or hetnatoma forrnation (false passage).
3) Cervical incompetence -- Repeated abortions.
4) Ascending infection -- endometritis, salpingitis, paranretritis and peritonitis.
Management of perforation of the uterus:
. Perforation in a clean operation e.g dysfunctional bleeding or spasmodic dysmenorrhea: stop the
operation * close observation of the patient.
. Pulse, temperature and blood pressure every'/, hour dLrring the lst 6 hours then every 1-2 hours
during 24 hours.
. Development of abdorninal pain or vomiting. As a rule nothing happen except the possibility of
uterine rupture in subsequent labor.
. If perforation occurred in infected cases antibiotics are given.
" If no improvement this rneans either peritonitis or bowel injury and if this is suspected
laparotomy and management according to the findings.

WhiteKnightLove

-l 83-
Freely you have received; freely give.

4. UTERINE CURETTES
(sEE APPENDTX 2)
Types:
1- Sharp curette (Fig. 6).
2- Double curette ISharp and blunt] (Fig. 7).

Fig. 6: Sharp Lrterine curette


Others:
3- Blunt flushing curette [Rheinstadter's curette] (Fig. 8).
4- Fundal curette (Fig. 9).
- It is designed to curette the furrdus of the uterus in fractional curettage for diagnosis of
endometrial carcinoma.
- It has 2 ends of different sizes.
5- Suction curette.
6- Biopsy curettes as Sharman's and Novak's.

,-

Fig. 9: Fundal curette

Indic
I. Diagnostic:
l- Irregular or excessive uterine hemorrhage to establish the histopathological pattern of the
endometrium.
2- To exclude or confirnr the presence of endometrial carcinorla.
3- To establish the occurrerrce of ovulation in infertile patierrt (if it occurred, secretory changes are seen).
4- To diagnose an intrauterine cause for bleeding as endometrial polypi, fibroid polypi or
retained products of conception.
5- To exclude intrauterine pathology in prolapse operations in which the uterus is not to be
removed.
6- To diagnose tuberculous or non specific endometritis.
II. Therapeutic:
l) To evacuate the utenrs in cases of abortion (lnevitable - lncomplete and missed).
2) To rernove retained products of conception.
3) To remove endometrial polypi or smalI myonratous polypi.
4) Sometimes it is effective in certain types of dysfunctional bleeding.

Complications: (SEE APPENDIX 2)


1- Sepsis
2- Hernorrhage
3- Perforation ofthe uterus
WhiteKnightLove

4- Permanent amenorrhea and infertility (over curretage removing the basal layer).
N.B.: Ashenrann's syndrome (endonretrial synac/riae) due to intrauterine adhesions leading to
amenorrhea and inferti I ity.

-184-
'It is more blessed to give than to receive.

5. VOLSELLUM FORCEPS
Types:
a. Single toothed (Fig. t0).
b, Multiple toothed (Fig. 1t):
- They are designed for grasping the cervix during vaginal operations, insufflation, hystero-
salpingorrhaphy and insertion of I.U.C.D.
- N.B: During vaginal circlage although there is pregnancy but we better grasp the cervix by
nrultiple toothed volsellunr instead of ring forceps to avoid its slipping and trauma to the
cervix from the repeated applications.

nU

Fig. 10: Volsellunr forceps (single toothed) Fig. l1: Volsellum forceps (multiple toothed)

6. BONNEY'S MYOMECTOMY CLAMP


(Fig. l2)
It is used for compression of the uterine vessels alongside the uterus just above the internal os
(isthmus), to ensure hemostasis during myomectomy for cornplete hemostasis intestinal clamps
are put on the overiopelvic (infundiblo-pelvic) ligaments to avoid bleeding from ovarian vessels.
The action of myomectomy clarnp could be achieved by encircling the cervix with a tightened
rubbertube passed through 2 holes in bloodless area in the base of the broad ligament.
The clamp should include both round ligaments in its grip.
The clarnp may be left safely for one hour without uterine infarction.
It could result in traunra to the uterine vessels.

Fig. 12: Bonney's myomectonry clarnp


WhiteKnightLove

-1 85-
Freely you have received; freely give.

7. MYOMECTOMY SCREW
(Fig. 13)
. It is used to steady and hold the myomatous uterus during rnyomectomy and hysterectomy.

Fig. 13: Myomectomy screw

8. INSUFFLATION CANNULA
(sEE APPENDTX 3)
Types:
I - The ordinary type (Fig. 14).
2- The cone shaped screw canula [Leech Wilkinson] of different sizes (Fig. l5).
3- Some canulae are kept in place on the cervix by suction.
4- They are used for insufflation or hysterosalpingoraphy.
Signs of tubal patency if insufflation (Co2 gases used):
l- Changes in pressures in manometer or kymography.
2- Hearing of bubbling sounds over the lower abdomen during the test.
3- The patient may experience referred pain at either shoulder-usually the right-when she sits up
after the test due to irritation of the diaphragm.
4- Radiography shows a cresentic gas shadow under diaphragm.

Fig. 14: Insufflation cannula (ordinary type) Fig. l5: Cone shaped screw cannula of different sizes

Contraindications:
1- During and imrnediately before or after the menstruation.
2- Evidence of infection in tlre lower genital tract and presence of purulent discharge,
3- Pregnancy.
4- History of recent salpingitis.
5- Suspected tuberculosis of the genital tract.
WhiteKnightLove

-1 86-
'It is more blessed to give than to receive.

9. UTERINE HOLDING FORCEPS


(Fig. 16)
. It is used to hold the rrterus during abdominal gynecological operations.

Fig. 16: Uterus holding forceps

10. INTRAUTERINE CONTRACEPTIVE DEVICES (IUCDS)


(Fig. 17) (SEE APPENDIX 4)

Fig. 17: Intrauterine contraceptive devices left to right are


lippes loop (size D), CLr 7, and copper T (380A)
a These are devices introduced inside the uterine cavity for contraception,
a The most commonly used are the polyethyelene types- impregnated into barium to be radio-
opaque as lippes loop, saf-T-coil, Margulies spiral and Binberg bow.
Other materials used for rnanufacturing the devices are: stainless-steel [as Majzlin spring and M.
devicel, nylon and wire.
The devices are sterilized by imrnersion in an antiseptic for l5 rninutes as l%o savlon or l:1.000
chlorhexidine ortincture iodine (old lines) or by gamma radiation as Saf-T-coil (used nowadays).
a There is a special applicator for the device.
a Recently medicated loops are used as:
(1) Copper Medicated Loops:
- The copper is either in the form of wire or multiple sleeves.
- The figure following the narne of the device means the surface area of the copper in the loop in mm2.
Types:
- Copper T loops: (the most commonly used nowadays is CuT 380A),
- Copper 7: Gravigand.
- Lippes loops with copper:
(2)Progestasert: Progesterone medicated T Ioop.
WhiteKnightLove

-l 87-
Freely you have received; freely give.

Contraindications: (SEE APPENDIX 4)


l- Pregnancy or suspicion ofpregnancy.
2- Pelvic inflammatory disease [PID] acute, subacute or recurrent.
3- Septic abortion or post parturn endonretritis in the last 3 months.
4- Disturbed configuration of the endometrial cavity by rnyoma or congenital anomaly in the
uterus.
5- Unexplained uterine bleeding.
6- Suspicion or presence of genitaI malignancy.
Instructions for the patient following application of the device:
l- During the first few months spotting or heavier menstrustion may occur.
2- The cramps are treated by niild analgesics and they will become rnilder during the following
periods.
3- She must report immediately if:
- The device is expeled partially or completely or the tlrreads are not felt protruding in the
vagina.
- There is rise in tenrperature.
- Any purulent vaginal discharge.
-The position of the device should be checked first after the two periods then every 6-12
months later on.
Complications:
l- Bleeding- the most frequent side effect. It is due to:
- Damage to the endornetrium by pressure.
- Increased vasculality and congestion of the endometrium.
- Local derangement of the blood clotting mechanism.
2- Pain: Crarnp-like pain-due to increased uterine contractility in a trial to expel the loop.
N.B: These 2 side affects may necessitate removalor the device if uncontrolled.
3- Expulsion: Partial or complete.
4- Pelvic inflamnratory disease [PID].
5- Ernbedding in the endometrium which result in:
- Reduction in the contraceptive efficacy.
- Difficult renroval.
6- Pregnancy:
7- Perforation of tlre uterlrs.

PREGNANCY WHILE IUCD IS IN SITU:


A. Intrauterine:
- With the possibility of abortion [spontaneous or septic] or continuation of pregnancy.
B. Extruterine: More than normal population.
Other considerations:
o If intrauterine pregnancy occurs there is no suggestion that the device cause congenital
abnorniality of the fetus.
. No evidence that tliere is increase in the incidence olcervical or endometrial malignancy with
the I.U.C.D.
o The fertility of the patient returns in a short time after removal of the device, but it may be
WhiteKnightLove

delayed due to infections as PID caused by IUD.

-l 88-
'It is more blessed to give than to receive.

PERFORATION OF THE UTERUS:


A. Fundal B. Cervical.

A. FUNDAL PERFORATION:
It occurs more if the uterus is soft and friable as in puerperal uterus or if the push-out
technique is used rather the withdrawal technique:
Management of Extrauterine IUD:
Diagnosis:
It is suspected when no threads are felt or seen in the cervical os
The diagnosis is confirnred by:
1- Outlining the uterine cavity by radio opaque liquid.
2- Dimensional X-ray are made if the loop appears beyond the limits of the filled of the uterus,
it is extrauterine.
3- Placing a metal sound in the uterine cavity and X-rays are made.
4- Sounding the uterine cavity to feel if it is present or not.
Surgical removal ifr
I- The device is of the closed type to avoid bowel strangulation.
2- The device is of the multifilament type [as Dalkon Shield and Maizlin Spring] as they are
farnous to cause sepsis.
3- The device is of the nredicated types-copper or progsterone-to avoid tissue reaction in the
peritoneum.
Methods of removal of the device:
l- Laparotomy
2- Posterior colopotomy
3- Laparoscopy
Could the device be left inside??
If the device is not of the previor.rs types and no anxiety fiom the patient's side, it can be left.

B. PERFORATION OF THE CERVIX:


. It occurs when the lower tip of the loop irnpinge on one of the endocervical region and
gradual ly perforating it.
. Sorne devices have an enlarged tip to mininrize this complication and also to prevent
downward d i sp lacernent.
. The treatment of cervical perforation is removal of the device after dislodging its tip by
pressing upwards on it.

WhiteKnightLove

-189-
Freely you have received; freely give.

11. FEMALE URINARY CATHETERS


(Fig. l8)
Types:
l- Female metal catheter.
2- Rubber catheter or plastic catheter (Neleton's catheter),
3- Special types e.g. (Foley's self retaining catheter).

rubbu
Fig. 18: Female catheter

Indications: the catheter is used in:


I' Postoperative or post partum evacuation of the bladder specially in perineal operations.
2- Cases of retention of urine as occurs in:
- Pelvic tumours (cervical fibroid, ovarian or broad ligarnent tumour, uterine fibroid).
- Pelvic heniatocele,
- Hematocolpos,
- Retroverted gravid uterus.
-During labor.
3- Evacuation of the bladder before any gynecological or obstetrical operation or examination.
4- Diagnosis of vesico-vaginal fistula:
a) By metal catheter click against a metal probe or sound introduced from the fistulous opening.
b) By methylyene blue test injected through the catheter in the bladder.
5- Diagnosis of urethral diverticulum.
6- Diagnosis of bladder conditions as injuries or stones,
7- Collection of urine samples for laboratory examination and cultures.

WhiteKnightLove

-190-
'It is more blessed to give than to receive.

APPENDIX 1

ELECTROCAUTARIZATION TO THE CERVIX


- It aims at destruction of the abnormal epithelium.
- Cryocoagulation and Iaser vaporization are alternative techniques.
Indications:
1. Chronic cervicitis.
2. Cervical erosions that cause synrptoms.
Technique:
l. Dilatation of the cervix up to No. 10 Hegar's dilator,
2. Diathermy needle: Linear incisions, at the longitudinal axis of the endocervix and about 8-10
in number, and 1/8 of an inch deep, Nabothian cysts are punctured.
3. Button electrode: to cautarize the portio vaginalis.
Complications of cauterization :
1 - Ascending infection.

2- Secondary hemorrhage.
3- Cervical stenosis.
4- Dysmenorrhea.
5- Infertility.
6- Cervical dystocia, during labor

WhiteKnightLove

- 191-
Freely you have received; freely give.

APPENDIX 2

DILATATION OF THE CERVICAL CANAL


Dilatation of the cervix aims at rnaking the cervical canal patulous, by stretching the muscle and
fibrous tissue of the cervix.
Indications:
A. Preliminary to other operations:
L Curettage of the endometrial cavity or the cervical canal.
2. Evacuation ofthe pregnant uterus.
3. Cauterization in cases of large chronic cervicitis.
4. Polypectomy.
5. Trachelorraphy.
6. Cervical biopsy, especially cone biopsy.
7. Amputation of the cervix:
8. Hysterosalpingography and insufflation: only when the cervical canal is less than the
diameter of the cannula.
9. Hysteroscopy: occasional[y when the canal is smaller than the hyseteroscope
10. Vaginal myomectorny.
I l. Excision of uterine septum through he lower genital tract.
12, Occasional in few cases before insertion or removal of the IUCD.
13. Insertion of hygroscopic devices for induction of labor or abortion.
14. Application of local radiotherapy.
B. Therapeutic:
l. Spasrrodic dysnrenorrhea:
2. Cervical stenosis:
- Traumatic: cervical synechia.
- Congenital: pin hole os.
3. Drainage of hematonretra and pyometra.
Contraindications:
- Active local infection: acute cervicitis, acute endometritis, pelvic inflammatory disease.
- Norrnal intrauterine pregnancy.
Complications:
1. Spread of local infection: to cause salpingitis or peritonitis.
2. Rupture tubal or ovarian abscess.
3. Rupture pelvic adhesions.
Methods of dilatation of the cervical canal:
Mechanical dilatation is the traditionaI and most comnron rnethod used. It is done by a series of
graduated metal dilators that are passed into the cervical canal. The smallest one in the set is
Nurnbu, 3.
The requisite amount of dilatation is related to the indication and No. l0 Hegar's dilator is
usually sufficient in most of the cases (7-10 for curettage, l2 for cervical amputation, 14 for
spasmodic dysrnenorrhea).
WhiteKnightLove

Mechanical dilators:
Hegar dilator: uniform thickness. May be single ended or double ended.

-192-
'It is more blessed to give than to receive.

Other Methods of dilatation of the cervical canal:


The following methods achieve softening and gentle and non-traumatic dilatation of the cervix.
- Prostaglandin derivatives: Soften the unripe (unfavorable) cervix with PGE2 or its analogue.
Local application of intra-cervicalor intra-vaginal preparations are available.
- Hygroscopic dilatation: They are used when the cervical canal is not effaced and closed
especially after the twelve weeks of pregnancy.
Mechanism of action of hygroscopic dilators:
l. Mechanical action: The devices have a hygroscopic action and slowly swell up to about three
times the original diameter.
2. Increase in blood levels of prostaglandin metabolites.

CURETTAGE
Principle:
It is scraping off the inner lining of the uterine cavity, The procedure is performed usually
through the cervical canal.
Indications:
. Diagnostic purposes:
- To determine the endometrial pattern when there is possible hormonal disorders.
o Abnormal uterine bleeding. Long streaks, smootlr glistening.
o Work-up in cases of amenorrhea/ oligomenorrhea.
o Evaluation of inferlile patients: Assessment of endornetrial changes (premenstrual
endorletrial biopsy PMB).
- Confirmation of local endometrial disease, such as:
o Malignant disease: No gritty sensation, and the obtained tissues are friable.
o Chronic endometritis: T.B., Schistosomiasis.
o Endometrial polyp.
. Therapeutic purposes:
L With evacuation of the pregnant uterus, to remove the decidua.
2. Other pregnancy related conditions: vesicular mole, postabortive bleeding.
3. Endometrial polyps.
4. Dysfunctional r-rterine bleeding: to arrest bleeding and for obtaining tissues for histopathology.
5. Membranous dysnrenorrhea.
6, Small submucous fibroid.
7. Removal of retained contraceptive device.
Types of uterine curettes:
l) Sharp uterine curette: used in most of the cases.
2) Blunt uterine curette: used in certain cases e,g. soft uterus as in malignancy, pregnancy, senile
endometritis.
3) Blunt flushing curette: a warnt antiseptic is used to wash the uterine cavity of any remnants after
cureffage.
4) Suction curette: used with negative pressure to help complete removaI of the intrauterine
contents in evacuation of pregnant uterus and used also with vesicular mole.
WhiteKnightLove

5) Fundal curette: used to reach the remote areas at the fundus, angles and lateral walls.
6) Biopsy curette: could be used without anesthesia and does not aim at removal of all the
endometrial tissue.

-193-
Freely you have received; freely give.

Fractional curettage: Separate samples frorn the cervical canal and each segment of the uterine
cavity are kept separate and labeled accordingly. The obtained information helps in staging and
planning therapy in case of cervical or uterine malignancy is suspected.

SURGICAL EVACUATION OF THE PREGNANT UTERUS


The minimum preoperative laboratory study:
1. Blood typing (ABO and Rh).
2. Hematocrit value.
Start intravenous drip to keep a vein open for emergency adnrinistration of medications.
I. Procedure up to twelve weeks gestation:
l. Dilatation of the cervical canal: Dilatation proceeds to reach a diameter roughly
correlated to the size of the products of the gestation i.e. Hegar 8 for 8 weeks
pregnancy.
2. Evacuation ofthe products ofconception:
a. Mechanical corrventional curettage which is rrechanical scraping out the intrauterine
contents. A sharp curette is more efficacious than dull curette and is not more dangerous,
as was thouglit.
b. Suction curettage, which is vacuum aspiration of tlre intrauterine contents
Advantages of the suction aspiration over meclranical curettage are the higher speed of
operation, lower perforation rate, and less blood loss.

Evacuation of the uterine cavity in incomplete abortion

lI. When the uterinc sizc is greater than 14 weeks, c'r,acuation of the uterus become more
difficult.
L Prostaglandins (PGE2a) or their analogues: Various preparations could be used to
induce effective uterine contractions that mini-labor is painful and subsequent surgical
evacuation of placental tissues is commonly indicated. Vaginal preparations, intra-
amniotic, extra-amniotic or intravenous routes of adnrinistration could be used.
2. Hysterotorny: Mini-cesarean section: The uterine incision is vertical midline as there is
no lower segment yet. The need for hysterotomy dropped sharply after the introduction
of prostagland ins.
WhiteKnightLove

-t94-
'It is more blessed to give than to receive.

DILATATION AND CURETTAGE (D&C)


Principles of the procedure:
l. Dilatation of the cervicaI canal.
2. Exploration of the endornetrial cavity.
3. Removal of the endornetrium.
Anesthesia: Dilatation of the cervical canal is painful and anesthesia is needed. The procedure
could be done under local or general anesthesia.
l. General anesthetic: it allows optimum pelvic examination.
2. Local anesthetic (para-cervical block with or without systemic analgesia).
Technique of D&C:
1. Placement of the patient in the dorsal lithotorny position.
2. Preparation of the operative area:
- Disinfection with general antiseptic solution e.g. chlorohexidine, the external genitalia,
vaginal canal and the ectocervix.
- Draping of the legs and the vulval area with towels.
3. Catheterization of the bladder.
4. EUS including bimanual palpation. Special attention to asceftain the size, consistency, position,
mobility, and direction of the uterus, and the condition of the appendages. Contraindications to
the procedure are looked for.
5. Self-retaining speculum (e.g, Auvard's speculum) is placed in position. Cleansing of the cervix
and the vaginal fornices is insured with iodine, after good visualization of the area.
6. Volsellurn application: For stabilization, manipulation and application of traction on the cervix.
Gentle traction helps to align the canal with the cavity to help easy introduction of the
instruments and to nrinimize the risk of perforation.
7. Sounding the uterus: To confinn the direction and length of the cervical canal and body of the
uterus. Careful rnanipulation may detect an abnormality inside the uterus e.g. fibroid polyp,
septum. The uterine sound is made of a maleable metal and its tip is blunt for fear of accidental
perforation of the uterus. It is graduated in centimeters.
8. Dilatation of the cervical canal: Attention is given to the direction of insertion, length, and the
pressure needed. The dilator is held gently and firmly like a pencil so that the slight curve
confirm with the direction of its axis, to fo[[ow the direction of the uterus. The main resistance
to the dilator is at the internal os. Steady pressure is applied until yielding and entrance of the
dilator inside the cavity. Avoid the sudden uncontrolled thrust of the dilator, especially if the
cervix relaxes suddenly. Each dilator is left in the canal for about half a minute to allow the
circular fibers to relax. Exact sequence is followed by inserting the next larger size
9. Polyp forceps is used to explore for and remove any polyps.
10. Curettings the wall of the uterus: Introduce the curette up to the fundus, then apply the curette
with some force against the uterine wall and draw from above downwards over the internal
surface of the uterus. Curetting is carried out in a systematic manner; the anterior then the
posterior and lastly the lateral surfaces. Particular care is taken at the fundus and cornua of the
uterus. The procedure is stopped when a gritty feel and sound are noticed, which indicate that
the myometrium has been approached. Preservation of the rnaterial obtained is according to the
needed examinatiorr; forrnaline 4 %o for histopathological studies. Culture is rarely needed when
tuberculosis endometritis is suspected
WhiteKnightLove

11. A grasping instrument e.g. sponge forceps or small ring forceps, is introduced into the uterine
cavity, to explore the existence of any pedunculated lesions, as curettage alone may not remove
intra-uterine polypi.

-195-
Freely you have received; freely give.

Complications of dilatation, curettage, and evacuation:


These minor surgical operations are considered invasive procedures, and should not be taken
lightly. They should not be abused or performed without good indications. Aseptic technique is
essential and prophylactic antibiotics are given when appropriate.

1. Uterine perforation: Perforation of the cervicalcanal, orthe body of the uterus, could happen
during introduction of the uterine sound, dilators or curette. Perforation is more likely with the
following conditions:
- Introduction of excessive length of an instrurnent, or wrong direction, or using excessive
force.
- Difficulty in finding the cervical os e,g. carcinoma of the cervix. A false passage could
be imposed on the cervical tissues.
- Soft uterus e.g. pregnant uterus, malignancy, pyometra, A blunt curette is used in these
circumstances.
- Retroverted uterus.
- Small menopausal uterus.
- Inexperience ofthe operator.
Recognition of perforation of the uterus:
- The instrument passes further than estimated.
- Unusual free mobility of the instrument inside the cavity.
- Unusual blood loss, which may be internal and may present as secondary hemorrhage.
- The recognition of onrentum or even intestine through the cervical canal.
Perforation may bc into one of the following sites:
- Extraperitonel and enter the broad ligament or uterovesical cellular tissue
- Pass into the peritoneaI cavity.
Management of uterine perforation:
. Conservative managerrent: Small rent with rninimal or no bleeding: e.g. produced by a
uterine sound or narrow dilator. Midline perforations are less serious than lateral perforations.
- Discontinue the procedure.
- Observations: Ceneral condition, temperature, pulse, fillin in the cul-de-sac.
- Rest in rnodified Fowler's position.
- Antibiotics.
- Special care in case ofsubsequent pregnancy, for fear ofrupture uterus.
. Laparoscopy/[aparotomy.
Indications:
- LateraI perforations.
- Parametrial expanding mass (hematoma).
- Instability of the vital signs.
- Intra-abdominal damage: caused by instruments passed through a uterine defect into
the peritoneal cavity, produced by suction curettes or mechanical sharp curettes.
Bowel or orlrentun.r may appear through the cervical canal.
Procedure:
- Examine the abdominaI contents, especially the bowel,
- Repair of the uterus and any injured tissues.
- Hysterectomy is considered,
WhiteKnightLove

o Infected cases: managed as pelvic peritonitis.

-196-
'It is more blessed to give than to receive.

2. Cervical laceration
Predisposing factors to cervical tears:
l. Too rapid dilatation: especially in nulliparae may lead to lateral tears
2. Forcing the instrunrent against unproperly dilated canal.
3. Very soft cervix (pregnancy), or rigid cervix (nultipara).
4. Too strong pull on the cervix.
5. Dilatation to diameters larger lhan 12 Hegar's.
Sequale of cervical tears:
l, Extension of the tear to the broad ligaments and rupture uterine vessels. May cause
hematoma, or severe bleeding.
2. Cervical infection and erosion.
3. Parameteritis may be acute or chronic.
4. Patulous internalos:this nray happen with cervical dilatation more than l0 mm dilatation.
Management of the cervical tears which complicate D. & C.:
- Irnmediate interrupted sutures.
- Cervical and vaginal packs.
3. Hemorrhage: The plegnant uterus is very vascular.'l'o limit blood loss uterotonic agents are
given intravenously to contract the uterus and close blood vessels penetrating the uterus.
Secondary hemorrhage is especially difficult to manage. It
is managed by pack or
hysterectomy.
4. Incomplete evacuation of the pregnant uterus: Retention of some products of conception is
related to inexperience and advanced pregnancy. Retention may cause bleeding and infection.
Surgical re-evacuation is needed, usually under generaI anesthesia.
5. Infection: Promote early treatment is essential to avoid sequale such as endometritis,
cervisitis, salpingitis, peritonitis, pelvic cellulitis.
6. Overcurettage: Leads to intrauterine synechiae as proplrylaxis avoid removal of the basal
layer of the endometrium or the decidua basalis in case of pregnancy and treat local infection,

WhiteKnightLove

-t97 -
Freely you have received; freely give.

APPENDIX 3

HYSTEROSALPINGOGRAPHY (HSG)

HYSTEROSALPINGO GRAPHY (HSG) :


. HSG is the radiological visualisation of the following:
1- The uterine cavity.
2- The lumens of the fallopian tubes.
3- The spill of the dye from the fimbriated ends into the peritoneal cavity.
4- The pattern of dispersion of the dye into the peritoneal cavity.
Indications of HSG:
o Infertility: This is the rnost common indication. [see infertility]
o Investigation of uterine disease:
I- Congenital malformations
2- Submucous myomas.
3- Endornetrial polyps.
4- Intrauterine adhesions.
Contraindications of HSG:
l- Pelvic mass of unknown nature.
2- Possibility of active pelvic inflammatory disease (PID). Suspicious evidences include tenderness
on bimanual palpation.
3- Lower genital tract infection.
4- Possibility of gynecological malignancy for fear of disserlination through the tubes or through
the vascular system.
5- Uterine bleeding or menstrual flow to avoid remobilization as vessels are opened and to avoid
iatrogenic endometriosi s.
6- Allergy to contrast material (iodine).
7- Possibility of pregnancy: i.e. in cases of amenorrhea pregnancy should be excluded.
8- If performed at the proliferative phase might give false negative results because of thick
endometrium that blocks the tubal cornua.

Cannula inserted into the cervical canal


WhiteKnightLove

-r98-
'It is more blessed to give than to receive.

Imrnediate film, normal findings at hysterosalpingography delayed film

Procedure of HSG:
l. Timing: HSG is performed duringthe early proliferative phase i.e.2-5 days afterthe end
of the menstrual flow, when the endometrium is least vascular.
2. Sterile technique is essential. HSG is perforrned in the Inragirrg Departrnent on the X-ray
table. No anesthetic is needed br-rt antiprostaglandin could be given to avoid uterotubal
spasm.
3. Radio-opaque contrast rnaterial is injected through the cervical canal into the uterine
cavity and the fallopian tubes. Vaginal speculum, volsellum, syringe and cervical cannula
are used to fill the genital tract. Tl-re average volume needed is 5-10 ml dependingon the
size of the uterus. The use of the fluoroscopy helps to judge accurately the amount
needed.
4. Two films at least are taken:
- Irnrnediate after injection: to show the uterus and tubes.
- Second film: to show the peritoneal spill 24 hours after injection of oil soluble media
or 30 minutes after injection of water soluble rnedia.
5. The radio-opaque material could be either of the following:
o Oil soluble (Lipidol: 40 o/o iodine in poppy seed oil).
o Water soluble (urographin: 40 %o iodine in water): used instead of the oil soluble
media for fear of oil embolism, whiclr is very rare. It is rapidly absorbed and the
shadow inside the peritoneum is not as clear as that r,vith oil rnedia (peritubal
adhesions may not be visualised clearly).
6. Fluoroscopic monitor and contrast films X-ray films are used for assessment of the flow
of the dye into the uterine cavity and the fallopian tLrbes and their passage to the
peritoneal cavity.
Normally the dye reach the firrbriated end of the tube and a spill appears into the pelvic cavity.
Limitations of HSG:
- It is not as accurate as laparoscopy. It has high false-positive rate because of transient tubal
blockages induced by smooth muscle spasm.
- It also has a false-negative rate.High false-positive rate because of transient tubal blockages
induced by smooth muscle spasrr.
Complications of HSG:
l- Pain and neurogenic shock in sensitive women.
2- Ascending infection that rnay leads to salpingitis, peritonitis. New pathogens might be
WhiteKnightLove

introduced or flaring of silent subclinical infection.


3- Chronic oil granulorra in the endometriutn or in the tubes.
4- It might exacerbate ongoing chronic infection especially salpingitis.
-r99-
Freely you have received; freely give.

5- If it is performed in the secretory phase it might induce the following complications:


- Disturb an early pregnancy.
- Causes reflux of fragments endometrial tissue that is detached and induce endometriosis.
- Oilembolism.
6- Traumatic lesions induced by rough manipulations: cervical lacerations by the volsellum,
perforation of the uterus by the cannula.
7- Possible allergic reaction to the dye (iodine),
Values of HSG in infertile patients:
o Evaluate tubal factor of infertility: HSG reveal tubal patency vs. occlusion. In case of occlusion it
clarif, the site of tubal obstruction, hydrosalpinx, and Salpingitis isthmica nodosa (SIN),
peritubal and pelvic adhesions. The dye accumulates proxirnal to the blockage
o Evaluate peritoneal factor of infertility: HSG reveals the nonnal peritoneal spill of the dye and
the normal distribution of the dye on delayed (after24 hours) radiograph. The delayed film rnay
show evidences of pelvic adhesions (peritubal adhesions).
o Evaluate uterine factors of infertility: HSG reveal the size, shape, and position of the uterus: It
documents any of the following: intrauterine adhesions, congenital uterine malformations,
submucous leiomyomas, intrauterine polyps.
o Occasionally it has a curative value: reports documented increased conception rate after the
procedure, probably because of possible clearance of the tubal lumens of thickened mucus or
relief of r.rterotubal spasm.

WhiteKnightLove

-200-
'It is more blessed to give than to receive.

APPENDIX 4

INTRAUTERINE DEVICES
Intrauterine devices (lUDs) are flexible polyethylene plastic devices inserted into the uterine
cavity for contraception. They are either Inert (rarely used nowadays except in China) or
Medicated (Copper or Hormone releasing IUDs).
Most intrauterine devices (lUDs) being inserled today are shaped Iike a "T" with copper wires
or bands on the plastic stem and arms. The TCu 3804. is currently one of the most widely used
copper IUDs in the world.
Mechanism of action:
- Neither ovulation nor steroidogenesis is affected.
- It is believed that the IUD prevents pregnancy primarily by causing local reaction in the
uterine cavity, making it hostile to sperm and possibly to egg.
- Some evidences suggest that the IUD acts before fertilization, possibly by affecting ovum
transport.

Intrauterine devices (lUDs)

Advantages of IUD:
- Very safe and highly effective (99%), Failure rate l-2/HWY,
- Easy to use, requiring no action at the tirne of intercourse or at any other time.
- Long-acting (10 years) but easily reversible, with return to fertility typically occurring very
soon after removal.
- No systemic side effects.
- Complications, such as perforation or expulsion, are rate.
- Can be used safely by breastfeeding women.
Side effects of IUDs:
. Cramping and increased or prolonged nrenstrual bleeding.
. Possible bleeding between nrenstrual periods.
. Side effects are nlore corrrnon during the first few nronths alter insertion of a copper IUD, but
they usually decrease with time.
Disadvantages of the IUDs:
- Can cause side effects.
- Trained health provider is needed to insert, follow up and remove the device.
- Is not recommended for women at risk of STIs as it offers no protection against these
WhiteKnightLove

infections including HIV.

-201-
Freely you have received; freely give.

IUD Complications:
l. Pelvic inflammatory disease (PID):
- IUD use has been associated with a small increased risk of developing PID during the first
month after insertion.
- Risk is presumed to be due to the introduction of bacteria fronr the Iower genitaltract into the
uterus as the IUD is being inserted,
- Risk can be substantially reduced if providers screen potential users for symptoms and signs
of current STIs, and assess for their STI risk. Also, sterile techniques during insertion should
be followed.
2. Perforation:
- Rare, but potentially serious event.
- Risk is directly Iinked to the skilland experience of the provider, and can be reduced through
supervised training.
- Risk is greater for postpartum insertions performed between 48 hours and 4 weeks after
delivery. For this reason, it is recommended that postpartum insertions be done within the first
48 hours after delivery, or postponed until 4 weeks after delivery.
3. Intrauterine pregnancy:
- A woman who presents with an IUD in place and an amenorrhea, should have a pregnancy
test and examination:
o If intrauterine pregnancy is diagnosed and the IUD strings are visible, the IUD should be
removed as soon as possible in order to prevent later septic abortion, premature rupture of
the membranes, and premature birth. Spontaneous abortion risk of 50%if the IUD is left
and 25%o if removed.
o Ifthe strings of tlie IUD are notvisible, an ultrasound examination should be performed to
localize the IUD and determine whether expulsiort has occurred. If the IUD is present,
there are two options for management: Continuation of the pregnancy with the device left
in place or therapeutic abortion after good counseling.
4. Ectopic pregnancy:
- Overall, TCu 380A protects frorn ectopic pregnancy. Incidence in users of TCu 380A is very
low, only 2 out of every 10.000 women.
- If a pregnancy does occur in an IUD user, the risk of it being ectopic is h igher than in women
using some other contraceptive methods.
5. Expulsions:
- Partial or unnoticed expulsion may result in irregular bleeding, pain or pregnancy.
- Factors affecting expulsion rates include provider's inserlion skills, a woman's (young) age
and nulliparity, length of time since insertion and timing of inserlion.
. Timing of IUD insertion:
- Interval insertion:
o During menstruation.
o Anytime during nrenstruaI cycle as long as there is a definite proof that the woman is not
pregnant.
- Postparlum insertion:
o Immediately after vaginal delivery (within l0 minutes after delivery of placenta) if
no infection or hemorrhage, or during cesarean section,
o After 6 weeks,
WhiteKnightLove

- Inseftions after abortion:


o Immediately if no infection.
o After 4 to 6 weeks if the pregnancy was 16 weeks or more.
-202-
'It is more blessed to give than to receive.

o Contraindications of IUD use:


- The copper IUD should not be used by women with:
. Current or recent STIs, PID or purulent cervicitis.
. Unexplained vaginal bleeding.
. Cervical, endometrial or ovarian cancer.
. Malignant trophoblastic disease or known pelvic tuberculosis.
. Uterine distortion that prevents correct IUD placenrent.
' Infection following childbirth, abortion or incomplete abortion.
- The copper IUD use is recommended for women rvith:
. Increased risk for STIs.
. Benign trophoblastic disease.
' Nulliparous wonten: This is a current recommendation in Egypt, however, nulliparous
women can use an IUD according to the WHO eligibility criteria,
Hormone releasing intrauterine system (IUS):
-
The hormone releasing intrauterine (lUS) is a newer form of Hormonal contraception.
Like the rnost widely used lUDs, it has a "T" shape and is inserted into the uterus. Unlike an IUD,
however, the tUS contains progestin that is released dilectll, into the uterus. The IUS is not a
substitute for the copper-bearing IUD. However, it can be used if a woman has: excessive bleeding
with a copper IUD that she can't tolerate, still rvants to use an intrauterine device and can afford the
high price of a honnone-releasing IUS. In this case, she should be counseled that she willbe using a
hormonal rather than a non-hormonal rnethod, and prior to irritiation the provider must rule out any
conditions that contraindicate IUS use.
Mechanisms of action of IUS:
- Thickening of cervical t.t-tucus, making it difficult for sperm to enter the uterine cavity,
- Partial suppression of ovulation.
Advantages of IUS:
- Safe and highly effective
- Easy to use; no action reqr-rired at the tirne of intercourse, or any other time
- Long-acting (5 years), but reversible
- Has non-contraceptive health benefits, such as reducing the duration and quantity of
menstrual bleeding and pain; possibly provides protection frorn pelvic inflammatory disease
(PrD)
Disadvantages of IUS:
- Provider's help needed irr order to initiate or discontinue
- Side effects, such as irregular bleeding and spotting, are colllnton
- Provides no protection fronr STIs, including HIV
- Expensive
- Not yet available in many countries
WhiteKnightLove

-203-
Freely you have received; freely give.

Fie. 1: Fenestrated Cusco's vaginal sDeculum Cusco's vaginal speculum

Fis. 2: Sim's soeculum Fis. 3: Auvard's vaeinal speculum

Fis. 4: The uterine sound Fie. 5: Hesar's dilator

Fie. 6: Sharp uterine curette Fie. 7: Double ended uterine curette

t-

Fis. 8: Blunt flushins curette Fis. 9: Fundal curette


WhiteKnightLove

-204-
'It is more blessed to give than to receive.

nU
10: Volsellum forceps (sinele toothed 11: Volsellum forceps (multiple toothed

Fis. 12: Bonney's mvomectomy clamp Fis. 15: Myomectomy screw


<nrt
Se'/r

Fig. 14: Insufflation cannula (ordinary type) Fis. 15: Cone shaped screw cannula of different sizes

Fig. 16: Uterus holding forceps Fig. l7: Intrauterine contraceptive devices Ieft to
right are lippes loop (size D), Cu 7, and copper T
(380A)

metal

rubber
Fis. 18: Female catheter
WhiteKnightLove

-205-
Freely you have received; freely give.

OBSTETRICAL INSTRUMETNS

L2. OVUM FORCEPS


(Fig. le)

Fig. 19: Ovum forceps

13. RING FORCEPS


(Fig.20)

. It is used to grasp the cervix especially during exploration after difficult instrumental or
destructive operation to detect cervical trauma.
. It is used instead of the ovum forceps to remove the products of conception during vaginal
evacuation. Also it is used to remove corporeal or cervical polypi after twisting them.

Fig. 20: Ring forceps

L4. THE OBSTETRIC FORCEPS


o It is an instrument devised for delivery of a Iiving fetus per vagina.
Types:
14. A. Long Forceps:
I. Long curved Forceps:
a. Without axis traction piece.
b. With axis traction piece: Neivelle-Simpson-Barnes (Fig. 21).
c. With axis traction rods: Milne-Murray's forceps.
WhiteKnightLove

Fig. 2l: Long curved obstetric forceps (Neivelle-Simpson-Barnes)

-206-
'It is more blessed to give than to receive.

II. Long sStraight Forceps:


Kielland's forceps (Fig.22)

Fig. 22: Kielland's forceps

14. B. Short Forceps:


I. Short curved (By Wrigley): With pelvic curve (Fig. 23).
II. Short straight (By Simpson): Without pelvic curve (Fig. 24).

Fig.23: Curved shorl forceps (by Wrigley) Fig.24: Straight short forceps (by Simpson)
The long curved forceps is formed of:
l. Handle.
2. Shank.
3, Lock.
a. English (double slot) (Fig. 25).
b. French (screw) (Fig. 26).
c. Combination of the above 2 types: German.

lock
Fig. 25: English lock Fig. 26: French lock

4. B lade.
1. The value of the pelvic curve is to obtain a good central grip, to avoid extension of the fetal
head and avoids misdirection of the line of traction.
2. The value of axis traction piece is to allow traction on the fetal head in the axis of the pelvis
whatever its level in the pelvis,
3. Its action could be simurlated by Pajot's maneuver.
4. In the straight types no right or left blade but any of tlrenr is applied to one side and the other
one to the other side of the head.
5. In the curved types there is left and right blades (according to the side of the maternal pelvis
to which the blade is applied) to be applied correctly, the pelvic curve must be directed
WhiteKnightLove

anteriorly and the cephalic curve medially.

-207-
Freely you have received; freely give.

14. C. Special types of forcePs:


I. Kielland's forcePs (Fie.22):
- It is more or less straight one as it has very slight pelvic curve the effect of which is
nullified by slight bent between the blade and shank'
- It has no right or Ieft blades but as it is used mainly in deep transverse arrest so one blade is
anterior and the other posterior which oriented according to the position gf-lhe.occiput to
which the 2 rnetaI knobs on the handles are directed at apptication (steps of its application).
- It has a stiding lock to allow its application to asyncletic head.
- It is mainty used for delivery by rotation and extraction of the head in deep transverse
arrest.
IL Piper's forcePs (Fig,27,28):
- It has a shank, with a backward curve and designed for detivery of the after coming head of
breech.

Fig.21: Piper's forceps applied to the after Fig. 28: Delivery of after coming head by
coming head, the head has entered the pelvis and Piper's forceps. Note the direction of movement
forceps have been aPPlied (arrow)

Action of the forcePs:


l- Traction: is the main action: A fair rneasure of force applied is traction that can be exerted by the
unaided effort of one forearrn.
2- Compressor
3- Rotator.

Indications of forcePs oPeration:


l- Prolonged second stage rvhich may be due to:
l- Malpresentations:
- OccipitoPosterior.
- DeeP transverse arrest.
- Face presentation.
2- Rigid perineum or Pelvic floor.
3- First degree cephalopelvic disproprotion.
4- Fetal distress or prolapsed pulsating cord in the second stage.
5- Maternal distress or disease (heart disease-eclampsia and previous, c.s)
WhiteKnightLove

6- Uterine inertia,
fetus]'
N.B: In recent obstetrics we avoid prolonged tabor [for sake of the nrother and the

-208-
'It is more blessed to give than to receive.

Conditions to be fulfillcd before forceps application:


l- Fully dilated cervix- no rim is felt in between uterine contractions. If done before full
dilatation:
a) Cervical tears occur which may extend to the lower segnrent Iincomplete ruptured uterus].
b) Failed forceps.
c) Prolapse-later on.
d) Intra cranial hemorrhage - stillbirth or neonatal death.
2- No Major disproportion.
3- Suitable presentation.
- All vertex presentations.
- Mentoanterior.
- After coming head of breech.
4- Engagement of the head.
5- Under anesthesia:
-General or
-Spinal or
-bilateral pudendaI nerve block
6- Empty bladder and rectum.
7- Complete asepsis.
8- The membranes ruptured.
9- The uterus is not in a state of complete inertia,
Types of forceps operations:
1- Low forceps operation (Fig. 29,30):
. In which a low head is delivered by a short or long curved forceps.
. The low head is that which tlie greatest circumference has passed the ischial spines and
rests on the perineurn with its saggital suture lies in the anteroposterior diameter of the
outlet i.e. rotated head.
. In this type of operation the cornplications [Maternal and fetal] are minimal and the best
type of forceps application is attained [cephalopelvic application].

Fig. 29: Low forceps operation: Fig. 30: Occiputoanterior delivery by low
Introduction of the left blade. forceps (Sirnpson) the direction of gentle
traction for delivery of the head is indicated

l- Cephalic:the blades applied to the sides of the head disregarding tlre pelvis.
2- Plevic: the blades applied to the sides of the pelvis disregarding the head"
WhiteKnightLove

Both carry risks (rnaternaI and fetat) so they are discarded and we use the safest (cephalo-pelvic
aaplication).

-209-
Freely you have received; freely give.

2- Mid-forceps operation :
- More difficult, as the head is higher than the low type.
- Needed rvhen the greatest diarneter of the head is at or just above the ischial spines.
- Usually the head needs rotation before forceps applications.
3- High forceps operation [obsolete]: We resort to C.S as a safer method or delivery both for
the mother and fetus.
RISKS OF THE FORCEPS OPERATION
I- Maternal
I - Trauma:
. Cervical tears:
- Unilateral [more to the left].
- Bilateral
- Stellate.
. Vaginal tears
. Perineal tears
. Ruptured lower segnrent
o To the bladder
2- Sepsis
3- Obstetric shock.
4- Post-partum hernorrhage [atonic or traumatic].
5- Complications of anesthesis.
6- Rare - bony injuries specially to the coccyx and lower end of sacrum.
II- Fetal:
1- Excessive compression - intracranial hemorrhage due to:
o Wrong application: the worst is the application to the occipito frontal diameter of the head.
. Applied to a prenrature fetus.
. Undue force is used,
. Applied too early irr labor before fulfillrnents are present. .
2- Fractures ofskull bones.
3- Cephalohematorna.
4- Nerve lesions;
. Facial nerve compression (Bell's palsy)
o Brachial palsy.
5- Asphyxia due to:
. Cord courpressiorr
o Anesthetic complication
FAILED FORCEPS: Carrses misdiagnosing the patient as fulfilling the criteria for forceps application as in:
l- Incomplete dilatation of the cervix.
2- Undiagnosed occipitoposterior.
3- Unsuspected pelvic contractiorr [at the outlet as well as the brirn].
4- Unsuspected malpresentation: as brow or mentoposterior.
5- Unsuspected hydrocephales.
6- Contraction ring,
7- Shoft cord.
WhiteKnightLove

8- Obstructed labor with huge caput succidanium which falsely diagnosed as engaged head while
the head is high or even abdorninal in its station.

-210-
'It is more blessed to give than to receive.

15. DOYEN'S ABDOMINAL RETRACTOR


(Fig.31)
. Used mainly in L.S. C. S. to retract the urinary bladder.

Fig. 3l: Doyen's abdominal retractor

t6. BOZEMANN'S PACKING FORCEPS


(Fig.32)
o It is used for packing the uterine cavity and or the vagina.

Fig. 32: Bozemann's packing forceps

17. METAL MUCUS CATHETER


(Fig, 33)
. Used to clear the mouth, nasopharynx and larynx of the fetus immediately after delivery from
lnucr.ls, aspirated am n iotic fluid and meconium.

Fig.33: Metal mucous catheter

18. THE VACUUM EXTRACTOR [VENTOUSE]


@ig.3a)
The idea of this instrument is to apply traction to the fetal head through a form of suction cup,
- It is introduced by Malmstrom (1957),
- It has a cup [3sizes 40, 50 and 60 mm diameter] attached to a vacuum pump.
- It could not be applied to face, brow, transverse lie or breech.
- The forceps is more suitable if urgency is needed [as fetal distress],
- It could be applied late in the first stage [before full dilatation of the cervix] and also on un'
WhiteKnightLove

rotated occipito posterior.

-2tt-
Freely you have received; freely give.

Fig. 34: Malmstrorn-vacuum extractor with rnetal cups, vacuum pump, and pressure gauge

Dangers:
l- Injury:
. To the cervix.
o To the vagina
. To the fetal head
- Scalp laceration
- Cephalohematoma.
2- Prolapse of the uterus later on.

19. METHODS OF HEARING FETAL HEART SOUNDS


l) Pinard's fetal stethoscope (Fig. 35). It is made of metal or plastic oi wooden material.
2) Ultrasonic Doppler effects [sonicad].
3) The traditional adult stethoscope: sometimes the fetal heart sounds could be heard with them.

Fig. 35: Pinard's fetal stethoscope

Significance of F.H.S. dctection:


1- Sure sign ofpregnancy.
2- To insure that the fetus is alive.
3- Diagnosis of fetal distress [Tachycardia above 160 rninute or bradycardia below 100/minute or
irregular rhythml.
N.B: The sonicaid is nrore sensitive for fetal heart sounds detection, In high risk pregnancythe
labor should be continuously monitored by recording the fetal heart beats in relation to uterine
contraction and fetal scalp pH measurement. If needed to diagnose fetal distress [see
aminoscope]
4- To confirm the diagnosis of fetal presentation and position by the determining the site of
maximum intensity of the heart sounds.
5- Help to confirnr multiple pregnancy.
WhiteKnightLove

-212-
'It is more blessed to give than to receive.

Fig. 19: Ovum forceps Fig. 20: Ring forceps

Fig. 21: Long curved obstetric forceps (Neivelle-Simpson-


Fig. 22: Kielland's forceps
Barnes

Fig.24: Straight short forceps (by


Fig. 23: Curved short forceps (by Wrigley)
Simpson)

Fig. 25: English lock Fig. 26: French lock

Fig, 28: Delivery of after coming head by


Fig,27: Piper's forceps applied to the after coming head, the
Piper's forceps. Note the direction of
head has entered the pelvis and forceps have been applied
movement (arrow)
WhiteKnightLove

-213-
Freely you have received; freely give.

Fig. 30: Occiputoanterior"dslivery by low


Fig.29: Low forceps operation: Introduction of
forceps (Simpson) the direction of gentle
the left blade.
traction for delivery of the head is indicated

Fig. 31: Doyen's abdominal retractor Fig.32: Bozemann's packing forcep

Fig. 34: Malmstrom-vaccum extractor with


Fig.33: Metal mucous catheter
metal cups, vaccum pump, and pressure gauge

Fig. 35: Pinard's fetal stethoscope

WhiteKnightLove

-214-

You might also like